Sunteți pe pagina 1din 412

Florida Appellate Practice*

Florida State University College of Law


Fall Semester 2011

Charles A. Stampelos
Adjunct Professor

*These materials were prepared by the Honorable Philip J. Padovano who serves
as a Judge on the First District Court of Appeal in Tallahassee, Florida, and used
by Judge Padovano when he taught this course, most recently this past spring at the
Florida State University College of Law.
Table of Contents
1. Jurisdictional Principles ........................................................................................... 1

A. Definitions .................................................................................................................. 1
B. Justiciability ................................................................................................................ 1
C. Mootness ..................................................................................................................... 1
1. Examples of Mootness ......................................................................................... 3
2. Exceptions to Mootness ....................................................................................... 4
D. Waiver or Consent .................................................................................................... 10
E. Jurisdiction Pending Review .................................................................................... 15
F. Extent of Jurisdiction ................................................................................................ 19
G. Objections ................................................................................................................. 23

II. Invoking Appellate Jurisdiction ................................................................................ 23

A. Jurisdictional Papers ................................................................................................. 23


B. Timeliness of Review ............................................................................................... 23
C. Rendition................................................................................................................... 34
D. Motions Extending Rendition .................................................................................. 39
E. Premature appeals ..................................................................................................... 49
F. Filing in the Proper Forum ........................................................................................ 51
G. Nonjurisdictional Matters ......................................................................................... 60

III. Subject Matter Jurisdiction ..................................................................................... 60

A. The Florida Supreme Court ...................................................................................... 60


1. Judgments Imposing the Death Penalty ............................................................. 65
2. Decisions Declaring State Laws Invalid ............................................................ 66
3. Bond Validation Proceedings ............................................................................ 66
4. Utility Cases – Rate or Service Orders .............................................................. 66

i
5. Decisions Declaring State Laws Valid .............................................................. 66
6. Construction of State or Federal Constitution ................................................... 67
7. Decisions Affecting a Class of Constitutional Officers ..................................... 67
8. Express and Direct Conflict of Decisions .......................................................... 67
9. Certified Decisions of District Courts................................................................ 77
10. Certified Decisions of Trial Courts .................................................................... 83
11. Questions Certified by Federal Courts ............................................................... 83
12. Prohibition .......................................................................................................... 83
13. Mandamus .......................................................................................................... 83
14. Quo Warranto ..................................................................................................... 83
15. Habeas Corpus ................................................................................................... 84
16. All Writs Jurisdiction ......................................................................................... 84
B. The District Courts of Appeal .................................................................................. 86
1. Final Orders of Trial Courts ............................................................................... 88
2. Nonfinal Orders of Circuit Courts ..................................................................... 89
3. Direct Review of Administrative Action ........................................................... 91
4. Certiorari ............................................................................................................ 92
5. Discretionary Review......................................................................................... 92
6. Extraordinary Writs............................................................................................ 92
C. Appellate Jurisdiction of the Circuit Courts ............................................................. 92
1. Final Orders of Lower Tribunals ....................................................................... 93
2. Nonfinal Orders of Lower Tribunals ................................................................. 93
3. Direct Review of Administrative Action ........................................................... 96
4. Discretionary Review of Nonfinal Orders ......................................................... 97
5. Extraordinary Writs............................................................................................ 97

ii
IV. Preservation of Error............................................................................................... 97

A. Contemporaneous Objection Rule............................................................................ 97


B. Motions in Limine .................................................................................................. 101
C. Offers of Proof ........................................................................................................ 105
D. Fundamental Error .................................................................................................. 108
E. Invited Error ............................................................................................................ 110

V. Initiating Review ....................................................................................................... 118

A. Proper Forum .......................................................................................................... 118


B. Appellate Districts .................................................................................................. 118
C. Correct Remedy ...................................................................................................... 120
D. Procedure ................................................................................................................ 121
1. Appeals............................................................................................................. 121
2. Discretionary Review....................................................................................... 128
3. Original Proceedings ........................................................................................ 130
E. Proceedings by Indigents ........................................................................................ 130

VI. Parties ....................................................................................................................... 131

A. Entitlement to Review ............................................................................................ 133


1. Parties in the lower tribunal ............................................................................. 133
2. Standing ........................................................................................................... 136
B. Waiver – Acceptance of Benefits Rule................................................................... 138
C. Joinder..................................................................................................................... 140
D. Substitution ............................................................................................................. 143
E. Intervention ............................................................................................................. 144

VII. Stays ........................................................................................................................ 145

A. General Procedure .................................................................................................. 147

iii
B. Supersedeas Bonds ................................................................................................. 151
C. Money Judgments ................................................................................................... 152
D. Review by Public Parties ........................................................................................ 153
E. Stay of Appellate Decisions .................................................................................... 165
F. Review of Stay Orders ............................................................................................ 166

VIII. The Record............................................................................................................ 167

A. Duty to Produce ...................................................................................................... 170


B. Time for Filing ........................................................................................................ 172
C. Contents of Record ................................................................................................. 174
D. Statement of the Evidence ...................................................................................... 175
E. Transcript of Proceedings ....................................................................................... 176
F. Supplementing the Record ...................................................................................... 177

IX. Motions ..................................................................................................................... 180

A. Time Limits ............................................................................................................ 181


B. Contents of Motions ............................................................................................... 182
C. Motions in the Lower Tribunal ............................................................................... 184
D. Motions for Review ................................................................................................ 185
E. Sanctions ................................................................................................................. 186

X. Dismissal .................................................................................................................... 190

A. Voluntary Dismissal ............................................................................................... 190


B. Settlement ............................................................................................................... 193
C. Involuntary Dismissal ............................................................................................. 193

XI. Briefs......................................................................................................................... 203

A. Duty to File ............................................................................................................. 206

iv
B. Form ........................................................................................................................ 207
C. Time for Service ..................................................................................................... 208
D. Appendix ................................................................................................................ 209
E. Notice of Supplemental Authority .......................................................................... 210
F. Sanctions ................................................................................................................. 214
G. Types of Briefs ....................................................................................................... 215

XII. Oral Argument ....................................................................................................... 217

A. Request for Oral Argument .................................................................................... 217

XIII. Scope of Review .................................................................................................... 218

A. Nature of the Proceeding ........................................................................................ 218


1. Final Orders...................................................................................................... 218
2. Nonfinal Orders................................................................................................ 221
3. Discretionary Review....................................................................................... 223
B. Decisions................................................................................................................. 228
C. Issues....................................................................................................................... 229
D. Arguments – The Tipsy Coachman Rule ............................................................... 230

XIV. Standards of Review ............................................................................................ 244

A. Decisions of Law .................................................................................................... 244


B. Decisions of Fact .................................................................................................... 255
C. Discretionary Decisions .......................................................................................... 260
D. Certiorari................................................................................................................. 274
1. Review of Appellate Decisions ........................................................................ 274
2. Review of Local Administrative Orders .......................................................... 278
3. Review of Unappealable Pretrial Orders ......................................................... 281
E. Harmless Error ........................................................................................................ 284

v
1. Civil Cases ....................................................................................................... 284
2. Criminal Cases ................................................................................................. 287
F. The Two-Issue Rule ................................................................................................ 303

XV. Disposition .............................................................................................................. 311

A. Applicable Law ...................................................................................................... 311


B. Affirmance .............................................................................................................. 312
C. Modification ........................................................................................................... 315
D. Reversal .................................................................................................................. 316
E. Decision and Opinion ............................................................................................. 317
F. Precedents ............................................................................................................... 323
G. Mandate .................................................................................................................. 329
H. Law of the Case ...................................................................................................... 338

XVI. Rehearing .............................................................................................................. 356

A. Grounds .................................................................................................................. 358


B. Time for Filing Motion ........................................................................................... 371
C. En Banc Review ..................................................................................................... 374

XVII. Costs and Attorney Fees..................................................................................... 376

A. Appellate Court Costs............................................................................................. 376


B. Appellate Attorneys’ Fees ...................................................................................... 384
C. Review of Cost and Fee Awards ............................................................................ 402

vi
1. Jurisdictional Principles

A. Definitions

Subject Matter Jurisdiction: “The power of the court to adjudicate the class of cases to
which the particular case belongs.” Hollywood, Inc. v. Clark, 15 So. 2d 175 (Fla. 1943).
The power to adjudicate a particular kind of case could also be characterized as potential
jurisdiction.
Territorial Jurisdiction: The limitation to the geographic area served by an appellate
court. See Egner v. Unemployment Appeals Com’n, 633 So. 2d 1157 (Fla. 1st DCA
1994).
Actual Jurisdiction: A court has actual jurisdiction if the case is within its subject matter
jurisdiction and if its potential judicial power has been invoked by the filing of a timely
notice of appeal or petition. See Dupree v. Elleman, 191 So. 65 (Fla. 1939).

B. Justiciability

The courts do not exercise appellate jurisdiction to resolve abstract issues of law. See
Allen v. Martinez, 573 So. 2d 987 (Fla. 1st DCA 1991). Justiciability requires the
existence of a case or controversy.
Justiciability requires at least two parties who are asserting opposing interests. See
Sarasota-Fruitville Drainage Dist. v. Certain Lands, 80 So. 2d 335 (Fla. 1955).

C. Mootness

Merkle v. Guardianship of Jacoby


912 So. 2d 593 (Fla. 2d DCA 2005)
WALLACE, Judge.
This case is before us on a motion filed by the Department of Veterans
Affairs (the Department) requesting that we vacate our prior ruling FN1 on the
ground of mootness. On April 30, 2004, the Department filed its motion
suggesting that prior to our decision on the merits this case had become
moot by reason of the “Settlement Agreement” (the Agreement) that the
Department had concluded with the appellant, Mr. LeRoy H. Merkle, Jr., on
December 4, 2002. Upon receipt of the Department’s motion and
consideration of Mr. Merkle’s response, we issued an order on June 11,

1
2004, recalling our mandate. The subsequent history of this matter is
detailed in our “Order Imposing Sanctions on LeRoy H. Merkle, Jr., a
Member of The Florida Bar,” which we are issuing the same day as this
order.
FN1. Our prior ruling is reported as Merkle v. Guardianship of Jacoby,
862 So. 2d 906 (Fla. 2d DCA 2003).
[1] [2] We have concluded that the Agreement between Mr. Merkle and
the Department rendered this case moot before our decision on the merits.
Thus the question presented by the Department’s motion is whether we
should withdraw our prior opinion or if some other relief is more appropriate
under the unusual circumstances of this case. Initially, we note that the issue
of mootness does not raise a question about our jurisdiction to decide the
case. The reluctance of the Florida courts to decide moot questions is based
on policy reasons, not lack of jurisdiction. See Cook v. City of Jacksonville,
823 So. 2d 86, 87 n. 1 (Fla.2002); Pleasures II Adult Video, Inc. v. City of
Sarasota, 833 So. 2d 185, 186 (Fla. 2d DCA 2002); see generally Philip J.
Padovano, Florida Appellate Practice § 1.4 (2005 ed.). Thus we had
jurisdiction to decide the case even though it had been settled without our
knowledge prior to our decision on the merits.
[3] A request for the withdrawal of an opinion that has already been
released to the parties and to law reporting services requires consideration of
what should be done once the cat is out of the bag. We believe that our
approach to this problem should be governed by practical considerations. If
we had been advised by the parties immediately after the issuance of our
opinion that the case was already moot by reason of a settlement, we could
have prevented the opinion’s appearance in permanent form in the various
reporting services by withdrawing it then. See, e.g., J.C. v. Garner, 874 So.
2d 694 (Fla. 4th DCA 2004); Seslow v. Seslow, 625 So. 2d 1248 (Fla. 4th
DCA 1993). Under those circumstances, withdrawing the opinion would
have substantially limited its circulation. At this point, the cat is not only out
of the bag, it has curled up and is nestled in one of the bound volumes of the
Southern Reporter. Whether or not we withdraw the opinion, it will be
available to the bench and bar for the foreseeable future and is likely to be
cited. See, e.g., Carolyn S. Grigsby, Lofton v. Kearney: Discrimination
Declared Constitutional in Florida, 21 St. Louis U. Pub.L.Rev. 199, 209 n.
74 (2002) (citing to an opinion of the First District Court of Appeal that
appears in a bound volume of the Southern Reporter despite the fact that the

2
opinion was withdrawn because the appeal had been dismissed as moot).
Thus an announcement now that we are withdrawing our opinion would be
futile. Taking this fact into account, together with the unusual circumstances
of this case, we decline to withdraw our prior opinion.
Of course, Mr. Merkle’s settlement of the case with the Department
before the issuance of our opinion precludes granting him the relief ordered
by our decision. Therefore, the mandate that we will issue on the remand of
this case will direct the circuit court to have such further proceedings as may
be appropriate in view of the Agreement between Mr. Merkle and the
Department. We will not direct the circuit court to grant relief to Mr. Merkle
as ordered by our prior decision.
Upon consideration thereof, IT IS ORDERED that the Department’s
motion is granted in part and denied in part as follows:
1. The Department’s motion is granted to the extent that this court will
not direct the circuit court on remand of this case to grant relief to Mr.
Merkle as ordered by our prior decision.
2. The Department’s motion is denied to the extent that it seeks the
withdrawal of our prior opinion in this case.
NORTHCUTT and DAVIS, JJ., Concur.

1. Examples of Mootness

A. A challenge to the constitutionality of a statute can be rendered moot by


intervening legislation modifying or repealing the statute. See Walker v. Pendarvis, 132
So. 2d 186 (Fla. 1961).
B. The voluntary payment of a money judgment can render an appeal from a
judgment moot. See Feinstein v. New Bethel Missionary Baptist, 938 So. 2d 562 (Fla. 3d
DCA 2006); Frank Silvestri Investments, Inc. v. Sullivan, 486 So. 2d 20 (Fla. 5th DCA
1986); but see Khazaal v. Browning, 707 So. 2d 399 (Fla. 5th DCA 1998) (questioning
the wisdom of this rule).
C. Enforcement of an order that has not been stayed pending appeal can render an
appeal from the order moot. See Coursen v. City of South Daytona, 127 So. 2d 905 (Fla.
1st DCA 1961).

3
D. An appeal from a temporary order can be moot if the order has expired by its
own terms by the time the appeal is considered. See Gray v. Gray, 958 So. 2d 955 (Fla.
1st DCA 2007).

2. Exceptions to Mootness

Godwin v. State of Florida


593 So. 2d 211 (Fla. 1992)
HARDING, J.
We have for plenary review Godwin v. State, 557 So. 2d 955, 956 (Fla.
1st DCA 1990), which certified the following question of great public
importance:
When an individual seeking review of an order of involuntary
commitment has been released from that commitment prior to disposition of
the appeal on the merits, what showing must she make to avoid dismissal of
the appeal on grounds of mootness?
We rephrase the question as follows:
Does an appeal from a civil commitment order under The Baker Act,
section 394.467, Florida Statutes (1989), become moot solely because the
person subject to that order has already been released?
We granted jurisdiction, pursuant to article V, section 3(b)(4) of the
Florida Constitution, and answer the question in the negative.
On July 19, 1989, the trial court below ordered Shirley Godwin
involuntarily committed to the Florida State Hospital. Godwin filed a notice
of appeal attacking her commitment; however, the hospital discharged
Godwin before the First District Court of Appeal decided her appeal. The
State then moved to dismiss Godwin’s appeal on the grounds that Godwin’s
subsequent release made the appeal of the commitment moot. The First
District Court ordered the dismissal, but expressed reservations about this
procedure and certified the question to this Court. Godwin, 557 So. 2d at
956.

4
***
[1] An issue is moot when the controversy has been so fully resolved that
a judicial determination can have no actual effect. Dehoff v. Imeson, 153 Fla.
553, 15 So. 2d 258 (1943). A case is “moot” when it presents no actual
controversy or when the issues have ceased to exist. Black’s Law Dictionary
1008 (6th ed.1990). A moot case generally will be dismissed.
[2] Florida courts recognize at least three instances in which an otherwise
moot case will not be dismissed. The first two were stated in Holly v. Auld,
450 So. 2d 217, 218 n. 1 (Fla.1984), where we said: “[i]t is well settled that
mootness does not destroy an appellate court’s jurisdiction ... when the
questions raised are of great public importance or are likely to recur.” Third,
an otherwise moot case will not be dismissed if collateral legal consequences
that affect the rights of a party flow from the issue to be determined. See
Keezel v. State, 358 So. 2d 247 (Fla. 4th DCA 1978).
Here, we address the issue of collateral legal consequences flowing from
an involuntary commitment. Section 394.457(8), Florida Statutes (1989),
states that “[f]ees and fee collections for patients in treatment facilities shall
be according to s. 402.33.” In turn, section 402.33(8), Florida Statutes
(1989), reads as follows:
(8)(a) Unpaid fees for services provided by the department to a client
constitute a lien on any property owned by the client or the client’s
responsible party which property is not exempt by s. 4, Art. X of the State
Constitution. * * *
The imposition of a lien under section 402.33(8) on the property of an
involuntarily committed person is a collateral legal consequence. In all
probability, a lien will be filed by the Department of Health and
Rehabilitative Services (HRS) long after the expiration of the time for filing
an appeal from an order of commitment. In fact, the discretion as to whether
and when to file the lien rests solely with HRS. Because section 402.33(8)
affects a person involuntarily committed beyond the person’s initial release,
the statute has collateral legal consequences. * * *
The State asserts that Godwin can challenge the lien at the time that HRS
files it, and, until the lien is filed, Godwin has not been subjected to
collateral legal consequences. There are two reasons, however, for allowing
Godwin’s appeal of her involuntary commitment to proceed in the Court.
First, there is no statutory means provided to challenge a lien imposed after
an improper commitment short of challenging the validity of that

5
commitment. Second, Godwin’s appeal is timely. If a person is allowed to
challenge the involuntary commitment only after the lien has been imposed,
the courts will be faced with the problem of reconstructing the record of the
involuntary commitment sometime in the future. The validity of the initial
commitment is best examined while a record is readily available.
In State v. Kinner, 398 So. 2d 1360 (Fla.1981), we held that a person’s
release from confinement made the issue of evidence supporting an
involuntary commitment moot. However, in Kinner, the parties did not raise
the issue of collateral legal consequences, and the Court did not address the
issue which is presented here. Thus, we distinguish Kinner.
[3] Godwin’s appeal is not moot because section 402.33(8) allows for
the imposition of a lien for unpaid fees flowing from an involuntary
commitment, and HRS has not indicated a waiver of its right to impose a
lien. We recognize that other consequences may follow an involuntary
commitment under The Baker Act, such as the stigma that society may
attach, as well as some restrictions on a person’s privileges and
opportunities. See, e.g., § 322.05(5), Fla.Stat. (1989) (restriction on drivers’
licenses); § 97.041(3)(a), Fla.Stat. (1989) (restriction on right to vote); §
790.06(10), Fla.Stat. (1989) (restriction on right to carry a concealed
weapon). While we recognize these consequences are significant, we hold
that they do not rise to the level of collateral legal consequences.
Restrictions imposed as the result of a commitment can be or are removed
when the patient is discharged or released from active treatment. See §
394.469(4), Fla.Stat. (1989).
Thus, we answer the rephrased question in the negative. We quash the
opinion below and remand this case for further proceedings consistent with
this opinion.
It is so ordered.
OVERTON, McDONALD and GRIMES, JJ., concur.
KOGAN, J., concurs in part and dissents in part with an opinion, in
which SHAW, C.J. and BARKETT, J., concur. KOGAN, Judge,
concurring in part, dissenting in part.
***

6
Matter of Dubreuil
629 So. 2d 819 (Fla. 1993)
BARKETT, Chief Judge.
We review In re Dubreuil, 603 So.2d 538 (Fla. 4th DCA 1992), which
held that a married but separated woman who chose not to receive a blood
transfusion for religious reasons could be compelled to receive medical
treatment because her death would cause the abandonment of four minor
children. We quash the district court’s decision because there was no
abandonment proved in this case to override the patient’s constitutional
rights.FN1
FN1. We have jurisdiction pursuant to article V, section 3(b)(3) of the
Florida Constitution to review the district court’s express construction of the
Florida Constitution.
I. The Facts
The parties have agreed on the essential facts in this case. In the late
evening of Thursday, April 5, 1990, Patricia Dubreuil was admitted to
Memorial Hospital in Hollywood, Florida, through its emergency room.FN2
Patricia was in an “advanced stage” of pregnancy. At the time of her
admission, she did not have a private attending physician, so Memorial
Hospital assigned an obstetrician from its staff to render necessary
obstetrical services. Upon admission, Patricia signed a standard consent
form agreeing to the infusion of blood if it were to become necessary.
FN2. Memorial Hospital is a public health care facility owned and
operated by the South Broward Hospital District, a special taxing district
established under Florida law.
By the early morning hours of April 6, physicians determined that
Patricia was ready to deliver her child and that a Caesarean section delivery
would be appropriate. She consented to the Caesarean section, but
notwithstanding the routine consent form she had signed, she withheld
consent to the transfusion of blood on the basis of her values and religious
convictions as a Jehovah’s Witness. Michael Dubreuil was subsequently
delivered by Caesarean section at approximately 5:30 a.m. on April 6.
At the time of delivery Patricia experienced a significant loss of blood
because of a severe blood condition that prevents her blood from clotting
properly. Attending physicians determined that a blood transfusion was
required to save her life, but Patricia still refused to consent. Because of the

7
extreme medical emergency that existed on the morning of April 6, medical
authorities, with police assistance, contacted Luc Dubreuil, Patricia’s
estranged husband. He had not accompanied Patricia when she went to the
hospital hours earlier. When Luc arrived shortly thereafter, he consented to
the blood transfusion. Physicians relied upon Luc’s written consent and
transfused a quantity of blood into Patricia during the morning of April 6.
Luc and Patricia were still married but were separated and living apart
when this incident arose. They are the natural parents of the newborn infant,
Michael, and three other minor children, Cary, Tina, and Tracy, who at the
time, respectively, were twelve, six, and four years old and living with their
mother. Luc was not a Jehovah’s Witness. Luc’s consent was supported by
Patricia’s two brothers, who were not Jehovah’s Witnesses, while Patricia’s
mother, who is a Jehovah’s Witness, backed her daughter’s decision.
After the transfusion early on April 6, physicians apparently believed that
transfusions would continue to be needed. Unsure of its legal obligations and
responsibilities under these circumstances, the hospital petitioned the circuit
court for an emergency declaratory judgment hearing to determine the
hospital’s authority or duty to administer blood transfusions to Patricia over
her objections. A hearing was scheduled for 3 p.m. on April 6. The parties
do not know whether the trial court was aware that a transfusion had already
been given at the time of the hearing, but they believe the trial court was
aware that transfusions would continue to be needed throughout the day.
The trial court conducted the hearing as scheduled, attended by counsel
representing Patricia and the hospital. No testimony was taken, but during
the hearing the hospital’s counsel received a telephone call advising that
Patricia, who had been unconscious, had just become conscious, appeared
lucid, and was able to communicate. When asked at that time whether she
would consent to a blood transfusion, Patricia again refused.
At 3:30 p.m. on April 6, the trial court orally announced judgment in
favor of the hospital, allowing it to administer blood as physicians deemed
necessary. Subsequently, according to an affidavit later executed by Patricia,
the hospital continued to administer blood, and Patricia survived.
The trial court issued a written order on April 11, concluding that there
has been no suggestion as to the means or methods of caring for the four
minor children of Patricia Dubreuil, if she should die. In the absence of some
suggestion or showing as to the availability of proper care and custody of the
four minor children, in the event of the death of Patricia Dubreuil, this court

8
believes that the demands of the state (and society) outweigh the wishes of
Patricia Dubreuil and that every medical effort should be made to prolong
her life so that she can care for her four minor children until their respective
majorities.
In re Dubreuil, No. 90-10561(21), Order at 10 (Fla. 17th Cir.Ct., Apr. 11,
1990). Patricia moved for rehearing, indicating that she continued to object
to blood transfusion and that she had an “extended family as well as friends
who are willing to assist in the rearing of [her] minor children in the event of
her demise.” The Circuit Court denied rehearing on April 12. The Fourth
District affirmed by a 2-1 vote.
[1] Patricia sought discretionary review here, arguing that the decision
below violates her state and federal constitutional rights of privacy, bodily
self-determination, and religious freedom. We recognize that the present
case is moot given that Patricia received blood and was released from the
hospital. However, we accept jurisdiction because the issue is one of great
public importance, is capable of repetition, and otherwise might evade
review. See In re Guardianship of Browning, 568 So. 2d 4, 8 n. 1
(Fla.1990); In re T.W., 551 So. 2d 1186, 1189 (Fla.1989); Holly v. Auld,
450 So. 2d 217, 218 n. 1 (Fla.1984); In re Dubreuil, 603 So. 2d at 540; Wons
v. Public Health Trust of Dade County, 500 So. 2d 679, 684 (Fla. 3d DCA
1987), approved, 541 So. 2d 96 (Fla.1989).
II. The Rights of Privacy and Free Exercise of Religion
We begin our analysis with the overarching principle that article I,
section 23 of the Florida Constitution guarantees that “a competent person
has the constitutional right to choose or refuse medical treatment, and that
right extends to all relevant decisions concerning one’s health.” In re
Guardianship of Browning, 568 So. 2d 4, 11 (Fla.1990); see also In re T.W.,
551 So. 2d 1186 (Fla.1989); Public Health Trust of Dade County v. Wons,
541 So. 2d 96 (Fla.1989). In cases like this one, the privacy right overlaps
with the right to freely exercise one’s religion to protect the right of a person
to refuse a blood transfusion because of religious convictions. Art. I, §§ 3,
23, Fla. Const.
* * * [after a lengthy and comprehensive analysis of the privacy rights at
issue, the court concluded that the lower court’s were in error in requiring
Mrs. Dubriel to receive a transfusion] * * *
For the foregoing reasons, we quash the district court’s decision.
It is so ordered.

9
SHAW, GRIMES, KOGAN and HARDING, JJ., concur.
OVERTON, J., dissents with an opinion. * * *
McDONALD, J., dissents with an opinion, in which OVERTON, J.,
concurs. * * *

Note: The “capable of repetition yet evading review” exception was applied
in Roe v. Wade, 410 U.S. 113 (1973) (concerning the right to an abortion in
the first trimester of pregnancy); and in Nebraska Press Ass’n v. Stuart, 427
U.S. 539 (1976) (involving a prior restraint on publication by the news
media).

D. Waiver or Consent

Peltz v. District Court of Appeal, Third District


605 So. 2d 865 (Fla. 1992)
GRIMES, Judge.
This is a petition for prohibition which seeks to prevent the Third District
Court of Appeal from considering an appeal. We have jurisdiction. Art. V, §
3(b)(7), Fla. Const.
On February 6, 1991, the circuit court entered an order on exceptions to
general master’s report. Arvin Peltz, acting pro se, filed a notice of appeal
from this order on March 11, 1991. On March 15, 1991, the Third District
Court of Appeal entered an order to show cause why the appeal should not
be dismissed as untimely filed. Three days later, Suzanne Peltz served a
notice of cross appeal which was filed on March 21, 1991. On April 1, 1991,
Arvin Peltz served a notice of voluntary dismissal. The district court of
appeal entered an order on April 12, 1991, accepting the voluntary dismissal
but stating that the cross appeal would remain pending. On June 25, 1991,
Arvin Peltz filed a motion to dismiss the cross appeal because of lack of
subject-matter jurisdiction. This motion was denied. Thereafter, Arvin Peltz,
through counsel, filed a renewed motion to dismiss cross appeal. The
appellate court also denied this motion and directed Arvin Peltz to file his
brief by March 27, 1992.
[1] [2] [3] The untimely filing of a notice of appeal precludes the
appellate court from exercising jurisdiction. Hawks v. Walker, 409 So. 2d

10
524 (Fla. 5th DCA 1982); Pitts v. State, 225 So. 2d 352 (Fla. 1st DCA
1969). Further, subject-matter jurisdiction cannot be conferred by waiver or
consent. Board of Trustees of Internal Improvement Trust Fund v. Mobil Oil
Corp., 455 So. 2d 412 (Fla. 2d DCA 1984), approved in part, quashed in
part on other grounds, 492 So. 2d 339 (Fla.1986), cert. denied, 479 U.S.
1065, 107 S.Ct. 950, 93 L.Ed.2d 999 (1987); Ringling Bros.-Barnum &
Bailey Combined Shows, Inc. v. State, 295 So. 2d 314 (Fla. 1st DCA), cert.
denied, 303 So. 2d 644 (Fla.1974). On the other hand, the time for filing a
notice of cross appeal is not jurisdictional and may be extended by the
appellate court. Agrico Chemical Co. v. Department of Environmental
Regulation, 380 So. 2d 503 (Fla. 2d DCA 1980); Brickell Bay Club
Condominium Ass'n v. Forte, 379 So. 2d 1334 (Fla. 3d DCA 1980).
[4] Arvin Peltz’s original notice of appeal was clearly untimely. While
the notice of cross appeal was filed within ten days of the filing of the notice
of appeal, it was not filed within thirty days of the order on exceptions to
general master’s report. Therefore, it could not provide an independent basis
for jurisdiction. Moreover, because Arvin Peltz’s original notice of appeal
did not vest the appellate court with jurisdiction to proceed, there was no
jurisdictional basis upon which the notice of cross appeal could be based.
The Third District Court of Appeal itself, while addressing the issue in a
slightly different context in Breakstone v. Baron’s of Surfside, Inc., 528 So.
2d 437, 439 (Fla. 3d DCA 1988), explained:
This court’s jurisdiction to entertain an appeal is invoked solely by the
notice of appeal which must timely seek review of an appealable trial court
order or orders. Hawks v. Walker, 409 So. 2d 524 (Fla. 5th DCA 1982);
Dibble v. Dibble, 377 So. 2d 1001 (Fla. 3d DCA 1979); § 59.081(2),
Fla.Stat. (1985). The notice of cross appeal, on the other hand, is not a
jurisdiction-invoking document, but instead is in the nature of a cross
assignment of error. See supra cases collected at note 1. It therefore follows
that the cross appeal must necessarily “piggy back” jurisdictionally on the
notice of appeal....
[5] This Court’s prohibition power is properly invoked when it appears
that a district court of appeal is about to act in excess of its jurisdiction. State
ex rel. Cantera v. District Court of Appeal, Third District, 555 So. 2d 360
(Fla.1990); State ex rel. Sarasota County v. Boyer, 360 So. 2d 388
(Fla.1978). The Third District Court of Appeal has no jurisdiction to proceed
in this case. Therefore, we grant prohibition.

11
It is so ordered.
BARKETT, C.J., and OVERTON, McDONALD, SHAW, KOGAN and
HARDING, JJ., concur.

Polk County v. Sofka


702 So. 2d 1243 (Fla. 1997)
PER CURIAM.
[1] This case is before us on a question certified by the district court as
one of great public importance. Polk County v. Sofka, 675 So. 2d 615 (Fla.
2d DCA 1996). However, because we conclude that the district court lacked
jurisdiction to hear the appeal, we do not address the certified question.
Instead, we quash the decision of the district court, and remand with
directions that the district court dismiss the appeal for lack of jurisdiction.
Respondent, Donna M. Sofka, sued petitioner, Polk County, in tort, to
recover for personal injuries sustained in an automobile accident. She
alleged that the County had created a dangerous intersection, tantamount to a
hidden trap of which she was unaware, but of which the County was (or
should have been) aware, yet failed to warn. A jury trial resulted in a verdict
for Sofka.
The County filed a motion requesting a new trial, which was granted. The
parties subsequently executed a settlement agreement by which they agreed,
among other things, that a final judgment would be entered in favor of
Sofka, following which the County might seek appellate review of two
issues:
[1] The Trial Court’s refusal to grant POLK COUNTY’s Motion to
Dismiss, to enter summary judgment for POLK COUNTY, or to direct a
verdict against SOFKA, by virtue of POLK COUNTY’s sovereign
immunity, which POLK COUNTY asserts immunizes it from any liability
for the accident....
[2] The Trial Court’s refusal to direct a verdict against SOFKA, by virtue
of POLK COUNTY’s assertion that SOFKA failed to adduce sufficient
evidence showing any alleged fault of POLK COUNTY was the proximate
cause of the accident or any of SOFKA’s damages stemming therefrom.
The agreement provided, further, “that the intermediate appellate court
has jurisdiction to hear POLK COUNTY’s appeal of the Stipulated Final

12
Judgment”; that “[t]he record on appeal shall be the record as it exists at the
time of the entry of the Stipulated Final Judgment”; and that, “if the
intermediate appellate court, for any reason, determines there is no
jurisdiction or standing, or if the appeal is not dispositive of the issue of
POLK COUNTY’s liability for the accident, ... the Stipulated Final
Judgment shall be void,” and “the parties shall be entitled to again proceed
to trial” pursuant to the order granting the County’s motion for a new trial.
The trial court entered the judgment contemplated by the agreement,
incorporating by reference all of the terms of that agreement.
After oral argument in this Court, the parties were asked to brief the
following question:
Whether the District Court of Appeal had jurisdiction to hear Polk
County’s appeal from rulings made before its motion for new trial had been
granted because it had entered into a stipulated final judgment permitting it
to seek such review, notwithstanding that the order granting the motion for
new trial had not been vacated and that one is generally deemed to have
waived the right to review of rulings made prior to, or during, a previous
trial by moving for and receiving a new trial.
The parties responded with a joint brief in which they asserted that the
district court had jurisdiction because (1) the stipulated final judgment
incorporated the settlement agreement, which expressly provided “that the
intermediate appellate court has jurisdiction over the appeal”; (2) the parties
had agreed to waive any “procedural hurdles” to appellate review; (3) the
stipulated final judgment “nullifies or makes the issue of the new trial
moot”; and (4) a contrary conclusion “will result in a waste of judicial
resources.”
By their settlement agreement, the terms of which were incorporated into
the judgment, the parties stipulated that “[t]he record on appeal shall be the
record as it exists at the time of the entry of” the judgment. Prior to the entry
of the judgment, neither party requested that the order granting a new trial be
set aside, and that the County be permitted to withdraw its motion for a new
trial. Accordingly, but for the settlement agreement, the County would not
have been permitted to seek review of the denial of its motions to dismiss,
for summary judgment or for directed verdict based on sovereign immunity,
or the denial of its motion for directed verdict because the evidence was
legally insufficient to establish proximate cause. This is because, having
requested and received a new trial, the County is deemed to have waived its

13
right immediately to seek appellate review of rulings made prior to, or
during, the previous trial. It would have had to await the outcome of the new
trial it had requested and, if adverse, then seek review. See Atlantic Coast
Line R.R. Co. v. Boone, 85 So. 2d 834 (Fla.1956) (because an order granting
a new trial has the effect of completely revitalizing the case for further
proceedings, the party requesting, and receiving, a new trial may not seek
immediate review of an earlier order denying its motion for a directed
verdict); Crown Pontiac, Inc. v. Bell, 547 So. 2d 290 (Fla. 2d DCA 1989) (a
party requesting a judgment notwithstanding the verdict or, alternatively, a
new trial may not seek review of the order denying the motion for judgment
notwithstanding the verdict following the granting of its alternative motion
for a new trial).
[3] It is clear that the parties have stipulated to the district court’s
jurisdiction. However, it is equally clear “that the parties cannot stipulate to
jurisdiction over the subject matter where none exists.” Cunningham v.
Standard Guar. Ins. Co., 630 So. 2d 179, 181 (Fla.1994). See also Snider v.
Snider, 686 So. 2d 802, 804 (Fla. 4th DCA 1997) (“Subject matter
jurisdiction is conferred upon a court by a constitution or statute, and cannot
be created by waiver, acquiescence or agreement of the parties.”). Thus, the
fact that the parties stipulated to the district court’s jurisdiction has no
bearing on whether, in fact, such jurisdiction existed.
The parties’ agreement expressly provides that “[t]he record on appeal
shall be the record as it exists at the time of the entry of the Stipulated Final
Judgment.” When the judgment was entered, the order granting the County’s
motion for a new trial remained in effect, as neither party requested that it be
vacated, and that the motion for a new trial be treated as withdrawn. Thus,
based upon the record to which the parties agreed, the district court lacked
jurisdiction to hear the appeal, notwithstanding the parties' attempt to confer
such jurisdiction.
[4] It is true, as the parties state, that this conclusion “will result in a
waste of judicial resources.” However, “[c]ourts are bound to take notice of
the limits of their authority and if want of jurisdiction appears at any stage of
the proceedings, original or appellate, the court should notice the defect and
enter an appropriate order.” West 132 Feet v. City of Orlando, 80 Fla. 233,
239, 86 So. 197, 198-99 (1920). This is because the limits of a court’s
jurisdiction are of “primary concern,” requiring the court to address the issue
“sua sponte when any doubt exists.” Mapoles v. Wilson, 122 So. 2d 249, 251

14
(Fla. 1st DCA 1960). Thus, while the resulting “waste of judicial resources”
is regrettable, in the absence of jurisdiction, it is unavoidable.
We quash the decision of the district court and remand with directions
that the district court dismiss the appeal for lack of jurisdiction.
It is so ordered.
KOGAN, C.J., OVERTON, SHAW, HARDING and ANSTEAD, JJ.,
and KAHN and WEBSTER, Associate Justices, concur.
GRIMES and WELLS, JJ., recused.

E. Jurisdiction Pending Review

Florida Rules of Appellate Procedure

Rule 9.600. Jurisdiction of Lower Tribunal Pending Review


(a) Concurrent Jurisdiction. Only the court may grant an extension of
time for any act required by these rules. Before the record is transmitted,
the lower tribunal shall have concurrent jurisdiction with the court to render
orders on any other procedural matter relating to the cause, subject to the
control of the court.
(b) Further Proceedings. If the jurisdiction of the lower tribunal has been
divested by an appeal from a final order, the court by order may permit the
lower tribunal to proceed with specifically stated matters during the
pendency of the appeal.
(c) Family Law Matters. In family law matters:
(1) The lower tribunal shall retain jurisdiction to enter and enforce
orders awarding separate maintenance, child support, alimony, attorneys'
fees and costs for services rendered in the lower tribunal, temporary
attorneys' fees and costs reasonably necessary to prosecute or defend an
appeal, or other awards necessary to protect the welfare and rights of any
party pending appeal.
(2) The receipt, payment, or transfer of funds or property under an
order in a family law matter shall not prejudice the rights of appeal of any
party. The lower tribunal shall have the jurisdiction to impose, modify, or
dissolve conditions upon the receipt or payment of such awards in order to
protect the interests of the parties during the appeal.
(3) Review of orders entered pursuant to this subdivision shall be
by motion filed in the court within 30 days of rendition.

15
(d) Criminal Cases. The lower tribunal shall retain jurisdiction to consider
motions pursuant to Florida Rules of Criminal Procedure 3.800(b)(2) and
in conjunction with post-trial release pursuant to rule 9.140(h).

Rule 9.020. Definitions


***
(c) Court. The supreme court; the district courts of appeal; and the circuit
courts in the exercise of the jurisdiction described by rule 9.030(c),
including the chief justice of the supreme court and the chief judge of a
district court of appeal in the exercise of constitutional, administrative, or
supervisory powers on behalf of such courts.
***

Raynor v. Florida State Lodge


987 So. 2d 152 (Fla. 1st DCA 2008)
BROWNING, C.J.
These consolidated appeals are from two orders of the circuit court, one
vacating an arbitration award, and the other confirming a second arbitration
award. We reverse the first order, and quash the second, for the reasons
below.
Background and Facts
Appellant Victor Raynor was a police officer accused of using
inappropriate force. He was at first counseled “verbally” and recommended
for anger counseling, but after an internal investigation he was fired. He
grieved the decision, and, under the applicable collective bargaining
agreement (CBA), the case went to arbitration. After a hearing, the arbitrator
found that Raynor was guilty of the misconduct of which he had been
accused, but that the job termination was an inappropriate punishment.
In support, the arbitrator produced an extensive and thoughtful order.
Therein he noted the tension between articles 9.1 and 11.2(d) of the CBA,
which read as follows:
The Employer has the right to discharge, suspend or otherwise discipline
employees for just cause. The Employer shall consider, among other things,
the seriousness and frequency of offenses when determining the appropriate
discipline, which may include a warning, suspension or immediate

16
discharge. Employees are not entitled to a particular number of warnings
prior to the imposition of discipline, including discharge.
***
If the subject of the grievance submitted to arbitration concerns
disciplinary measures (including discharge) taken against one or more
employees, the Arbitrator is only empowered to pass upon whether the
employee or employees concerned actually committed, participated in, or
were responsible for the act of misconduct. The Arbitrator has no authority
to pass upon the nature, extent, or severity of the disciplinary measure(s)
taken, such determination being solely a managerial prerogative. If the
Arbitrator finds that the employee has not committed, participated in, or was
not responsible for, the act of misconduct for which he has discipline [sic],
the Arbitrator has the power to make the employee or employees whole,
including ordering back pay (less compensation received from any other
sources) for time lost, and reinstatement when applicable.
The arbitrator acknowledged that while article 9.1 requires there be just
cause for discipline, article 11.2(d) seems to limit an arbitrator’s ability to
consider the severity of the disciplinary measure taken. However, the
arbitrator reconciled the two by noting it was “well-settled” that arbitrators
have the authority and obligation to modify or rescind a penalty where the
employer’s decision was arbitrary or violated due process, and by
recognizing colleagues’ interpretations of similar provisions to mean
arbitrators must examine due process concerns and the employer’s good
faith. In applying these thoughts, the arbitrator found the misconduct was
unintentional and isolated, that the employer did not follow the CBA’s
requirements that it consider the “seriousness and frequency” of the offense
or the possible use of progressive discipline, and that double jeopardy
concerns precluded termination.
Despite the arbitration award, the police department did not reinstate
Raynor. Raynor and the Fraternal Order of Police together petitioned the
circuit court to confirm the award, and the police department filed a
counterclaim and motion to vacate, alleging that the arbitrator exceeded his
powers under the CBA by conducting a “just cause” analysis, that double
jeopardy does not apply here, and that the award was against public policy.
The circuit court granted the police department’s motion, reasoning that the
arbitrator had exceeded his powers. The circuit court based its decision on
section 682.13(1)(c), Florida Statutes, which provides that “[u]pon

17
application of a party, the court shall vacate an award when ... [t]he
arbitrators or the umpire in the course of her or his jurisdiction exceeded
their powers.”
Raynor filed a timely notice of appeal, but, notwithstanding the fact that
an appeal was pending, the arbitrator entered an amended award affirming
the termination and setting aside the previous award. Raynor and the
Fraternal Order of Police moved to vacate the amended arbitration award,
but the circuit court granted the police department’s counter-motion to
confirm the award.
Analysis
[1] [2] We reverse the circuit court’s ruling on the first motion to vacate
the amended arbitration award because the arbitrator did not exceed the
scope of his authority. “[An] arbitrator’s award settling a dispute with
respect to the interpretation or application of a labor agreement must draw
its essence from the contract....” United Paperworkers Int’l Union, AFL-CIO
v. Misco, Inc., 484 U.S. 29, 38, 108 S.Ct. 364, 98 L.Ed.2d 286 (1987). Thus,
the proper inquiry here is whether the arbitrator was arguably construing or
applying the terms of the CBA. See id. “So far as the arbitrator’s decision
concerns construction of the [CBA], the courts have no business overruling
him because their interpretation of the contract is different from his.” United
Steelworkers of Am. v. Enter. Wheel & Car Corp., 363 U.S. 593, 599, 80
S.Ct. 1358, 4 L.Ed.2d 1424 (1960).
[3] In conducting his analysis, the instant arbitrator interpreted the
language of the CBA to mean he could determine whether the discipline was
justified only if he could also consider the severity of the discipline. Many
Florida state courts have taken the view that such a “just cause” analysis is a
permissible interpretation of a collective bargaining agreement. See, e.g.,
Commc'ns Workers of Am. v. City of Largo, 463 So. 2d 454 (Fla. 2d DCA
1985); Amalgamated Transit Union v. Hillsborough Area Reg’l Transit
Auth., 450 So.2d 590 (Fla. 2d DCA 1984). Although the CBAs in those
cases may have had slightly different wording from the instant CBA, the
circumstances are similar enough to show that, arguably, the instant
arbitrator was attempting to construe the CBA when he made his ruling.
Similarly, the arbitrator’s analysis of due process and the principle of
double jeopardy did not exceed the scope of his powers. The arbitrator
arguably may have construed Article 9.1 of the CBA to require the employer
to make a thorough and impartial evaluation of the seriousness of the

18
offense. And application of the double jeopardy concept ultimately “draw[s]
its essence from the contract” because the CBA incorporates by reference
section 112.532(4)(a), Florida Statutes, requiring written notice of
disciplinary action.
[4] Accordingly, we reverse the circuit court’s order vacating the first
arbitration award and remand for further proceedings on that award. We
quash the circuit court’s second order because it was entered without
jurisdiction. While the appeal of the first order was pending in this Court, the
circuit court had no jurisdiction to continue to act on this case. Florida Rule
of Appellate Procedure 9.600(a) provides that a lower tribunal has
concurrent jurisdiction with this Court only for procedural matters; this
matter was not procedural. And Rule 9.600(b) provides that this Court “by
order may permit the lower tribunal to proceed with specifically stated
matters during the pendency of the appeal”; this Court never ordered the
circuit court to do so.
We therefore REVERSE AND REMAND the circuit court’s order dated
March 1, 2007, and QUASH the circuit court’s order dated June 27, 2007.
KAHN and THOMAS, JJ., concur.

F. Extent of Jurisdiction

Florida Rules of Appellate Procedure

Rule 9.110. Appeal Proceedings to Review Final Orders of Lower


Tribunals and Orders Granting New Trial in Jury and Non-Jury Cases
***
(h) Scope of Review. The court may review any ruling or matter occurring
before filing of the notice. Multiple final orders may be reviewed by a
single notice, if the notice is timely filed as to each such order.
***

19
Saul v. Basse
399 So. 2d 130 (Fla. 2d DCA 1981)
OTT, Judge.
Following entry of final judgment in this declaratory relief action, the
trial court ordered appellants to pay a fee to appellees’ attorney. We reverse
that order.
The original developer of Dolphin Towers, a Sarasota condominium
complex, retained title to the recreation areas, which were then leased to the
association of condominium owners for 99 years. By a series of assignments,
appellants succeeded to the interest of the developer. Appellants also held a
mortgage on the entire complex, securing the loan for construction. They
foreclosed that mortgage against everyone, including the association of
condominium owners and excluding only the individual unit owners.
Appellants purchased the property at the foreclosure sale “free and clear of
all liens, claims, or interest.”
Appellants then attempted to enforce the 99-year lease of the recreation
areas, but the unit owners refused to pay rent and brought this action seeking
a declaration that the foreclosure sale terminated the lease. During the
pendency of the action the trial court ruled that the “prevailing party” would
be entitled to an attorney’s fee under section 718.303, Florida Statutes.
[FN1]
FN1. 718.303 Obligations of owners.
(1) Each unit owner and each association shall be governed by, and shall
comply with the provisions of, this chapter, the declaration, the documents
creating the association, and the association bylaws. Actions for damages or
for injunctive relief, or both, for failure to comply with these provisions may
be brought by the association or by a unit owner against:
(a) The association.
(b) A unit owner.
(c) Directors designated by the developer, for actions taken by them prior
to the time control of the association is assumed by unit owners other than
the developer.
(d) Any director who willfully and knowingly fails to comply with these
provisions.
The prevailing party is entitled to recover reasonable attorney’s fees. This
relief does not exclude other remedies provided by law.

20
In 1979 a final judgment was entered in favor of appellees, declaring that
the 99-year lease had been terminated by the foreclosure sale, and further
providing that appellees were entitled to attorneys' fees, with the court
reserving jurisdiction to set the amount of the fee. Appellants appealed that
judgment to this court, but voluntarily dismissed it. Saul v. Basse, 377 So. 2d
179 (Fla. 2d DCA 1979). Subsequently, the trial court held a hearing and
entered a “Supplemental Final Judgment” awarding appellees' attorney a fee
of $22,800.
[1] [2] The award of a fee was error because (1) this lawsuit had nothing
whatever to do with the litigation addressed by section 718.303, namely
actions between unit owners and/or their condominium associations for
breach of the statutory or contractual provisions governing such
associations, (2) no other law, and no contract between the parties, permits
the assessment of such a fee, and (3) in the absence of statutory or
contractual authority, parties to litigation must pay their own attorneys’ fees.
Fred Howland, Inc. v. Gore, 152 Fla. 781, 13 So. 2d 303, 309 (1943).
[3] Although never argued to the trial court, appellees now assert that the
award was proper because appellants’ cross-complaint was for rent due
under the lease, which contained a provision for attorneys’ fees for
appellants had they prevailed. Therefore argue appellees, a fee for their own
attorney for successfully resisting the cross-complaint is authorized by
section 718.125, Florida Statutes. It’s a bit late for appellees to advance such
an argument, but in any event it has no merit because section 718.125,
enacted in 1978, could not operate to alter the obligations of the 1974 lease
involved here. Commodore Plaza at Century 21 Condominium Association
v. Cohen, 378 So. 2d 307, 309 (Fla. 3d DCA 1980).
Our conclusion that the award of fees was error is not completely
dispositive of this appeal. Appellees contend that appellants waived that
error when they dismissed their appeal from the final judgment, which had
established appellees’ entitlement to the fee and reserved for adjudication
only the amount to be awarded. The flaw in that reasoning is that it
disregards well established precedent.
[4] [5] [6] [7] Whether a judgment is final or merely interlocutory is not
determined by its title. Nowlin v. Pickren, 131 So. 2d 894, 895 (Fla. 2d DCA
1961). An award of attorneys’ fees may be made after entry of a judgment
finally determining the merits of a controversy, and that award is sufficiently
substantive, per se, to support an independent appeal not related to the final

21
judgment previously rendered. Department of Citrus v. Griffin, 332 So. 2d
54, 57 (Fla. 2d DCA 1976). Here, the question is whether the final judgment
on the merits of the case was also a final adjudication of appellants’ liability
for a fee for appellees’ attorney. Generally, if a court reserves jurisdiction to
make further adjudications, the judgment is only interlocutory. Olin’s, Inc. v.
Avis Rental Car System, 100 So. 2d 825, 826 (Fla. 3d DCA 1958). We hold
that the final judgment entered in 1979 was only interlocutory insofar as it
concerned attorneys’ fees.
[8] The question remains whether the interlocutory order regarding fees
was appealable under rule 9.130(a)(3)(C)(iv), Rules of Appellate Procedure
(1977), since it purports to determine liability for the fee, and is in favor of
appellees. We need not answer that question because in any event an appeal
from a final order calls up for review all necessary interlocutory steps
leading to that final order, whether they were separately appealable or not.
Auto-Owners Insurance Co. v. Hillsborough County Aviation Authority, 153
So. 2d 722, 724 (Fla.1963). In other words, failure to utilize the right to take
an interlocutory appeal does not restrict the scope of appellate review when
the final order is appealed. Rule 9.130(g), Fla.R.App.P. (1977).
If appellants had prosecuted their previous appeal to a decision on the
merits, it is unarguable that they would not be entitled to a second appeal of
any portion of that judgment, whether final or interlocutory. In that event,
the scope of any appeal from the subsequent award of a specific fee would
necessarily be restricted to questioning the reasonableness of the fee or the
propriety of the proceedings taken in connection with setting it.
But here, as we have noted, the first appeal was dismissed, thereby
precluding an adjudication on the merits. In legal effect, it was just as though
the appeal had never been taken. Upon the dismissal those portions of the
judgment which were final in nature became final; those that were
interlocutory remained interlocutory, subject to review on the appeal from
the subsequent monetary award.
We conclude that the present appeal was timely and, for the reasons
expressed herein, the order granting the fee to appellees’ attorney is
REVERSED.
HOBSON, Acting C. J., and DANAHY, J., concur.

22
G. Objections

An appellate court has authority to dismiss a case for lack of jurisdiction, on its own
motion, without an objection by a party. See Gleicher v. Claims Verification, Inc., 908
So. 2d 560 (Fla. 4th DCA 2005).

II. Invoking Appellate Jurisdiction

A. Jurisdictional Papers

The jurisdiction of an appellate court is invoked by filing a timely notice of appeal or a


timely petition for extraordinary relief. The discretionary jurisdiction of the Florida
Supreme Court to review a decision by a district court of appeal is invoked by filing a
notice to invoke discretionary jurisdiction.
An appellate court acquires jurisdiction at the moment the notice or petition is filed. See
Vasilinda v. Lozano, 631 So. 2d 1082 (Fla. 1994).

B. Timeliness of Review

Florida Rules of Appellate Procedure

Rule 9.110. Appeal Proceedings to Review Final Orders of Lower


Tribunals and Orders Granting New Trial in Jury and Non-Jury Cases
***
(b) Commencement. Jurisdiction of the court under this rule shall be
invoked by filing an original and 1 copy of a notice, accompanied by any
filing fees prescribed by law, with the clerk of the lower tribunal within 30
days of rendition of the order to be reviewed.
***

Rule 9.120. Discretionary Proceedings to Review Decisions of


District Courts of Appeal
***
(b) Commencement. The jurisdiction of the supreme court described in

23
rule 9.030(a)(2)(A) shall be invoked by filing 2 copies of a notice,
accompanied by the filing fees prescribed by law, with the clerk of the
district court of appeal within 30 days of rendition of the order to be
reviewed.
***

Rule 9.130. Proceedings to Review Non-Final Orders and Specified


Final Orders
* * *.
(b) Commencement. The jurisdiction to seek review of orders described
in subdivisions (a)(3)–(a)(6) shall be invoked by filing 2 copies of a notice,
accompanied by the filing fees prescribed by law, with the clerk of the
lower tribunal within 30 days of rendition of the order to be reviewed.
***

Rule 9.140. Appeal Proceedings in Criminal Cases


***
(b) Appeals by Defendant.
(1) Appeals Permitted.A defendant may appeal
***
(3) Commencement.The defendant shall file the notice prescribed by rule
9.110(d) with the clerk of the lower tribunal at any time between rendition
of a final judgment and 30 days following rendition of a written order
imposing sentence. Copies shall be served on the state attorney and
attorney general.
***

(c) Appeals by the State.


(1) Appeals Permitted.The state may appeal an order
***
(3) Commencement.The state shall file the notice prescribed by rule
9.110(d) with the clerk of the lower tribunal within 15 days of rendition of
the order to be reviewed; provided that in an appeal by the state under
rule 9.140(c)(1)(K), the state's notice of cross-appeal shall be filed within
10 days of service of defendant's notice or service of an order on a motion
pursuant to Florida Rule of Criminal Procedure 3.800(b)(2). Copies shall

24
be served on the defendant and the attorney of record. An appeal by the
state shall stay further proceedings in the lower tribunal only by order of
the lower tribunal.
***

Rule 9.100. Original Proceedings


***
(b) Commencement; Parties. The original jurisdiction of the court shall
be invoked by filing a petition, accompanied by a filing fee if prescribed by
law, with the clerk of the court deemed to have jurisdiction. If the original
jurisdiction of the court is invoked to enforce a private right, the proceeding
shall not be brought on the relation of the state. If the petition seeks review
of an order entered by a lower tribunal, all parties to the proceeding in the
lower tribunal who are not named as petitioners shall be named as
respondents.
(c) Exceptions; Petitions for Certiorari; Review of Non-Final Agency
Action. The following shall be filed within 30 days of rendition of the order
to be reviewed:
(1) A petition for certiorari.
(2) A petition to review quasi-judicial action of agencies, boards, and
commissions of local government, which action is not directly appealable
under any other provision of general law but may be subject to review by
certiorari.
(3) A petition to review non-final agency action under the Administrative
Procedure Act.
(4) A petition challenging an order of the Department of Corrections
entered in prisoner disciplinary proceedings.

State ex rel Cantera v. District Court of Appeal, Third District


555 So. 2d 360 (Fla. 1990)
GRIMES, Justice.
Eduardo Cantera filed this petition for writ of prohibition seeking to
prevent the Third District Court of Appeal from considering an appeal from
a judgment he obtained against the Boatsmen’s National Bank of St. Louis,
formerly known as Centerre Bank, N.A. Pursuant to article V, section
3(b)(7), of the Florida Constitution, this Court may entertain by prohibition

25
any case pending in a district court of appeal when it appears that that court
is about to act in excess of its jurisdiction. State ex rel. Sarasota County v.
Boyer, 360 So. 2d 388 (Fla.1978). Upon consideration of the response to our
order to show cause, we now grant the petition.
Final judgment was entered on October 11, 1988. The bank filed a
motion for rehearing on October 21, 1988. The trial judge entered an order
denying the motion for rehearing on November 15, 1988. However, on
November 21, 1988, the trial court entered an order vacating the order
denying the motion for rehearing and by a separate order directed the parties
to file memoranda directed to the issues raised by the bank’s motion for
rehearing. After the submission of memoranda, the trial court entered an
order denying the motion for rehearing on December 28, 1988. On January
17, 1989, the bank filed a notice of appeal to the Third District Court of
Appeal. Cantera filed a motion to dismiss or quash the appeal, contending
that when the trial judge originally denied rehearing he lost jurisdiction to
enter the subsequent order. Thus, Cantera argued that the notice of appeal
should have been filed no later than thirty days from November 15, 1988. In
a split decision, the Third District Court of Appeal denied the motion to
dismiss. 544 So. 2d 341.
[2] The case turns on whether the trial judge had the authority to vacate
the order denying rehearing. Ironically, a recent decision of the same Third
District Court of Appeal suggests the lack of such authority. In Capital Bank
v. Knuck, 537 So. 2d 697 (Fla. 3d DCA 1989), the trial judge entered an
order denying a motion for rehearing but gave the movant an additional ten
days to file a new motion for rehearing. In prohibiting the trial court from
entertaining further proceedings with respect to the new motion, the Third
District Court of Appeal said:
It is apparent that the unqualified denial of Bulas’s appropriate post-
judgment motion constituted a final disposition of that motion. Because,
notwithstanding that the order undertook to do so, the trial court has no
authority either to permit the filing of any further motion for rehearing
beyond the one authorized by Florida Rule of Civil Procedure 1.530, or to
extend the time for filing that motion, the quoted order therefore terminated
the trial court’s jurisdiction over the cause....
In sum, the lower court could do nothing after the appropriate disposition
of the single authorized post-trial motion. Id. at 698 (footnotes omitted;

26
citations omitted). See also Markevitch v. Van Harren, 429 So. 2d 1255 (Fla.
3d DCA 1983).
The bank seeks to justify the order vacating the denial of the motion for
rehearing as an order entered pursuant to Florida Rule of Civil Procedure
1.540(a) which reads:
(a) Clerical Mistakes. Clerical mistakes in judgments, decrees or other
parts of the record and errors therein arising from oversight or omission may
be corrected by the court at any time on its own initiative or on the motion of
any party and after such notice, if any, as the court orders. During the
pendency of an appeal such mistakes may be so corrected before the record
on appeal is docketed in the appellate court and thereafter while the appeal is
pending may be so corrected with leave of the appellate court.
However, it is clear that the order of November 21, 1988, was not
intended to correct a clerical mistake. The first order denying rehearing
recited that the motion contained no legal argument not previously presented
to the court and that the order was being entered in conformity with
Hernandez v. Ward, 437 So. 2d 781 (Fla. 2d DCA 1983), and Boddie v.
Connecticut, 401 U.S. 371, 91 S.Ct. 780, 28 L.Ed.2d 113 (1971). By making
reference to these cases, it is apparent that the trial judge had concluded that
it was unnecessary to hold a hearing or receive memoranda in order to
dispose of the motion. While he evidently reconsidered his position six days
later, the entry of the order of November 15, 1988, denying rehearing cannot
be deemed a mistake or error of the kind contemplated by rule 1.540(a).
The trial judge lost jurisdiction when he denied the motion for rehearing.
Consequently, the notice of appeal filed more than two months later was
untimely. An appellate court cannot exercise jurisdiction over a cause where
a notice of appeal has not been timely filed. Hawks v. Walker, 409 So. 2d
524 (Fla. 5th DCA 1982); Pitts v. State, 225 So. 2d 352 (Fla. 1st DCA
1969). We, therefore, grant prohibition.
It is so ordered.
EHRLICH, C.J., and OVERTON, McDONALD, SHAW, BARKETT,
GRIMES and KOGAN, JJ., concur.

27
In Interest of T.D.
623 So. 2d 851 (Fla. 1st DCA 1993)
PER CURIAM.
The appellant has sought review of a final order rendered May 13, 1993,
which terminated her parental rights. The notice of appeal was filed on June
17, 1993. As it appeared the notice was untimely, appellant was ordered to
show cause why the appeal should not be dismissed. Finding no good cause,
we dismiss the appeal for lack of jurisdiction.
In response to the show cause order, counsel for appellant states that he
filed with the trial court a motion for extension of time to file the appeal.
This motion represented that counsel was appointed for appellate purposes
on June 4, 1993, but that counsel was in receipt of neither discovery,
transcripts nor a copy of the order entered by the court, and therefore could
not adequately prepare an appeal. The trial court entered an order on June
16, 1993, which purported to extend the time for filing the notice of appeal
to June 24, 1993. Counsel seeks to rely on this order to avoid dismissal.
The time for taking an appeal is a jurisdictional requirement established
by Florida Rule of Appellate Procedure 9.110(b), which provides that to
invoke the appellate court’s jurisdiction, a notice of appeal must be filed
with the clerk of the lower tribunal within 30 days of rendition of the order
to be reviewed. A trial court has no authority to extend the time for taking an
appeal. Ramagli Realty Co. v. Craver, 121 So.2d 648 (Fla.1960); Gordon v.
Green, 382 So.2d 1344 (Fla. 5th DCA 1980). Florida Rule of Civil
Procedure 1.090(b) provides for the enlargement of time to perform acts
under the rules, but specifically states that a trial court “may not extend the
time for ... taking an appeal....” The trial court was without authority to
attempt to extend the jurisdiction of this court.
As for the grounds counsel cited in his request for extension, the
preparation and filing of a simple notice of appeal does not require a review
of discovery material nor the transcripts, which are not normally
immediately available in any circumstance. In the time it took counsel to file
his unauthorized motion for extension, he could have prepared and filed the
notice of appeal, thereby preserving the mother’s right to appeal.
We are frankly surprised by the need to restate this basic rule of the law.
Until recently, the late filing of a notice of appeal in a civil proceeding was
an insurmountable jurisdictional defect which no one could correct, not even
the court. Hawks v. Walker, 409 So.2d 524 (Fla. 5th DCA 1982). The

28
supreme court has recently created a narrow exception to this rule when a
parent seeks an untimely appeal of an order which terminates parental rights.
In the Interest of E.H., 609 So.2d 1289 (Fla.1992). The court provided a
collateral remedy by petition for writ of habeas corpus in the trial court
which would permit a resolution of any factual issues as well as any
defenses. The court did not grant the relief based on precedent, but on the
significant policy interest in ensuring that a parent and child are not
separated without a thorough review of the merits of the case. Id. at 1291.
This appeal is hereby dismissed for lack of jurisdiction without prejudice
to appellant’s right to file a petition for writ of habeas corpus in the trial
court pursuant to E.H.
APPEAL DISMISSED.
ZEHMER, C.J., and JOANOS and BARFIELD, JJ., concur.

Breakstone v. Baron’s of Surfside, Inc.


528 So. 2d 437 (Fla. 3d DCA 1988)
ON MOTION TO DISMISS CROSS APPEAL
HUBBART, Judge.
The central question presented by appellants’ motion to dismiss is
whether this court has jurisdiction to entertain a cross appeal from a trial
court order denying appellee’s motion for attorney’s fees where, as here, (1)
appellants’ notice of appeal states that the main appeal is taken from a final
judgment and amended final judgment which were “rendered upon the
denial of post-trial motions . . .”, and (2) the order denying post-trial motions
is an omnibus trial court order which (a) denies appellants’ timely motion
for new trial and judgment n.o.v. and (b) denies appellee’s motion for
attorney’s fees. For the reasons which follow, we hold that this court has no
jurisdiction to entertain the above-stated cross appeal and grant appellants’
motion to dismiss same.
The relevant facts are undisputed. The plaintiff/appellee Baron’s of
Surfside, Inc. brought an action below against the defendants/appellants
Arthur Breakstone, Beach Enterprises, Ltd., and David Lehr to recover a
broker’s commission. The jury returned a verdict in favor of the
plaintiff/appellee and a final judgment was entered thereon dated November

29
30, 1987. All the defendants/appellants filed timely motions for new trial
and judgment n.o.v.; the plaintiff/appellee filed a motion for attorney’s fees
as the prevailing party. An amended final judgment was thereafter entered,
dated December 31, 1987, which reaffirmed the prior final judgment and
added a sum for costs and prejudgment interest. In an omnibus order dated
January 5, 1988 and filed January 8, 1988, the trial court disposed of the
parties’ post-trial motions by denying them all-including (a)
defendants/appellants’ timely motions for new trial and judgment n.o.v., and
(b) plaintiff/appellee’s motion for attorney’s fees.
On February 3, 1988, the defendants/appellants Arthur Breakstone and
Beach Enterprises, Ltd. filed below the following notice of appeal:
“NOTICE IS GIVEN THAT Arthur Breakstone and Beach Enterprises,
Ltd., defendants, appellants, appeal to the District Court of Appeal of
Florida, Third District, the orders of this Court entered on November 30,
1987, and December 31, 1987, and rendered upon the denial of post-trial
motions on January 5, 1988. The nature of the first order is a final judgment
in favor of the plaintiff, and the nature of the second order is an amended
final judgment in favor of the plaintiff.” (emphasis supplied).
On February 9, 1988, the plaintiff/appellee filed below the following
notice of cross appeal:
“NOTICE IS GIVEN THAT BARONS OF SURFSIDE, INC., Plaintiff
and Cross-Appellant, appeals to the District Court of Appeal of Florida,
Third District, the orders of this Court rendered on January 5, 1988,
denying Plaintiff/Cross Appellant attorneys fees and the Order of the Trial
Court granting Defendant/Cross-Appellees Motion for Directed Verdict as to
Count IV of the Amended Complaint at the conclusion of Plaintiff’s
evidence at trial. The nature of the orders appealed from are Final Orders.”
(emphasis supplied).
The defendants/appellants Arthur Breakstone and Beach Enterprises, Ltd.
now move this court to dismiss the italicized portion of the cross appeal
which seeks review of the trial court’s order denying the plaintiff/appellee’s
motion for attorney’s fees.
Fla.R.App.P. 9.110(g) provides that “[a]n appellee may cross appeal by
serving a notice within 10 days of service of the appellant’s notice or within
the time prescribed in section (b) of this rule [i.e., within 30 days of
rendition of order to be reviewed], whichever is later.” The filing of a cross
appeal notice was intended “to replace the cross assignments of error” which

30
were provided for under the “old” rules of appellate procedure, In re
Emergency Amendments to Rules of Appellate Procedure, 381 So. 2d 1370,
1382 (Fla.1980); Fla.R.App.P. 9.110(g) committee notes para. 8 (1977 rev.);
see Fla.R.App.P. 3.5 b (1962 rev.). Because the time limit set for filing such
cross assignments of error under the “old” appellate rules was considered
non-jurisdictional in nature and could therefore be extended, so too it has
been held that the time limit set for filing a notice of cross appeal under the
current rule is also non-jurisdictional in nature and may be extended by the
appellate court.FN1 It is also plain that a notice of cross appeal, like the “old”
cross assignments of error, must identify with particularity the exact adverse
trial court order or ruling which the appellee claims is error.
FN1. County Sanitation v. Ross, 389 So. 2d 1247 (Fla. 1st DCA 1980);
Safeco Ins. Co. v. Rochow, 384 So. 2d 163 (Fla. 5th DCA 1980); Agrico
Chem. Co. v. Department of Envtl. Regulation, 380 So. 2d 503 (Fla. 2d DCA
1980); Brickell Bay Club Condominium Ass'n v. Forte, 379 So. 2d 1334
(Fla.3d DCA 1980).
[2] It does not follow, however, that this court has jurisdiction to
entertain a cross appeal from every order or ruling made in the case which is
adverse to the appellee. This court’s jurisdiction to entertain an appeal is
invoked solely by the notice of appeal which must timely seek review of an
appealable trial court order or orders. Hawks v. Walker, 409 So. 2d 524 (Fla.
5th DCA 1982); Dibble v. Dibble, 377 So. 2d 1001 (Fla.3d DCA 1979);
§59.081(2), Fla.Stat. (1985). The notice of cross appeal, on the other hand, is
not a jurisdiction-invoking document, but instead is in the nature of a cross
assignment of error. See supra cases collected at note 1. It therefore follows
that the cross appeal must necessarily “piggy back” jurisdictionally on the
notice of appeal, and is, accordingly, confined to those trial court orders or
rulings adverse to the appellee which either “merge” into or are an inherent
part of the order or orders which are properly under review by the main
appeal-much as the main appeal is confined to similar trial court orders or
rulings which are adverse to the appellant.
We have recently stated:
“The rule [Fla.R.App.P. 9.110(g) ] allowing for a cross-appeal
contemplates an appeal from the same judgment from which the original
appeal is taken. [Citations omitted]. The function of a cross-appeal is to call
into question error in the judgment appealed, which, although substantially
favorable to the appellee, does not completely accord the relief to which the

31
appellee believes itself entitled. It is not the function of a cross-appeal to
seek review of a distinct and separate judgment, albeit rendered in the same
case below, favorable to the appellant.”
Webb Gen. Contracting, Inc. v. PDM Hydrostorage, Inc., 397 So. 2d
1058, 1059-60 (Fla.3d DCA 1981). By this we mean that a cross appeal is
not a separate appeal in itself but “rides along” with the main appeal-that is,
the cross appeal contemplates, jurisdictionally speaking, an appeal from the
same judgment from which the original appeal is taken. The function of the
cross appeal, then, is to call into question error in certain trial court orders or
rulings adverse to the appellee which “merge” into or are an inherent part of
the order or orders under review by the main appeal-although the latter may
be favorable or substantially favorable to the appellee. It is not, however, the
function of a cross appeal to seek review of a distinct and separate
appealable order which does not otherwise “merge” into the order or orders
from which the main appeal is taken. It has, accordingly, been held that a
district court of appeal has no jurisdiction to entertain a cross appeal from a
separately appealable, post-trial final order denying a motion for attorney’s
fees-where the main appeal is taken solely from the underlying final
judgment-because such an attorney’s fee order cannot, by definition,
“merge” into the final judgment from which the main appeal is taken. Miller
v. Nassofer, 484 So. 2d 619, 621 (Fla. 5th DCA 1986).
[3] [4] Turning to the instant case, we have no trouble in concluding that
this court lacks jurisdiction to entertain that portion of the cross appeal
which seeks review of the trial court’s ruling denying the appellee’s motion
for attorney’s fees, as contained in the omnibus order denying post-trial
motions in the cause. This is so because the main appeal is taken solely from
the final judgment and amended final judgment entered below-and the
subject attorney’s fee ruling is a separately appealable final order which does
not “merge” into the final judgment and amended final judgment appealed
from. Based on the established law of this state, as reviewed above, our sole
jurisdiction, therefore, is to review the final judgment and amended final
judgment entered below; we have no jurisdiction under the main appeal, and
therefore no jurisdiction under the cross appeal, to review the separately
appealable final order denying attorney’s fees.FN2 Moreover, we cannot treat
the plaintiff/appellee’s notice of cross appeal as a notice of appeal from this
separately appealable order because the notice was untimely filed below
more than thirty days after the rendition of the said order. Free Unitholders

32
of Outdoor Resorts at Orlando, Inc. v. Outdoor Resorts of America, Inc.,
460 So. 2d 382 (Fla.2d DCA 1984); Webb Gen. Contracting, Inc. v. PDM
Hydrostorage, Inc., 397 So. 2d at 1060; Fla.R.App.P. 9.110(b).
FN2. It should be noted, however, that the defendants/appellants have,
quite correctly, not challenged this court’s jurisdiction to entertain that
portion of the cross appeal which seeks review of the order of the trial court
granting the defendants/appellants’ motion for directed verdict as to count
IV of the amended complaint. Plainly, we have jurisdiction to entertain this
portion of the cross appeal because the granting of the subject motion for
directed verdict was adverse to the plaintiff/appellee and “merged” into the
final judgment and amended final judgment from which the main appeal is
taken.
To avoid this inexorable result, however, the appellee contends that it has
a right to cross appeal the adverse attorney’s fee ruling herein because it is
contained in the order denying post-trial motions which, it is urged, the
appellants have appealed. We do not agree because the appellants have not
appealed from the order denying post-trial motions; they have appealed
solely from the final judgment and amended final judgment entered below. It
is true that the order denying post-trial motions is mentioned in the notice of
appeal, but that was for the sole purpose of reflecting the rendition date of
the two judgments which were appealed, as Fla.R.App.P. 9.900(a) requires
as a matter of form. The mere mention of that order in the notice of appeal
for the purpose of reflecting the rendition date of the judgments appealed
from cannot be construed as an appeal from the said order.
For the above-stated reasons, the appellants’ motion to dismiss is granted
and that portion of the cross appeal which seeks review of the denial of
appellee’s motion for attorney’s fees is
Dismissed.

33
C. Rendition

Florida Rules of Appellate Procedure

Rule 9.020. Definitions


***
(f) Order. A decision, order, judgment, decree, or rule of a lower tribunal,
excluding minutes and minute book entries.
***
(h) Rendition (of an Order). An order is rendered when a signed, written
order is filed with the clerk of the lower tribunal. * * *

An order is rendered when it has been reduced to writing, signed by the judge, and filed
in the court file. See Casto v. Casto, 404 So. 2d 1046 (Fla. 1981).
It follows from these elements that an oral order cannot be rendered. See Owens v. State,
579 So. 2d 311 (Fla. 1st DCA 1991).

State v. Wagner
863 So. 2d 1224 (Fla. 2004)
BELL, J.
We review State v. Wagner, 825 So. 2d 453 (Fla. 5th DCA 2002), which
expressly and directly conflicts with State v. Tremblay, 642 So. 2d 64 (Fla.
4th DCA 1994). FN1 *** For the reasons that follow, we quash the Fifth
District’s decision in Wagner and approve the decision of the Fourth District
in Tremblay, which held that a signed court status form does not constitute a
final, appealable order under the Florida Rules of Appellate Procedure. Our
approval of Tremblay, however, is limited to its holding. We disapprove the
dicta expressed in that opinion.

I.
The State sought to have Alfred Wagner involuntarily committed as a
sexually violent predator pursuant to the Jimmy Ryce Act. §§ 394.910-.931,

34
Fla. Stat. (2002). On January 23, 2002, the circuit court held a hearing on
Wagner’s motion to dismiss the State’s involuntary commitment petition.
The court denied the motion to dismiss but concluded that Wagner should be
released from custody pending resolution of the commitment proceedings.
The judge then signed the “open court minutes” form, which contained the
following notation: “Defendant to be released immediately from custody.”
At the hearing, the judge instructed Wagner’s attorney to prepare the
formal order. This instruction was also noted on the court minutes form:
“[Wagner’s attorney] to prepare an Order.” The judge explained to the
attorneys that Wagner’s attorney was to send a copy of the drafted order to
the State’s attorney and that a follow-up hearing would be held if the State’s
attorney objected to the content of the drafted order.FN2 As it turned out, the
State did object to portions of Wagner’s drafted order, arguing that it did not
accurately reflect the judge’s prior oral pronouncements. A hearing on the
matter was held on March 20, 2002. On March 26, 2002, the court issued its
formal written order releasing Wagner from custody pending resolution of
the involuntary commitment proceedings.
FN2. The transcript of the hearing reveals the following colloquy:
THE COURT: ... So what I’m going to do at this point, Mr. Kutsche
[Wagner’s attorney], is I’m going-you’re going to have to draw the order,
but what I’m going to do, at this moment, subject to a formal order, I’m
going to deny your motion to dismiss the petition in the case without
prejudice for you to raise the additional arguments that you have wanted,
that you’ve filed here today at a later time, and I’m going to order Mr.
Wagner to be released immediately from the custody of the Department of
Children and Families.
....
THE COURT: And I’d ask you to draw the order, and I’ll find the fifteen
days [sic] for the State to appeal to begin upon the rendition of the order.
[WAGNER’s ATTORNEY]: Yes sir.
[STATE’s ATTORNEY]: I would ask Mr. Kutsche to somehow notify
me so that I can, when the order is drafted and when it’s signed by you so I
know what my appeal time is.
THE COURT: Well, what will happen is ... [b]efore he sends me an order
to sign, he’ll send you a copy of the order so you can approve it as to form.
[STATE’s ATTORNEY]: Right.

35
THE COURT: And then you’ll know when it’s been signed.
[STATE’s ATTORNEY]: I just wanted that on the record so I know that
I’m going to receive that copy.
THE COURT: In other words, once you draft the order, send it over, let
her look at it, if she thinks you’ve got something in there that I didn’t say,
then you can,-we even have hearings sometimes when they disagree as to the
form of the order. Basically, I want to give you something to appeal more
than what I just said, more than some form order that the clerk prepared. By
then, the Supreme Court may decide that Atkinson was wrongly decided and
we can all take it back.
....
[WAGNER’s ATTORNEY]: Your honor, is he-I take it that he actually
is not going to be released until the formal order is signed and transmitted.
THE COURT: I just signed an order saying that he’s to be released from
custody immediately. We'll give this to these gentleman that brought him
here, and they can head on back without him unless he wants to go back
with them to get his stuff and be released from Martin County, that’s up to
him. I guess they’ll give him a ride, I don’t know. I don’t know if their
insurance will cover it or not.
On April 25, 2002, the State petitioned the Fifth District for a writ of
certiorari. The district court dismissed the petition as untimely filed.
Wagner, 825 So. 2d at 453-54. It concluded that the signed court minutes
form, which “gave a clear written direction to release Wagner immediately,”
constituted an appealable order. Id. at 455. This conclusion was premised
upon the determination that the order was rendered that same day when it
was filed with the court clerk. Id. Consequently, the State’s certiorari
petition was deemed untimely because it was filed more than thirty days
thereafter. Id. at 455-56.
The State invoked this Court’s discretionary jurisdiction, claiming that
the district court’s opinion expressly and directly conflicted with Tremblay.
In Tremblay, the Fourth District held that a signed court status form, which
indicated that a particular charge against the defendant had been dismissed,
did not constitute a final, appealable order under the Florida Rules of
Appellate Procedure. We exercised our discretionary jurisdiction to resolve
the conflict.
II.

36
Wagner argues that the trial court rendered its order FN3 on January 23,
which was the date on which it orally pronounced that Wagner was to be
immediately released and on which it then signed and filed with the clerk the
court minutes form, which indicated that Wagner was to be immediately
released. The State, on the other hand, contends that the trial court’s order
was not rendered until March 26, the date on which the trial court issued its
formal written order. If Wagner is correct, then the State’s certiorari petition
was properly dismissed by the district court as untimely filed. FN4 If the State
is correct, then its petition was timely filed and improperly dismissed by the
district court.
FN3. “An order is rendered when a signed, written order is filed with the
clerk of the lower tribunal.” Fla.R.App.P. 9.020(h).
FN4. A petition for a writ of certiorari must be filed “within 30 days of
rendition of the order to be reviewed.” Fla.R.App.P. 9.100(c)(1).
The Florida Rules of Appellate Procedure define an “order” as “[a] decision,
order, judgment, decree, or rule of a lower tribunal, excluding minutes and
minute book entries.” Fla.R.App.P. 9.020(f). The rules explicitly exclude
“minutes and minute book entries” from the definition of “order.” FN5 The
Committee Notes to rule 9.020 state that “[m]inute book entries are excluded
from the definition in recognition of the decision in Employers' Fire Ins. Co.
v. Continental Ins. Co., 326 So. 2d 177 (Fla.1976). It was intended that this
rule encourage the entry of written orders in every case.”
FN5. Instead of looking to rule 9.020(f), the Fifth District looked to a
legal dictionary to define an “order” as “a command, direction, or
instruction.” Wagner, 825 So. 2d at 455 (quoting Black’s Law Dictionary
1123 (7th ed.1999)).
In Employers’ Fire, we noted that “[t]he inevitable variations in the way
judges complete court minutes suggest that these entries would generally be
an unreliable guide by which to measure either appellate or limitations
time.” 326 So. 2d at 180. We also noted the “wide disparity in the practices
of our trial courts as to the time, manner and completeness of minute book
entries,” id., and foresaw “a whole range of legal problems arising from
signings on a day subsequent to the day of the actual activity recorded, and
from the completeness of the information contained in minute book
recitations.” Id. at 180 n. 8. On the other hand, we also recognized “that
there is an arbitrary aspect to a measuring event which depends upon the

37
willingness of trial counsel to supply a form called ‘final judgment’ to the
trial judge.” Id. at 180.
Ultimately, we concluded in Employers’ Fire that “a distinction should
be drawn between the time for commencing an appeal and the time for
commencing a statute of limitations on a money judgment.” Id.FN6
FN6. Although the issue before us in Employers’ Fire dealt expressly
with “judgments,” we noted that “[b]oth parties argue from decisions fixing
the time for commencement of an appeal.” Id. at 179-80.
With respect to an appeal, the arbitrary delay between a determination of
liability and the signing of a judgment document is not a significant cause
for concern. The rendition of an adverse money judgment in a lawsuit, and
the desire to suspend the effect of the loss as promptly as possible, provide
sufficient incentives for counsel to prepare and submit the necessary paper
work to end the trial court’s labor and lay the predicate for appellate review.
Equally important is the fact that a trial court’s decision is more clearly
articulated, and therefore more reviewable, in a final judgment document
than in a minute book notation. For these reasons, the time for taking an
appeal should be governed by the rendition of a formal document of
judgment by the trial judge ... rather than by the signed entry in a minute
book. Id. at 180-81 (emphasis added).
We believe that the rationale of Employers’ Fire, which was explicitly
included in the Florida Rules of Appellate Procedure, controls our resolution
of this case. For purposes of Florida Rule of Appellate Procedure
9.100(c)(1), which requires a petition for a writ of certiorari to be filed
“within 30 days of rendition of the order to be reviewed,” a court minutes
form, even if signed by the judge, does not constitute a reviewable order.
The text of Florida Rule of Appellate Procedure 9.020(f), the Committee
Notes, and the rationale of Employers’ Fire clearly dictate this result. A
formal written order will articulate the trial court’s decision and its
supporting reasoning much more clearly than a clerk’s notation in a court
minutes form. The result reached by the Fifth District in Wagner would not
serve the policy of efficient and thorough appellate review. It would also
create situations of inadvertent rendition where the judge had signed a court
minutes form and filed it with the clerk unbeknownst to the parties.FN7 A
regime engendering so much uncertainty is anything but workable.
FN7. For example, in State v. Brown, 629 So. 2d 980 (Fla. 5th DCA
1993), the Fifth District held that a “court minutes/order” form, signed by

38
the judge and filed with the court clerk, constituted a rendered order. As
opposed to the court minutes form at issue here, and the court status form at
issue in Tremblay, the form signed by the judge and filed with the clerk in
Brown was explicitly designated by the court as an “order.” Brown, 629 So.
2d at 980 (“The trial court X’d the blanks preceding both the words ‘Court
Minutes’ and the words ‘Order (Motion Hearing),’ indicating that the subject
form was both the court minutes and an order of the court.”).
III.
Because the signed court minutes form was not an order within the
definition of the Florida Rules of Appellate Procedure, the act of filing it
with the court clerk did not amount to the rendition of an order. The
appellate jurisdictional clock of Florida Rule of Appellate Procedure
9.100(c)(1) did not begin ticking in this case until March 26, the date on
which the formal written order was signed by the judge and filed with the
court clerk. It follows, therefore, that the State’s petition for a writ of
certiorari was timely filed. Accordingly, we quash the decision of the Fifth
District in the case below.
With respect to Tremblay, we approve the holding of the Fourth District
that a signed court status form does not constitute a final, appealable order
under the Florida Rules of Appellate Procedure. However, we disapprove
the court’s statement that under certain peculiar circumstances a court status
form could be found to be a final, appealable order.
It is so ordered.
ANSTEAD, C.J., and WELLS, PARIENTE, LEWIS, QUINCE, and
CANTERO, JJ., concur.

D. Motions Extending Rendition

Florida Rules of Appellate Procedure

Rule 9.020. Definitions


***
(h) Rendition (of an Order). An order is rendered when a signed, written
order is filed with the clerk of the lower tribunal. However, unless another
applicable rule of procedure specifically provides to the contrary, if a final

39
order has been entered and there has been filed in the lower tribunal an
authorized and timely motion for new trial, for rehearing, for certification, to
alter or amend, for judgment in accordance with prior motion for directed
verdict, for arrest of judgment, to challenge the verdict, to correct a
sentence or order of probation pursuant to Florida Rule of Criminal
Procedure 3.800(b)(1), to withdraw a plea after sentencing pursuant to
Florida Rule of Criminal Procedure 3.170(l), or to vacate an order based
upon the recommendations of a hearing officer in accordance with Florida
Family Law Rule of Procedure 12.491, the following exceptions apply:
(1) If such a motion or motions have been filed, the final order shall
not be deemed rendered with respect to any claim between the movant
and any party against whom relief is sought by the motion or motions until
the filing of a signed, written order disposing of all such motions between
such parties.
(2) If such a motion or motions have been filed, a signed, written
order granting a new trial shall be deemed rendered when filed with the
clerk, notwithstanding that other such motions may remain pending at the
time.
(3) If such a motion or motions have been filed and a notice of
appeal is filed before the filing of a signed, written order disposing of all
such motions, all motions filed by the appealing party that are pending at
the time shall be deemed abandoned, and the final order shall be deemed
rendered by the filing of the notice of appeal as to all claims between
parties who then have no such motions pending between them. However,
a pending motion to correct a sentence or order of probation or a motion
to withdraw the plea after sentencing shall not be affected by the filing of a
notice of appeal from a judgment of guilt. In such instance, the notice of
appeal shall be treated as prematurely filed and the appeal held in
abeyance until the filing of a signed, written order disposing of such
motion.
(i) Rendition of an Appellate Order.If any timely and authorized motion
under rule 9.330 or 9.331 is filed, the order shall not be deemed rendered
as to any party until all of the motions are either abandoned or resolved by
the filing of a written order.

40
Fire & Cas Ins. Co. of Conn. v. Sealey
810 So. 2d 988 (Fla. 1st DCA 2002)
PADOVANO, J.
We dismiss the appeal for lack of jurisdiction. The notice of appeal was
not filed within thirty days of the date of rendition of the final judgment.
Although the notice would have been timely if measured from the date of a
subsequent order denying the defendant’s post trial motions, these motions
were all ineffective to suspend the running of the time for an appeal.
The plaintiff asserted an uninsured motorist claim against the defendant
and ultimately prevailed at trial. The jury returned a verdict for damages in
favor of the plaintiff on February 9, 2001. Approximately three months later
on May 10, 2001, the trial court entered a final judgment based on the
verdict. The court apportioned the plaintiff’s comparative fault and made the
appropriate adjustments for prejudgment interest and collateral source
benefits, and then entered a judgment of $1,878, 687.35 in favor of the
plaintiff. The defendant had not filed any motions challenging the verdict
before the entry of the judgment.
On May 21, 2001, the defendant served a motion for remittitur, a motion
for new trial, and a motion for judgment notwithstanding the verdict. In the
motion for remittitur the defendant made four main arguments: (1) the
plaintiff failed to present evidence of causation, (2) the verdict was
influenced by passion, prejudice, and sympathy, (3) the plaintiff failed to
present evidence of lost wages, and (4) the plaintiff failed to present
evidence of his future medical expenses. These arguments are directed
exclusively to the jury’s verdict. The opening line of the motion states that
the defendant is moving “for a remittitur of the verdict.”
The motion for a new trial presents the same four arguments raised in the
motion for remittitur, but it also contains an argument that the verdict is
contrary to the manifest weight of the evidence, and an argument that the
trial court erred in denying the defendant’s motion for directed verdict.
Although the motion for judgment notwithstanding the verdict was
submitted as a separate pleading, it is identical to the motion for a new trial.
On August 3, 2001, the trial judge entered an order denying all three of
the defendant’s post-trial motions. The defendant filed a notice of appeal on
August 31, 2001, to seek review in this court. Because it was not clear from
the notice of appeal whether the post-trial motions were timely, and
therefore capable of suspending rendition of the final judgment, we issued

41
an order directing the defendant to show cause why the appeal should not be
dismissed as untimely. The defendant responded with an argument that the
court has jurisdiction. The plaintiff moved to dismiss the appeal and the
defendant submitted a response in opposition to the motion to dismiss.
[1] We are governed by the fundamental principle that the time for
initiating an appeal is jurisdictional. If the notice of appeal is not filed within
the time set by the applicable rule, the appellate court must dismiss the
appeal. See State ex rel. Cantera v. District Court of Appeal, Third District,
555 So. 2d 360, 362 (Fla.1990); Int’l Studio Apt. Ass’n v. Sun Holiday
Resorts, Inc., 375 So. 2d 335, 336 (Fla. 4th DCA 1979). The method of
initiating an appeal from a final order in a civil case is outlined in rule
9.110(b) of the Florida Rules of Appellate Procedure. This rule states that
the notice of appeal must be filed “within 30 days of the date of rendition of
the order to be reviewed.”
Rule 9.020(h) provides that “[a]n order is rendered when a signed,
written order is filed with the clerk of the lower tribunal.” See Casto v.
Casto, 404 So. 2d 1046, 1047 (Fla.1981); State v. Moore, 563 So. 2d 115
(Fla. 2d DCA 1990). Here, the judgment was reduced to writing, signed, and
filed on May 10, 2001. The time for an appeal began to run from that date.
[2] The date of rendition, and thus the beginning of the jurisdictional
time limit for filing an appeal, may be delayed by the filing of certain
motions in the lower court after entry of the final judgment. Rule 9.020(h)
identifies these motions and sets forth the requirements for suspending the
date of rendition.FN1 A motion is generally not effective to suspend rendition
of the time for an appeal unless it is one of the motions identified in the rule.
See Klemba v. State, 490 So.2d 1050 (Fla. 4th DCA 1986); Joseph v. State,
437 So. 2d 245, 246 (Fla. 5th DCA 1983); Culpepper v. Britt, 434 So. 2d 31,
32 (Fla. 2d DCA 1983). If a party files a motion that is authorized in the
proceeding, but not listed in rule 9.020(h), the time for filing an appeal will
run uninterrupted from the date the final order was reduced to writing,
signed, and filed with the clerk.
FN1. The motions listed in rule 9.020(h) are: (1) a motion for new trial,
(2) a motion for rehearing, (3) a motion for clarification, (4) a motion for
certification, (5) a motion to alter or amend, (6) a motion for judgment in
accordance with a prior motion for directed verdict, (7) a motion for arrest of
judgment, (8) a motion to challenge the verdict, (9) a rule 3.800(b)(1)
motion to correct a sentence or probation order, (10) a rule 3.170( l ) motion

42
to withdraw a plea, and (11) a rule 12.492 motion to vacate an order based
on recommendations of a family law hearing officer.
[3] Filing one of the motions listed in rule 9.020(h) will suspend
rendition of the final order only if the motion is timely under the rules
applicable to the proceeding in the lower court. See Salam v. Benmelech, 622
So. 2d 592, 593 (Fla. 3d DCA 1993) (untimely motion to alter or amend);
Bailey v. Mobile Home Park Realty, Inc., 579 So. 2d 198, 199, 200 (Fla. 2d
DCA 1991) (untimely motion in arrest of judgment); Americare Biologicals,
Inc. v. Technical Chemicals & Products, Inc., 766 So. 2d 284 (Fla. 4th DCA
2000) (untimely motion for rehearing); see also Howard v. Farm Bureau
Ins. Co., 467 So. 2d 442 (Fla. 5th DCA 1985); White v. State, 416 So. 2d 39
(Fla. 2d DCA 1982). Hence, a motion that is untimely does not toll the time
for an appeal, even if the motion is one of those identified in rule 9.020(h) as
a motion that is otherwise capable of suspending rendition.
[4] To determine the timeliness of the defendant’s post-trial motions, we
must refer to the rules governing each of those motions. Rule 1.530(b) of the
Florida Rules of Civil Procedure states in pertinent part that a motion for a
new trial must be served “not later than 10 days after the return of the verdict
in a jury action.” The same time limit applies to the service of a motion for
judgment notwithstanding the verdict, which is now more often referred to
as a motion for judgment in accordance with a prior motion for directed
verdict.FN2 Rule 1.480(b) states that “[w]ithin 10 days after the return of a
verdict, a party who has timely moved for a directed verdict may serve a
motion to set aside the verdict and any judgment entered thereon and to enter
judgment in accordance with the motion for a directed verdict.”
FN2. Some Florida courts have held that a motion for a judgment
notwithstanding the verdict is obsolete and should now be styled as a motion
in accordance with a prior motion for directed verdict. See La Rosa Del
Monte Express, Inc. v. G.S.W. Enters. Corp., 483 So. 2d 472, 473 n. 1 (Fla.
3d DCA 1986); Meus v. Eagle Family Discount Stores, 499 So. 2d 840, 842
n. 6 (Fla. 3d DCA 1986). However, other courts have continued to use the
term “judgment notwithstanding the verdict” even after the adoption of rule
1.480(b). See Lindenfield v. Dorazio, 606 So. 2d 1255 (Fla. 4th DCA 1992);
Dean Witter Reynolds, Inc. v. Hammock, 489 So. 2d 761 (Fla. 1st DCA
1986). One commentator maintains that there is a distinction between a
motion for a judgment in accordance with a prior motion for directed verdict
and a motion for judgment notwithstanding the verdict. Henry P. Trawick,

43
Jr., Florida Practice and Procedure, § 26-4 (2001). For the purpose of this
appeal, we assume the defendant’s motion would qualify as a motion for
judgment in accordance with a prior motion for directed verdict and that it
would have been capable of suspending rendition, had it been timely.
It follows that both the motion for new trial and the motion for judgment
notwithstanding the verdict were ineffective to suspend rendition of the final
judgment. The time for service of each motion began on February 9, 2001,
the day the jury returned its verdict. Both motions had to be served within
ten days after the return of the verdict, but neither was served until May 21,
2001, more than three months later.
[5] This leaves only the defendant’s motion for remittitur, but it did not
delay the running of the appeal time either. A motion for remittitur is
essentially a conditional motion for a new trial. See Poole v. Veterans Auto
Sales & Leasing Co., 668 So. 2d 189 (Fla.1996). As we have explained, a
motion for new trial must be served within ten days after the return of the
verdict. The motion for remittitur, like the other two post-trial motions, was
served more than three months after the verdict. Hence, it was untimely and
therefore ineffective to suspend rendition.
[6] The defendant contends that the motion for remittitur should be
treated as a motion to alter or amend the judgment, one of the motions that
can operate to suspend rendition under rule 9.020(h). If we were to reclassify
the motion in this way it would be timely and thus it would delay rendition
of the final judgment until August 3, 2001, when the motion was denied.
Rule 1.530(g) provides that a motion to alter or amend the judgment “shall
be served not later than 10 days after entry of the judgment” and the motion
in this case was served within that time. However, we cannot treat the
motion for remittitur as if it were a motion to alter or amend the judgment.
[7] We agree that the true nature of a motion must be determined by its
content and not by the label the moving party has used to describe it. See
Magnum Towing, Inc. v. Sunbeam Television Corp., 781 So. 2d 379, 380
(Fla. 3d DCA 1998) (treating a motion styled as a “motion for
reconsideration” as a motion for rehearing); Olson v. Olson, 704 So. 2d 208,
210 (Fla. 5th DCA 1998) (construing a motion to set aside a final judgment
as a motion for rehearing). When we consider the content of the motion for
remittitur, though, we conclude that it was, in fact, a conditional motion for a
new trial, not a motion to alter or amend the judgment. We can look beyond

44
the label the defendant used, but we cannot make the motion into something
it was not.
A motion to alter or amend is most often used to correct a substantive
error in the judgment itself. For instance, it would be proper to file a motion
to alter or amend if the judgment did not accurately reflect the decision of
the court. See Fisher v. Fisher, 787 So. 2d 926, 929 (Fla. 2d DCA 2001) (the
judgment did not conform to the provision of the marital settlement
agreement); Sanchez v. Sanchez, 435 So. 2d 347, 349 (Fla. 3d DCA 1983)
(the court ordered alimony in the amount of $100 per month, but the
judgment erroneously required payments of $100 per week). The function of
the motion in this class of cases is to inform the court that an issue was not
adjudicated in the judgment in the same way that it was decided.
It would also be proper to file a motion to alter or amend to correct an
error in the adjudication of a post-verdict issue. If the trial judge has erred in
reducing the jury award to account for collateral source benefits, for
example, the error could be addressed in a motion to alter or amend. See
Nova Univ. v. Katz, 636 So. 2d 729 (Fla. 4th DCA 1993). In this situation,
the motion seeks to correct an error reflected only in the judgment and not in
the underlying jury verdict. The aggrieved party could not present the issue
in a motion for a new trial, because the error is not one that would have been
committed at that time.
[8] Finally, a motion to alter or amend may be used to correct a judgment
to the extent that it was based on an error of law. See Aagaard-Juergensen v.
Lettelier, 579 So. 2d 404, 405 (Fla. 5th DCA 1991); Barrios v. Draper, 423
So. 2d 1002, 1003 (Fla. 3d DCA 1982). As an illustration of this point, if the
trial court enters a judgment in excess of the legal limit of the defendant’s
liability, the defendant can correct the error by filing a motion to alter or
amend the judgment. See Kenilworth Ins. Co. v. Pizarro, 369 So. 2d 995,
996 (Fla. 3d DCA 1979); cf. Florida Patient’s Comp. Fund v. Scherer, 558
So. 2d 411, 415 (Fla.1990).
The common feature of these cases is that the motion to alter or amend
was used in each case to correct an error in the judgment itself, and not an
error in the course of the proceeding leading up to the judgment. All prior
errors in a civil case can be corrected on appeal from the final judgment; but
the scope of a motion to alter or amend the judgment is much more limited.
A motion to alter or amend must address an issue relating directly and
exclusively to the judgment.

45
The defendant argues that the motion for remittitur should be treated as a
motion to alter or amend because it goes to “the heart of the judgment,” but
that is simply not the case. The defendant’s motion raised four arguments,
all of which were directed exclusively to the jury’s verdict. Hence, the
defendant’s argument that the motion goes to the heart of the judgment is
correct only in the general sense that the judgment was based on the disputed
verdict.
If the defendant’s position were correct, an untimely motion for a new
trial could be revived simply by reclassifying it as a motion to alter or amend
the judgment. The moving party would always have the same argument the
defendant makes here; the motion attacks the correctness of the verdict and
therefore it goes to the heart of the judgment rendered on the verdict. The
danger of accepting this argument, however, is that it would effectively
nullify the time limit for service of a motion that is truly directed to the
verdict.
We consider it essential to maintain the distinction between post-trial
motions that challenge the verdict and those that challenge the judgment. A
motion for new trial or a motion for judgment in accordance with a prior
motion for directed verdict must be served within ten days after the return of
the verdict. See Fla.R.Civ.P. 1.530(b); 1.480(b). In contrast, a motion to alter
or amend the judgment must be served within ten days of the judgment. See
Fla.R.Civ.P. 1.530(g). If a motion for a new trial could simply be treated as
if it were a motion to alter or amend, there would be little point in placing a
time limit on service of a motion for a new trial.
In summary, we hold that the defendant’s post-trial motions were
ineffective to suspend rendition of the final judgment. Because the notice of
appeal was untimely when measured from the date of the final judgment,
this court lacks jurisdiction.
Appeal dismissed.
BROWNING and LEWIS, JJ., concur.

46
Goodwin v. State
826 So. 2d 1022 (Fla. 3d DCA 2001)
Before COPE, FLETCHER and SORONDO, JJ.
PER CURIAM.
Charles Goodwin and William Kemp petition this court for a writ of
prohibition directing the circuit court to dismiss the State’s appeal.
Petitioners allege that the State failed to timely file a notice of appeal from a
suppression order of the county court. We agree and grant prohibition.
Petitioners were each charged in county court with driving under the
influence of alcoholic beverages, causing property damage. See § 316.193,
Fla. Stat. (Supp.1998). Petitioners sought suppression of the breath test
results. The county court granted the suppression motions.
Ten days after the filing of the orders with the clerk’s office, the State
filed motions for rehearing. One week later, the court denied the State’s
motions for rehearing. Fourteen days later (and thirty-one days after
rendition of the order granting defendants’ motions to suppress), the State
filed its notices of appeal, thereby initiating an appeal to the circuit court.
Petitioners filed motions in the circuit court to dismiss the State’s appeals
as untimely in that the State failed to file the notices within fifteen days
following rendition of the court’s orders granting the motions to suppress.
See Fla. R.App. P. 9.140(c)(3). The trial court denied petitioners’ motions.
The court reasoned that the time for filing the notices of appeal was tolled by
the filing of the motions for rehearing. The petitioners then sought
prohibition here.
[1] Only an “authorized and timely” motion for rehearing can toll the
period for filing a notice of appeal. Id. R. 9.020(h). The State concedes that
the Florida Rules of Criminal Procedure do not “authorize” motions for
rehearing directed to motions to suppress. The State nonetheless argues that
the motions for rehearing were “authorized” pursuant to case law because a
suppression order is an interlocutory ruling which can always be revisited by
the court. Savoie v. State, 422 So. 2d 308, 312 Fla.1982) (determining that a
trial court has the power to reconsider a ruling made on a motion to suppress
upon motion by either party while the court has jurisdiction of the case);
Obregon v. State, 601 So. 2d 616, 616 (Fla. 3d DCA 1992)(same).
[2] While it is true that the court has the power to reconsider an
interlocutory ruling on a motion to suppress, this does not mean that the

47
filing of such a motion tolls the time for filing an appeal. Wagner v. Bieley,
Wagner & Assocs., Inc., 263 So. 2d 1, 4 (Fla.1972). Under Rule 9.020(h),
only an “authorized” motion suspends rendition of an order, and the rules do
not authorize a motion for rehearing of an order on a motion to suppress.
The appeals were untimely filed and must be dismissed.
Prohibition granted.

Coats v. Climatic Products Corporation


756 So. 2d 1104 (Fla. 1st DCA 2000)
PER CURIAM.
Climatic Products Corporation and its owner brought an action against
certain defendants, including J.D. Coats, Jr., individually and d/b/a Auto Air
of Tallahassee, to recover on open accounts. The matter was tried without a
jury and a judgment was entered for the plaintiffs against all defendants on
February 4, 1999. J.D. Coats, Jr., timely moved for rehearing, arguing that
there was insufficient evidence that he was responsible for the indebtedness.
Rehearing was denied on July 9, 1999, and a notice of appeal, purporting to
seek review of the judgment on behalf of all defendants, was filed on August
6, 1999.
Appellees move to dismiss the appeal of all defendants except J.D. Coats,
Jr, arguing this court’s jurisdiction was not timely invoked, relying upon
Florida Rule of Appellate Procedure 9.020(h)(1). This rule subdivision
provides that if a timely and authorized motion for rehearing has been filed,
“the final order shall not be deemed rendered with respect to any claim
between the movant and any party against whom relief is sought by the
motion or motions until the filing of a signed, written order disposing of all
such motions between the parties.” Appellants oppose the motion, arguing
that failure to name all defendants as seeking rehearing in the trial court was
excusable neglect which should be disregarded in light of timely invocation
of this court’s appellate jurisdiction.
We find appellees’ motion to be well-taken. The Committee Note to this
rule subdivision, added in 1992, leaves no doubt as to the proper application
of Rule 9.020(h)(1) in this setting:
Subdivision (g)(1) [now (h)(1)] has been added to clarify the date of
rendition when post-judgment motions have been filed. If there is only 1

48
plaintiff and 1 defendant in the case, the filing of a post-judgment motion or
motions by either party (or both parties) will postpone rendition of the entire
final order as to all claims between the parties. If there are multiple parties
on either or both sides of the case and less than all parties file post-judgment
motions, rendition of the final order will be postponed as to all claims
between moving parties and parties moved against, but rendition will not be
postponed with respect to claims disposed of in the final order between
parties who have no post-judgment motions pending between them with
respect to any of those claims. See, e.g., Phillips v. Ostrer, 442 So. 2d 1084
(Fla. 3d DCA 1983).
We have also carefully reviewed the motion for rehearing filed in the trial
court and find that it unambiguously sought rehearing only on behalf of J.D.
Coats, Jr.
Accordingly, we grant the motion and dismiss this appeal insofar as
review of the judgment is sought by J.D. Coats, individually and d/b/a Auto
Air of Tallahassee, David Wild, individually and d/b/a Auto Air Too, and
Auto Air of Tallahassee, Inc. The appeal shall proceed with J.D. Coats, Jr.,
individually and d/b/a Auto Air of Tallahassee as the sole appellant.
IT IS SO ORDERED.
JOANOS, LAWRENCE and DAVIS, JJ., concur.

E. Premature appeals

Florida Rules of Appellate Procedure

Rule 9.110. Appeal Proceedings to Review Final Orders of Lower


Tribunals and Orders Granting New Trial in Jury and Non-Jury Cases
***
(l) Premature Appeals. If a notice of appeal is filed before rendition of a
final order, the appeal shall be subject to dismissal as premature.
However, if a final order is rendered before dismissal of the premature
appeal, the premature notice of appeal shall be considered effective to
vest jurisdiction in the court to review the final order. Before dismissal, the
court in its discretion may permit the lower tribunal to render a final order.

49
Benton v. Moore
655 So. 2d 1272 (Fla. 1st DCA 1995)
PER CURIAM.
Tommy Lee Benton’s circuit court complaint sought to recover damages
for alleged negligence on the part of certain officials of the Department of
Corrections in the handling of a disciplinary matter. On July 14, 1994, the
trial court entered an order which provides in pertinent part:
THIS CAUSE came on before me on Defendants’ Motion to Dismiss and
Plaintiff’s Response to Defendants’ Motion to Dismiss. After reviewing the
Defendants’ Motion and Plaintiff’s Response, it is
ORDERED AND ADJUDGED that Defendants’ Motion to Dismiss is
hereby granted.
Plaintiff filed a timely notice of appeal.
After the record on appeal was filed, this court directed appellant to show
cause why the appeal should not be dismissed for lack of jurisdiction. That
is, it has been held that an order which grants a motion to dismiss is neither a
final order nor an appealable non-final order. Board of County
Commissioners v. Grice, 438 So. 2d 392 (Fla.1983); Johnson v. First City
Bank, 491 So. 2d 1217 (Fla. 1st DCA 1986). Appellant, in his response to
the order to show cause, simply argues that the order was final.
[1] [2] Appellant has failed to distinguish Grice or Johnson or persuade
us that the order is final or otherwise appealable. Accordingly, we dismiss
the appeal for lack of jurisdiction. In doing so, we are not unmindful of
Florida Rule of Appellate Procedure 9.110(m) which provides that if a
notice of appeal is filed before rendition of a final order, “[b]efore dismissal,
the court in its discretion may permit the lower tribunal to render a final
order.” We find that a proper exercise of our discretion in this and most
cases in this posture is dismissal of the appeal at the time this court is called
upon to resolve the jurisdictional issue unless a final order has been rendered
by the trial court in the interim. In the absence of some exceptional
circumstance, the appeal would be dismissed without prejudice, of course, to
the right of the appealing party to file a timely notice of appeal after a final
order has been rendered by the trial court.FN1 As no exceptional
circumstances are present in the case at bar, this appeal is dismissed for lack
of jurisdiction.

50
FN1. In Latin Express Service, Inc. v. State, Department of Revenue, No.
95-510, --- So. 2d ---- (Fla. 1st DCA May 18, 1995), this court found that an
order which did not advise the appellant of its right to appeal directly to the
district court was not final and did not commence the running of the 30-day
period for filing of the notice of appeal. Jurisdiction was relinquished
pursuant to rule 9.110(m) for entry of a final order. We find Latin Express
distinguishable from the instant case because it arose from an administrative
tribunal and because the jurisdictional defect was the result of the
agency/appellee’s issuance of a defective order.
WOLF, WEBSTER and LAWRENCE, JJ., concur.

F. Filing in the Proper Forum

Florida Rules of Appellate Procedure

Rule 9.040. General Provisions


***
(b) Forum.
(1) If a proceeding is commenced in an inappropriate court, that court shall
transfer the cause to an appropriate court.
(2) After a lower tribunal renders an order transferring venue, the
appropriate court to review otherwise reviewable non-final orders is as
follows:
(A) After rendition of an order transferring venue, the appropriate
court to review the non-final venue order, all other reviewable non-final
orders rendered prior to or simultaneously with the venue order, any order
staying, vacating, or modifying the transfer of venue order, or an order
dismissing a cause for failure to pay venue transfer fees, is the court that
would review non-final orders in the cause, had venue not been
transferred.
(B) After rendition of an order transferring venue, the appropriate
court to review any subsequently rendered reviewable non-final order,
except for those orders listed in subdivision (b)(2)(A), is the court which
would review the order, if the cause had been filed in the lower tribunal to
which venue was transferred.
(C) The clerk of the lower tribunal whose order is being reviewed

51
shall perform the procedures required by these provisions regarding
transfer of venue, including accepting and filing a notice of appeal. If
necessary to facilitate non-final review, after an order transferring venue
has been rendered, the clerk of the lower tribunal shall copy and retain
such portions of the record as are necessary for review of the non-final
order. If the file of the cause has been transferred to the transferee
tribunal before the notice of appeal is filed in the transferring tribunal, the
clerk of the transferee tribunal shall copy and transmit to the transferring
tribunal such portions of the record as are necessary for review of the non-
final order.
(c) Remedy. If a party seeks an improper remedy, the cause shall be
treated as if the proper remedy had been sought; provided that it shall not
be the responsibility of the court to seek the proper remedy.
***

Johnson v. Citizens State Bank


537 So. 2d 96 (Fla. 1989)
BARKETT, Justice.
We have for review Johnson v. Citizens State Bank, 518 So. 2d 410 (Fla.
1st DCA 1988), and Spector v. Trans World Airlines, Inc., 523 So. 2d 704
(Fla. 4th DCA 1988), in which the district courts certified the following
question to be of great public importance: FN1
FN1. We have discretionary jurisdiction under article V, section 3(b)(4),
Florida Constitution.
When a party seeks appellate review of a nonappealable order, and
assuming that the notice of appeal is timely filed in the lower tribunal, must
the notice of appeal be filed in the appellate court within 30 days of
rendition of the order in order for the appellate court to have jurisdiction to
treat the notice as a petition for writ of certiorari?
We answer the certified question in the negative and quash the decisions
of the district courts.
Initially, Johnson and the Spectors sought relief in their respective county
courts, lost, and then appealed to the circuit court. Unsuccessful in the circuit
court, they then attempted to perfect a plenary appeal in the district court by
filing a notice of appeal with the circuit court clerk. Although the notices of
appeal were filed within thirty days, the circuit court clerks did not transmit
the notices to the respective district courts within thirty days from the
rendition of the orders sought to be reviewed. The Johnson court dismissed

52
the appeal because its jurisdiction had not been timely invoked. The Spector
court did likewise, relying upon Johnson.
All parties now agree that the orders were not reviewable by plenary
appeal and that review is available, if at all, by certiorari under article V,
section 4(b)(3), Florida Constitution. Appellate certiorari is the appropriate
remedy to review “final orders of circuit courts acting in their review
capacity.” Fla.R.App.P. 9.030(b)(2)(B). Original jurisdiction of the appellate
court is invoked for this purpose “by filing a petition . . . with the clerk of
the court deemed to have jurisdiction,” that is, with the clerk of the district
court. Fla.R.App.P. 9.100(b).
Citizens State Bank and Trans World Airlines contend that because the
notices of appeal were not timely filed in the district court, Johnson and the
Spectors are not entitled to relief. Johnson and the Spectors argue that under
Florida’s Constitution a timely filing of a notice of appeal with the circuit
court clerk is legally effective to vest jurisdiction in the district court. We are
compelled to agree.
Article V, section 2(a), of our Constitution, provides:
The supreme court shall adopt rules for the practice and procedure in all
courts including ... the transfer to the court having jurisdiction of any
proceeding when the jurisdiction of another court has been improvidently
invoked, and a requirement that no cause shall be dismissed because an
improper remedy has been sought.
In response to that provision, the Court adopted Florida Rule of Appellate
Procedure 9.040(b) and (c). Subsection (b) provides that “[i]f a proceeding is
commenced in an inappropriate court, that court shall transfer the cause to an
appropriate court,” and subsection (c) provides that “[i]f a party seeks an
improper remedy, the cause shall be treated as if the proper remedy had been
sought....”
There is no question that an appellate court has jurisdiction to review a
cause even though the form of appellate relief is mischaracterized. Thus,
district courts have considered as petitions for writs of certiorari, erroneously
titled notices of appeal. See, e.g., Hillsborough County v. Marchese, 519 So.
2d 728 (Fla. 2d DCA 1988), cause dismissed, 526 So. 2d 75 (Fla.1988);
Nunez v. Gonzalez, 456 So. 2d 1336, 1339 (Fla. 2d DCA 1984); Sunshine
Dodge, Inc. v. Ketchem, 445 So. 2d 395, 396 (Fla. 5th DCA 1984); Wingate
v. Department of Highway Safety and Motor Vehicles, 442 So. 2d 1023,
1024 (Fla. 5th DCA 1983); Hackenberg v. Artesian Pools of East Florida,

53
Inc., 440 So. 2d 475 (Fla. 5th DCA 1983); Radio Communications Corp. v.
Oki Electronics of America, Inc., 277 So. 2d 289, 290 (Fla. 4th DCA
1973).FN2 Indeed, a district court shall not dismiss a timely filed notice of
appeal, if upon consideration, the court concludes that relief would be
warranted under a petition. Art. V, § 2(a), Fla. Const.; Fla.R.App.P.
9.040(c).
FN2. District courts have also considered as notices of appeal,
erroneously titled petitions for writs of certiorari. See, e.g., Pearce v.
Parsons, 414 So. 2d 296, 296 n. 1 (Fla. 2d DCA 1982).
In these cases, as in Johnson and Spector, the notice of appeal would
have been filed in the circuit court since the applicants erroneously
perceived their appeal to be one of right. Obviously, filing of the notice of
appeal in these cases was sufficient to confer jurisdiction on the district court
to consider the appropriate remedy. Once the district court’s jurisdiction has
been invoked, it cannot be divested of jurisdiction by a hindsight
determination that the wrong remedy was sought. See Art. V, § 2(a), Fla.
Const. Nor does the district court’s jurisdiction depend on the timely
transmission of the appeal papers to the district court by the clerk of the
circuit court, for it is the action of the claimant which invokes the
jurisdiction of a court. See Fla.R.App.P. 9.040.
Accordingly, we hold that article V, section 2(a) prohibits a district court
from dismissing as untimely a timely notice of appeal filed with the clerk of
the circuit court, which should be considered as a petition for a writ of
certiorari. We note that the district courts below relied upon Lampkin-Asam
v. District Court of Appeal, 364 So. 2d 469 (Fla.1978), which in turn relied
upon Southeast First National Bank of Miami v. Herin, 357 So. 2d 716
(Fla.1978). To the extent of conflict with our decision today, we recede from
Lampkin-Asam and Southeast First National Bank of Miami.
We answer the certified question in the negative, quash the decisions
below, and remand to the district courts to consider petitioners’ claims on
the merits and determine whether certiorari is otherwise warranted.
It is so ordered.
EHRLICH, C.J., and OVERTON, McDONALD, SHAW, GRIMES and
KOGAN, JJ., concur.

54
Kaweblum v. Thornhill Estates Homeowners Ass’n, Inc.
755 So. 2d 85 (Fla. 2000)
PER CURIAM.
Petitioner Y. Aaron Kaweblum petitions this Court for writ of
mandamus. He alleges that the Fourth District Court of Appeal wrongfully
dismissed his appeal for lack of jurisdiction. Because a petition for writ of
mandamus may be used to test the correctness of a dismissal for lack of
jurisdiction by a lower court, we have jurisdiction. See art. V, § 3(b)(8), Fla.
Const.; see also Pino v. District Court of Appeal, 604 So. 2d 1232, 1233
(Fla.1992); State ex rel. Gaines Constr. Co. v. Pearson, 154 So. 2d 833, 835
(Fla.1963). For the reasons expressed below, we grant Kaweblum’s petition.
The Palm Beach County Circuit Court entered summary judgment
against Kaweblum in a civil suit. The order was rendered on December 7,
1998, when the clerk of court docketed the order. On December 30, 1998,
Kaweblum incorrectly filed a notice of appeal with the Broward County
Circuit Court.FN1 The Broward County Circuit Court forwarded the notice of
appeal to the Palm Beach County Circuit Court, which docketed the notice
on January 7, 1999. On January 21, 1999, the Fourth District Court of
Appeal ordered Kaweblum to show cause why his appeal should not be
dismissed as untimely.FN2 After Kaweblum filed a response, the district court
dismissed his appeal, citing Upshaw v. State, 641 So.2d 451 (Fla. 1st DCA
1994). Subsequently, Kaweblum filed a petition for writ of mandamus with
this Court arguing that the Fourth District Court of Appeal should not have
dismissed his appeal because under Florida Rule of Appellate Procedure
9.040(b), his notice of appeal must be treated as timely filed even though it
was filed with the wrong circuit court.
FN1. Kaweblum claims that the erroneous filing was solely due to a
clerical error in “inadvertently” mailing the notice of appeal to Broward
County Circuit Court when it was intended to be filed in Palm Beach County
Circuit Court. With his petition for writ of mandamus, Kaweblum includes a
copy of the notice of appeal, which properly lists Palm Beach County Circuit
Court in the caption of the pleading.
FN2. To be timely, Kaweblum’s notice of appeal must have been filed
within 30 days of the December 7, 1998, rendition of the trial court’s order,
which would have been no later than Wednesday, January 6, 1999. See Fla.
R.App. P. 9.110(b).

55
In order to evaluate the correctness of the district court’s decision, we
first examine Upshaw. The petitioner in Upshaw filed a “Motion for Writ of
Mandamus” with the First District Court of Appeal, arguing that the district
court was required to consider his appeal even though his notice of appeal
was timely filed with the wrong circuit court.FN3 See id. at 452. The First
District held that for appellate jurisdiction to be properly invoked, a notice of
appeal must be timely filed either in the lower court that entered the order to
be reviewed, or in the appellate court where review is sought. See id. at 452.
Because the appellant in Upshaw filed his notice of appeal with the wrong
circuit court, the First District concluded that it was without jurisdiction to
hear the appeal and denied the motion. See id. at 453. In so holding, the
district court relied upon this Court’s opinion in Alfonso v. Department of
Environmental Regulation, 616 So. 2d 44, 47 (Fla.1993).
FN3. Upshaw sought review of an order entered by the Circuit Court of
the Eighth Judicial Circuit, in and for Baker County. In his petition, Upshaw
alleged that he filed notices of appeal in both the Baker County Circuit Court
and the Alachua County Circuit Court. See Upshaw v. State, 641 So. 2d 451,
452 (Fla. 1st DCA 1994). The Baker County Circuit Court never received
the notice of appeal that Upshaw alleged he sent. See id. The Alachua
County Circuit Court received Upshaw’s notice of appeal and forwarded it
to the Baker County Circuit Court, but this notice of appeal was also not
received by the Baker County Circuit Court. See id.
In Alfonso, the following question was certified to this Court:
[W]hether a district court of appeal has jurisdiction to entertain an appeal
from a final judgment of a circuit court where, as here, (1) the appellant
erroneously files a notice of appeal with the district court, rather than the
circuit court, and (2) the appellant takes no corrective action to file the
notice of appeal in the circuit court within thirty days of the rendition of the
final judgment. 616 So. 2d at 45.
This Court answered the question affirmatively, holding that an appellate
court has jurisdiction when the notice is filed “in either the lower court that
issued the order to be reviewed or the appellate court which would have
jurisdiction to review the order.” Id. at 47.
However, we conclude that Upshaw applies this Court’s holding in
Alfonso too narrowly. In Alfonso, the certified question was expressly
limited to the situation where a notice of appeal was erroneously filed with
the appellate court rather than the trial court. Nowhere in Alfonso did this

56
Court explicitly hold that the only situation where a misfiled notice of appeal
properly invokes the appellate court’s jurisdiction is when the notice is
erroneously filed with the appellate court.
[1] [2] To resolve this issue, we turn to the Florida Constitution and the
Rules of Appellate Procedure, both of which are the foundation of our
opinion in Alfonso. Article V, section 2(a) of the Florida Constitution states
that “no cause shall be dismissed because an improper remedy has been
sought.” That directive is implemented by Florida Rule of Appellate
Procedure 9.040(b) and (c), which provides:
(b) Forum. If a proceeding is commenced in an inappropriate court, that
court shall transfer the cause to an appropriate court.
(c) Remedy. If a party seeks an improper remedy, the cause shall be
treated as if the proper remedy had been sought; provided that it shall not be
the responsibility of the court to seek the proper remedy.
The use of the word “shall” under rule 9.040(b) demonstrates that
transfer of an improperly filed cause is mandatory, not discretionary. See
Chaky v. State, 651 So. 2d 1169, 1172 (Fla.1995) (construing “shall” to be
mandatory and “may” to be directory in a rule of procedure); see also
Sternfield v. Jewish Introductions, Inc., 581 So. 2d 987, 988 (Fla. 4th DCA
1991) (finding that the circuit court had departed from essential
requirements of law when it denied petitioner’s request to transfer a petition
for writ of certiorari erroneously filed with that court to the district court of
appeal).
[3] Further, the 1977 committee notes to rule 9.040(b) provide that:
[A] case will not be dismissed automatically because a party seeks an
improper remedy or invokes the jurisdiction of the wrong court. The court
must instead treat the case as if the proper remedy had been sought and
transfer it to the court having jurisdiction. All filings in the case have the
same legal effect as though originally filed in the court to which transfer is
made. (Emphasis supplied.)
Although the committee notes to rule 9.040(b) are only persuasive
authority and are not part of the rule, see D.K.D. v. State, 470 So. 2d 1387,
1389 (Fla.1985), this Court may look to the notes as a means of determining
the clear intent of the rule. See, e.g., State v. Salzero, 714 So. 2d 445, 447
(Fla.1998) (finding that “strict adherence to [the time requirements of a rule
of criminal procedure] would not comport with the clear intent of this

57
section as evident from the committee notes to the 1984 amendment of the
rule”).
These committee notes indicate that while appellants must conform to the
rules adopted by this Court in seeking an appeal, they should not lose their
opportunity to present what may be meritorious claims for relief simply
because they have either sought the wrong form of relief, or have sought the
proper relief in the wrong court. Thus, both the language of the rule and the
committee notes support an interpretation that under rule 9.040(b) a notice
of appeal timely filed in the wrong court must be transferred to the proper
court and treated as timely filed in that court.
Further, this Court has construed the rule similarly in the past by holding
that when a document is filed in the wrong court and the document is
transferred to the proper court, the date of filing is the date the document
was filed in the wrong court, not the date it was received by the proper court.
See Alfonso, 616 So. 2d at 47. For example, in Alfonso this Court noted that
pursuant to rule 9.040(b), a notice of appeal erroneously filed with the
district court “should be transferred to the appropriate court with the date of
filing being the date the document was filed in the wrong court.” 616 So. 2d
at 47. More recently, in a slightly different context, this Court found that in
cases where trial court venue has been changed during the course of the
proceedings, “the district courts of appeal should liberally transfer their
appellate jurisdiction when it appears that the appeal or petition has been
timely filed but directed to the wrong appellate court.” Vasilinda v. Lozano,
631 So. 2d 1082, 1087 n. 3 (Fla.1994) (citing article V, section 2(a), Fla.
Const.).
The dilemma that would arise were we to find the date of filing to be the
date that the proper court receives the notice of appeal is highlighted by the
following anomaly. If a notice of appeal is filed with the wrong trial court
and that court forwards the notice to the appropriate court within the 30-day
time limit, the district court must treat the notice as timely filed and hear the
appeal. On the other hand, if the forwarded notice arrives at the proper court
even one day late because of a delay on the part of the forwarding clerk’s
office, then the appeal would be dismissed for lack of jurisdiction. The
timeliness of a misfiled notice of appeal should not depend on how quickly
the court where the notice was improperly filed transfers the notice to the
appropriate court.FN4

58
FN4. Further, in the instant case, the Broward County Circuit Court did
not return Kaweblum’s improperly filed notice of appeal or otherwise notify
him of the misfiling. Rather, the court assumed the duty of forwarding the
notice to the Palm Beach County Circuit Court pursuant to the mandate of
rule 9.040(b). Had Kaweblum been notified of his misfiling, he could have
independently filed the notice of appeal with the Palm Beach County Circuit
Court to ensure that it would be timely filed. We do not imply any
impropriety on the part of the Broward County Clerk’s Office; however, we
believe that Kaweblum should not be prejudiced by any delay that occurred
in the transfer of his notice by the circuit court because Kaweblum had no
notification of his misfiling until a motion to dismiss was filed with the
Fourth District Court of Appeal.
[4] Accordingly, we conclude that consistent with the intent of the
Florida Constitution and the Florida Rules of Appellate Procedure, when a
notice of appeal is timely filed in the wrong court, and there is no indication
that the misfiling was intentionally done for the purpose of convenience or
delay, the notice of appeal must be treated as timely filed in the appropriate
court. We disapprove Upshaw to the extent it is inconsistent with this
opinion.
Because there is no indication that Kaweblum’s notice of appeal was
intentionally filed with the wrong court for the purpose of convenience or
delay, we hold that the Fourth District Court of Appeal’s dismissal of
Kaweblum’s appeal for lack of jurisdiction was improper. Accordingly, we
conclude that the Fourth District should have treated Kaweblum’s notice of
appeal as timely filed and that the court erred in dismissing his appeal for
lack of jurisdiction. We grant the petition and order that the Fourth District
Court of Appeal reinstate Kaweblum’s appeal.FN5 However, because we trust
that the Fourth District Court of Appeal will fully comply with the dictates
of this order, we withhold issuance of the writ.
FN5. Even though in the instant case we find that Kaweblum has a right
to have his notice of appeal treated as timely filed, we refer this issue to the
Appellate Rules Committee for the purpose of determining whether
clarification of rule 9.040(b) is necessary.
It is so ordered.
HARDING, C.J., and SHAW, ANSTEAD, PARIENTE, LEWIS and
QUINCE, JJ., concur.

59
WELLS, J., dissents with an opinion.
***

G. Nonjurisdictional Matters

Florida Rules of Appellate Procedure

Rule 9.040. General Provisions


***
(h) Non-Jurisdictional Matters. Failure of a clerk or a party timely to file
fees or additional copies of notices or petitions or the conformed copy of
the order or orders designated in the notice of appeal shall not be
jurisdictional; provided that such failure may be the subject of appropriate
sanction.

Payment of the filing fee is not jurisdictional. See Weintraub v. Alter, 482 So. 2d 454
(Fla. 3d DCA 1986).
The clerk of the court may not decline to accept a notice of appeal on the ground that the
appellant did not pay the filing fee. See Jones v. Peninsula Motor Club, Inc, 558 So. 2d
517 (Fla. 1st DCA 1990).

III. Subject Matter Jurisdiction

A. The Florida Supreme Court

The Florida Constitution

Art. V, § 3(b), Fla. Const.


(b) Jurisdiction.--The supreme court:
(1) Shall hear appeals from final judgments of trial courts imposing the
death penalty and from decisions of district courts of appeal declaring
invalid a state statute or a provision of the state constitution.
(2) When provided by general law, shall hear appeals from final judgments

60
entered in proceedings for the validation of bonds or certificates of
indebtedness and shall review action of statewide agencies relating to
rates or service of utilities providing electric, gas, or telephone service.
(3) May review any decision of a district court of appeal that expressly
declares valid a state statute, or that expressly construes a provision of
the state or federal constitution, or that expressly affects a class of
constitutional or state officers, or that expressly and directly conflicts with
a decision of another district court of appeal or of the supreme court on
the same question of law.
(4) May review any decision of a district court of appeal that passes upon
a question certified by it to be of great public importance, or that is certified
by it to be in direct conflict with a decision of another district court of
appeal.
(5) May review any order or judgment of a trial court certified by the district
court of appeal in which an appeal is pending to be of great public
importance, or to have a great effect on the proper administration of
justice throughout the state, and certified to require immediate resolution
by the supreme court.
(6) May review a question of law certified by the Supreme Court of the
United States or a United States Court of Appeals which is determinative
of the cause and for which there is no controlling precedent of the
supreme court of Florida.
(7) May issue writs of prohibition to courts and all writs necessary to the
complete exercise of its jurisdiction.
(8) May issue writs of mandamus and quo warranto to state officers and
state agencies.
(9) May, or any justice may, issue writs of habeas corpus returnable
before the supreme court or any justice, a district court of appeal or any
judge thereof, or any circuit judge.

Florida Rules of Appellate Procedure

Rule 9.030. Jurisdiction of Courts


(a) Jurisdiction of Supreme Court.
(1) Appeal Jurisdiction.
(A) The supreme court shall review, by appeal
(i) final orders of courts imposing sentences of death;

61
(ii) decisions of district courts of appeal declaring invalid a state
statute or a provision of the state constitution.
(B) If provided by general law, the supreme court shall review
(i) by appeal final orders entered in proceedings for the validation of
bonds or certificates of indebtedness;
(ii) action of statewide agencies relating to rates or service of
utilities providing electric, gas, or telephone service.
(2) Discretionary Jurisdiction.The discretionary jurisdiction of the supreme
court may be sought to review
(A) decisions of district courts of appeal that
(i) expressly declare valid a state statute;
(ii) expressly construe a provision of the state or federal
constitution;
(iii) expressly affect a class of constitutional or state officers;
(iv) expressly and directly conflict with a decision of another district
court of appeal or of the supreme court on the same question of law;
(v) pass upon a question certified to be of great public importance;
(vi) are certified to be in direct conflict with decisions of other district
courts of appeal;
(B) orders and judgments of trial courts certified by the district court of
appeal in which the appeal is pending to require immediate resolution by
the supreme court, and
(i) to be of great public importance, or
(ii) to have a great effect on the proper administration of justice;
(C) questions of law certified by the Supreme Court of the United States or
a United States court of appeals that are determinative of the cause of
action and for which there is no controlling precedent of the Supreme
Court of Florida.
(3) Original Jurisdiction.The supreme court may issue writs of prohibition
to courts and all writs necessary to the complete exercise of its
jurisdiction, and may issue writs of mandamus and quo warranto to state
officers and state agencies. The supreme court or any justice may issue
writs of habeas corpus returnable before the supreme court or any justice,
a district court of appeal or any judge thereof, or any circuit judge.
***

62
Ansin v. Thurston
101 So. 2d 808 (Fla. 1958)
DREW, Justice.
The petitioner, defendant in the trial court, seeks review by certiorari of a
decision of the District Court of Appeal, Third District, by which certain
judgments against him in tort actions for negligence were affirmed.
The only point presented in the appeal below was the alleged error of the
trial court in denying defendant’s motion for directed verdict on the issue of
liability. The actions against him, both arising out of the same
circumstances, were by the respondent individually and as administrator of
the estate of his minor son, who died by drowning in a ‘rockpit’ on land
owned by defendant. In each case the cause of action was dependent upon
proof of facts sufficient to come within the doctrine of tort liability usually
referred to as attractive nuisance.
[1] It was the opinion of the district court that the facts alleged and
proved, details of which appear fully in the published report of the case in
that court, were sufficient to present a jury question under the rule
enunciated in the case of Allen v. William P. McDonald Corporation, Fla.,
42 So.2d 706, to the effect that the maintenance of an artificial body of water
where there exists some unusual element of danger not present in ponds or
natural bodies of water generally may constitute actionable negligence
supporting recovery for injury or death by drowning of a minor child upon
the theory noted above.
The petition herein is necessarily prosecuted under that portion of
amended Article V, Section 4(b), (4(2), F.S.1957) of the Florida
Constitution, F.S.A., authorizing review by certiorari in this Court of ‘any
decision of a district court of appeal * * * that is in direct conflict with a
decision of another district court of appeal or of the supreme court on the
same point of law * * *’, and the corresponding provision of Rule 2.1, subd.
a(5)(b) of the Florida Appellate Rules.
[2] Petitioner contends that the decision below is not in accord with the
rule of the case relied upon by the district court, and that it conflicts with
two subsequent decisions where this Court affirmed judgments for defendant
in such actions, but did not purport to overrule the earlier case. Newby v.
West Palm Beach Water Co., Fla., 47 So. 2d 527; Lomas v. West Palm
Beach Water Co., Fla., 57 So. 2d 881. In the brief much attention is devoted
to the character of the banks surrounding the body of water involved, and

63
argument is addressed primarily to the point that the present case is
distinguishable upon the facts from Allen v. McDonald Corp., supra. The
very fact that petitioner finds it necessary in a proceeding of this nature to
review with such particularity the evidence in the various cases, and to refer
to authorities elsewhere to bolster his position, would indicate that the
argument is primarily upon the merits of the decision attacked as opposed to
any contention that it brings into existence a conflict of authority in this
jurisdiction. These considerations, among others, impel our conclusion that
the writ should be denied for failure to show direct conflict between the
decision in question and a previous ruling ‘on the same point of law.’ Rule
2.1, subd. a(5)(6) supra.
[3] [4] We have heretofore pointed out that under the constitutional plan
the powers of this Court to review decisions of the district courts of appeal
are limited and strictly prescribed. Diamond Berk Insurance Agency, Inc., v.
Goldstein, Fla., 100 So. 2d 420; Sinnamon v. Fowlkes, Fla., 101 So. 2d 375.
It was never intended that the district courts of appeal should be intermediate
courts. The revision and modernization of the Florida judicial system at the
appellate level was prompted by the great volume of cases reaching the
Supreme Court and the consequent delay in the administration of justice.
The new article embodies throughout its terms the idea of a Supreme Court
which functions as a supervisory body in the judicial system for the State,
exercising appellate power in certain specified areas essential to the
settlement of issues of public importance and the preservation of uniformity
of principle and practice, with review by the district courts in most instances
being final and absolute.
To fail to recognize that these are courts primarily of final appellate
jurisdiction and to allow such courts to become intermediate courts of appeal
would result in a condition far more detrimental to the general welfare and
the speedy and efficient administration of justice than that which the system
was designed to remedy.
[5] The suggestion is inevitable that the detailed consideration given the
issues here presented and the exposition of reasons by written opinion,
contrary to the customary appellate practice in denying certiorari, involves
the expenditure of quite enough judicial labor to have enabled the Court to
dispose of this controversy on its merits, and so far as the particular
litigation is concerned our efforts might more logically be so directed. But it
is of obvious importance that there should be developed consistent rules for

64
limiting issuance of the writ of certiorari to ‘cases involving principles the
settlement of which is of importance to the public, as distinguished from that
of the parties, and in cases where there is a real and embarrassing conflict of
opinion and authority’ between decisions. See Layne & Bowler Corp. v.
Western Well Works, 261 U.S. 387, 43 S.Ct. 422, 423, 67 L.Ed. 712. While
the court in the latter case dealt with rules couched in varying language, the
conclusion is inescapable that our own constitutional provision has the same
general objectives. A limitation of review to decisions in ‘direct conflict’
clearly evinces a concern with decisions as precedents as opposed to
adjudications of the rights of particular litigants.
Similar provisions in the court systems of other states have been so
construed: ‘A conflict of decisions * * * must be on a question of law
involved and determined, and such that one decision would overrule the
other if both were rendered by the same court; in other words, the decisions
must be based practically on the same state of facts and announce
antagonistic conclusions.’ 21 C.J.S. Courts s 462.
The general import of these pronouncements should be of benefit in
charting a course of practice under amended Article V, and considered in
relation to the instant case they serve to sustain and explain our conclusion
herein.
Writ denied.
TERRELL, C. J., THORNAL and O'CONNELL, JJ., and WIGGINTON,
District Judge, concur.

1. Judgments Imposing the Death Penalty

Florida Rules of Appellate Procedure

Rule 9.142. Procedures for Review in Death Penalty Cases


***
(6) Scope of Review. In death penalty cases, whether or not insufficiency
of the evidence or proportionality is an issue presented for review, the
court shall review these issues and, if necessary, remand for the
appropriate relief.

65
Article V, Section 3(b)(1). The Florida Supreme Court’s jurisdiction to hear an appeal
from a judgment of the circuit court imposing the death penalty is mandatory. See Castro
v. State, 547 So. 2d 111 (Fla. 1989). The court’s jurisdiction is invoked by filing a notice
of appeal in the circuit court.
The Florida Supreme Court must engage in a full review of the record even if the
defendant has not challenged the conviction or sentence of death. See Maxwell v. State,
443 So. 2d 967 (Fla. 1983).

2. Decisions Declaring State Laws Invalid

Article V, Section 3(b)(1). The Florida Supreme Court’s jurisdiction to hear an appeal
from a decision of district court of appeal declaring invalid a state statute or a provision
of the state constitution is mandatory. See Bush v. Holmes, 919 So. 2d 392 (Fla. 2006).
The court’s jurisdiction is invoked by filing a notice of appeal in the district court.

3. Bond Validation Proceedings

Article V, Section 3(b)(2). This provision is not a self-executing grant of jurisdiction but
rather it depends on the existence of a general law providing for the appeal. The
legislature implemented this constitutional provision by enacting a general law. See §
75.08. Fla. Stat.

4. Utility Cases – Rate or Service Orders

Article V, Section 3(b)(2). This provision is not a self-executing grant of jurisdiction but
rather it depends on the existence of a general law providing for the appeal. This
constitutional provision is implemented by three statutes. See §§ 350.128; 366.10;
364.381 Fla. Stat.

5. Decisions Declaring State Laws Valid

Article V, Section 3(b)(3). The Florida Supreme Court has discretionary jurisdiction to
review a decision by a district court of appeal that expressly declares a state statute to be
valid. Jurisdiction is invoked by filing in the district court a document styled as a Notice
to Invoke Discretionary Jurisdiction.

66
6. Construction of State or Federal Constitution

Article V, Section 3(b)(3). The Florida Supreme court has discretionary jurisdiction to
review a decision by a district court of appeal that expressly construes a provision of the
state or federal constitution. Jurisdiction is invoked by filing in the district court a
document styled as a Notice to Invoke Discretionary Jurisdiction.

7. Decisions Affecting a Class of Constitutional Officers

Article V, Section 3(b)(3). The Florida Supreme Court has discretionary jurisdiction to
review a decision by a district court of appeal expressly affecting a class of constitutional
officers. Jurisdiction is invoked by filing in the district court a document styled as a
Notice to Invoke Discretionary Jurisdiction.

8. Express and Direct Conflict of Decisions

Article V, Section 3(b)(3). The Florida Supreme Court has jurisdiction to review a
decision by a district court of appeal that expressly and directly conflicts with the
decision of another district court of appeal or of the supreme court. Jurisdiction is
invoked by filing in the district court a document styled as a Notice to Invoke
Discretionary Jurisdiction.

The Florida Star v. B.J.F.


530 So. 2d 286 (Fla. 1988)
BARKETT, Justice.
This case is before us on the following question of Florida law certified
by the United States Supreme Court:
Whether the Florida Supreme Court had jurisdiction, pursuant to Article
V, § 3(b)(3) FN1 of the Florida Constitution or otherwise, to hear Appellant’s
appeal [petition for review] in this cause from the Florida First District Court
of Appeal?
FN1. This section vests the Florida Supreme Court with subject-matter
jurisdiction over cases that manifest express and direct conflict with
opinions of another district court or of this Court. For convenience, this type
of jurisdiction usually is called “conflict jurisdiction.”

67
The Florida Star v. B.J.F., --- U.S. ----, 108 S.Ct. 499, 499, 98 L.Ed.2d
498 (1987). We have jurisdiction. Art. V, § 3(b)(6), Fla. Const. As delimited
by this opinion, we answer in the affirmative.
The Florida Star, a Jacksonville newspaper, published the name of a rape
victim that police erroneously had included in material released to the press
pretrial. There is no dispute that the name should not have been released and
that the newspaper itself had a policy against the publication of rape victims’
names. Publication appeared to be a criminal violation under section 794.03,
Florida Statutes (1985).FN2
FN2. The statute states:
No person shall print, publish, or broadcast, or cause or allow to be
printed, published, or broadcast, in any instrument of mass communication
the name, address, or other identifying fact or information of the victim of
any sexual offense within this chapter. An offense under this section shall
constitute a misdemeanor of the second degree, punishable as provided in s.
775.082, s. 775.083, or s. 775.084.
B.J.F., the rape victim, brought a civil action premised on a statutory duty
arising from section 794.03. The Florida Star moved for dismissal based on
the ground that the theory of recovery violated the first and fourteenth
amendments. In denying this motion, the trial court ruled that no such
violation would occur, and it specifically upheld the constitutionality of
section 794.03.
On appeal, the Florida Star again challenged the constitutionality of the
statute. The district court affirmed but did not discuss section 794.03 except
to quote it verbatim, nor did it expressly uphold the statute against
appellant’s constitutional challenge.
The Florida Star subsequently filed a jurisdictional brief with this Court,
seeking discretionary review. Review summarily was denied. The Florida
Star v. B.J.F., 509 So.2d 1117 (Fla.1987).
On August 26, 1987, the Florida Star sought review in the United States
Supreme Court. Appellee filed a motion to dismiss on grounds the appeal
was untimely. Appellee argued that the Florida Supreme Court lacked
jurisdiction to review the case, and that the opinion of the First District thus
was the final decision of the highest state court empowered to hear the
cause. Under this argument, the Florida Star should have appealed to the
United States Supreme Court within ninety days of the First District’s

68
opinion. The United States Supreme Court then certified the instant question
to this Court.
We do not read the question presented by the Supreme Court as a request
to explain the internal mechanism of the court or to attempt the impossible
task of second-guessing the original panel’s decision on jurisdiction. Nor do
we believe, as appellee suggests, that the present court should reexamine the
question and decide anew whether conflict existed.
Instead, we limit our answer to the context in which the question was
posed. For that sole purpose, we answer the question in the affirmative. This
Court in the broadest sense has subject-matter jurisdiction under article V,
section 3(b)(3) of the Florida Constitution, over any decision of a district
court that expressly addresses a question of law within the four corners of
the opinion itself.FN3 That is, the opinion must contain a statement or citation
effectively establishing a point of law upon which the decision rests. The
opinion in B.J.F. unquestionably met this requirement.
FN3. This Court does not, however, have subject-matter jurisdiction over
a district court opinion that fails to expressly address a question of law, such
as opinions issued without opinion or citation. Thus, a district court decision
rendered without opinion or citation constitutes a decision from the highest
state court empowered to hear the cause, and appeal may be taken directly to
the United States Supreme Court. Moreover, there can be no actual conflict
discernible in an opinion containing only a citation to other case law unless
one of the cases cited as controlling authority is pending before this Court,
or has been reversed on appeal or review, or receded from by this Court, or
unless the citation explicitly notes a contrary holding of another district court
or of this Court. See Jollie v. State, 405 So. 2d 418, 420 (Fla.1981).
We premise our holding on our conclusion that article V, section 3(b)(3)
creates and defines two separate concepts. The first is a general grant of
discretionary subject-matter jurisdiction, and the second is a constitutional
command as to how the discretion itself may be exercised. In effect, the
second is a limiting principle dictated to this Court by the people of Florida.
While our subject-matter jurisdiction in conflict cases necessarily is very
broad, our discretion to exercise it is more narrowly circumscribed by what
the people have commanded:
(b) JURISDICTION.-The supreme court:
....

69
(3) May review any decision of a district court of appeal ... that expressly
and directly conflicts with a decision of another district court of appeal or of
the supreme court on the same question of law.
Art. V, § 3(b)(3), Fla. Const.
Thus, it is not necessary that conflict actually exist for this Court to
possess subject-matter jurisdiction, only that there be some statement or
citation in the opinion that hypothetically could create conflict if there were
another opinion reaching a contrary result. This is the only reasonable
interpretation of this constitutional provision. As the final authority on the
meaning of the Florida Constitution, see Art. IV, § 1(c), and Art. V, §
3(b)(1), (3), Fla. Const., this Court has the final and inherent power to
determine what constitutes express and direct conflict. No other authority
exists, except the people pursuant to their power to amend the constitution,
that may nullify this Court’s pronouncements on that question.
This, by definition, is discretion, not jurisdiction. As noted in Black’s
Law Dictionary 419 (5th ed. 1979), discretion is the exercise of judicial
judgment, based on facts and guided by law.... It is a legal discretion to be
exercised in discerning the course prescribed by law and is not to give effect
to the will of the judge, but to that of the law.
Subject-matter jurisdiction, on the other hand, is defined as:
Power of a particular court to hear the type of case that is then before it....
jurisdiction over the nature of the cause of action and relief sought.... Id. at
767. While this Court has subject-matter jurisdiction to hear any petition
arising from an opinion that establishes a point of law, we have operated
within the intent of the constitution’s framers, as we perceive it, in refusing
to exercise our discretion where the opinion below establishes no point of
law contrary to a decision of this Court or another district court.
We thus conclude that we had complete subject-matter jurisdiction to
hear B.J.F. and decide the case on its merits with finality. This jurisdiction
must be regarded as complete until the time the petition for review was
denied. Moreover, the denial of review in B.J.F. did not operate to deprive
this Court of its subject-matter jurisdiction retroactively, but merely
constituted the point in time at which jurisdiction, for whatever reason, had
ended.
We confess that we are much influenced in this holding by the procedural
quagmire that would result from a negative answer. To seek review of a state
court judgment in the United States Supreme Court, a litigant first must

70
exhaust all avenues of review available in the courts of the state. The fact
that review in the highest court is discretionary is irrelevant; the litigant still
must seek such review in order to proceed to the United States Supreme
Court. American Ry. Express v. Levee, 263 U.S. 19, 20-21, 44 S.Ct. 11, 12-
13, 68 L.Ed. 140 (1923); Stratton v. Stratton, 239 U.S. 55, 56-57, 36 S.Ct.
26, 27, 60 L.Ed. 142 (1915).
It is therefore essential to the preservation of a litigant’s right to United
States Supreme Court review that he or she know with certainty the avenues
of appellate review required by the courts of the state. If, after the fact, we
held in a case such as this that there was no jurisdiction, litigants would be
placed in a needlessly burdensome position. A party would have to try to
predict which court ultimately would recognize jurisdiction in the case and
file a petition for review accordingly. A party who files only in the United
States Supreme Court, however, would risk the objection that he or she has
not exhausted state court remedies. On the other hand, a party who files only
in the Florida Supreme Court and is denied review, would risk the objection
made by B.J.F. here that the later appeal to the United States Supreme Court
is untimely.
Alternatively, a party could file a petition for review in both the Florida
Supreme Court and the United States Supreme Court simultaneously, simply
to protect his or her rights. This solution would entail double work, separate
briefs, and much more expense for the clients. Moreover, a stay would likely
need to be sought in the United States Supreme Court because the appeal in
that Court would not be ripe for review unless and until this Court denied
review or otherwise disposed of the case. The situation is further aggravated
by the fact that, on some occasions, this Court agrees to review a case on the
merits, but after briefing or argument determines that review was
improvidently granted.
No purpose is served by this duplication of effort, added expense and
uncertainty. We do not believe the people intended such a result when they
amended the constitution in 1980.
For the foregoing reasons, we answer the certified question in the
affirmative. This opinion shall be transmitted forthwith to the United States
Supreme Court.
It is so ordered.
EHRLICH, C.J., and OVERTON, McDONALD, SHAW, GRIMES and
KOGAN, JJ., concur.

71
Jenkins v. State
385 So. 2d 1356 (Fla. 1980)
SUNDBERG, Justice.
We here address the question whether this Court currently has
jurisdiction to review a decision of a district court of appeal which reads in
its entirety “Per Curiam Affirmed” where a dissenting opinion is filed in the
case. We answer the question in the negative.
Review of the decision of the District Court of Appeal, Fourth District,
[FN*] was sought in this cause by notice to invoke the certiorari jurisdiction
of this Court filed April 11, 1980. By his application petitioner asserts that
the decision of the district court is in conflict with decisions of other districts
or with Supreme Court decisions upon the issue of whether uncorroborated
hearsay information from a confidential informant, who had not divulged the
source of his information, was sufficient to establish probable cause for a
warrantless search of a vehicle. Prior to trial, petitioner moved to suppress
evidence seized in a search of his vehicle. The trial court denied the motion
to suppress. Petitioner subsequently entered a plea of nolo contendere
preserving his right to appeal the trial court ruling. On review the district
court affirmed the ruling of the trial court without opinion. One member of
the three-judge panel dissented to the decision of the majority in a
comprehensive opinion which recited the facts extensively and concluded
that under prevailing law the search violated petitioner’s fourth amendment
rights.
FN* Jenkins v. State, 382 So.2d 83 (Fla. 4th DCA 1980).
After ratification by the people of this state at an election held on March
11, 1980, article V, section 3 of the Florida Constitution pertaining to the
jurisdiction of the Supreme Court was substantially revised. In particular,
section 3(b)(3) underwent a dramatic change. Prior to April 1, 1980 (the
effective date of the amendment), the provisions of section 3(b)(3) relating
to review of conflicting decisions read as follows:
May review by certiorari any decision of a district court of appeal . . . that
is in direct conflict with a decision of any district court of appeal or of the
supreme court on the same question of law. . . .
Post April 1, 1980, that section reads with respect to review of conflicting
decisions:

72
May review any decision of a district court of appeal . . . that expressly
and directly conflicts with a decision of another district court of appeal or of
the supreme court on the same question of law. . . . (Emphasis supplied.)
[1] The constitutional amendment must be viewed in light of the
historical development of the decisional law extant at the time of its
adoption and the intent of the framers and adopters. Our inquiry must begin
with the amendment to article V of the Florida Constitution occurring in
1956, whereby the district courts of appeal were created. In grappling with
the significance of the revised jurisdiction of this Court, a tone was set early
on. In Ansin v. Thurston, 101 So.2d 808, 810 (Fla.1958), speaking through
Justice Drew, the Court said:
We have heretofore pointed out that under the constitutional plan the
powers of this Court to review decisions of the district courts of appeal are
limited and strictly prescribed. Diamond Berk Insurance Agency, Inc. v.
Goldstein, Fla., 100 So. 2d 420; Sinnamon v. Fowlkes, Fla., 101 So. 2d 375.
It was never intended that the district courts of appeal should be intermediate
courts. The revision and modernization of the Florida judicial system at the
appellate level was prompted by the great volume of cases reaching the
Supreme Court and the consequent delay in the administration of justice.
The new article embodies throughout its terms the idea of a Supreme Court
which functions as a supervisory body in the judicial system for the State,
exercising appellate power in certain specified areas essential to the
settlement of issues of public importance and the preservation of uniformity
of principle and practice, with review by the district courts in most instances
being final and absolute.
To fail to recognize that these are courts primarily of final appellate
jurisdiction and to allow such courts to become intermediate courts of appeal
would result in a condition far more detrimental to the general welfare and
the speedy and efficient administration of justice than that which the system
was designed to remedy.
[2] This was followed by Lake v. Lake, 103 So.2d 639 (Fla.1958), where
Justice Thomas again reviewed the history of and purposes for the 1956
amendment to article V and held that in order to fulfill those purposes, a “per
curiam” decision without opinion of a district court of appeal would not be
reviewed by this Court upon petition for certiorari based on “direct conflict”
jurisdiction except in those rare cases where the “restricted examination
required in proceedings in certiorari (revealed) that a conflict had arisen with

73
resulting injustice to the immediate litigant.” Id. at 643. Some seven years
later, however, in an opinion which observed that the rule of Lake v. Lake
had been eroded de facto if not de jure by subsequent actions of the Court, a
majority of the Court determined that there was jurisdictional power under
section 3(b)(3) to review district court decisions rendered “per curiam”
without opinion if from the “record proper” conflict with another decision
could be discerned. Foley v. Weaver Drugs, Inc., 177 So. 2d 221 (Fla.1965).
In the interim the Court had already concluded that conflict certiorari
jurisdiction could be founded on a dissenting opinion to a per curiam
majority decision rendered without opinion. Huguley v. Hall, 157 So. 2d 417
(Fla.1963). This position was adopted by a majority of the Court without
discussion or rationale and has been subsequently followed without
amplification of reasoning. E. g., Autrey v. Carroll, 240 So. 2d 474
(Fla.1970); Commerce Nat’l Bank in Lake Worth v. Safeco Ins. Co., 284 So.
2d 205 (Fla.1973). In the Commerce National Bank decision, however, the
impediments to relying on the factual statement contained in a dissenting
opinion to establish conflict jurisdiction were observed:
When facts and testimony are set forth in a majority opinion, they are
assumed to be an accurate presentation upon which the judgment of the
court is based. However, a dissent does not rise to a similar level of dignity
and is not considered as precedent; note, for example, that West Publishing
Company does not offer headnotes for dissents, regardless of their legal
scholarship. By definition, a dissent contains information, interpretations or
legal analysis which has been rejected in whole or part, by the majority. It is
also possible that the majority accepts matters set forth in the dissent, but for
other reasons declines to follow its line of thought. The majority is under no
compulsion to respond to a dissent or to set out the measure of their
reluctance to agree. The issuance of a per curiam opinion without comment
or citation of authority remains the prerogative of the majority. Id. at 207.
More recently, the wisdom of the jurisdictional policies expressed in
Foley and Huguley have been brought into question by several members of
this Court. See Florida Greyhound Owners & Breeders Ass’n, Inc. v. West
Flagler Associates, Ltd., 347 So. 2d 408, 408 (Fla.1977) (England, J.,
concurring; Overton, C. J., concurring specially); Golden Loaf Bakery, Inc.
v. Charles W. Rex Constr. Co., 334 So. 2d 585, 586 (Fla.1976) (England, J.
and Overton, C. J., concurring); AB CTC v. Morejon, 324 So. 2d 625, 628
(Fla.1975) (England and Overton, JJ., dissenting).

74
It was against this jurisprudential backdrop and in the face of a staggering
case load that in November, 1979, this Court urged the legislature, meeting
in special session, to enact a proposed amendment to section 3 of article V of
the Florida Constitution to limit the jurisdiction of the Supreme Court.
Times were not unlike the year 1956 when the challenge confronting the
drafters of that amendment to the judicial article was described thus:
The means and procedure required to accomplish the improvement were
difficult, complicated, tedious and onerous.
Yet the determination was not lacking for congestion in the court of last
resort had become almost intolerable. The time had come when the court,
working at top speed, with cases, except extremely emergent ones, set in the
order of their maturity, was hearing arguments as late as fourteen months
after the cases were ready for oral presentation.
For about eighteen months after its creation the (Judicial) Council, in
periodic meetings, debated and deliberated the method which might most
effectively modernize a system that by overloading had ceased to function as
it should to assure litigants justice without undue, or even ruinous, delay.
The words of Gladstone were often heard: “Justice delayed is justice
denied.” Lake v. Lake, 103 So. 2d 639, 640-41. The legislature responded
through enactment of Senate Joint Resolution No. 20-C, which forms the
language of the current section 3 of article V.
At hearings before the legislature and in countless meetings with
representatives of The Florida Bar, The Conference of Circuit Judges of
Florida, the Appellate Judges’ Conference, The League of Women Voters as
well as other interested organizations too numerous to recount, members of
this Court represented that one of the intents and effects of the revision of
section 3(b)(3) was to eliminate the jurisdiction of the Supreme Court to
review for conflict purposes per curiam decisions of the district courts of
appeal rendered without opinion, regardless of the existence of a concurring
or dissenting opinion. These same representations were made consistently to
the public at large preceding the ballot on the proposed amendment. There
can be little doubt that the electorate was informed as to this matter, because
opponents of the amendment broadcast from one end of this state to the
other that access to the Supreme Court was being “cut off,” and that the
district courts of appeal would be the only and final courts of appeal in this
state. With regard to review by conflict certiorari of per curiam decisions
rendered without opinion, they were absolutely correct.

75
The pertinent language of section 3(b)(3), as amended April 1, 1980,
leaves no room for doubt. This Court may only review a decision of a
district court of appeal that expressly and directly conflicts with a decision
of another district court of appeal or the Supreme Court on the same
question of law. The dictionary definitions of the term “express” include: “to
represent in words”; “to give expression to.” “Expressly” is defined: “in an
express manner.” Webster’s Third New International Dictionary, (1961 ed.
unabr.). The single word “affirmed” comports with none of these definitions.
Furthermore, the language and expressions found in a dissenting or
concurring opinion cannot support jurisdiction under section 3(b)(3) because
they are not the decision of the district court of appeal. As stated by Justice
Adkins in Gibson v. Maloney, 231 So. 2d 823, 824 (Fla.1970), “(i)t is
conflict of decisions, not conflict of opinions or reasons that supplies
jurisdiction for review by certiorari.” (Emphasis in original.)
[3] Accordingly, we hold that from and after April 1, 1980, the Supreme
Court of Florida lacks jurisdiction to review per curiam decisions of the
several district courts of appeal of this state rendered without opinion,
regardless of whether they are accompanied by a dissenting or concurring
opinion, when the basis for such review is an alleged conflict of that
decision with a decision of another district court of appeal or of the Supreme
Court. The application for review in the instant case having been filed
subsequent to March 31, 1980, it is therefore dismissed.
ENGLAND, C. J., and BOYD, OVERTON, ALDERMAN and
McDONALD, JJ., concur.
ENGLAND, C. J., concurs specially with an opinion.
***
ADKINS, J., dissents with an opinion.
***
The Florida Supreme Court lacks jurisdiction under section 3(b)(3) to review a per
curiam affirmance that merely cites a precedent, even if the precedent is in conflict with a
decision of another district court of appeal or of the supreme court. See Dodi Publ. Co. v.
Editorial America, S.A, 385 So. 2d 1369 (Fla. 1980).
The Florida Supreme Court has jurisdiction under section 3(b)(3) to review a per curiam
affirmance that cites a precedent if the precedent has been reversed, see Jollie v. State,
405 So. 2d 418 (Fla. 1981), or if the precedent is pending on review by the supreme
court. See Walker v. State, 682 So. 2d 555 (Fla. 1996).

76
9. Certified Decisions of District Courts

Article V, Section 3(b)(4). The Florida Supreme Court has jurisdiction to review a
decision by a district court of appeal if the district court has certified that its decision
passes upon a question of great public importance or that its decision conflicts with a
decision by another district court of appeal. Jurisdiction is invoked by filing in the
district court a document styled as a Notice to Invoke Discretionary Jurisdiction.

Floridians for a Level Playing Field v. Floridians against Expanded


Gambling
967 So. 2d 832 (Fla. 2007)
PARIENTE, J.
We have for review the decision in Floridians Against Expanded
Gambling v. Floridians for a Level Playing Field, 945 So. 2d 553 (Fla. 1st
DCA 2006), in which the First District Court of Appeal certified a question
to be of great public importance. Initially, the Court accepted review
pursuant to article V, section 3(b)(4) of the Florida Constitution, which gives
us discretionary jurisdiction to review “any decision of a district court of
appeal that passes upon a question certified by it to be of great public
importance.” Upon further consideration, we conclude that jurisdiction was
improvidently granted and therefore discharge this case for the two reasons
explained below.
[1] [2] In order to have discretionary jurisdiction based on a certified
question, there are essentially three prerequisites that must be met. First, it is
essential that the district court of appeal pass upon the question certified by
it to be of great public importance. We have previously discharged
jurisdiction where the district court of appeal has not in fact passed upon the
question certified. * * * Second, there must be a district court “decision” to
review. See art. V, § 3(b)(4), Fla. Const. For instance, where a district court
is unable to reach a clear majority decision on an issue and elects to certify a
question without resolving the merits, we are without jurisdiction to answer
such a question under article V, section 3(b)(4) of the Florida Constitution.
See Boler v. State, 678 So.2d 319, 320 n. 2 (Fla.1996) (stating that if a
district court is evenly split on a legal issue and specifically withholds a
decision on the merits, there is no “decision” on which to base certified
conflict review under article V, section 3(b)(4)). Third, and most important

77
for this case, the question must be in fact “certified” by a majority decision
of the district court. For the same reasons that we are without jurisdiction
under article V, section 3(b)(4) if there is no majority decision on the merits,
we are equally without jurisdiction if there is no clear majority on the
decision to certify. Accordingly, we conclude that under article V, section
3(b)(4) of the Florida Constitution, it is required that a majority of those
judges participating in the case concur in the decision to certify.
[3] [4] In this case, only six of the twelve judges that participated in the
en banc decision concurred in certification. * * * Five judges, although
concurring in the majority decision, disagreed with the decision to certify.
Importantly, Judge Benton did not fully concur with the majority decision;
rather, Judge Benton only concurred in the judgment. This vote indicates
Judge Benton’s agreement only with the judgment of the majority; that is, its
decision to reverse the summary judgment and remand. See Home Dev. Co.
of St. Petersburg v. Bursani, 168 So. 2d 131, 134 (Fla.1964) (distinguishing
between the judgment, which is essentially the ultimate decision in the case,
and the opinion, which sets “forth the theory and reasoning upon which a
decision” is reached); see also Black’s Law Dictionary 858 (8th ed. 2004)
(defining judgment as “[a] court’s final determination of the rights and
obligations of the parties in a case”). In this sense, “concurring in the
judgment” is akin to “concurring in result only,” which “expresses
agreement with the ultimate decision but not the opinion.” Rowe v. Winn-
Dixie Stores, Inc., 714 So. 2d 1180, 1181 (Fla. 1st DCA 1998), disapproved
on other grounds by Owens v. Publix Supermarkets, Inc., 802 So. 2d 315
(Fla.2001); accord Harry Lee Anstead, Gerald Kogan, Thomas D. Hall &
Robert Craig Waters, The Operation and Jurisdiction of the Supreme Court
of Florida, 29 Nova L.Rev. 431, 460 (2005) (“A concurring in result only
opinion indicates agreement only with the decision, that is, the official
outcome and result reached, but a refusal to join in the majority’s opinion
and its reasoning.”).
[5] Conversely, certification is separate from the judgment of the court
and its reasoning for the judgment as expressed in its opinion. This is
evident by comparing the definition of judgment, i.e., “[a] court’s final
determination of the rights and obligations of the parties in a case,” with the
definition of a certified question, i.e., “[a] point of law on which a[n] ...
appellate court seeks guidance ” from a higher court by the procedure of
certification. Black’s Law Dictionary at 858, 241 (emphasis supplied).

78
Moreover, there are separate and specific rules that both permit a party to
file a motion seeking certification and authorize the district court to certify
the question. Compare Fla. R.App. P. 9.030(a)(2)(A)(v) (granting
discretionary jurisdiction to the Court if a district court decision certifies a
question), with Fla. R.App. P. 9.330(a) (authorizing a party to a file motion
for certification to the district court); cf. Floridians, 945 So. 2d at 562
(reversing summary judgment and remanding for a trial, and then certifying
two questions to the Court, specifically in accordance with article V, section
3(b)(4) of the Florida Constitution and Florida Rule of Appellate Procedure
9.030(a)(2)(A)(v)). We conclude that by “concurring in the judgment” and
failing to indicate his agreement with the decision to certify, Judge Benton’s
vote cannot be counted as agreeing with the certification. Cf. Hadden v.
State, 670 So. 2d 77, 83 (Fla. 1st DCA 1996) (Benton, J., concurs in result
and in certification) (indicating Judge Benton’s agreement with the result
and the decision to certify, but disagreement with the reasoning of the
majority opinion).
[6] Even assuming that we had jurisdiction based on either a certification
of a question of great public importance or express and direct conflict under
article V, section 3(b)(3) of the Florida Constitution, we would nevertheless
decline to exercise our jurisdiction. After having reviewed the opinions from
this Court that deal with post-election challenges based on various issues
regarding the election process, we have determined that it is preferable that
the facts of this case be developed regarding the allegations and
pervasiveness of the fraud before we articulate a rule of law regarding the
election cure doctrine in such circumstances. This is the position advocated
by the Secretary of State, Kurt S. Browning, and the Department of State
represented by its General Counsel and the Attorney General. As stated in its
answer brief:
The long-standing principle that an election cures irregularities in the
process and thereby promotes finality and administrative efficiency is a
weighty one; similarly, the ability of citizens to amend the state constitution
through the initiative process without fraud is extremely important. This
Court should avoid making rulings affecting the application of these
principles until the specific allegations of fraud are adjudicated. A fully-
developed record with a proven set of facts will allow this Court to carefully
consider and balance the competing legal principles; that cannot properly be
done based on speculative, unproven factual allegations.

79
We believe that the position of the Attorney General is a sound one and
conclude, as an alternative basis for discharging jurisdiction, that review in
this case is premature. * * * Accordingly, we hereby discharge
jurisdiction and dismiss this review proceeding.
It is so ordered.
LEWIS, C.J., and WELLS, ANSTEAD, QUINCE, CANTERO, and
BELL, JJ., concur.

Department of Law Enforcement v. House


678 So. 2d 1284 (Fla. 1996)
PER CURIAM.
We have for review an unpublished order of the First District Court of
Appeal in Florida Department of Law Enforcement v. House, No. 95-965
(Fla. 1st DCA Oct. 13, 1995), dismissing an appeal for lack of jurisdiction.
The court certified conflict with the opinion in Department of
Transportation v. Wallis, 659 So. 2d 429 (Fla. 5th DCA 1995). We have
jurisdiction. Art. V, § 3(b)(4), Fla. Const.
This Court recently disapproved Wallis in Department of Education v.
Roe, 679 So. 2d 756 (Fla. 1996). Accordingly, we approve the dismissal of
the appeal.
It is so ordered.
KOGAN, C.J., and OVERTON, SHAW, GRIMES, HARDING, WELLS
and ANSTEAD, JJ., concur.

State v. Vickery
961 So. 2d 309 (Fla. 2007)
CANTERO, J.
We review three cases in which the Fourth and Fifth District Courts of
Appeal acknowledged (but did not certify) conflict with the First District
Court of Appeal. These are: Charles v. State, 890 So. 2d 542 (Fla. 4th DCA
2005), James v. State, 881 So. 2d 85 (Fla. 5th DCA 2004), and Vickery v.
State, 869 So. 2d 623 (Fla. 5th DCA 2004). The issue is whether a claim that
alleges ineffective assistance of counsel for failure to request an instruction
on a lesser-included offense may be summarily denied. See Sanders v. State,
847 So. 2d 504 (Fla. 1st DCA 2003) (en banc), approved, 946 So. 2d 953

80
(Fla.2006). In acknowledging conflict, the Fifth District in James and
Vickery cited Sanders, while the Fourth District in Charles cited Willis v.
State, 840 So. 2d 1135 (Fla. 4th DCA 2003) (on motion for rehearing and
motion for certification of conflict), quashed, 946 So. 2d 953 (Fla.2006), in
which it had earlier certified conflict with Sanders. When the Fourth and
Fifth Districts issued their respective decisions in Charles, James, and
Vickery, both Sanders and Willis were pending review in this Court. We
have jurisdiction and consolidate Charles, James, and Vickery for purposes
of this opinion. See art. V, § 3(b)(3), Fla. Const.; Jollie v. State, 405 So. 2d
418 (Fla.1981).
JURISDICTION
[1] Before deciding these cases, we address a jurisdictional issue. The
district courts in these cases only acknowledged, but did not certify, their
conflict with the First District. For purposes of our jurisdiction, this is an
important distinction. While it is a district court’s prerogative to
acknowledge rather than certify conflict, such an approach does not give us
jurisdiction under article V, section 3(b)(4) of the Florida Constitution
(establishing this Court’s discretionary jurisdiction to review “any decision
of a district court of appeal that ... is certified by it to be in direct conflict
with a decision of another district court of appeal”) (emphasis added).
As already informally recognized, “district court opinions accepted [for
review as certified conflict cases under article V, section 3(b)(4) of the
Florida Constitution] ... almost uniformly meet two requirements: they use
the word ‘certify’ or some variation of the root word ‘certif.-’ in connection
with the word ‘conflict;’ and, they indicate a decision from another district
court upon which the conflict is based.” Harry Lee Anstead, Gerald Kogan,
Thomas D. Hall, & Robert Craig Waters, The Operation and Jurisdiction of
the Supreme Court of Florida, 29 Nova L.Rev. 431, 529 (2005) (footnote
omitted). However, “all of the cases-with few exceptions-in which the
district court has merely ‘acknowledged’ conflict are treated as petitions for
[review based on] ‘express and direct’ conflict [under article V, section
(3)(b)(3) of the Florida Constitution], and some are accepted for review on
that basis.” Id. at 530 (footnote omitted).
[2] [3] [4] We thus hold that district court decisions that simply
acknowledge, discuss, cite, suggest, or in any other way recognize conflict
do not provide a proper basis for a party to seek this Court’s review under
our “certified conflict” jurisdiction. See art. V, § 3(b)(4), Fla. Const. To

81
support such review, conflict must be “certified.” Of course, this does not
mean that we lose all jurisdiction to review the case. As occurred with the
three cases here, jurisdiction may nevertheless exist under our “express and
direct conflict” jurisdiction, see art. V, § 3(b)(3), Fla. Const. (granting this
Court jurisdiction to review district court opinions that “expressly and
directly” conflict with the decision of another district court of appeal or with
a decision of the Florida Supreme Court), or on some other basis. The
difference is that a certification of conflict provides us with jurisdiction per
se. On the other hand, when a district court does not certify the conflict, our
jurisdiction to review the case depends on whether the decision actually
“expressly and directly” conflicts with the decision of another court. We
therefore advise district courts that when they intend to certify conflict under
article V, section 3(b)(4) of the Florida Constitution, they use the
constitutional term of art “certify.”
MERITS
[5] As stated earlier, the three decisions under review acknowledged
conflict with the First District’s decision in Sanders, 847 So. 2d at 504, on
whether a claim of ineffective assistance of counsel for failure to request an
instruction on a lesser-included offense may be summarily denied. We
ultimately approved Sanders, holding that “a claim alleging ineffective
assistance of counsel for failure to request an instruction on a lesser-included
offense may be summarily denied.” Sanders v. State, 946 So. 2d 953, 960
(Fla.2006). We accordingly issued orders directing the respondents in
Charles, James, and Vickery to show cause why we should not accept
jurisdiction, summarily quash the decisions being reviewed, and remand for
reconsideration in light of our decision in Sanders. No responses were filed
in Vickery or James, and the response in Charles ultimately “prays that this
Honorable Court would quash the decision being reviewed, and remand for
further proceeding[s].” All three respondents have thus constructively or
otherwise conceded that we should quash and remand for reconsideration in
light of Sanders. We now do so.
We grant the petitions for review in Charles, James, and Vickery. As to
each case, we quash the decision below and remand the case to the
respective district court for reconsideration in light of our decision in
Sanders.
It is so ordered.
WELLS, ANSTEAD, PARIENTE, and BELL, JJ., concur.

82
LEWIS, C.J., and QUINCE, JJ., concur in result only.

10. Certified Decisions of Trial Courts

Article V, Section 3(b)(5). The Florida Supreme Court has discretionary jurisdiction to
review a decision by a trial court pending on appeal to a district court of appeal if the
district court has certified that the decision is of great public importance and that there is
a need for immediate resolution. Jurisdiction is invoked by filing in the district court a
document styled as a Notice to Invoke Discretionary Jurisdiction.

11. Questions Certified by Federal Courts

Article V, Section 3(b)(6). The Florida Supreme Court has discretionary jurisdiction to
review a question of law certified by the United States Supreme Court or a United States
Court of Appeals. There are two prerequisites to the exercise of this jurisdiction: (1) the
issue must be one for which there is no controlling precedent of the Florida Supreme
Court; and (2) the issue must be determinative of the cause.

12. Prohibition

Article V, Section 3(b)(7). The Florida Supreme Court has jurisdiction to issue writs of
prohibition to lower courts. This does not include other kinds of tribunals, such as
administrative agencies. See Chiles v. Public Employees Relations Com’n, 630 So. 2d
1093 (Fla. 1994).

13. Mandamus

Article V, Section 3(b)(8). The Florida Supreme Court has jurisdiction to issue writs of
mandamus to state officers and state agencies.

14. Quo Warranto

Article V, Section 3(b)(8). The Florida Supreme Court has jurisdiction to issue writs of
quo warranto to state officers and state agencies.

83
15. Habeas Corpus

Article V, Section 3(b)(9). Jurisdiction to issue writs of habeas corpus exists at three
levels of the Florida court system. It is unlikely the Florida Supreme Court will issue a
writ of habeas corpus if the issue is one that could be resolved in a lower court. See
Harvard v. Singletary, 733 So. 2d 1020 (Fla. 1999).

16. All Writs Jurisdiction

Article V, Section 3(b)(7). The Florida Supreme Court has jurisdiction to issue all writs
necessary to the complete exercise of its jurisdiction. Writs issued under this section are
sometimes called constitutional writs or stay writs. In most cases, they are used to
preserve the status quo until the Florida Supreme Court can exercise jurisdiction under
another constitutional basis.

Chiles v. Public Employees Relations Commission


630 So. 2d 1093 (Fla. 1994)
McDONALD, Judge.
The State of Florida petitions for a writ of prohibition directed to the
Public Employees Relations Commission (PERC). We find that this Court
does not have jurisdiction to issue the writ, and, therefore, we deny the
petition.
On March 23, 1993, the State Employees Attorneys Guild (SEAG) filed a
petition with PERC, pursuant to section 447.307(2), Florida Statutes (1991),
seeking certification of a bargaining unit composed of attorneys who are
employed by the State of Florida. PERC entered an order finding reasonable
cause to believe the petition sufficient and ordered an evidentiary hearing on
questions concerning representation and unit determination. On April 12,
1993, the State filed a response to the petition, contending that the proposed
bargaining unit was an unconstitutional attempt to regulate and alter the
practice of law, in derogation of the Supreme Court’’s exclusive jurisdiction
under article V, section 15 of the Florida Constitution. The State also
requested that PERC issue a stay of the administrative proceeding pending
the State’s decision to file a petition for writ of prohibition with this Court.
PERC denied the request for stay, but this Court granted the stay. The State

84
now seeks entry of an order prohibiting PERC from proceeding further with
certification of a bargaining unit for state employed attorneys.
[1] [2] The writ of prohibition is an extraordinary writ that may be
granted only when a lower court is without jurisdiction or attempts to act in
excess of jurisdiction. English v. McCrary, 348 So. 2d 293 (Fla.1977).
Article V, section 3(b)(7) of the Florida Constitution provides that the
Supreme Court “may issue writs of prohibition to courts and all writs
necessary to the complete exercise of its jurisdiction.” Because the plain
language of this provision specifically limits the issuance of writs of
prohibition to courts, we do not have jurisdiction to issue a writ of
prohibition to a state agency like PERC.
[3] This Court may also exercise jurisdiction via the “all writs” provision
of article V, section 3(b)(7) of the Florida Constitution. In Florida Senate v.
Graham, 412 So. 2d 360 (Fla.1982), we held that this Court may issue all
writs necessary to aid the Court in exercising its “ultimate jurisdiction.” Id.
at 361. Article V, section 15 of the Florida Constitution vests this Court with
the “exclusive jurisdiction to regulate the admission of persons to the
practice of law and the discipline of persons admitted.” Because the
regulation of attorneys falls within the Court’s ultimate power of review, the
all writs clause could arguably be invoked as a basis for this Court’s
jurisdiction. The pivotal issue in the instant case, however, is whether
PERC’s statutory certification process infringes on this Court’s jurisdiction
over attorneys. We find that collective bargaining by state employed
attorneys does not encroach upon this Court’s jurisdiction over the
admission of attorneys to the practice of law or the discipline of attorneys.
Under section 447.307, Florida Statutes (1991), PERC is required to
process the representation-certification petition of SEAG. Because PERC
has not yet held a hearing on SEAGs petition, as required by the statute, the
State’s petition in this proceeding is premature. Under ordinary
circumstances, once PERC has issued a final order on the matter and a
notice of appeal has been filed, the district court of appeal shall have
jurisdiction and may grant such relief as it deems just and proper. § 447.504,
Fla.Stat. (1991). Of course, it is always possible that PERC’s subsequent
rulings in this proceeding could interfere with this Court’s regulation of the
practice of law, thereby triggering our jurisdiction, but it is clear that nothing
like this has yet occurred. Accordingly, the State’s petition is denied.
It is so ordered.

85
BARKETT, C.J., and OVERTON, SHAW, GRIMES, KOGAN and
HARDING, JJ., concur.

B. The District Courts of Appeal

The Florida Consitution

Article V, Section 4(b), Florida Constitution


(b) JURISDICTION. –
(1) District courts of appeal shall have jurisdiction to hear appeals, that
may be taken as a matter of right, from final judgments or orders of trial
courts, including those entered on review of administrative action, not
directly appealable to the supreme court or a circuit court. They may
review interlocutory orders in such cases to the extent provided by rules
adopted by the supreme court.
(2) District courts of appeal shall have the power of direct review of
administrative action, as prescribed by general law.
(3) A district court of appeal or any judge thereof may issue writs of
habeas corpus returnable before the court or any judge thereof or before
any circuit judge within the territorial jurisdiction of the court. A district
court of appeal may issue writs of mandamus, certiorari, prohibition, quo
warranto, and other writs necessary to the complete exercise of its
jurisdiction. To the extent necessary to dispose of all issues in a cause
properly before it, a district court of appeal may exercise any of the
appellate jurisdiction of the circuit courts.

Florida Rules of Appellate Procedure

Rule 9.030. Jurisdiction of Courts


***
(b) Jurisdiction of District Courts of Appeal.
(1) Appeal Jurisdiction.District courts of appeal shall review, by appeal
(A) final orders of trial courts,12 not directly reviewable by the
supreme court or a circuit court, including county court final orders
declaring invalid a state statute or provision of the state constitution;

86
(B) non-final orders of circuit courts as prescribed by rule 9.130;
(C) administrative action if provided by general law.
(2) Certiorari Jurisdiction. The certiorari jurisdiction of district courts of
appeal may be sought to review
(A) non-final orders of lower tribunals other than as prescribed by
rule 9.130;
(B) final orders of circuit courts acting in their review capacity.
(3) Original Jurisdiction. District courts of appeal may issue writs of
mandamus, prohibition, quo warranto, and common law certiorari, and all
writs necessary to the complete exercise of the courts' jurisdiction; or any
judge thereof may issue writs of habeas corpus returnable before the court
or any judge thereof, or before any circuit judge within the territorial
jurisdiction of the court.
(4) Discretionary Review. District courts of appeal, in their discretion, may
review by appeal
(A) final orders of the county court, otherwise appealable to the
circuit court under these rules, that the county court has certified to be of
great public importance;
(B) non-final orders, otherwise appealable to the circuit court under
rule 9.140(c), that the county court has certified to be of great public
importance.

The district courts have jurisdiction to hear appeals from final orders of trial courts, that
are not directly appealable to the supreme court or a circuit court, and they have the
power to review interlocutory orders to the extent provided by rules adopted by the
supreme court. See Art. V. § 4(b)(1) Fla. Const.
The district courts have jurisdiction to hear appeals from orders of the county court
declaring state statutes unconstitutional, because such orders are not appealable to the
circuit court. See Section 26.012(1), Florida Statutes.

87
1. Final Orders of Trial Courts

Fieselman v. State
566 So. 2d 768 (Fla. 1990)
PER CURIAM.
We have for review two cases: Fieselman v. State, 537 So. 2d 603 (Fla.
3d DCA 1988), based upon conflict with Baker v. State, 518 So. 2d 457 (Fla.
5th DCA 1988); and Williams v. State, 540 So. 2d 229 (Fla. 5th DCA 1989),
based upon conflict with Fieselman and Mitchell v. State, 538 So. 2d 106
(Fla. 4th DCA 1989). We have jurisdiction over these consolidated cases.
Art. V, § 3(b)(3), Fla. Const. We approve the opinion of the district court
below in Fieselman and quash that in Williams.
WILLIAMS
Williams was charged in county court with littering; the court dismissed
the charge, ruling the statute unconstitutional. The state appealed to the
circuit court, which reversed and remanded for reinstatement of the charges.
Williams petitioned for certiorari before the district court, which declined to
exercise review. The court based its decision upon its prior ruling in Baker,
wherein it said that “an order denying a motion to dismiss or a circuit court
opinion reversing an order granting a motion to dismiss, both ... amount to
the same thing. An adequate remedy by appeal, if conviction ensues, is
available.” Baker, 518 So. 2d at 458.
[1] We granted review of Williams based upon conflict. Upon review,
however, we have determined that this case was improperly before the
circuit court. The circuit court lacked jurisdiction to review the county court
order finding the anti-litter law unconstitutional.FN1 Only the district courts
can hear such appeals from county courts. Art. V § 4(b)(1), Fla. Const.
Accordingly, we declare the circuit court decision in Williams void; we
quash the district court decision and direct that the appeal of the county court
order be transferred to the district court.
FN1. Circuit courts lack jurisdiction to hear appeals from county court
decisions declaring statutes invalid. See § 26.012(1), Fla.Stat. (1987).
FIESELMAN
Fieselman was charged in county court with being in actual physical
control of a vehicle while under the influence of alcoholic beverages. The
county court dismissed the charge, and the state appealed to the circuit court,

88
which reversed and remanded. Fieselman petitioned for writ of certiorari
before the district court, which held that the decision was reviewable. The
case was certified to us as being in conflict with Baker, wherein the court
held that a district court should not review on certiorari a circuit court
opinion reversing an order granting a motion to dismiss.
[2] This case presents the following issue: Is the decision of a circuit
court reversing a county court’s order granting a motion to dismiss
reviewable on certiorari before a district court? We conclude that it is.
[3] [4] No certiorari review may ordinarily be had of a trial court order
denying a motion to dismiss because the party has available to it an eventual
plenary appeal of the final judgment. See Martin-Johnson, Inc. v. Savage,
509 So. 2d 1097 (Fla.1987). This must be distinguished, however, from the
situation where a county court grants a motion to dismiss and a circuit court,
sitting in its appellate capacity, reverses. The decision of a trial court
denying dismissal affects only the immediate parties and they can seek
eventual redress through plenary appeal of the final judgment. When a
circuit court reverses a county court order of dismissal, on the other hand,
the circuit court is acting in its appellate capacity and its decision is binding
on all county courts within the circuit. The decision thus affects parties
outside the original litigation.
We approve that portion of the decision in Fieselman that concerns
certiorari review, and disapprove Baker.
It is so ordered.
SHAW, C.J., and McDONALD, EHRLICH, BARKETT, GRIMES and
KOGAN, JJ., concur.
OVERTON, J., concurs in result only.

2. Nonfinal Orders of Circuit Courts

Florida Rules of Appellate Procedure

Rule 9.130. Proceedings to Review Non-Final Orders and Specified


Final Orders
(a) Applicability.
(1) This rule applies to appeals to the district courts of appeal of the non-

89
final orders authorized herein and to appeals to the circuit court of non-
final orders when provided by general law. Review of other non-final
orders in such courts and non-final administrative action shall be by the
method prescribed by rule 9.100.
(2) Appeals of non-final orders in criminal cases shall be as prescribed by
rule 9.140.
(3) Appeals to the district courts of appeal of non-final orders are limited to
those that
(A) concern venue;
(B) grant, continue, modify, deny, or dissolve injunctions, or refuse
to modify or dissolve injunctions;
(C) determine
(i) the jurisdiction of the person;
(ii) he right to immediate possession of property, including
but not limited to orders that grant, modify, dissolve or refuse to
grant, modify, or dissolve writs of replevin, garnishment, or
attachment;
(iii) the right to immediate monetary relief or child custody in
family law matters;
(iv) the entitlement of a party to arbitration, or to an appraisal
under an insurance policy;
(v) that, as a matter of law, a party is not entitled to workers'
compensation immunity;
(vi) that a class should be certified;
(vii) that, as a matter of law, a party is not entitled to absolute
or qualified immunity in a civil rights claim arising under federal law;
or
(viii) that a governmental entity has taken action that has
inordinately burdened real property within the meaning of section
70.001(6)(a), Florida Statutes.
(D) grant or deny the appointment of a receiver, and terminate or
refuse to terminate a receivership.
(4) Non-final orders entered after final order on motions that suspend
rendition are not reviewable; provided that orders granting motions for new
trial in jury and non-jury cases are reviewable by the method prescribed in
rule 9.110. Other non-final orders entered after final order on authorized
motions are reviewable by the method prescribed by this rule.

90
(5) Orders entered on an authorized and timely motion for relief from
judgment are reviewable by the method prescribed by this rule. Motions
for rehearing directed to these orders will not toll the time for filing a notice
of appeal.
(6) Orders that deny motions to certify a class may be reviewed by the
method prescribed by this rule.
(b) Commencement.The jurisdiction to seek review of orders described in
subdivisions (a)(3)–(a)(6) shall be invoked by filing 2 copies of a notice,
accompanied by the filing fees prescribed by law, with the clerk of the
lower tribunal within 30 days of rendition of the order to be reviewed.
(c) Notice.The notice, designated as a notice of appeal of non-final order,
shall be substantially in the form prescribed by rule 9.900(c). Except in
criminal cases, a conformed copy of the order or orders designated in the
notice of appeal shall be attached to the notice.
(d) Record.A record shall not be transmitted to the court unless ordered.
(e) Briefs.Appellant's initial brief, accompanied by an appendix as
prescribed by rule 9.220, shall be served within 15 days of filing the notice.
Additional briefs shall be served as prescribed by rule 9.210.
(f) Stay of Proceedings.In the absence of a stay, during the pendency of
a review of a non-final order, the lower tribunal may proceed with all
matters, including trial or final hearing; provided that the lower tribunal
may not render a final order disposing of the cause pending such review.
(g) Review on Full Appeal.This rule shall not preclude initial review of a
non-final order on appeal from the final order in the cause.
(h) Scope of Review. Multiple non-final orders that are listed in rule
9.130(a)(3) may be reviewed by a single notice if the notice is timely filed
as to each such order.

The right to appeal a nonfinal order of the circuit court to a district court of appeal
depends on the existence of a court rule authorizing the appeal. See Blore v. Fierro, 636
So. 2d 1329 (Fla. 1994).

3. Direct Review of Administrative Action

See Section 120.68(2), Florida Statutes.

91
4. Certiorari

A nonfinal order that is not appealable under rule 9.130 is reviewable in the district court
only by certiorari. See Fla.R.App.P. 9.030(b)(2)(A).
A decision by a circuit court on appeal from the county court is reviewable by certiorari
to the district court of appeal.

5. Discretionary Review

District courts of appeal have discretionary jurisdiction to hear appeals from county court
orders that are certified to be of great public importance.

6. Extraordinary Writs

District courts of appeal have jurisdiction to issue writs of mandamus, prohibition, quo
warranto, certiorari, habeas corpus, and all writs necessary to the complete exercise of
their jurisdiction.

C. Appellate Jurisdiction of the Circuit Courts

The Florida Constitution

Article V, Section 5(b), Florida Constitution


(b) JURISDICTION. ---
The circuit courts shall have original jurisdiction not vested in the county
courts, and jurisdiction of appeals when provided by general law. They
shall have the power to issue writs of mandamus, quo warranto, certiorari,
prohibition, and habeas corpus, and all writs necessary or proper to the
complete exercise of their jurisdiction. Jurisdiction of the circuit courts
shall be uniform throughout the state. They shall have the power of direct
review of administrative action prescribed by general law.

92
Florida Rules of Appellate Procedure

Rule 9.030. Jurisdiction of Courts


***
(c) Jurisdiction of Circuit Courts.
(1) Appeal Jurisdiction.The circuit courts shall review, by appeal
(A) final orders of lower tribunals as provided by general law;12
(B) non-final orders of lower tribunals as provided by general law;
(C) administrative action if provided by general law.
(2) Certiorari Jurisdiction. The certiorari jurisdiction of circuit courts may be
sought to review non-final orders of lower tribunals other than as
prescribed by rule 9.130.
(3) Original Jurisdiction. Circuit courts may issue writs of mandamus,
prohibition, quo warranto, common law certiorari, and habeas corpus, and
all writs necessary to the complete exercise of the courts' jurisdiction.

1. Final Orders of Lower Tribunals

The jurisdiction of the circuit court to hear an appeal from a final order depends on the
existence of a general law. Section 26.012(1) Florida Statutes provides that the circuit
courts shall have jurisdiction to hear appeals from orders of the county courts except
appeals of county court orders or judgments declaring invalid a state statue, or a
provision of the Florida Constitution.

2. Nonfinal Orders of Lower Tribunals

Blore v. Fierro
636 So. 2d 1329 (Fla. 1994)
OVERTON, Justice.
James Blore seeks review of Blore v. Fierro, 618 So. 2d 762 (Fla. 3d
DCA 1993), in which the district court held that a breathalyzer test is a
“search” as contemplated by rule 9.140(c)(1)(B), Florida Rules of Appellate
Procedure, so that, pursuant to the rule, the State could appeal a trial court

93
order that suppressed the results of Blore’s breath test. The district court
acknowledged that the Second District Court of Appeal had come to a
contrary conclusion in State v. Gemignani, 545 So. 2d 929 (Fla. 2d DCA
1989), and State v. Townsend, 479 So. 2d 306 (Fla. 2d DCA 1985), and
certified conflict. We have jurisdiction. Art. V, § 3(b)(4), Fla. Const.
For the reasons expressed, we hold that a breathalyzer test is a “search”
within the meaning of rule 9.140(c)(1)(B) and approve the district court’s
conclusion that the State could appeal the trial court’s suppression order.
However, we also find that the State’s authority to appeal the county court
suppression order is governed not by rule 9.140, as stated by the district
court, but by section 924.071(1), Florida Statutes (1991). This latter finding
is based on the distinction between the allocation of jurisdiction for the
district courts of appeal contained in article V, section 4, Florida
Constitution, and the jurisdiction provided to the circuit courts in article V,
section 5.
The record reflects that the Florida Marine Patrol arrested James Blore
for operating a vessel while he was under the influence of alcohol. At the
time of the arrest, a breathalyzer test was used by a marine patrol officer to
measure Blore’s blood alcohol level. Although the officer complied with
most of the applicable Department of Health and Rehabilitative Services
rules governing breathalyzer testing, testimony revealed that the officer
failed to properly check the calibration of the breathalyzer apparatus prior to
its use on Blore.
In a pretrial evidentiary ruling, the county court judge found that the
marine patrol officer’s failure to perform the calibration check impaired the
scientific reliability of the breathalyzer test and granted Blore’s motion to
suppress the test results. The State then filed a notice of appeal to the circuit
court pursuant to rule 9.140(c)(1)(B), Florida Rules of Appellate Procedure.
This rule provides that “[t]he state may appeal an order ... suppressing before
trial confessions, admissions, or evidence obtained by search and seizure.”
Blore moved to dismiss the State’s appeal on the ground that a breathalyzer
test is not a search as contemplated by the rule. The circuit court, acting in
its appellate capacity, denied Blore’s motion to dismiss.
Blore then filed a petition for a writ of prohibition in the Third District
Court of Appeal to prevent the circuit court from acting in excess of its
jurisdiction. The district court denied the petition and held that, because the
United States Supreme Court had established that a breathalyzer test is a

94
“search” under the Fourth Amendment, see Skinner v. Railway Labor
Executives’ Association, 489 U.S. 602, 109 S.Ct. 1402, 103 L.Ed.2d 639
(1989), the State was entitled to appeal the suppression order under rule
9.140(c)(1)(B). Blore then petitioned this Court for review.
[1] In Skinner, the United States Supreme Court stated that “a
breathalyzer test, which generally requires the production of ... “deep lung”
breath for chemical analysis ... implicates similar concerns about bodily
integrity and, like the blood-alcohol test ... should also be deemed a search.”
489 U.S. at 616-17, 109 S.Ct. at 1413. Although the Second District Court
of Appeal had concluded otherwise in Townsend and Gemignani, we note
that one of these decisions predated Skinner and that the other decision was
released just shortly thereafter. In accordance with Skinner, we find that a
breathalyzer test is a search.
[2] Although the district court upheld the State’s appeal based on rule
9.140, we find that section 924.071(1), Florida Statutes (1991), rather than
Rule 9.140, Rules of Appellate Procedure, provides the authority for the
State’’s interlocutory appeal in this instance. The Florida Constitution
establishes a controlling distinction between the appellate jurisdiction of the
district courts of appeal and the appellate jurisdiction of the circuit court.
Article V, section 4(b)(1), of the Florida Constitution, which allocates
jurisdiction to the district courts of appeal, reads as follows:
District courts of appeal shall have jurisdiction to hear appeals, that may
be taken as a matter of right, from final judgments or orders of trial courts,
including those entered on review of administrative action, not directly
appealable to the supreme court or a circuit court. They may review
interlocutory orders in such cases to the extent provided by rules adopted by
the supreme court.
Art. V, § 4(b)(1) (emphasis added).
Article V, section 5(b), Florida Constitution, which sets forth the
jurisdiction of the circuit courts, reads as follows:
The circuit courts shall have original jurisdiction not vested in the county
courts, and jurisdiction of appeals when provided by general law. They shall
have the power to issue writs of mandamus, quo warranto, certiorari,
prohibition and habeas corpus, and all writs necessary or proper to the
complete exercise of their jurisdiction. Jurisdiction of the circuit court shall
be uniform throughout the state. They shall have the power of direct review
of administrative action prescribed by general law.

95
Art. V, § 5(b) (emphasis added).
[3] It is important to note that, while this Court is given exclusive
rulemaking authority over interlocutory appeals to the district courts of
appeal, the Constitution does not provide this Court with such authority for
appeals from the county court to the circuit court. The authority for appeals
to the circuit court is established solely by general law as enacted by the
legislature. The legislature has enacted section 924.071, which provides:
“The state may appeal from a pretrial order dismissing a search warrant or
suppressing evidence, however obtained.... The appeal must be taken before
the trial.” Clearly, this statute permits the State to appeal a county court
order that suppresses the results of a breathalyzer test to the circuit court.
[4] In so holding, we acknowledge that we have previously held that
sections 924.07 and 924.071, Florida Statutes, are invalid as applied to
interlocutory appeals from the circuit court to the district court of appeal. See
R.J.B. v. State, 408 So. 2d 1048 (Fla.1982); State v. Smith, 260 So. 2d 489
(Fla.1972). These decisions were correct based on article V, section 4(b)(1),
of the Florida Constitution; however, neither decision controls the appeal in
this case. In this instance, article V, section 5, of the Florida Constitution
gives the legislature the exclusive authority to provide for the manner of
appeals from the county court to the circuit court, and sections 924.07 and
924.071 are valid as far as these sections pertain to appeals to the circuit
court.
Accordingly, for the reasons expressed, we approve the result reached by
the district court in this case, and we disapprove the decisions of the Second
District Court of Appeal in Townsend and Gemignani.
It is so ordered.
GRIMES, C.J., and McDONALD, SHAW, KOGAN and HARDING, JJ.,
concur.
3. Direct Review of Administrative Action

The circuit courts have jurisdiction to hear appeals from administrative action when
provided by general law. The Florida Administrative Procedure Act provides that
appeals from final agency action shall be to the district courts of appeal. If the
administrative tribunal does not qualify as a state agency, review would be in the circuit
court. The circuit court often reviews orders by local administrative tribunals, for
example, local zoning boards.

96
4. Discretionary Review of Nonfinal Orders

The circuit courts have discretionary jurisdiction to review by certiorari unappealable


orders of the county courts.

5. Extraordinary Writs

The circuit courts have jurisdiction to issue writs of prohibition, mandamus, quo
warranto, habeas corpus, certiorari, and all writs necessary to the complete exercise of
their jurisdiction.

IV. Preservation of Error

A. Contemporaneous Objection Rule

Castor v. State
365 So. 2d 701 (Fla. 1978)
ENGLAND, Chief Justice.
The decision of the First District Court of Appeal in this case, reported at
351 So. 2d 375, is properly brought to us to decide whether a
contemporaneous objection is essential for appellate review when a trial
judge re-instructs in response to a jury’s request in a homicide case but omits
a portion of the homicide instructions which we have declared essential for
the jury’s complete understanding. The district court essentially held that
review is foreclosed when defense counsel has failed to “clearly request” re-
instruction on homicide exclusions, despite our announcement in Hedges v.
State, 172 So. 2d 824 (Fla.1965), that a failure to do so is error.
In Hedges we said:
“(I)n order to supply a complete definition of manslaughter as a degree of
unlawful homicide it is necessary to include also a definition of the
exclusions. . . .
It is proper for a judge to limit the repetition to the charges requested.
(Citation omitted.) However, the repeated charges should be complete on the
subject involved. The giving of a partial instruction fails to inform the jury
fully and often leads to undue emphasis on the part given as against the part
omitted. . . .

97
In the instant case when the judge repeated his charges on degrees of
homicide he should have included the requested definitions of justifiable and
excusable homicide. Failure to do so erroneously left with the jury an
incomplete, and, potentially misleading instruction. [FN2]
FN2. 172 So. 2d at 826. In Hedges, the defendant was tried for first
degree murder, and the judge originally gave instructions on all degrees of
unlawful homicide and on justifiable and excusable homicide. After some
deliberation, the jury requested that the instructions be read again; the trial
judge then repeated his charges to the jury on unlawful homicide offenses,
but refused the defendant’s request that the definitions of justifiable and
excusable homicide also be re-read. We reversed that decision.
Charles Castor was charged by information with second degree murder.
At the close of the evidence, the trial court properly instructed the jury on
second and third degree murder, on manslaughter, on lesser included
offenses,[FN3] on justifiable and excusable homicide, and on self-defense.
After deliberating for approximately thirty minutes, the jury indicated that it
desired re-instruction. Before the jury was brought back into the courtroom,
this colloquy was held between the court and counsel:
FN3. Instructions were given on aggravated battery, aggravated assault,
battery, and assault.
“The Court: Counsel, do you think that we should that I should read all of
the definitions to them or would both of you stipulate and agree that only
those that they request would be read to them are necessary to read?
(Castor’s attorney): I think we should read them all.
The Court: All right, bring them in.
(Castor’s attorney): You’re talking about all lesser included offenses, not
the entire instructions.
(Prosecuting attorney): We’re not sure exactly what they want yet.
The jury then assembled in the courtroom and asked “to have the
definition of the various charges read to us again, second degree, third
degree, manslaughter.”
The trial judge re-instructed on second and third degree murder, on
manslaughter, and on the lesser included offenses, aggravated battery,
aggravated assault, battery and assault. He did not, however, re-instruct on
justifiable and excusable homicide. Following further deliberations the jury
returned a verdict finding Castor guilty of murder in the third degree.

98
[1] Castor appealed to the district court the trial court’s failure to re-
instruct on justifiable and excusable homicide. The district court found the
point not properly preserved and affirmed Castor’s conviction.[FN4] * * *
Castor asserts here that his trial counsel did all that could reasonably be
required to request the omitted instructions, and that he was barred from
doing more by the trial court’s failure to follow our requirement, found in
the rules of criminal procedure,[FN5] * * * that the court submit to counsel
all proposed responses to a jury’s questions.
[2] [3] As a general matter, a reviewing court will not consider points
raised for the first time on appeal. Dorminey v. State, 314 So. 2d 134
(Fla.1975). Where the alleged error is giving or failing to give a particular
jury instruction, we have invariably required the assertion of a timely
objection. Febre v. State, 158 Fla. 853, 30 So.2d 367 (1947); see Williams v.
State, 285 So.2d 13 (Fla.1973). The requirement of a contemporaneous
objection is based on practical necessity and basic fairness in the operation
of a judicial system. It places the trial judge on notice that error may have
been committed, and provides him an opportunity to correct it at an early
stage of the proceedings. Delay and an unnecessary use of the appellate
process result from a failure to cure early that which must be cured
eventually.
[4] To meet the objectives of any contemporaneous objection rule, an
objection must be sufficiently specific both to apprise the trial judge of the
putative error and to preserve the issue for intelligent review on appeal. See
Rivers v. State, 307 So. 2d 826 (Fla. 1st DCA), Cert. denied, 316 So. 2d 285
(Fla.1975); York v. State, 232 So. 2d 767 (Fla. 4th DCA 1969).
These considerations compel us to conclude that in re-instruction cases
like the present one, objections must be explicit. They must direct the
attention of the trial judge to the purported error in a way which will allow
him to respond in a timely fashion.
[5] [6] The record in the case before us highlights the problems posed by
counsel’s inexactitude. Trial counsel for Castor stated to the judge that the
jury should be recharged on all legal definitions the jury may want and any
lesser included charges, but he neither signaled the judge before nor after re-
instruction that, for completeness, Hedges required that the instructions on
justifiable and excusable homicide should also be restated. Nor did trial
counsel object, before or after re-instruction, to the trial court’s failure to
follow our rule regarding the procedure for submitting to counsel all

99
responses to a jury’s questions. His failure to do either not only prevented
the judge from correcting an inadvertent error, but it produced the delay and
systemic cost which result from invoking both levels of the state’s appellate
structure for the application of a legal principle which was known and
unambiguous at the time of trial. Except in the rare cases of fundamental
error, moreover, appellate counsel must be bound by the acts of trial
counsel.[FN6]
FN6. We recognized and reaffirmed this principle most recently in Clark
v. State, 363 So. 2d 331 (Fla.1978), in which we announced the requirement
of a contemporaneous objection for comments on a defendant’s exercise of
his right to remain silent.
[7] [8] As a final point, Castor urges that, in reliance on Bagley v. State,
119 So. 2d 400 (Fla. 1st DCA 1960), the trial court’s failure to provide the
jury with the complete re-instruction required by Hedges is indeed
fundamental error.[FN7] Bagley, however, involved the court’s initial
instructions to the jury, not re-instructions in response to the jury’s specific
question. The difference is crucial if the doctrine of fundamental error is to
remain a limited exception to the requirement that a trial judge must be
given an opportunity to correct his own errors. Brown v. State, 124 So. 2d
481 (Fla.1960); Jackson v. State, 307 So. 2d 232 (Fla. 4th DCA 1975). And
see Henry v. State, 359 So. 2d 864 (Fla.1978), pointing out (as was
recognized in Hedges ) the significance of the differences between original
instructions to a jury and subsequent re-instruction.
FN7. For an error to be so fundamental that it may be urged on appeal
though not properly preserved below, the asserted error must amount to a
denial of due process. State v. Smith, 240 So. 2d 807 (Fla.1970).
For the case before us, we hold that where a jury has once been fully
instructed on the applicable law of the case and later requests additional
instructions, an appellate court will not review an alleged re-instruction error
unless a timely and explicit objection is made to the trial judge. [FN8]
Inasmuch as trial counsel for Castor did not present the trial court with an
opportunity to cure a legal but non-fundamental error,[FN9] we affirm the
decision of the district court declining to consider the error on appeal, and
discharge the writ.
FN8. Recently in Henry v. State, 359 So. 2d 864 (Fla.1978), Justice
Sundberg traced the development of jury re-instruction principles and the
doctrine, articulated in Hedges, that limited re-instructions are permissible if

100
complete. He concluded, however, and the Court agreed, that it is not an
abuse of discretion for a trial judge to limit re-charge to a direct response to
the jury’s specific request. The need for a contemporaneous objection was
not an issue in the Henry case.
FN9. There is no assertion by Castor that, except for the alleged re-
instruction error, he did not otherwise receive a fair trial.
It is so ordered.
BOYD, OVERTON, HATCHETT and ALDERMAN, JJ., concur.

The issue presented to the trial court must be the same as the issue presented on
appeal. See W.R. Grace & Co. v. Dougherty, 636 So. 2d 746 (Fla. 2d DCA 1994)
(the appellant argued that a deposition was used improperly because it was not
shown that the witness was unavailable but the appellate court refused to consider
the issue because that was not the objection in the trial court).

B. Motions in Limine

The Florida Evidence Codes provides that “[i]f the [trial] court has made a definitive
ruling on the record admitting or excluding evidence, either at or before trial, a party need
not renew an objection or offer of proof to preserve a claim of error for appeal.” See §
90.104(1) Fla. Stat.

USAA Casualty Insurance Company v. Allen


17 So. 3d 1270 (Fla. 4th DCA 2009)
WARNER, J.
We affirm the final judgment rendered after a jury verdict in favor of the
plaintiff in an automobile negligence case. Two of the issues raised by
appellant were not preserved for appeal. As to the third, the trial court did
not abuse its discretion in admitting evidence.
[1][2] Appellant failed to preserve its objection to the trial court's denial
of appellant's use of a peremptory challenge against an African-American
juror under the procedure required by Melbourne v. State, 679 So. 2d 759,
765 (Fla.1996). In order to preserve the issue of whether the trial court's
ruling on a peremptory challenge constitutes reversible error, the appellant
must accept the juror, or panel, subject to its prior objection and/or renew the

101
objection before the jury is sworn. See id.; Joiner v. State, 618 So. 2d 174,
176 (Fla.1993); Glinton v. State, 956 So. 2d 497, 500 (Fla. 4th DCA 2007).
Because the appellant accepted the jury without renewing its objection to the
challenged juror, the issue was not preserved. We reject its contention that a
challenge would have been futile. Without restating the objection to the trial
court, the court cannot know that the party still maintains the previously
voiced objection. The objection was also not so close to the end of jury
selection such that it could be considered preserved without renewing the
objection, as in Gootee v. Clevinger, 778 So. 2d 1005, 1009 (Fla. 5th DCA
2000).
[3] Appellant's claim that the court erred in denying its motion in limine
to prevent the mention of surveillance evidence was also not properly
preserved. Section 90.104(1), Florida Statutes, provides: “If the court has
made a definitive ruling on the record admitting or excluding evidence,
either at or before trial, a party need not renew an objection or offer of proof
to preserve a claim of error for *1272 appeal.” See also Tillman v. State, 964
So. 2d 785, 787-88 (Fla. 4th DCA 2007) (holding that the trial court's
granting the state's pre-trial motion in limine constituted “a definitive ruling
on the record admitting evidence, meaning that Tillman need not have
renewed his objection to preserve his claim of error for appeal”). In this
case, the court never issued a definitive ruling on the motion, noting twice
that it had deferred its ruling. It was thus incumbent on appellant to object
when the appellees sought to introduce the evidence, which it did not do.
Therefore, appellant failed to preserve the issue for appeal.
[4] As to the admission of post-crash vehicle photographs, the trial court
did not abuse its discretion.
Affirmed.

Ocwen Financial Corp. v. Kidder


950 So. 2d 480 (Fla. 4th DCA 2007)
KLEIN, J.
Ocwen appeals a jury verdict awarding two of its former employees
approximately $1 million dollars each in compensatory damages for
violating the Florida Civil Rights Act, the Florida Whistleblower Act, and
for negligent supervision and retention of an employee. We affirm on all
issues and address two, whether the “send a message” closing argument was

102
improper, and whether a new trial argument was preserved where there was
an objection, but no motion for mistrial.
The factual basis for the main claim was that a male employee had been
peeping through the ceiling of the women’s restroom, had made revealing
videotapes, had shared the information with other employees, and that the
employer had permitted the workplace to be turned into a hostile
environment of sexual harassment. During closing argument, the plaintiffs
suggested to the jury that it should send a message “loud and clear from this
courtroom that you are not going to permit these corporations to treat these
people this way.” The trial court overruled objections to the comments, and
plaintiff’s counsel continued to pursue that theme. The cases relied on by
Ocwen for its argument that this was error include Kloster Cruise Ltd. v.
Grubbs, 762 So. 2d 552, 554-55 (Fla. 3d DCA 2000) (granting a new trial
after counsel encouraged the jury to send a message to the defendant by its
verdict), and Murphy v. Murphy, 622 So. 2d 99, 102 (Fla. 2d DCA 1993)
(closing argument encouraging the jury to “send a message” is
“unquestionably improper and highly prejudicial”).
Plaintiffs respond by explaining that the cases relied on by Ocwen did not
include punitive damage claims, and in the present case, although the jury
did not award punitive damages, claims for punitive damages were
submitted to the jury. In its reply brief Ocwen argues that the “send a
message” is never proper, and relies primarily on language in this court’s
opinion in Airport Rent-A-Car, Inc. v. Lewis, 701 So. 2d 893 (Fla. 4th DCA
1997), in which plaintiff’s counsel stated:
Tomorrow you get to go home and tell everyone about this case that you
sat on and I want you to tell them what you get in Broward County if a taxi
driver runs a red light and injures somebody ... I wish you could punish
them, but you can’t.
This court reversed, explaining:
We hold that this comment constituted an impermissible conscience of
the community argument. A conscience of the community argument
“extends to all impassioned and prejudicial pleas intended to evoke a sense
of community law through common duty and expectation.” Blue Grass
Shows, Inc. v. Collins, 614 So. 2d 626, 627 (Fla. 1st DCA), rev. denied, 624
So. 2d 264 (Fla.1993). Generally where an argument is not otherwise
inflammatory or egregious, an isolated statement is only harmful if coupled
with an argument for punitive damages. See Florida Crushed Stone Co. v.

103
Johnson, 546 So. 2d 1102 (Fla. 5th DCA 1989) ... Counsel’s caveat to the
jury, “I wish you could punish [appellant], but you can’t,” did not mitigate
the prejudicial impact of his statement. Rather, we conclude that counsel’s
statement planted the seed to motivate the jury to include a punitive aspect in
the damage award. (emphasis added.)
[1] Ocwen reads the emphasized statement as prohibiting “conscience of
community” or “send a message” arguments in punitive damage cases.
Lewis, however, did not involve a claim for punitive damages. The point we
were making in Lewis is that such an argument could motivate a jury to be
punitive. Ocwen’s argument that this type of argument cannot be used in a
punitive damage case ignores that “[p]unishment and deterrence are the
policies underlying punitive damages.” W.R. Grace & Co. v. Waters, 638
So. 2d 502, 504 (Fla.1994). Ocwen cites no punitive damage cases which
prohibit this type of argument.
[2] The second issue relates to a motion in limine by Ocwen to preclude
any reference to the fact that the peeping employee had been arrested, which
the court granted. When an Ocwen manager was testifying as an adverse
witness during plaintiffs’ case, however, he blurted out that the employee
had been arrested. Ocwen objected, asked the court for a curative
instruction, and the court gave one. The arrest came out several additional
times during the trial, and each time Ocwen’s objections were sustained and,
if requested, a curative instruction was given. Significantly, Ocwen did not
move for a mistrial, but now argues that this was so prejudicial as to require
a new trial.
The plaintiffs argue that, because Ocwen did not move for a mistrial, any
error involving the testimony about the employee’s arrest was not preserved.
They cite cases such as Grau v. Branham, 761 So. 2d 375 (Fla. 4th DCA
2000), and Weise v. Repa Film International, 683 So. 2d 1128 (Fla. 4th
DCA 1996), which hold that a motion for mistrial is necessary to preserve
for review an issue relating to an improper comment, if an objection to the
comment has been sustained in the trial court. See also Roundtree v. State,
362 So. 2d 1347 (Fla.1978).
[3] Ocwen argues that it is no longer necessary to move for a mistrial, if
an objection has been sustained, because of section 90.104(1)(b), Florida
Statutes (2003). The statute provides that “... if the court has made a
definitive ruling on the record ... excluding evidence, either at or before trial,
a party need not renew an objection ... to preserve a claim of error for

104
appeal.” If Ocwen had believed that the reference to the employee having
been arrested was so prejudicial as to be incurable by objection and curative
instruction, it had the option of moving for a mistrial, a far more drastic
remedy. The statute, however, only addresses objections, not motions for
mistrial. If the legislature had meant to relieve a party from having to move
for a mistrial it would have said so, and in the absence of such a statement
we cannot assume that the cases requiring a motion for mistrial have been
overruled.
We have considered the other issues raised by Ocwen and find that they
are without merit.
Affirmed.

C. Offers of Proof

Brantley v. Snapper Power Equipment


665 So. 2d 241 (Fla. 3d DCA 1995)
Before SCHWARTZ, C.J., and NESBITT and COPE, JJ.
PER CURIAM.
[1] This is an appeal from a defense verdict and judgment in a strict
products liability case based on the plaintiffs’ claim that the explosion of a
riding mower the plaintiff was using was caused by a defective weld in the
battery. The primary issue concerns the trial court’s ruling, in accordance
with its granting the defendants’ in limine motion to that effect, excluding
certain evidence of post-manufacture, pre-accident discovery of the
defective weld problem-and the possibility of consequent explosions-which
had been found in some of the mowers. The basis for the ruling was the
claimed effect of section 90.407, Florida Statutes (1991), which precludes
evidence of “subsequent remedial measures” in a negligence case. Section
90.407 was invoked because the defective weld problem had been
documented in notices, service bulletins, and correspondence concerning the
defect which set forth corrective measures to mitigate the explosion risk.
There is no doubt that the determination below was entirely, prejudicially
erroneous. As the Fourth District has recently and squarely held, section
90.407 does not apply to the pre-accident evidence involved in this case.
Keller Industries v. Volk, 657 So. 2d 1200, 20 Fla.L.Weekly D1460 (Fla. 4th
DCA June 21, 1995). Indeed, Keller Industries presents an a fortiori

105
situation because it involves a design rather than, as in this case, a
manufacturing defect. See Harley-Davidson Motor Co., Inc. v. Carpenter,
350 So. 2d 360, 361 (Fla. 2d DCA 1977), and cases cited; Millette v.
Radosta, 84 Ill.App.3d 5, 39 Ill.Dec. 232, 404 N.E.2d 823 (1980). We
conclude that plaintiffs’ position on this issue is properly preserved for
appellate review at least with respect to the limitations imposed on plaintiffs’
cross-examination of the defense expert witness,FN1 and that reversal of the
judgment is required.FN2
FN1. Plaintiffs attempted to cross-examine the defense expert regarding
the defective weld problem and Snapper’s response thereto. The lengthy
discussion at sidebar makes clear what the expert’s testimony would have
been, had the expert been allowed to answer.
FN2. Assuming arguendo that section 90.407 were applicable here, the
statute provides that “[e]vidence of measures taken after an event, which
measures if taken before it occurred would have made the event less likely to
occur, is not admissible to prove negligence or culpable conduct in
connection with the event.” The statute does not, however, preclude
admission of such evidence when “offered to prove a material issue other
than negligence or culpability; i.e., credibility of a witness, feasibility of
precautionary measures, and existence of a defect.” Charles W. Ehrhardt,
Florida Evidence § 407.1, at 215-16 (1995)(emphasis added; footnotes
omitted).
[2] Because the issue may recur on remand, we also address how to make
an offer of proof where the trial court excludes documents from evidence.
See § 90.104(1)(b), Fla.Stat. (1993). In this case the trial court’s order in
limine had the effect of excluding from evidence certain post-manufacture,
pre-accident notices, service bulletins, and correspondence on which the
plaintiffs desired to rely. Plaintiffs made no offer of proof of the excluded
documents at trial, nor were the excluded documents made part of the record
of the motion in limine hearing.
Plaintiffs contend that they were excused from any obligation to make an
offer of proof, because the trial court’s order in limine forbade any mention
of the documents at trial. Plaintiffs argue that they could not have proffered
the excluded documents at trial without violating the order in limine. We
disagree.
When the trial court excludes evidence, an offer of proof is necessary
(with some exceptions not applicable here) if the claimed evidentiary error is

106
to be preserved for appellate review. See § 90.104(1)(b), Fla.Stat. (1993).
This can be done without violating the order in limine by offering the
excluded documents at trial outside the presence of the jury. See Michael H.
Graham, Phillip A. Hubbart, Hugo L. Black, Jr. & Edward J. Imwinkelried,
Florida Evidentiary Foundations, at 20 (1991). “Excluded documents ...
should be marked for identification with a number and described fully in the
record. This makes a record of the excluded evidence available to an
appellate court so it can determine if error was committed in excluding the
evidence and also makes it available for post trial motions.” Henry P.
Trawick, Jr., Trawic’s Florida Practice & Procedure § 22-10, at 333 (1994)
(footnote omitted); see also § 90.104(1)(b), Fla.Stat. (1993); Fla.R.Civ.P.
1.450(b); Charles W. Ehrhardt, Florida Evidence §§ 104.3, 104.5 (1995). “In
considering the form in which the offer of proof is to be made, a distinction
should be made between testimony and documents or other tangible items.
Documents and other exhibits are usually marked for identification and
become part of the record on appeal even if excluded.” Charles A. Wright &
Kenneth W. Graham, Federal Practice & Procedure: Evidence § 5040
(1977). The courtroom clerk will maintain the excluded documents along
with any other exhibits offered at trial “but not admitted for one reason or
another.” Florida Evidentiary Foundations, supra, 1995 Cum.Supp., at 6 n.
9. Alternatively, if an adequate record of excluded evidence has been made
at the hearing on the motion in limine, it is not necessary to make an offer of
proof at trial. See Bender v. State, 472 So. 2d 1370, 1372-73 (Fla. 3d DCA
1985); FN3 see also Charles W. Ehrhardt, supra, § 104.5.
FN3. Bender contains dictum stating that once the trial court has entered
an order in limine excluding testimony at trial, the proponent of the evidence
“need not-and, indeed, should not-attempt to elicit such testimony to
preserve the error of the trial court’s earlier ruling.” 472 So. 2d at 1373.
Bender simply made the obvious point that the proponent of the excluded
evidence is not allowed to violate the order in limine by offering the
excluded evidence at trial in the presence of the jury. Bender does not
preclude a proffer of the excluded evidence at trial outside of the hearing of
the jury. Bender proceeds on the unspoken assumption that an adequate
record of the excluded evidence has been made during the motion in limine
hearing; where that has not been done, an offer of proof must be made at
trial.

107
It should also be noted that where the order in limine is one admitting
evidence (rather than an order excluding evidence), the objecting party must
renew the objection at trial, else the objection is waived. E.g., Correll v.
State, 523 So. 2d 562, 566 (Fla.), cert. denied, 488 U.S. 871, 109 S.Ct. 183,
102 L.Ed.2d 152 (1988).
The final judgment is reversed and the cause remanded for a new trial.

A formal presentation of the evidence is not required if the offer of proof would be
a useless ceremony; if the evidence is rejected as a class; or if the court indicates
that it is not necessary. See O’shea v. O’shea, 585 So. 2d 405 (Fla. 1st DCA 1991).
An alleged error is not preserved for review unless the complaining party has
obtained an adverse ruling in the trial court. See Hamilton v. R. L. Best Int’l, 996
So. 2d 233 (Fla. 1st DCA 2008). The appellate court reviews the correctness of
rulings and orders.

D. Fundamental Error

Sanford v. Rubin
237 So. 2d 134 (Fla. 1970)
ADKINS, Justice.
[ * * *]
Thereupon, the Third District Court of Appeal required the parties to file
additional briefs and participate in an additional oral argument limited to this
constitutional question. The Third District Court of Appeal then entered its
opinion and judgment (Rubin et al. v. Sanford and Rubin et al. v. Shapiro,
218 So. 2d 177), which is now before us for review by petition for writ of
certiorari. The District Court held that the failure to include the right to
attorneys’ fees in the title made Section 8 of the Act unconstitutional and
said:
***
[1] Numerous cases were cited by the District Court of Appeal sustaining
the statement that the constitutionality of a statute could be raised for the
first time on appeal as fundamental error. Upon examination of the

108
authorities cited by the District Court of Appeal and those cited by
respondents in their brief, it appears that the constitutional issue in each case
involved fundamental error. These cases do not hold that every
constitutional issue amounts to fundamental error cognizable initially upon
appeal. Constitutional issues, other than those constituting fundamental
error, are waived unless they are timely raised.
[2] [3] Fundamental error, which can be considered on appeal without
objection in the lower court, is error which goes to the foundation of the case
or goes to the merits of the cause of action. The Appellate Court should
exercise its discretion under the doctrine of fundamental error very
guardedly. See Holman v. State, 97 Okl.Cr. 279, 262 P.2d 456; State v.
Heisler, 58 N.M. 446, 272 P.2d 660; Goodhue v. Fuller, 193 S.W. 170, 172
(Tex.Civ.App.).
[4] The above-extended statement of facts in the cases Sub judice is
included to show that the allowance of the attorneys’ fee did not go to the
merits of the case or the foundation of the case. This extended litigation
involved the right of petitioners to retain their employment and receive
compensation for the time of their suspension by virtue of an erroneous
ruling of the Personnel Board.
In Nelson et al. v. Selden Cypress Door Co., 78 Fla. 203, 83 So. 286
(1919), the final judgment in a mechanic’s lien foreclosure suit included a 10
per cent attorneys’ fee. On petition for rehearing in the Appellate Court, it
was claimed that under the law attorneys’ fees were not recoverable in suits
to foreclose mechanic’s lien. This Court held that the question was waived,
saying:
‘While the judgment awards attorney’s fees to the amount of 10 per cent.
of the recovery, no objection was made to this in any of the proceedings
shown by the transcript. The illegality, if any, in the judgment, because of
the allowance of attorney’s fees was not questioned, and consequently the
point was waived. As the question of attorney’s fees was not presented for
determination when the case was heard on its merits, such question cannot
now be presented, on an application for rehearing, as a matter overlooked by
the court in affirming the judgment.’
St. Paul Fire & Marine Insurance Company v. Hodor, 200 So. 2d 205
(Fla.App.3rd, 1967) was an action by the insured against the insurer for
breach of contract to defend under an insurance policy. After trial, the trial
court entered a summary judgment and found the insurer liable for certain

109
attorneys’ fees. On appeal to the Third District Court of Appeal, the
appellant insurer argued that the statute pertaining to recovery of attorneys’
fees from the insurer in an action against it by its insured was
unconstitutional. The Court in its opinion said:
‘Appellant’s seventh point raises a constitutional issue as to the validity
of s 627.0127 Fla.Stat., F.S.A. Inasmuch as the trial court did not expressly
rule on the issue, and the point is not argued in the brief, we do not consider
that the judgment directly passed upon the validity of the statute. See Lipe v.
City of Miami, Fla.1962, 141 So. 2d 738, 743.’ (p. 207)
There being no fundamental error, the District Court of Appeal in the
case Sub judice improperly considered the constitutionality of Ch. 27735,
Laws of Florida, Special Acts 1951. This objection having been waived, this
Court at this late date will not pass upon the constitutionality of Sec. 8, Ch.
27735, Special Acts of 1951.
[ * * *]
The decision and opinion of the Third District Court of Appeal is
quashed and the cause is remanded to the Third District Court of Appeal
with directions to further remand said cause to the trial court for the purpose
of reinstating the judgment granting the attorneys’ fee.
It is so ordered.
ERVIN, C.J., and DREW, THORNAL and BOYD, JJ., concur.

E. Invited Error

Sheffield v. Superior Ins. Co.


800 So. 2d 197 (Fla. 2001)
PARIENTE, J.
We have for review Sheffield v. Superior Insurance Co., 741 So. 2d 533
(Fla. 1st DCA 1999), a decision from the First District Court of Appeal that
expressly and directly conflicts with the Third District Court of Appeal’s
decisions in Smith v. Hooligan’s Pub & Oyster Bar, Ltd., 753 So. 2d 596
(Fla. 3d DCA 2000), and Porter v. Vista Building Maintenance Services, 630
So.2d 205 (Fla. 3d DCA 1993). We have jurisdiction. See art. V, § 3(b)(3),
Fla. Const.
BACKGROUND

110
Petitioner Mary Ann Sheffield sustained soft tissue injuries as a
passenger in an automobile that was hit from the rear while the vehicle was
stopped at a traffic light. See Sheffield, 741 So. 2d at 534. After settling with
the tortfeasor, Sheffield sued her uninsured motorist insurance carrier,
respondent Superior Insurance Company (“Superior”). See id.
Prior to trial, Sheffield filed a motion in limine seeking to exclude
collateral source evidence regarding both insurance and other benefits
provided by her employer. The trial court denied Sheffield’s motion. The
parties then stipulated that Sheffield would have a standing objection to the
introduction of collateral source evidence and would not have to
contemporaneously or spontaneously object during trial in order to preserve
the objection for appeal. Thereafter, Sheffield introduced collateral source
evidence in her case-in-chief, including the fact that she had group medical
insurance to defray some of the costs of treatment.FN1
FN1. Superior also agreed on the record that Sheffield had not waived
her objection by raising the collateral source issue during voir dire and
opening statement. See Sheffield, 741 So. 2d at 537 n. 6.
Although the jury returned a verdict in Sheffield’s favor, the jury did not
find that Sheffield sustained a permanent injury and only awarded her past
medical expenses and $6,554.61 for future medical expenses over a five-
year period. After reducing the amount of the verdict with set-offs from
collateral sources and the settlement with the tortfeasor, the trial court
entered final judgment for Sheffield in the total amount of $4,170.22.
Sheffield filed a motion for a new trial on the issue of damages, arguing that
the trial court erred in denying her motion for directed verdict on the issue of
the permanency of her injuries and that the trial court erred in denying her
motion to exclude evidence of collateral sources of recovery. The trial court
denied the motion.
Sheffield appealed to the First District, which affirmed the denial of her
motion for a new trial, finding that “her own introduction of such [collateral
source] evidence precludes reversal for a new trial on that ground.”
Sheffield, 741 So. 2d at 534. The First District reasoned that the trial court’s
failure to grant the motion in limine did not confer a right on Sheffield to
“build error into the trial so as to guarantee two bites at the apple.” Id. at
537. The First District explained that Sheffield “invited the error now
invoked as a reason for a new trial.” Id.

111
Judge Browning dissented, concluding that Sheffield did not waive any
error in light of the stipulation with Superior that Sheffield would have a
standing objection to the introduction of collateral source evidence. See id. at
541 (Browning, J., dissenting). Judge Browning also objected to the
majority’s reliance on the “invited error” reasoning, noting that Superior had
never advanced the argument that Sheffield had waived the collateral source
issue and that Superior argued only that the introduction of the evidence was
harmless error that did not affect the amount of the jury’s verdict. See id. at
539-40. In response to the majority position that Sheffield opened the door
by introducing the improper evidence herself, the dissenting opinion
explained that because Sheffield was faced with the trial court’s incorrect
ruling, Sheffield “had every right to attempt to defuse the issue and initially
present collateral-source evidence.” Id. at 540.
Judge Browning analogized the instant case to the Third District’s
decision in Porter, a case involving the plaintiff’s introduction of his
previous alcohol abuse after the trial court denied his motion in limine on the
matter:
[P]laintiff’s counsel’s attempt to diminish the prejudicial impact of the
damaging evidence did not, contrary to appellee’s contentions, waive the
error, or render the error harmless. A party cannot be penalized for his good-
faith reliance on a trial court’s incorrect ruling.
Sheffield, 741 So. 2d at 540 (quoting Porter, 630 So. 2d at 206). In
addition, Judge Browning concluded that because Gormley v. GTE Products
Corp., 587 So. 2d 455 (Fla.1991),FN2 constituted a landmark decision that the
trial court ignored, “no excuse exists for trying a case on a misapplication of
such a universally known and accepted principle of law.” Sheffield, 741 So.
2d at 540. Finally, Judge Browning concluded that “[i]f the admitted error of
the trial judge is considered on its impact on the trial rather than on the basis
of nonpreservation, the burden of proving that the error was ‘harmless’ is
borne by Superior, which induced the trial court to commit reversible error.”
Id. Judge Browning expressed concern that
FN2. The Court in Gormley held that the introduction of collateral source
benefits in a liability trial, over objection, constitutes reversible error. 587
So. 2d at 458.
[t]he majority’s decision will encourage litigants to seek clever
misapplications of law to gain an advantage, and then after so doing, if
confronted by appeal, admit to error and claim it to be harmless, or claim

112
that the opponent waived the error while struggling within the confines of a
patently unfair proceeding. Id. at 541.
DISCUSSION
[1] [2] The First District majority opinion acknowledges, and we agree,
that the trial court committed “clear error” in denying Sheffield’s motion in
limine to exclude the introduction of collateral sources of payment at trial.
See Sheffield, 741 So. 2d at 536. Upon proper objection, the collateral source
rule prohibits the introduction of any evidence of payments from collateral
sources. See Gormley, 587 So. 2d at 457; § 768.76, Fla. Stat. (2000).FN3
Because the subject of Sheffield’s motion in limine covered collateral source
benefits from her employer, there is no question that the trial court erred in
denying Sheffield’s motion in limine.
FN3. The common-law rule prohibited both the introduction of evidence
of collateral benefits received and the setoff of any collateral source benefits
from the damage award. See Rollins v. Pizzarelli, 761 So. 2d 294, 300
(Fla.2000). The common-law rule has been altered in this State by statute.
See § 768.76, Fla. Stat. (2000). Although collateral source evidence may not
be introduced before the jury, after entry of the verdict the trial court is
required to reduce the amount of damages by the amount of all collateral
sources for which no right of subrogation exists. See § 768.76(1), Fla. Stat.
(2000). However, because this was a motor vehicle accident, the jury would
not award damages for personal injury protection (“PIP”) benefits that were
paid or payable. See § 627.736(3), Fla. Stat. (2000); see generally Rollins,
761 So. 2d at 294.
Given this error, the issue before this Court is whether Sheffield’s
counsel, by introducing collateral source evidence in Sheffield’s case-in-
chief after the trial court erroneously denied her motion in limine, waived
any objection for appellate review. Florida appellate courts are divided on
this issue.
In Porter, the plaintiff in a slip-and-fall case filed a pretrial motion to
prevent the defendant from introducing evidence concerning the plaintiff’s
previous alcohol abuse. 630 So. 2d at 206. The trial court denied the motion
and during opening statement the plaintiff’s counsel mentioned plaintiff’s
prior alcoholism “in an effort to diffuse its impact.” Id. The Third District
first concluded that the trial court erroneously denied the motion, because
there was no dispute that the plaintiff was sober at the time of the injury and
the prejudicial effect of the evidence outweighed its probative value. See id.

113
Next, the Third District held that “plaintiff’s counsel’s attempt to diminish
the prejudicial impact of the damaging evidence did not, contrary to
[defendant’s] contentions, waive the error, or render the error harmless. A
party cannot be penalized for his good-faith reliance on a trial court’s
incorrect ruling.” Id.
Subsequently, in Smith, the Third District, in reliance on Porter, held that
a party who is wrongfully denied a motion in limine does not “open the
door” upon subsequently admitting this evidence in the party’s case-in-chief.
Smith, 753 So. 2d at 599-600. In Smith, the parents of a bar patron, who was
shot and killed by another bar patron, brought a wrongful death action
against the bar for negligent failure to provide adequate security. Id. at 598.
Before trial, the parents moved for a motion in limine to exclude as
irrelevant and immaterial all evidence regarding their son’s character. See id.
The trial court denied the motion and the parents introduced evidence
concerning their son’s good character. Id. at 599.
On appeal, the parents claimed that the trial court erred in denying the
motion in limine. See id. In response, the bar claimed that the introduction of
character evidence was appropriate rebuttal to the parents’ evidence of their
son’s good nature, which the parents introduced to prove the measure of
their loss. See id. The Third District agreed with the parents’ argument,
concluding that “Smiths’ counsel presented good character testimony in
anticipation of [the bar’s] bad character evidence and simply attempted to
minimize the latter’s prejudicial impact.” Id. at 600.
In Duffell v. South Walton Emergency Services, Inc., 501 So. 2d 1352,
1353 (Fla. 1st DCA 1987), Judge Ervin, in a concurring in part dissenting in
part opinion, espoused a similar approach. In Duffell, a personal injury
action, the plaintiff sought to exclude evidence of her drug use following her
discharge from the hospital. See id. at 1353. The trial court ruled that the
defendants could mention the plaintiff’s drug use to the jury in the
defendant’s opening statement. See id. at 1353. Consequently, the plaintiff’s
counsel alluded to the plaintiff’s prescribed drug use during the plaintiff’s
opening statement, and asked the jury to ignore this evidence as irrelevant to
the issue of negligence. See id. The defense, in its opening statement,
advised the jury that the plaintiff had a drug problem, and that the problem
continued through the accident. See id. The plaintiff contended that the
admission of her drug use was prejudicial and constituted grounds for
reversal, but the defendant claimed that plaintiff invited the error. See id. at

114
1357. Although the First District in a per curiam opinion found “no
reversible error” occurred,FN4 id. at 1352, Judge Ervin explained:
FN4. Because the entire majority opinion consists of the one-line
statement that no reversible error occurred, we cannot discern whether the
First District majority reached this result because it concluded that the error
was not properly preserved or because it concluded that there was no error in
the evidentiary ruling regarding the drug history.
I think it obvious that the conduct of [plaintiff’s] trial counsel in making
disclosure of his client’s drug history was a last desperate attempt to salvage
a case that had already been fatally infected by the lower court’s rulings and
the trial tactics of the [defense’s] attorney. It was only after her specific
objections to the admissibility of such evidence had been overruled that
appellant submitted evidence relating to her drug dependency. By analogy,
“the doctrine of waiver does not apply where the appellant makes a proper
objection of the matters included in the court’s general charge, and when
such objection is overruled, necessarily offers an instruction on such
matters.” 3 Fla. Jur.2d, Appellate Review § 294 at 348-349 (1978). No
lawyer-as was [plaintiff’s] below-should ever be forced to make such a
Hobson’s choice: either to go forward with the trial with no reference to his
client’s drug habits, after nearly all of his objections to clearly prejudicial
evidence had been overruled, or, faced with the adverse ruling, to appeal to
the jurors’ sense of justice and fairness by “telling all”, in the hopes that his
client’s openness and candor would add credibility to the witness’s
testimony bearing upon disputed factual issues.
Id. at 1358 (emphasis added).
We note, however, that in a case not cited by either party or by the First
District, Chenoweth v. Kemp, 396 So. 2d 1122, 1125 (Fla.1981), this Court
rejected, without any citation or analysis, a finding of trial court error in
admitting into evidence the written findings of a mediation panel because the
“appellants put the findings before the jury first, by reading the entire
mediation panel report in their opening arguments after having been denied a
motion in limine. We recognize that this was a tactical decision, but
appellants are bound by it.” Yet in a cogent dissenting opinion, Justice
Sundberg explained:
My colleagues hold that appellant has waived his right to contest the
clearly erroneous inclusion in evidence of the actual written finding of the
mediation panel because appellant first read the finding to the jury during

115
opening argument. This ignores the fact that in the course of denying
appellant’s pretrial motion in limine on this subject, the trial judge
specifically noted that he was going to allow written finding to be admitted
in evidence. In light of this ruling, appellant had every right, and indeed had
no choice, but to comment upon the evidence in an attempt to mitigate the
damage soon to be done by the erroneous inclusion of the written finding.
Trial court error, not tactics, dictated appellant’s actions.
Id. at 1127 (Sundberg, C.J., dissenting) (emphasis added). We agree with
Justice Sundberg’s reasoning.
[3] [4] When a party, after receiving an adverse evidentiary ruling, seeks
to minimize its prejudicial impact, the situation is not, contrary to the First
District’s reasoning, an attempt to get two bites at the apple. The concept of
“invited error” does not apply where, as here, the trial court makes an
unequivocal ruling admitting evidence over the movant’s motion in limine,
and the movant subsequently introduces the evidence in an attempt to
minimize the prejudicial impact of the evidence. “Under the invited-error
doctrine, a party may not make or invite error at trial and then take
advantage of the error on appeal.”Goodwin v. State, 751 So. 2d 537, 544 n. 8
(Fla.1999); see also Terry v. State, 668 So. 2d 954, 962 (Fla.1996).
As the Third District has recognized, once the party is faced with the
knowledge that the jury will hear the evidence, it is legitimate trial strategy
for a party to introduce the evidence at trial in an attempt to mitigate the
harm and diffuse the prejudicial impact of the evidence. See Smith, 753 So.
2d at 600. In many cases, the only way to attempt to minimize the
“dynamite” impact of the adverse evidence will be to question the jury about
it in voir dire, discuss it in opening statement, and thereafter introduce the
evidence.
[5] We thus agree with Judge Browning’s dissenting opinion in this case
and with the Third District’s decisions in Porter and Smith. We hold that
once a trial court makes an unequivocal ruling admitting evidence over a
movant’s motion in limine, the movant’s subsequent introduction of that
evidence does not constitute a waiver of the error for appellate review.
THIS CASE
[6] In this case, the trial court denied the motion in limine regarding the
collateral source evidence. Sheffield and Superior thereafter entered into a
stipulation that Sheffield’s introduction of collateral source evidence would
not act as a waiver of her objection and that Sheffield would have a standing

116
objection to evidence of collateral sources. We thus conclude that based
upon the facts of this case, Sheffield’s counsel did not waive the error for
appellate review by introducing collateral source evidence after the trial
court erroneously denied Sheffield’s motion in limine.
[7] [8] Moreover, we agree with Judge Browning that the collateral
source evidence introduced in this case was not harmless error. With regard
to the introduction of collateral source evidence, this Court held in Gormley
that:
Equity and logic demand that the burden of proving such an error
harmless must be placed on the party who improperly introduced the
evidence. Putting the burden of proof on the party against whom the
evidence is used ... would simply encourage the introduction of improper
evidence. 587 So. 2d at 459. The burden of proving that the admission of the
collateral source evidence was harmless rests on Superior. As the Fourth
District observed, “when a trial lawyer leads a judge into an obvious error ...
cries of harmless error on appeal are likely to fall on deaf ears.” Mattek v.
White, 695 So. 2d 942, 944 (Fla. 4th DCA 1997). Although we did not
announce a per se rule of reversal in Gormley, we recognized the inherently
damaging effect of the jury hearing collateral source evidence both on the
issues of liability and on issues of damages. 587 So. 2d at 458; see also
Parker v. Hoppock, 695 So. 2d 424, 429 (Fla. 4th DCA 1997).
[9] Superior claims that Sheffield minimized any prejudice with regard to
future benefits by testifying that there was no guarantee that she would
receive collateral source benefits in the future. However, given the
inherently prejudicial effect of such evidence, which is the very reason the
collateral source rule was first established, we cannot conclude that in this
case the introduction of collateral source evidence was harmless. The jury
certainly could have concluded that because Sheffield had group insurance
available to cover future medical expenses, there would be no need to award
substantial damages for the future.
Accordingly, we approve Porter and Smith, recede from Chenoweth to
the extent it is inconsistent with this opinion, quash the First District’s
decision and remand for proceedings consistent with this opinion. FN5
FN5. Because our quashing of the First District decision will require a
new trial, we decline to address the second issue raised by Sheffield and
addressed in the First District’s opinion, i.e., whether the trial court erred in

117
denying Sheffield’s motion for directed verdict on the issue of permanency
of injury, which is not a basis for this Court’s jurisdiction.
It is so ordered.
WELLS, C.J., and SHAW, ANSTEAD, LEWIS, and QUINCE, JJ.,
concur.
HARDING, J., dissents with an opinion.
HARDING, J., dissenting.
I respectfully dissent. I would approve the opinion of the First District
Court of Appeal in Sheffield v. Superior Insurance Co., 741 So. 2d 533 (Fla.
1st DCA 1999).

V. Initiating Review

A. Proper Forum

Florida Rules of Appellate Procedure

Rule 9.040. General Provisions


***
(b) Forum.
(1) If a proceeding is commenced in an inappropriate court, that court shall
transfer the cause to an appropriate court.

B. Appellate Districts

Wovas v. Tousa Homes, Inc.


940 So. 2d 1166 (Fla. 2d DCA 2006)
LaROSE, Judge.
Edward J. Wovas and Tom and Elaine Mitchell (collectively, Lot
Owners) petition this court to issue a writ of prohibition stopping the circuit
court from ruling upon a petition for writ of prohibition filed by Tousa
Homes, Inc., and Royal-Tee Homeowners Association, Inc. (collectively,
Developer). The Lot Owners argue that the circuit court does not have
appellate jurisdiction over the decisions of the Department of Business and

118
Professional Regulation (Department); accordingly, the circuit court lacks
jurisdiction to issue an extraordinary writ. We agree and grant the petition.
The Lot Owners own residential lots in a subdivision. Pursuant to section
720.311, Florida Statutes (2005), they sought mandatory binding arbitration
in the Department’s Division of Land Sales, Condominiums and Mobile
Homes (Division). Essentially, the Lot Owners seek an order requiring the
Developer to relinquish control of a homeowners’ association to the
residents. The Lot Owners characterize their dispute with the Developer as
an election dispute. Section 720.311 mandates binding arbitration of such
disputes.
The Developer filed a petition for writ of prohibition or, alternatively, a
petition for all writs, in the circuit court. Claiming that, among other things,
section 720.311 is unconstitutional, the Developer sought to prohibit the
Division from conducting arbitration. The circuit court issued an order to
show cause. The Lot Owners then filed their own petition for writ of
prohibition in this court. They argue that the circuit court lacks jurisdiction
to consider the Developer’s prohibition petition.
[1] [2] Article V, section 5(b) of the Florida Constitution states:
The circuit courts shall have original jurisdiction not vested in the county
courts, and jurisdiction of appeals when provided by general law. They shall
have the power to issue writs of mandamus, quo warranto, certiorari,
prohibition and habeas corpus, and all writs necessary or proper to complete
exercise of jurisdiction. Jurisdiction of the circuit court shall be uniform
throughout the state. They shall have the power of direct review of
administrative action prescribed by general law.
A circuit court may issue an extraordinary writ only where it has original
or appellate jurisdiction. A circuit court may exercise appellate jurisdiction
over administrative action where authorized by the legislature. Section
26.012, Florida Statutes (2005), grants the circuit courts appellate
jurisdiction over “final administrative orders of local government code
enforcement boards.” That jurisdiction does not extend to the Division, a
state agency. Prime Orlando Props., Inc. v. Dep’t of Bus. Regulation, Div. of
Land Sales, Condos. & Mobile Homes, 502 So. 2d 456, 458 (Fla. 1st DCA
1986).
[3] Review of state administrative agency action is proper “in the
appellate district where the agency maintains its headquarters or where a
party resides or as otherwise provided by law.” § 120.68(2)(a), Fla. Stat.

119
(2005). A petition for issuance of an extraordinary writ “challenging matters
in an administrative action is properly filed in [the District Court of Appeal],
not the circuit court.” Dep’t of Health, Bd. of Dentistry v. Barr, 882 So. 2d
501, 501 (Fla. 1st DCA 2004). In Barr, the Board of Dentistry sought a writ
of prohibition in the First District seeking to restrain the circuit court from
considering Barr’s petition for writ of prohibition seeking to halt disciplinary
proceedings against him. The First District granted the petition, holding that
the circuit court did not have jurisdiction to issue an extraordinary writ
where it did not have appellate jurisdiction over the subject matter. Id.
Similarly, in Florida Real Estate Commission v. Anderson, 164 So. 2d 265
(Fla. 2d DCA 1964), two real estate brokers filed a petition for writ of
prohibition in the circuit court alleging that the Commission lacked
jurisdiction to proceed against them. See id. at 267. The supreme court held
that the circuit court did not have jurisdiction to consider prohibition
proceedings where it did not have appellate jurisdiction over the
Commission. Id. at 268.
We conclude, therefore, that the circuit court does not have jurisdiction to
consider the Developer’s petition for writ of prohibition. We grant the
petition and quash the circuit court’s order to show cause.
Petition granted.
SALCINES and STRINGER, JJ., Concur.

C. Correct Remedy

Florida Rules of Appellate Procedure

Rule 9.040. General Provisions


***
(c) Remedy. If a party seeks an improper remedy, the cause shall be
treated as if the proper remedy had been sought; provided that it shall not
be the responsibility of the court to seek the proper remedy.

120
D. Procedure

1. Appeals

Florida Rules of Appellate Procedure

Rule 9.110. Appeal Proceedings to Review Final Orders of Lower


Tribunals and Orders Granting New Trial in Jury and Non-Jury Cases
***
(b) Commencement. Jurisdiction of the court under this rule shall be
invoked by filing an original and 1 copy of a notice, accompanied by any
filing fees prescribed by law, with the clerk of the lower tribunal within 30
days of rendition of the order to be reviewed.
(c) Exception; Administrative Action.In an appeal to review final orders of
lower administrative tribunals, the appellant shall file the original notice
with the clerk of the lower administrative tribunal within 30 days of
rendition of the order to be reviewed, and file a copy of the notice,
accompanied by any filing fees prescribed by law, with the clerk of the
court.
(d) Notice of Appeal.The notice of appeal shall be substantially in the
form prescribed by rule 9.900(a). The caption shall contain the name of
the lower tribunal, the name and designation of at least 1 party on each
side, and the case number in the lower tribunal. The notice shall contain
the name of the court to which the appeal is taken, the date of rendition,
and the nature of the order to be reviewed. Except in criminal cases, a
conformed copy of the order or orders designated in the notice of appeal
shall be attached to the notice together with any order entered on a timely
motion postponing rendition of the order or orders appealed.
***

Rule 9.130. Proceedings to Review Non-Final Orders and Specified


Final Orders
***
(b) Commencement.The jurisdiction to seek review of orders described in
subdivisions (a)(3)–(a)(6) shall be invoked by filing 2 copies of a notice,
accompanied by the filing fees prescribed by law, with the clerk of the
lower tribunal within 30 days of rendition of the order to be reviewed.
(c) Notice.The notice, designated as a notice of appeal of non-final order,
shall be substantially in the form prescribed by rule 9.900(c). Except in

121
criminal cases, a conformed copy of the order or orders designated in the
notice of appeal shall be attached to the notice.
***

Norm Burg Const. Corp. v. Jupiter Inlet Corp.


514 So. 2d 1102 (Fla. 1987)
PER CURIAM.
The Fourth District Court of Appeal certified to this Court its decision in
this case, Jupiter Inlet Corp. v. Brocard, 489 So. 2d 49 (Fla. 4th DCA 1986),
as presenting a question of great public importance. The decision under
review is the district court’s denial of petitioner Norm Burg Construction
Corporation’s motion to be dismissed from an appeal where it was named an
appellee. We have jurisdiction. Art. V, § 3(b)(4), Fla. Const.
The plaintiff, the surviving spouse of a deceased construction worker,
filed a wrongful death action against respondent Jupiter Inlet Corporation
(Jupiter Inlet). Jupiter Inlet filed a third-party claim against petitioner Norm
Burg Construction Corporation (Norm Burg) seeking common law and
contractual indemnity. The claim for contractual indemnity was severed for
subsequent determination by the trial judge alone. The third-party claim
based on common-law indemnity was tried to the jury along with the
plaintiff’s negligence claim.
The jury found in favor of the plaintiff and against Jupiter Inlet on the
wrongful death claim and in favor of Norm Burg and against Jupiter Inlet on
the common-law indemnity claim. Jupiter Inlet filed a motion for new trial,
which addressed both of the jury’s findings.
After denial of the motion for new trial, the trial court rendered a
judgment in the wrongful death action, adjudging that the plaintiff should
recover money damages in a specified amount from Jupiter Inlet. The
judgment did not adjudicate any portion of the dispute between Jupiter Inlet
and Norm Burg, and neither Norm Burg nor the issue of indemnification is
referred to in that final judgment. Jupiter Inlet filed a notice of appeal of this
judgment, naming Norm Burg as an appellee.
Following the rendition of judgment in the wrongful death action, the
severed claim for contractual indemnity was tried. A second final judgment
was rendered adjudicating the claim of Jupiter Inlet for indemnity from
Norm Burg, based in part on the jury’s verdict on the common-law

122
indemnity claim and in part on the judge’s determination of the contractual
indemnity claim. The second final judgment denied recovery to Jupiter Inlet
from Norm Burg on its claims for indemnity. Jupiter Inlet did not appeal the
second final judgment.
[1] Norm Burg filed a motion to be dismissed from Jupiter Inlet’s appeal
of the first final judgment on the ground that the final judgment did not
affect any legal rights or liabilities of Norm Burg. The district court of
appeal denied the motion to dismiss, reasoning that the notice of appeal of
the first final judgment vested jurisdiction in the appellate court as to Burg
and the issue of common-law indemnity, even though adjudication of the
third-party claim for indemnity was not final until the rendition of the
second final judgment, which Jupiter Inlet did not appeal. The district court
certified the following question:
Does a notice of appeal filed after jury verdict but before an appropriate
final judgment remain in limbo as to any aspect of the jury verdict which is
not reflected in such final judgment filed thereafter, but is eventually
reflected in a subsequently rendered final judgment?
Jupiter Inlet Corp., 489 So. 2d at 51. We answer the question in the
negative and quash the decision of the district court.
The district court of appeal upheld the position of Jupiter Inlet “that its
notice of appeal addressed to the final judgment of February 15 reached any
issue inhering in the jury verdict, whether or not enunciated in the final
judgment, and therefore constitutes an appeal of the adjudication on the
issue of common-law indemnification.” Id. at 50. The district court stated
that one of the functions of a notice of appeal is to give notice to the adverse
party that an appeal is being taken. The court found this function had been
adequately performed in that Norm Burg was on notice that Jupiter regarded
its appeal as encompassing the matter of the common-law indemnity claim.
The other function that a notice of appeal must fulfill, the district court said,
is to vest jurisdiction in the appellate court. In support of its finding that
jurisdiction was vested, the district court discussed this Court’s decision in
Williams v. State, 324 So. 2d 74 (Fla.1975).
Petitioner argues that Williams v. State arose in an entirely different
situation and should not be applied in this case. Petitioner maintains that a
notice of appeal directed to one final judgment cannot vest appellate
jurisdiction to review a wholly separate and independent final judgment
rendered subsequently.

123
In Williams v. State, the defendant was found guilty of a criminal offense
by jury verdict on August 13, 1973. On August 24, 1973, post-trial motions
were denied, judgment was announced, sentence was imposed, and appeal
bond was set in open court. Written judgment and sentence were signed that
same day in chambers. Notice of appeal was also filed that same day.
However, the judgment was not considered “rendered,” in the sense of being
filed for recording, until August 28. The state moved to dismiss the appeal
on the ground that a notice filed four days before the judgment was rendered
was ineffective to vest jurisdiction of the appeal.
The Court acknowledged that the notice of appeal was “premature.” Id. at
79. However, the Court noted that the judgment had “conditioned the grant
of supersedeas bond upon the filing of the notice of appeal” thus placing the
defendant in “an untenable position,” so that it was “understandable that the
defendant would file his notice of appeal so as to be eligible for such bond at
the earliest possible time.” Id.
Because of this situation, we now hold that a defendant may, for the
purposes of obtaining supersedeas bond, file his notice of appeal at any time
after oral judgment or sentence is pronounced and before it is rendered, i.e.,
filed for recording. Prior to the judgment, the notice of appeal shall not be
effective to vest jurisdiction in the appellate court, but will allow the
defendant to obtain supersedeas bond. At the time when the judgment and/or
sentence is rendered (filed for recording) the notice of appeal shall be
effective to vest jurisdiction in the appellate court. It must be noted that
notice of appeal so filed (to obtain supersedeas before judgment is rendered)
shall not be subject to dismissal either by motion of the parties or of the
Court.
Leaving aside the question of obtaining supersedeas bond, we also hold
that a notice of appeal which is prematurely filed shall not be subject to
dismissal. Rather, such a notice of appeal shall exist in a state of limbo until
the judgment in the respective civil or criminal case is rendered. At the time
of rendition, the notice of appeal shall mature and shall vest jurisdiction in
the appellate court.
Thus, a notice of appeal which is filed after the oral pronouncement of
judgment and/or sentence, but before rendition thereof, is not to be
dismissed on the grounds that it is premature. This rule shall apply to such
situations as when the defendant filed his notice of appeal:

124
1. After oral pronouncement of judgment, but before the judgment is
reduced to writing and signed.
2. After the written judgment is signed, but before it is rendered (filed for
recording).
3. After the written judgment is filed for recording, but before a post-trial
motion is decided.
Id. at 79-80.
We agree with petitioner that Williams is simply inapplicable to this case.
First, this is not a case where a party was required to file a premature notice
of appeal in order to obtain a supersedeas bond. Second, here there was no
notice of appeal addressed to a judgment that had been orally pronounced or
signed but not yet formally rendered. Here there was a timely notice of
appeal addressed to a final judgment that had been rendered but which did
not include an adjudication of the issues involved in the litigation in which
Norm Burg was a party. That litigation had not been concluded. The
situations are entirely different. Williams is not authority for holding that a
notice of appeal addressed to one final judgment can be effective as a notice
of appeal directed to a subsequently rendered, separate final judgment
adjudicating a different cause of action affecting a different party.
Petitioner correctly argues that respondent’s notice of appeal could not
obtain appellate review of the jury’s verdict and judicial actions in
connection therewith, pertaining to the common-law indemnity claim,
because the first final judgment did not dispose of the latter matter. Jury
verdicts are not appealable. See, e.g., Menfi v. Exxon Co., 433 So. 2d 1327
(Fla. 3d DCA 1983) (rendition of verdict is distinct from rendition of
judgment; the latter can be appealed but the former cannot).
Seeking to sustain the decision of the district court, respondent relies
upon a series of decisions of this Court and various district courts of appeal.
The cases cited support the general proposition that technical defects in
notices of appeal, which do not affect the appellate court’s jurisdiction and
do not mislead or prejudice the adversary party, do not require the dismissal
of an appeal. Ratner v. Miami Beach First National Bank, 362 So. 2d 273
(Fla.1978); Milar Galleries, Inc. v. Miller, 349 So. 2d 170 (Fla.1977);
Brown v. Winn-Dixie Stores, Inc., 267 So. 2d 78 (Fla.1972); Eggers v.
Narron, 238 So. 2d 72 (Fla.1970); State ex rel. Poe v. Allen, 196 So. 2d 745
(Fla.1967); Bay Area News, Inc. v. Poe, 364 So. 2d 830 (Fla. 2d DCA 1978),
cert. denied, 373 So. 2d 456 (Fla.1979); Casino, Inc. v. Kugeares, 354 So.

125
2d 936 (Fla. 2d DCA 1978); Bowen v. Bowen, 352 So. 2d 166 (Fla. 1st DCA
1977), dismissed, 360 So. 2d 1247 (Fla.1978); Burlingham v. Allen, 295 So.
2d 684 (Fla. 1st DCA 1974).
In Milar Galleries, Inc. v. Miller and in Burlingham v. Allen, a single
notice of appeal was held effective to secure appellate review of two
separate judgments referenced therein. The present case is distinguishable
because here the notice that was filed addressed a judgment that did not
affect Norm Burg, and did not address the judgment subsequently entered
that did involve Norm Burg. In Eggers v. Narron and State ex rel. Poe v.
Allen, the notices of appeal were timely filed with regard to the judgments
appealed, but the notices erroneously made reference to trial court orders
other than the final judgments. This Court found the deficiencies to be
technical rather than jurisdictional. Neither case dealt with a separate and
independent final judgment as in this case. In Ratner v. Miami Beach First
National Bank, the appellee moved to dismiss the appeal on the ground that
the assignments of error were insufficient as a matter of law to support an
appeal. This Court found that any deficiency shown was technical rather
than jurisdictional and not grounds for dismissal. No similarity to the present
case exists.
In Bowen v. Bowen, the court declined to dismiss an appeal where the
objection was that the notice of appeal was directed to the denial of a motion
for new trial rather than the final judgment. It does not appear that the case
involved separate causes of action with separate and independent final
judgments.
In Casino, Inc. v. Kugeares, the court held dismissal of an appeal was
improper after finding that the notice was timely filed measured from the
time of rendition of the final judgment of which appellate review was
sought. The court rejected the appellee’s argument that the notice was
untimely when measured from the time of rendition of a nonfinal partial
judgment. The decision does not support respondent’s position here.
Bay Area News, Inc. v. Poe is another case in which an appellant directed
its notice of appeal to the wrong action of the lower court but in a timely
manner measured from the time of rendition of the final appealable order.
The appellate court found that the defect was technical, not jurisdictional, so
that in the absence of prejudice to the other side, the appeal should not be
dismissed.

126
Respondent also relies upon Brown v. Winn-Dixie Stores, Inc. There the
trial court entered judgments in favor of each of two defendants. The
judgments were rendered on the same day. The plaintiff filed a notice of
appeal that expressly referred to only one of the judgments, but other appeal
documents demonstrated a clear intent to appeal both judgments. One of the
appellees filed a motion to dismiss on the ground that the judgment in its
favor had not been specifically referred to. This Court held that the appeal
encompassed both judgments, that the omission was a mere technical defect,
and that there was no prejudice. The present case is distinguishable because
here when the notice was filed there was no final judgment adjudicating or
affecting any right or liabilities of the named appellee Norm Burg
Construction Corporation.
A judgment is the means by which a court renders its decision. It is the
pronouncement by the court of its conclusions and decision upon the matter
submitted to it for adjudication. 32 Fla.Jur.2d Judgments & Decrees § 1
(1981).
The first judgment, from which the appeal was taken, awarded plaintiff
money damages against Jupiter Inlet. At that point in time, the third-party
claim had not been concluded and until it was concluded no final judgment
could be entered. While the jury had made its findings of fact on the
common-law indemnity claim, the third-party claim was not ready for final
adjudication because the trial court had not tried the severed contractual
indemnity issue. The second judgment adjudicated the third-party indemnity
claim between Jupiter Inlet and Norm Burg and, while it adjudicated an
issue resolved by the jury, it also adjudicated an issue decided by the trial
judge. Each judgment was separate and distinct. The notice of appeal was
directed to the first judgment and an earlier order denying Jupiter Inlet’s
motion for summary judgment. It made no reference to the adjudication yet
to be made of the third party claim, and it is that adjudication which
respondent seeks to have addressed by the previously filed notice of appeal.
This is not a mere technical defect. It goes to the matter of jurisdiction. The
district court of appeal had jurisdiction of all issues adjudicated in the first
judgment between the plaintiff and the defendant. It did not have jurisdiction
of the indemnity issue, which was the subject of the second judgment.
A timely filed notice of appeal is essential to vest jurisdiction of an
appeal in an appellate court. An appellate court lacks jurisdiction unless the
notice of appeal is filed within the time and in the manner prescribed by the

127
rules. State ex rel. Diamond Berk Insurance Agency v. Carroll, 102 So. 2d
129 (Fla.1958); Counne v. Saffan, 87 So. 2d 586 (Fla.1956); Lehmann v.
Cloniger, 294 So. 2d 344 (Fla. 1st DCA 1974); Sparks v. State, 262 So. 2d
251 (Fla. 4th DCA 1972); Brick v. Brick, 258 So. 2d 7 (Fla. 4th DCA 1971).
[2] We hold that an appeal taken from one final judgment cannot provide
a basis for appellate review of a subsequently rendered, separate and
independent final judgment. We therefore answer the certified question in
the negative. The decision of the district court of appeal is quashed and the
case is remanded with directions to dismiss the appeal filed against Norm
Burg construction Corporation.
It is so ordered.
McDONALD, C.J., and OVERTON, EHRLICH, SHAW and
BARKETT, JJ., concur.

2. Discretionary Review

Florida Rules of Appellate Procedure

Rule 9.120. Discretionary Proceedings to Review Decisions of


District Courts of Appeal
***
(b) Commencement. The jurisdiction of the supreme court described in
rule 9.030(a)(2)(A) shall be invoked by filing 2 copies of a notice,
accompanied by the filing fees prescribed by law, with the clerk of the
district court of appeal within 30 days of rendition of the order to be
reviewed.
(c) Notice. The notice shall be substantially in the form prescribed by rule
9.900. The caption shall contain the name of the lower tribunal, the name
and designation of at least 1 party on each side, and the case number in
the lower tribunal. The notice shall contain the date of rendition of the
order to be reviewed and the basis for invoking the jurisdiction of the court.
***

128
Taggart, Corp. v. Benzing
434 So. 2d 964 (Fla. 4th DCA 1983)
LETTS, Chief Judge.
[***]
We would be remiss in not referring to the opposite view expressed in
Newcombe v. South Florida Business Negotiators, Inc., 340 So. 2d 1192
(Fla. 2d DCA 1976), and Lhamon v. Retail Development, Inc., 422 So. 2d
993 (Fla. 5th DCA 1982). Once again these two latter cases can be factually
distinguished inasmuch as they concern jury rather than non-jury
proceedings. However, we believe the principle involved to be identical.
Accordingly, perceiving conflict with our sister courts and in any event
deeming the matter to be of great public importance, we hereby certify the
following question.
WHERE ATTORNEY’s FEES ARE PLED IN A SUCCESSFUL SUIT
FOR SPECIFIC PERFORMANCE UNDER A REAL ESTATE
CONTRACT PROVIDING FEES FOR THE PREVAILING PARTY, CAN
THE PROOF OF SAME BE PRESENTED FOR THE FIRST TIME
AFTER FINAL JUDGMENT IF JURISDICTION TO DO SO IS
REQUESTED?
In answering the above, we hope the Court will also address the question
of whether the issue of attorney’s fees must be pled, which we certainly
think it should be, and also the question of whether it makes any difference
whether the fees are allowable by statute as distinct from the terms of the
contract.
[2] Finally, although not necessary to our conclusion, we are reliably
informed that many lawyers do not realize that questions do not
automatically proceed upwards to the Supreme Court merely because we
certify them. The losing party at our level still has to exercise the initiative
and go forward, otherwise the questions die for lack of appropriate
presentation.
REVERSED AND REMANDED IN ACCORDANCE HEREWITH.
BERANEK and WALDEN, JJ., concur.

129
3. Original Proceedings

Florida Rules of Appellate Procedure

Rule 9.100. Original Proceedings


***
(b) Commencement; Parties.The original jurisdiction of the court shall be
invoked by filing a petition, accompanied by a filing fee if prescribed by
law, with the clerk of the court deemed to have jurisdiction. If the original
jurisdiction of the court is invoked to enforce a private right, the proceeding
shall not be brought on the relation of the state. If the petition seeks review
of an order entered by a lower tribunal, all parties to the proceeding in the
lower tribunal who are not named as petitioners shall be named as
respondents
***

E. Proceedings by Indigents

Florida Rules of Appellate Procedure

Rule 9.430. Proceedings by Indigents


(a) Appeals. A party who has the right to seek review by appeal without
payment of costs shall, unless the court directs otherwise, file a signed
application for determination of indigent status with the clerk of the lower
tribunal, using an application form approved by the Supreme Court for use
by circuit court clerks. The clerk of the lower tribunal's reasons for denying
the application shall be stated in writing and are reviewable by the lower
tribunal. Review of decisions by the lower tribunal shall be by motion filed
in the court.
(b) Original Proceedings. A party who seeks review by an original
proceeding under rule 9.100 without the payment of costs shall, unless the
court directs otherwise, file with the court a motion to proceed in forma
pauperis. If the motion is granted, the party may proceed without further
application to the court.
(c) Incarcerated Parties.
(1) Presumptions.In the absence of evidence to the contrary, an appellate
court may, in its discretion, presume that

130
(A) assertions in an application for determination of indigent status
filed by an incarcerated party under this rule are true, and
(B) in cases involving criminal or collateral criminal proceedings, an
incarcerated party who has been declared indigent for purposes of
proceedings in the lower tribunal remains indigent.
(2) Non-Criminal Proceedings.Except in cases involving criminal or
collateral proceedings, an application for determination of indigent status
filed under this rule by a person who has been convicted of a crime and is
incarcerated for that crime or who is being held in custody pending
extradition or sentencing shall contain substantially the same information
as required by an application form approved by the Supreme Court for use
by circuit court clerks. The determination of whether the case involves an
appeal from an original criminal or collateral proceeding depends on the
substance of the issues raised and not on the form or title of the petition or
complaint. In these non-criminal cases, the clerk of the lower tribunal shall
require the party to make a partial prepayment of court costs or fees and
to make continued partial payments until the full amount is paid.

VI. Parties

Florida Rules of Appellate Procedure

Rule 9.020. Definitions


***
(g) Parties.
(1) Appellant. A party who seeks to invoke the appeal jurisdiction of
a court.
(2) Appellee. Every party in the proceeding in the lower tribunal
other than an appellant.
(3) Petitioner. A party who seeks an order under rule 9.100 or rule
9.120.
(4) Respondent. Every other party in a proceeding brought by a
petitioner.
***

131
Rule 9.360. Parties
(a) Joinder. A party to the cause in the lower tribunal who desires to join
in a proceeding as a petitioner or appellant shall serve a notice to that
effect no later than the latest of the following: (i) within 10 days of service
of a timely filed petition or notice of appeal; (ii) within the time prescribed
for filing a notice of appeal; or (iii) within the time prescribed in rule
9.100(c). The original and 1 copy of the notice of joinder, accompanied by
any filing fees prescribed by law, shall be filed either before service or
immediately thereafter in the same manner as the petition or notice of
appeal.
(b) Attorneys, Representatives, and Guardians Ad Litem. Attorneys,
representatives, and guardians ad litem in the lower tribunal shall retain
their status in the court unless others are duly appointed or substituted;
however, for limited representation proceedings under Florida Family Law
Rule of Procedure 12.040, representation terminates upon the filing of a
notice of completion titled “Termination of Limited Appearance” pursuant
to rule 12.040(c).
(c) Substitution of Parties.
(1) If substitution of a party is necessary for any reason, the court
may so order on its own motion or that of a party.
(2) Public officers as parties in their official capacities may be
described by their official titles rather than by name. Their successors in
office shall be automatically substituted as parties.
(3) If a party dies while a proceeding is pending and that party's
rights survive, the court may order the substitution of the proper party on
its own motion or that of any interested person.
(4) If a person entitled to file a notice dies before filing and that
person's rights survive, the notice may be filed by the personal
representative, attorney of record, or, if none, by any interested person.
Following filing, the proper party shall be substituted.

132
A. Entitlement to Review

1. Parties in the lower tribunal

Smith v. Chepolis
896 So. 2d 934 (Fla. 1st DCA 2005)
PADOVANO, J.
In this appeal we must decide whether the judge of compensation claims
had authority to hold the owner of a corporation liable for workers’
compensation benefits claimed by an employee of the corporation. Because
the owner was not named as a party in the petition for benefits and did not
receive notice that he could be held personally responsible for the injured
worker’s claim against the corporation, we conclude that the judge erred in
holding him individually liable.
The claimant, Karl Chepolis, was injured on July 29, 2002, in the course
of his employment with Tower Contracting of Miami, a Florida corporation
owned and operated by the appellant, Timothy Smith. Chepolis filed a
petition for benefits naming Tower Contracting as his employer. Smith was
not named in the petition as a party but he participated in the case to some
extent, in his capacity as the president of the company.
The petition for benefits proceeded to a hearing on the merits and the
claimant ultimately proved that his injury was compensable. Following the
hearing, the judge of compensation claims entered an order directing “[t]he
employer Tower Contracting and/or Timothy Smith” to pay a total of
$72,312.78 in benefits, fees, costs, penalties, and interest. This was the first
time Smith was named as an individual. The petition for benefits, the pretrial
stipulation and the notice of final hearing all listed only Tower Contracting
as the employer.
Smith filed a motion to vacate the final order, contending that he could
not be required to pay workers’ compensation benefits because he was not
the claimant’s employer. The judge of compensation claims rejected this
argument and declined to vacate the compensation order. The judge
reasoned that the Workers’ Compensation Law defines the term “employer”
broadly to include corporate officers like Smith who are in actual control of
a corporation. Smith filed a timely appeal to this court to seek review of the
ruling.

133
[1] [2] Our first task is to identify the proper appellate remedy. Smith is
no doubt aggrieved by the order awarding benefits, but it is questionable
whether he has a right to a plenary appeal, given the fact that he was not a
party. As a general principle, the right to appeal is limited to those who were
parties to the proceeding in the lower tribunal. See Penabad v. A.G.
Gladstone Associates, Inc., 823 So. 2d 146 (Fla. 3d DCA 2002); Stas v.
Posada, 760 So. 2d 954 (Fla. 3d DCA 1999). This rule was designed to
prevent an appeal by a new litigant who did not participate in the case in the
lower tribunal and was not directly affected by the order.
[3] However, the rule limiting appellate review to the parties in the lower
tribunal should not be applied in a purely mechanical way to deny the right
to appeal in all circumstances. In some situations, the right to appeal extends
to a nonparty. For example, a litigant who is denied the right to intervene
may appeal the order denying intervention even though he is not a party. See
J.R. v. R.M., 679 So. 2d 64 (Fla. 4th DCA 1996); City of Dania v. Broward
County, 658 So. 2d 163 (Fla. 4th DCA 1995). An order denying a right to
intervene directly adjudicates the legal rights of the prospective intervenor.
This case presents a different situation, but the argument for allowing a
plenary appeal is no less compelling. It would be illogical to conclude that
Smith was denied due process of law because he was not joined in the
litigation and had no notice that a judgment might be entered against him,
and at the same time to conclude that he has no right to appeal because he is
not a party. The very point of his argument on appeal is that he should not
have been treated as if he were a party.
We have recognized that a nonparty who is adversely affected by an
order may be entitled to review by certiorari. See Ahlers v. Wilson, 867 So.
2d 524 (Fla. 1st DCA 2004); State ex rel. Boyles v. Florida Parole and
Probation Commission, 436 So. 2d 207 (Fla. 1st DCA 1983). It is a short
step from these decisions to conclude that a nonparty whose rights are
directly adjudicated in a final order has a right to a plenary appeal. In this
situation, the nonparty should not be left with only a discretionary appellate
remedy like certiorari. Because the order at issue directly adjudicates
Smith’s rights by imposing personal liability against him, we hold that he
has a right to appeal.
[4] On the merits of the case, we find no basis for the imposition of
personal liability against Smith. The claimant might have prevailed in an
action against Smith had he named him as a party and presented evidence

134
that his corporation was not a legitimate business entity, but that was not the
case. Smith was not named as a party in the petition for benefits and he had
no notice that the court might order him to pay the claimant workers’
compensation benefits. Consequently, the order requiring him to pay the
claim does not meet the basic requirements of due process. * * *
[5] The claimant argues that Smith qualified as an employer under the
definition in section 440.02(16), Florida Statutes (2002), but the part of the
statute the claimant relies on is inapplicable here. The statute provides a
general definition of the term “employer” and then in the last sentence offers
a much broader definition that applies only in certain kinds of penalty
proceedings. This final sentence is the one that was used as a justification for
the order in this case:
If the employer is a corporation, parties in actual control of the
corporation, including, but not limited to, the president, officers who
exercise broad corporate powers, directors, and all shareholders who directly
or indirectly own a controlling interest in the corporation, are considered the
employer for the purposes of ss. 440.105 or 440.106.
§ 440.02(16), Fla. Stat. (2002) (emphasis added). It is apparent from this
language that an officer or shareholder can be treated as an “employer” only
in an action under section 440.105 or 440.106. These statutes establish
criminal and administrative remedies that may be pursued by the Division of
Insurance Fraud or by a state attorney. They do not authorize an injured
worker to recover workers’ compensation benefits from a shareholder or
director of a corporation in an ordinary petition for benefits like the one in
the present case.
The judge of compensation claims found that Timothy Smith had evaded
his legal obligations, that he had deceived the claimant into believing that he
had insurance, and that he had made various misrepresentations to the court.
All of that may be true, but the problem in this case is that Smith was not a
party to the action, and he was not given notice that his personal liability was
at issue in the case.
For these reasons we reverse the order to the extent that it imposes
liability against Timothy Smith in his individual capacity. The part of the
order that awards workers’ compensation benefits and other remedies
against Tower Contracting of Miami has not been challenged here and it
remains in force.
Reversed.

135
ALLEN and VAN NORTWICK, JJ., concur.

2. Standing

Crown Pontiac, Inc. v. Bell


547 So. 2d 290 (Fla. 2d DCA 1989).

ALTENBERND, Judge.
Crown Pontiac, Inc., appeals the trial court’s order which granted
Crown’s own motion for a new trial. Because Crown requested this order
and is not aggrieved by the order, we dismiss this appeal.
Randy and Maria Bell filed suit against Crown seeking rescission of a
used car sales contract, as well as compensatory and punitive damages. They
based their requests for relief upon breach of implied warranty and fraud.
The Bells purchased from Crown a used 1984 Trans Am. Crown represented
the car to the Bells as a “dealer’s demonstration” model which had been
driven only by Crown’s employees. The Bells alleged, however, that the car
actually had been stolen from Crown, driven several thousand miles,
recovered by the Pinellas County Sheriff’s Department, and refurbished by
Crown. The Bells experienced several problems with the car, which Crown
was unable to correct. The Bells further alleged that they would not have
purchased the car had they been informed about the car’s history.
During trial, Crown moved for a directed verdict arguing that the Bells
had failed to prove any damages. The trial court took this motion under
consideration and eventually denied it. The jury rejected the Bells’ breach of
warranty claim, but found in their favor on the fraud claim. The verdict
awarded the Bells $7,000 in compensatory damages and $100,000 in
punitive damages. After the verdict, Crown moved for a new trial on all
issues of fraud and renewed its motion for directed verdict. The trial court
granted the new trial and denied the renewed motion for directed verdict.
Thus, no final judgment has been entered by the trial court.
Initially we denied the Bells’ motion to dismiss without prejudice to their
raising this jurisdictional argument in their brief. Now that the case is fully
briefed and the record is before us, it is clear that Crown is merely
attempting to use this appeal as a vehicle to challenge the trial court’s
decision denying Crown’s motion for directed verdict.

136
This case is controlled by our reasoning in Hill v. American Medical
Affiliates, Inc., 387 So. 2d 1056 (Fla. 2d DCA 1980), review denied, 397 So.
2d 778 (Fla.1981). In Hill, we held that:
[T]he losing party may not appeal an order granting a new trial unless
that party claims that it was error to grant a new trial on any issue. Where, as
in the case before us, such error is not asserted, the order granting a new trial
may not serve as the vehicle for review by this court of prior rulings by the
trial judge adverse to that party.
We are not asked in this case simply to reverse a new trial order and to
reinstate part or all of the jury’s verdict. Instead, Crown asks that we reverse
the denial of a directed verdict. Crown is appealing an order, which it does
not want us to reverse.
The issue which Crown wishes to appeal must await a final judgment.
Such a final judgment cannot be entered until the conclusion of the new trial.
Crown may be correct in arguing that judicial efficiency would be
accomplished if it could appeal this issue before the second trial. On the
other hand, the resolution of the second trial may moot this issue and render
any appeal unnecessary. Since our jurisdiction is established by the Florida
Constitution and the rules of the Supreme Court of Florida, such policy
arguments can only be addressed in other forums.
Dismissed.
DANAHY, A.C.J., and LEHAN, J., concur.

Appellate review is generally limited to persons aggrieved by the order or judgment. See
King v. Brown, 55 So. 2d 187 (Fla. 1951).
An order is not reviewable if it is entirely favorable to the party seeking appellate review.
See Florida Com’n on Hurricane Loss Projection Methodology v. Dept of Ins., 716 So.
2d 345 (Fla. 1st DCA 1998).
A party is not aggrieved by and may not therefore appeal an order entered by consent.
See Sierra v. Public Health Trust of Dade County, 661 So. 2d 1296 (Fla. 3d DCA 1995).

137
B. Waiver – Acceptance of Benefits Rule

Carter v. Carter
141 So. 2d 591 (Fla. 1st DCA 1962).
WIGGINTON, Judge.
This cause is before the court on appellee’s motion to dismiss the appeal.
Appellee husband instituted suit for divorce against appellant who
answered, denying the allegations of the complaint and counterclaiming in
her own behalf. She prayed for a decree of divorce, custody of the children,
alimony and support money, an interest in the property accumulated by the
parties during their marriage, court costs and counsel fees. During the
pendency of the suit in the trial court, the parties entered into a written
agreement which recognized the divorce proceeding then in litigation, and
recited that they were desirous of settling between themselves the personal
and property rights growing out of their marriage relationship. Under the
terms of this contract it was agreed that appellant should be awarded custody
of the children; that appellee would pay to appellant a stated sum of money
as alimony and support for the children in the event a final decree of divorce
is entered; that appellee would convey to appellant certain described real and
personal property; that appellee would pay to appellant stipulated sums of
money on stated future dates and would procure and maintain certain
described insurance policies for the benefit of appellant and the children;
that appellee would provide a college education for each of the children and
would pay an agreed sum as attorney’s fees for the services rendered by
appellant’s attorney in the cause.
At the final hearing appellee offered testimony in support of the
allegations of his complaint, and although appellant was present and
represented by counsel, she offered no evidence in opposition to that
submitted by appellee, nor did she offer any evidence in support of her
counterclaim. The agreement mentioned above was received in evidence as
part of appellee’s proof. The chancellor entered a decree awarding appellee a
divorce, ratifying and confirming in all respects the agreement entered into
between the parties, and ordered appellee to promptly discharge the
obligations assumed by him under the terms of the agreement by transferring
and paying over to appellant the property, money and other benefits called
for therein.

138
Subsequent to the rendition of the final decree appellee fully executed the
agreement by complying in all respects with the duties and obligations
devolving upon him thereunder. All money, property and other benefits
accruing to appellant as contemplated by the agreement were received and
accepted by her. Appellant thereafter appealed and assigned as error that
provision of the final decree which awarded appellee a divorce, but has not
assigned as error any of the remaining provisions of the decree under which
she received substantial benefits.
Appellee has moved to dismiss the appeal on the ground that appellant
has accepted the benefits accruing to her under the final decree in
accordance with the terms of their agreement, and she is now estopped to
attack the validity of the provision of the decree which granted appellee a
divorce.
[1] It is a well established principle of law prevailing in this state that
where a party recovering a judgment or decree accepts the benefits thereof
voluntarily and knowing the facts, he is estopped to afterwards seek a
reversal of such judgment or decree on appeal. His conduct amounts to a
release of errors. His acceptance of payment or enforcement of the judgment
or decree is a waiver of error, and estops the successful party from
appealing. If under such circumstances an appeal were permitted which
resulted in a reversal of the judgment or decree subjected to review, there
would be no means by which the status quo of the parties could be restored,
and would give the prevailing party an inequitable and unjust advantage over
his unsuccessful adversary. The foregoing principle applies to appeals from
decrees awarding alimony or questioning the propriety of a divorce or other
provisions of the decree the same as it does to other civil suits, in the
absence of a contrary statute or court rule.
***
[2] It cannot be questioned but that the agreement entered into between
the parties to this appeal was in recognition of the divorce proceeding then
pending, and was in anticipation that a final decree of divorce would
ultimately be rendered. Appellant’s entitlement to the benefits accruing to
her under the agreement could rightfully be claimed by her only in the event
that the bonds of matrimony then existing between the parties were
dissolved by decree of the court. The provisions of the final decree which
required appellee to pay over and transfer to appellant the money, property
and other benefits enumerated in the agreement were directly related to and

139
dependent upon that provision of the decree which granted plaintiff a
divorce. The wife has now accepted all and singular the benefits to which
she is presently entitled under the decree, and the parties’ rights under the
contract have now been fully and finally adjudicated. Should we entertain
this appeal from only that provision of the decree which grants appellee a
divorce, and ultimately conclude that the decree is erroneous and should be
set aside, there would be no way in which the status quo of the parties could
be restored. Such a reversal would work an inequitable and unjust hardship
upon appellee, and would defeat rather than promote the ends of justice.
For the foregoing reasons the motion of appellee to dismiss the appeal is
granted, and an appropriate order will accordingly be entered.
CARROLL, DONALD K., C. J., and STURGIS, J., concur.

The acceptance of the benefits rule does not apply if: (1) the relief denied is separate and
severable form the relief granted; or (2) the appellant is entitled in any event to at least
the amount received. See Dance v. Tatum, 629 So. 2d 127 (Fla. 1993).

C. Joinder

Florida Rules of Appellate Procedure

Rule 9.360. Parties


(a) Joinder. A party to the cause in the lower tribunal who desires to join
in a proceeding as a petitioner or appellant shall serve a notice to that
effect no later than the latest of the following: (i) within 10 days of service
of a timely filed petition or notice of appeal; (ii) within the time prescribed
for filing a notice of appeal; or (iii) within the time prescribed in rule
9.100(c). The original and 1 copy of the notice of joinder, accompanied by
any filing fees prescribed by law, shall be filed either before service or
immediately thereafter in the same manner as the petition or notice of
appeal.
***

140
Premier Industries, Inc. v. Mead
595 So. 2d 122 (Fla. 1st DCA 1992).
PER CURIAM.
[1] This is an appeal to review a workers’ compensation order. Carol D.
Mead, the claimant and appellee in this appeal, has filed a suggestion to
strike the answer brief filed by Northbrook Property and Casualty Company,
one of the carriers for the employer. Claimant contends that, since
Northbrook’s position in the tribunal below and in its brief on appeal is
aligned with that of the appellants, Premier Industries, the employer, and
Sentry Claims Service, another carrier, and since Northbrook did not file a
notice of joinder in appellants’ appeal, the argument in its answer brief is
impermissible and should be struck. We agree and strike the brief.
The employee filed a claim for benefits against Premier and Sentry.
Sentry controverted the claim and filed a notice of controversy naming
Northbrook Property and Casualty as the carrier that provided coverage
when the injury allegedly occurred. Northbrook appeared below and was
joined in the proceeding by order dated January 3, 1991. Northbrook
participated in the taking of depositions and discovery and attended the
merits hearing where its attorney cross examined witnesses and offered
exhibits. The parties stipulated that the controversy between Sentry and
Northbrook would be continued until the judge had ruled in the case in chief
and that the controversy between carriers would not delay the client’s
benefits, as Sentry would pay all benefits awarded without prejudice to its
right to seek contribution and apportionment from Northbrook.
The order of the Judge of Compensation Claims, rendered on May 31,
1991, authorized medical care, determined entitlement to an attorney’s fee,
and directed that the employer/carrier pay temporary total disability benefits,
wage loss benefits, medical bills, and costs to the claimant. Premier
Industries and Sentry Claims filed a timely notice of appeal from this order.
Although served with a copy of the order, Northbrook did not file a notice of
appeal or cross appeal, see rule 9.110, Fla.R.App.P., and rule 4.160,
Fla.W.C.R.P., nor did Northbrook file a notice of joinder as an appellant as
prescribed in rule 9.360(a), Fla.R.App.P.
[2] After the record was filed, Premier and Sentry filed a 48-page initial
brief and claimant filed her answer brief in response thereto. Five days
before claimant filed her brief, Northbrook filed its 49-page “Answer Brief”

141
arguing in four points why the appealed order should be reversed. Claimant
then filed the instant suggestion to strike.
Northbrook has responded to the suggestion, stating that it was named in
the docketing statements as a real party in interest, had been furnished with a
copy of the record, and is an appellee in this proceeding. Northbrook argues
that unless allowed to appear and file the answer brief, it will have no
opportunity to be heard on the compensability issue. It requests in the
alternative, should the court conclude that Northbrook cannot file an answer
brief, that its brief be accepted as an amicus brief.
The claimant’s suggestion to strike is well taken for the following
reasons. Rule 9.020(f) defines the parties to an appellate proceeding. An
appellant is a party who seeks to invoke the appeal jurisdiction of a court.
Rule 9.020(f)(1). An appellee is defined as every party in the proceeding in
the lower tribunal other than an appellant. Rule 9.020(f)(2). The committee
notes to this rule explain that, “[t]he term ‘appellee’ has been defined to
include the parties against whom relief is sought and all others necessary to
the cause.” Rules 9.110 and 4.160 authorize a party aggrieved by a final
order to obtain appellate review thereof by filing a notice of appeal or cross
appeal. Rule 9.360(a) directs that “[a] party to the cause in the lower tribunal
who desires to join in a proceeding as a petitioner or appellant shall file a
notice to that effect within 10 days of service of the petition or notice or
within the time prescribed by Rule 9.110(b), whichever is later.” The evident
scheme and purpose of these rules, read in pari materia, is to align all parties
contesting an order on appeal as appellants and to align all parties supporting
or not contesting the order as appellees. These rule provisions are mandatory
and failure of a party to comply therewith constitutes a waiver of the right to
attack the validity of the order being reviewed on appeal.
[3] [4] Because Northbrook failed to invoke the appellate jurisdiction of
this court by filing a notice of appeal, notice of cross appeal, or notice of
joinder in the appeal by Premier and Sentry, it has remained an appellee and
is not authorized to use its status as such to argue positions as an aggrieved
party in derogation of the appealed order. To so permit would violate the
clear purpose of the cited provisions of the rules governing appellate practice
and would prejudice the claimant, who was unaware that Northbrook sought
to challenge the lower tribunal’s award until receiving its answer brief.
Under the rules, claimant is indisputably an appellee and is not authorized to
respond to another appellee’s answer brief. Clearly, claimant would be

142
prejudiced by permitting Northbrook to challenge the validity of the
appealed order in an answer brief to which claimant is not authorized to
respond.FN1 [ * * *]
[5] [6] [7] Northbrook’s request that it be treated as an amicus curiae
likewise is not well taken. An amicus curiae, although having a general
interest in a proceeding, is not bound by the decision of the court. See
generally, 39 Fla.Jur.2d, Parties § 4 (1982). An amicus curiae serves as
friend of the court to offer its views on a particular issue pending, but is not
directly affected by the outcome of a proceeding. It is clear that Northbrook
has a direct interest in the outcome of this appeal. In addition, the applicable
appellate rule states that “an amicus curiae brief shall be served within the
time prescribed for briefs of the party whose position is supported.” Rule
9.370, Fla.R.App.P. The time for filing the initial brief in support of
appellant is past.
The answer brief filed by Northbrook is ordered stricken. Northbrook’s
request for amicus curiae status in support of appellant is denied.
JOANOS, C.J., and ZEHMER and WOLF, JJ., concur.

D. Substitution

Florida Rules of Appellate Procedure

Rule 9.360. Parties


***
(c) Substitution of Parties.
(1) If substitution of a party is necessary for any reason, the court
may so order on its own motion or that of a party.
(2) Public officers as parties in their official capacities may be
described by their official titles rather than by name. Their successors in
office shall be automatically substituted as parties.
(3) If a party dies while a proceeding is pending and that party's
rights survive, the court may order the substitution of the proper party on
its own motion or that of any interested person.
(4) If a person entitled to file a notice dies before filing and that
person's rights survive, the notice may be filed by the personal

143
representative, attorney of record, or, if none, by any interested person.
Following filing, the proper party shall be substituted.

E. Intervention

Tallahassee Democrat, Inc. v. O’Grady


421 So. 2d 58 (Fla. 1st DCA 1982).
MILLS, Judge.
The petitioners ask us to reconsider our order dismissing this cause as
moot. We deny the motion for the reasons stated below.
[1] This case grows from the indictment of Christine Falling by the
Taylor County Grand Jury. Prior to the indictment, the respondent entered an
administrative “gag” order. This order was admittedly entered without due
process. The Tallahassee Democrat petitioned for a writ of certiorari to
review this gag order. In response to our show cause order, the respondent
quashed the gag order. Upon the filing of the quashing order with this court,
we dismissed the petition as moot. The Tampa Tribune sought intervenor
status as a petitioner. Intervention is not authorized at the appellate level;
instead a petition for a writ of certiorari coupled with a motion to consolidate
is appropriate. In spite of this, we have considered the Tribune’s petitions in
reaching our decision. The respondent judge then, without a hearing or
notice to the newspaper, entered the following order:
[M]embers of the Grand Jury and witnesses who testified before the
Grand Jury on matters concerning any alleged criminal activity upon the part
of Christine Falling, are hereby restrained and enjoined from discussing or
attempting to discuss with any person, in any manner whatsoever, any
testimony of a witness examined before the Grand Jury, except the witnesses
may disclose any testimony to the State Attorney and his staff and to the
defendant, her attorney and her attorney’s staff; said injunction to remain in
full force and effect until this court is convinced that such disclosure will not
interfere with the administration of justice.
This order is to be read with conjunction with Section 905.27 of the
Florida Statutes. A violation of this order or of Section 905.27, Florida
Statute will constitute criminal contempt of court.

144
The newspapers then petitioned for reconsideration of our dismissal.
They argue that the issues raised by them have not been resolved and that
they are still under a gag order. We disagree.
[2] When review of the second order became desirable, the better practice
would have been to file a second petition for a writ of certiorari, not attempt
to reopen a case seeking review of a quashed order. Nevertheless, we have
considered the petitions on their merits.
[3] The second order does not appear to go any further than Section
905.27, Florida Statutes (1981). No conduct is illegal under this order that
would not be also illegal under Section 905.27. The respondent has assured
us that:
[T]he new Order is but a public announcement of the restriction placed
upon the members of the grand jury and the witnesses from speaking about
the testimony and merely reflects the intent and purpose of § 905.27 Florida
Statutes.
We conclude that the newspapers are not “gagged”; nor is their access to
news sources in any way limited by the respondent’s second order; nor is
their ability to gather and publish news impaired or curtailed by the order,
except as limited by Section 905.27. We therefore deny the motions for
reconsideration.
ERVIN and WIGGINTON, JJ., concur.

VII. Stays

Florida Rules of Appellate Procedure

Rule 9.310. Stay Pending Review


(a) Application. Except as provided by general law and in subdivision (b)
of this rule, a party seeking to stay a final or non-final order pending
review shall file a motion in the lower tribunal, which shall have continuing
jurisdiction, in its discretion, to grant, modify, or deny such relief. A stay
pending review may be conditioned on the posting of a good and sufficient
bond, other conditions, or both.
(b) Exceptions.
(1) Money Judgments.If the order is a judgment solely for the

145
payment of money, a party may obtain an automatic stay of execution
pending review, without the necessity of a motion or order, by posting a
good and sufficient bond equal to the principal amount of the judgment
plus twice the statutory rate of interest on judgments on the total amount
on which the party has an obligation to pay interest. Multiple parties
having common liability may file a single bond satisfying the above criteria.
(2) Public Bodies; Public Officers.The timely filing of a notice shall
automatically operate as a stay pending review, except in criminal cases,
in administrative actions under the Administrative Procedure Act, or as
otherwise provided by chapter 120, Florida Statutes, when the state, any
public officer in an official capacity, board, commission, or other public
body seeks review; provided that an automatic stay shall exist for 48 hours
after the filing of the notice of appeal for public records and public meeting
cases. On motion, the lower tribunal or the court may extend a stay,
impose any lawful conditions, or vacate the stay.
(c) Bond.
(1) Defined.A good and sufficient bond is a bond with a principal
and a surety company authorized to do business in the State of Florida, or
cash deposited in the circuit court clerk's office. The lower tribunal shall
have continuing jurisdiction to determine the actual sufficiency of any such
bond.
(2) Conditions.The conditions of a bond shall include a condition to
pay or comply with the order in full, including costs; interest; fees; and
damages for delay, use, detention, and depreciation of property, if the
review is dismissed or order affirmed; and may include such other
conditions as may be required by the lower tribunal.
(d) Judgment Against a Surety. A surety on a bond conditioning a stay
submits to the jurisdiction of the lower tribunal and the court. The liability
of the surety on such bond may be enforced by the lower tribunal or the
court, after motion and notice, without the necessity of an independent
action.
(e) Duration. A stay entered by a lower tribunal shall remain in effect
during the pendency of all review proceedings in Florida courts until a
mandate issues, or unless otherwise modified or vacated.
(f) Review. Review of orders entered by lower tribunals under this rule
shall be by the court on motion.

146
A. General Procedure

A stay prevents enforcement but it does not “undo or set aside what the trial court has
adjudicated.” See City of Plant City v. Mann, 400 So. 2d 952 (Fla. 1981).

Alexander v. Adams
501 So. 2d 15 (Fla. 4th DCA 1986)
HERSEY, Chief Judge.
We grant certiorari and quash the order of the circuit court dismissing
petitioner’s appeal from an order of the county court. Respondent, Adams,
filed suit in county court to evict petitioner, his tenant, for failing to pay rent.
Petitioner interposed various defenses and prayed for judgment in her favor,
attorney’s fees, costs and “such other relief as the Court deems proper.”
Ultimately, a default was entered against petitioner for failure to comply
with section 83.60(2), Florida Statutes (1985). She was evicted and the
premises rerented.
Petitioner’s appeal in the circuit court was dismissed on the basis that, no
stay having been procured and the premises having been rerented as a
consequence, the appeal was moot.
[1] [2] It is well established that a party is not required to obtain a stay in
order to appeal an adverse judgment. Ronette Communications Corp. v.
Lopez, 475 So. 2d 1360 (Fla. 5th DCA 1985); See also Green v. Green, 254
So. 2d 802 (Fla. 3d DCA 1971), writ discharged, 264 So. 2d 838 (Fla.1972).
While the remedy initially sought by petitioner is apparently no longer
available, her demand for “other relief” makes a money judgment for
damages a form of alternative relief that would be available should she
prevail on the merits.
Finding that the order under review constituted a departure from the
essential requirements of law for which there is no adequate relief upon
plenary appeal, we grant the petition, issue the writ and quash the order.
CERTIORARI GRANTED; ORDER QUASHED.
ANSTEAD and GLICKSTEIN, JJ., concur.

147
Palm Beach Heights Dev. v. Decillis
385 So. 2d 1170 (Fla. 3d DCA 1980)

Before HENDRY, NESBITT and DANIEL S. PEARSON, JJ.


PER CURIAM.
Anthony and Mary Decillis were awarded a money judgment in the
amount of $6,128 against Palm Beach Heights Development and Sales
Corporation (PBH). PBH appealed. Five days later the Decillises, pursuant
to Rule 9.310, Florida Rules of Appellate Procedure, moved the lower court
to enter an order requiring PBH to post an appeal bond. The lower court
entered an Order Requiring Supersedeas Bond, which provided, inter alia:
“ORDERED AND ADJUDGED the Defendant (PBH) shall post a good
and sufficient bond to indemnify Plaintiffs for their costs, interest and fees as
well as the amount of the Final Judgment heretofore entered by this Court, if
the appeal of the Defendant/Appellant is dismissed or the Order appealed
from is affirmed, in the amount of $7,216.00.”
PBH sought and we granted review of the aforesaid order. On June 30,
1980, we vacated the lower court’s Order Requiring Supersedeas Bond
“without prejudice to the appellant’s right to procure stay pending review at
any time throughout the pendency of the appeal by complying with the
provisions of Florida Rule of Appellate Procedure 9.310(b)(1).” At PBH’s
request, we now clarify that order.
We vacated the lower court’s order for the reason that it required the
appellant to post a bond pending appeal. A trial court cannot impose such a
requirement upon an appellant. A party can appeal a money judgment
without posting a supersedeas bond, and the opposing party has the
concomitant right to seek execution upon the judgment. Fitzgerald v.
Addison, 287 So. 2d 151 (Fla.2d DCA 1973). On the other hand, should the
appellant desire a stay pending our review, such a stay is automatic, without
the necessity of a motion or order, on its posting of a good and sufficient
bond equal to the amount of the judgment, plus fifteen per cent thereof.
Fla.R.App.P. 9.310(b).
PBH asserts that it has a right to obtain a stay under Rule 9.310(a)
without posting a bond or by posting less of a bond than Rule 9.310(b)
requires, and that our order is too limiting. We disagree. PBH is entitled to a
stay of the final judgment only by the posting of the bond in the amount set

148
forth in Rule 9.310(b), and the trial court is not empowered to deprive the
Decillises of their right to execute on the judgment by ordering any lesser
bond or otherwise setting less onerous conditions. Barnett v. Barnett Bank of
Jacksonville, N.A., 338 So. 2d 888 (Fla.1st DCA 1976); Jenkins Trucking,
Inc. v. Emmons, 207 So. 2d 280 (Fla.3d DCA 1968).
Our order of June 30, 1980, as clarified, stands.

Superior Garlic Int’l v. E & A Produce Corp.


934 So. 2d 484 (Fla. 3d DCA 2004)

Before GREEN, RAMIREZ, and SHEPHERD, JJ.


PER CURIAM.
Petitioners seek a writ of mandamus to compel the circuit court below to
disburse monies held as a cash supersedeas bond securing a final judgment
that awarded attorneys fees to petitioners. The award has been affirmed on
appeal by this court and a mandate has issued. We have jurisdiction. Art. V,
§ 4(b)(3), Fla. Const.; Fla. R.App. Proc. 9.030(b)(3) (2004). We grant the
writ.
I. Procedural Posture
E & A Produce Corporation (“E & A”) filed a multi-count complaint
against Superior Garlic International, Inc. and its two shareholders, Silfredo
Trujillo and Nilda Olmo, that included a treble damage claim for civil theft
pursuant to § 772.11, Fla. Stat. (2000). Following the abandonment of the
claim by E & A the trial court entered an award of attorneys’ fees against E
& A and in favor of Superior Garlic, Trujillo and Olmo in the sum of
$9,123.75.FN1 E & A superseded the judgment by filing a cash supersedeas
bond with the clerk of the Circuit Court. This court subsequently affirmed
the fee award and a mandate has issued. E & A Produce Corp. v. Superior
Garlic Int'l, Inc., 864 So. 2d 449 (Fla. 3d DCA 2003). However, E & A has
refused to satisfy the judgment.
FN1. As we noted in an earlier appeal of the fee award, it is unclear
whether the fee award was made pursuant to § 772.11(a), Fla. Stat. or §
57.105, Fla. Stat. E & A Produce Corp. v. Superior Garlic Int'l, Inc., 864 So.

149
2d 449, 450 (Fla. 3d DCA 2003). However, that is immaterial for our
purposes.
After E & A refused to satisfy the judgment, Superior Garlic, Trujillo and
Olmo filed a motion in the trial court to have the judgment satisfied from the
supersedeas bond. However, before the motion could be heard, a separate
judgment was entered, apparently severally but not jointly, in favor of E & A
and against Superior Garlic and Trujillo FN2 in the sum of $24,000 each on
the remaining counts. A successor trial judge denied disbursement pending
post-trial proceedings relating to the recently rendered final judgment and
collectability thereon. Superior Garlic, Trujillo and Olmo ask this court to
issue its writ of mandamus to nevertheless compel disbursement of the
supersedeas bond.
FN2. Olmo had been successful in removing herself from the controversy
on a summary judgment motion. See E & A Produce Corp. v. Olmo, 864 So.
2d 447 (Fla. 3d DCA 2003).
II. Discussion
[1] Mandamus is appropriate where a petitioner “demonstrate[s] a clear
legal right to the performance of a ministerial duty by [a] respondent FN3 and
that no other adequate remedy exists.” Morse Diesel Int’l, Inc. v.2000 Island
Blvd., Inc., 698 So. 2d 309 (Fla. 3d DCA 1997). See also Chavis v. Dugger,
538 So. 2d 120 (Fla. 1st DCA 1989) (“Mandamus is available to enforce an
established right, but not to establish that right.”)
FN3. The successor trial judge, Judge Robert Pineiro, although not
included as a respondent in the caption of the petition, is a “formal party to
the petition for mandamus.” Fla. R.App. P. 9.100(e).
[2] [3] “The mandate of an appellate court is a final judgment in the
cause, and compliance [therewith] is a ministerial act to be performed by the
trial court.” Nicholson v. Ariko, 565 So. 2d 843 (Fla. 5th DCA 1990). Upon
issuance of a mandate, an automatic stay resulting from the posting of a
supersedeas bond is automatically lifted, and the judgment creditor has the
right to either execute on the judgment or proceed against the supersedeas
bond. See Fla. R.App. P. 9.310(e) (“A stay entered by a lower tribunal shall
remain in effect ... until the mandate issues.”). See also Freedom Insurors v.
M.D. Moody & Sons, Inc., 869 So. 2d 1283, 1285 (Fla. 4th DCA 2004)(A
supersedeas “does not undo or set aside what the trial court has adjudicated,
but simply stays further proceedings in relation to the judgment being

150
appealed from until the appellate court acts upon the decision in the lower
court”) (citing Bacon v. Green, 36 Fla. 313, 18 So. 866, 869 (1894)).
[4] [5] “It is the duty of the trial court to enforce the mandate and not
stray from it.” Nicholson, 565 So. 2d 843 at 844. Although not
unsympathetic to the trial court’s apparent concern that E & A may be
obligated to satisfy one judgment while being potentially on the verge of
finality on a larger one in its favor, disbursement of the supersedeas was
required upon issuance of the mandate. Id. Moreover, on the facts of this
case, E & A would not be entitled to a set-off of the attorneys’ fees judgment
suffered by it against the later judgment obtained by E & A against Superior
Garlic and Trujillo in any event. See generally Florida Gas Transmission
Co. v. Lauderdale Sand & Fill, Co., 813 So. 2d 1013, 1015 (Fla. 1st DCA
2002) (noting that “it is proper to include as setoffs amounts recovered by
the plaintiff in settlements with other defendants when the suits against all of
the defendants involve the same claims ... When the plaintiff has separate
and distinct claims against more than one defendant, however, a setoff is
inappropriate”).
We grant the writ of mandamus with directions that petitioners be
allowed to satisfy the attorneys’ fee award from the proceeds of the cash
supersedeas.

B. Supersedeas Bonds

Florida Rules of Appellate Procedure

Rule 9.310. Stay Pending Review


(a) Application. Except as provided by general law and in subdivision (b)
of this rule, a party seeking to stay a final or non-final order pending
review shall file a motion in the lower tribunal, which shall have continuing
jurisdiction, in its discretion, to grant, modify, or deny such relief. A stay
pending review may be conditioned on the posting of a good and sufficient
bond, other conditions, or both.
***
(c) Bond.
(1) Defined.A good and sufficient bond is a bond with a principal
and a surety company authorized to do business in the State of Florida, or

151
cash deposited in the circuit court clerk's office. The lower tribunal shall
have continuing jurisdiction to determine the actual sufficiency of any such
bond.
(2) Conditions.The conditions of a bond shall include a condition to
pay or comply with the order in full, including costs; interest; fees; and
damages for delay, use, detention, and depreciation of property, if the
review is dismissed or order affirmed; and may include such other
conditions as may be required by the lower tribunal.
***

C. Money Judgments

Florida Rules of Appellate Procedure

Rule 9.310. Stay Pending Review


***
(b) Exceptions.
(1) Money Judgments.If the order is a judgment solely for the payment of
money, a party may obtain an automatic stay of execution pending review,
without the necessity of a motion or order, by posting a good and sufficient
bond equal to the principal amount of the judgment plus twice the statutory
rate of interest on judgments on the total amount on which the party has
an obligation to pay interest. Multiple parties having common liability may
file a single bond satisfying the above criteria.
***

Campbell v. Jones
648 So. 2d 208 (Fla. 3d DCA 1994).
Before HUBBART, JORGENSON and GERSTEN, JJ.
PER CURIAM.
Luther Campbell filed a Notice of Appeal from a money judgment in
excess of $1.5 million; thereafter, he filed a motion in the trial court to stay
execution of that judgment pending appeal. Relying on Fla.R.Civ.P.
1.550(b), Campbell argued that he was not able to procure a supersedeas
bond, and that unless execution was stayed pending appeal, appellants’
business would be “dismembered” through execution. The trial court granted

152
a stay that was not conditioned upon the posting of any bond; the stay was to
be in effect for three months. In so doing, the trial court erred.
[1] [2] Under Fla.R.App.P. 9.310(b)(1), an appellant is not required to
post a supersedeas bond in order to perfect an appeal from a money
judgment. Palm Beach Heights Dev. & Sales Corp. v. Decillis, 385 So. 2d
1170, 1171 (Fla. 3d DCA 1980). However, if an appellant seeks to stay
execution of judgment pending the appeal, he may obtain a stay “only by the
posting of the bond in the amount set forth in Rule 9.310(b).” Id. See also
First Dev. Co. v. Bemaor, 449 So. 2d 290, 291 (Fla. 3d DCA 1983)
(appellant entitled to stay of judgment for attorney’s fees and costs only by
posting good and sufficient bond as required by Fla.R.App.P. 9.310(b)(1)).
Cf. Avirgan v. Hull, 125 F.R.D. 185 (S.D.Fla.1989) (Fed.R.Civ.P. 62(d)
provides that court can issue stay pending appeal only when judgment debtor
posts supersedeas bond; claim that posting bond would render defendant
insolvent not sufficient ground for granting stay without bond), aff'd, 932
F.2d 1572 (11th Cir.1991), cert. denied, 502 U.S. 1048, 112 S.Ct. 913, 116
L.Ed.2d 813 (1992).
Motion granted; stay vacated.

D. Review by Public Parties

Florida Rules of Appellate Procedure

Rule 9.310. Stay Pending Review


***
(b) Exceptions.
***
(2) Public Bodies; Public Officers.The timely filing of a notice shall
automatically operate as a stay pending review, except in criminal cases,
in administrative actions under the Administrative Procedure Act, or as
otherwise provided by chapter 120, Florida Statutes, when the state, any
public officer in an official capacity, board, commission, or other public
body seeks review; provided that an automatic stay shall exist for 48 hours
after the filing of the notice of appeal for public records and public meeting
cases. On motion, the lower tribunal or the court may extend a stay,
impose any lawful conditions, or vacate the stay. * * *

153
Mitchell v. State
911 So. 2d 1211 (Fla. 2005).
ANSTEAD, J.
We have for review a decision of a district court of appeal on the
following question, which the court certified to be of great public
importance:
WHETHER THE STATE IS ENTITLED TO THE BENEFIT OF THE
AUTOMATIC STAY PROVISION OF RULE 9.310(b)(2) ON APPEAL IN
A CIVIL COMMITMENT PROCEEDING BROUGHT PURSUANT TO
PART V OF CHAPTER 394, FLORIDA STATUTES, WHEN THE TRIAL
COURT HAS DISMISSED THE PETITION SEEKING COMMITMENT.
State v. Mitchell, 848 So. 2d 1209, 1211 (Fla. 1st DCA 2003). We have
jurisdiction. See art. V, § 3(b)(4), Fla. Const. We answer the question in the
affirmative and hold that the State is entitled to the benefit of the automatic
stay provision of Florida Rule of Appellate Procedure 9.310(b)(2) on appeal
in a civil commitment proceeding brought pursuant to part V of chapter 394,
Florida Statutes, when the trial court has dismissed the petition seeking
commitment. However, as we explain below, the respondent has the right to
seek a dissolution of the stay pursuant to our rules.
FACTS AND PROCEDURAL HISTORY
The State filed a petition to civilly commit Frank Mitchell as a sexually
violent predator pursuant to the Jimmy Ryce Act, sections 394.910-394.931,
Florida Statutes (2000) (entitled “Involuntary Civil Commitment of Sexually
Violent Predators”). Mitchell, 848 So. 2d at 1210. However, the record
reflects that the State did not file the petition for commitment until the day
before Mitchell was scheduled to be released from his incarceration pursuant
to a criminal conviction and sentence for possession of cocaine and
felonious possession of a concealed weapon. Id. Subsequent to the filing of
the petition, the trial court found that there was probable cause to believe
that Mitchell was a sexually violent predator. Mitchell then sought a
dismissal of the proceedings and the trial court entered an order dismissing
the petition for lack of jurisdiction, concluding that the Act did not permit
the State to file a petition for civil commitment against anyone not
incarcerated for a sexual offense. Id.
The State appealed to the First District Court of Appeal and gave notice
that it would rely on the automatic stay provision of rule 9.310(b)(2). Id.

154
Mitchell filed a motion with the trial court to vacate the automatic stay under
rule 9.310(b)(2). Id. However, the trial court denied the motion, stating that
the issue was one of first impression in Florida and determining that it was
unable to find that the State had no likelihood of success on appeal. Id.
Mitchell then moved the appellate court to dissolve the stay. Id. at 1209.
The First District concluded that because sexually violent predator
commitment proceedings are civil and because rule 9.310(b)(2) applies in
civil cases, the automatic stay provision was applicable:
The statute and the case law construing the Jimmy Ryce Act make it
clear that the commitment proceedings under the Jimmy Ryce Act are civil
in nature. Moreover, section 394.9155(1), Florida Statutes (2000), provides
that the Florida Rules of Civil Procedure apply to all civil commitment
proceedings for sexually violent predators unless otherwise specified. It has
also been observed that automatic stays are available only in civil cases and
that “[s]ubject to the exception for criminal cases, rule 9.310(b)(2) makes no
distinction as to the type of case in which a public party is entitled to an
automatic stay.” Although, the automatic stay provision has been held
inapplicable to discretionary review proceedings, in the instant case, the
State has the right to appeal the final order of dismissal. Accordingly,
because the commitment of sexually violent predators has been held to be
civil in nature and because rule 9.310(b)(2) applies to civil cases, we
conclude that rule 9.310(b)(2) is applicable to this appeal.
Id. at 1210 (citations omitted). After noting that the trial court had made a
finding of probable cause as to Mitchell’s status as a sexually violent
predator, the district court denied the motion to dissolve the stay. Id. at 1211.
In dissent, Judge Padovano expressed the view that the stay provisions of
the rule were never intended to apply to cases where liberty interests were at
stake, and the liberty interests here, being constitutionally protected, should
not be impaired by the mere operation of an automatic stay provision. Id. at
1212-13 (Padovano, J., dissenting). He also cited financial interests as being
the primary basis upon which the rule was originally enacted to permit
obviously solvent governmental entities to maintain the status quo while
their appeals were pending. Id. at 1213.
Thereafter, the First District decided the merits of the appeal, finding that
the Jimmy Ryce Act was not limited to persons currently serving sentences
for sexually violent offenses, but could also be applied to those, like
Mitchell, whose current prison sentences were for non-sexually related

155
crimes, but who had been previously convicted of a sexual offense. State v.
Mitchell, 866 So. 2d 776, 777 (Fla. 1st DCA 2004), review denied, No.
SC04-368, 907 So. 2d 1171, 2005 WL 1699507 (Fla. July 6, 2005). Mitchell
sought review of that decision in this Court and we stayed the review
proceedings pending resolution of the same issue in Hale v. State, 891 So.
2d 517 (Fla.2004), petition for cert. filed (U.S. Mar. 10, 2005) (No. 04-
9054). We subsequently decided in Hale that the Jimmy Ryce Act applies to
“all persons who are currently incarcerated and who at some point in the past
have been convicted of a sexually violent offense.” Hale, 891 So. 2d at 522.
Mitchell acknowledges that Hale controls the issue as to whether his prior
conviction subjects him to the Jimmy Ryce Act.
However, we now consider Mitchell’s petition for review based upon the
district court’s certification as to the question of whether the State is entitled
to the benefit of the automatic stay provision of rule 9.310(b)(2) on appeal
from the dismissal of proceedings brought pursuant to the Jimmy Ryce Act.
ANALYSIS
[1] [2] [3] [4] “The same principles of construction apply to court rules
as apply to statutes.” Gervais v. City of Melbourne, 890 So. 2d 412, 414
(Fla. 5th DCA 2004) (citing Rowe v. State, 394 So. 2d 1059 (Fla. 1st DCA
1981)). If the language of a statute or rule is plain and unambiguous, it must
be enforced according to its plain meaning. See Fla. Dept. of Revenue v. Fla.
Mun. Power Agency, 789 So. 2d 320, 323 (Fla.2001). Legislative history is
not needed to determine intent when the language is clear. Goldenberg v.
Sawczak, 791 So. 2d 1078, 1083 (Fla.2001).
Consistent with Judge Padovano’s concerns in dissent, Mitchell asserts
that the automatic stay provisions of rule 9.310(b)(2) should not apply
because his very liberty is at stake, and the rule was never meant to apply to
civil commitments, but only to judgments or orders affecting the financial
interests of the State or another governmental body. Mitchell also cites the
decision in State v. Ducharme, 881 So. 2d 70 (Fla. 5th DCA 2004), review
dismissed, 895 So. 2d 405 (Fla.2005), and review denied, No. SC05-290,
908 So. 2d 1057 (Fla. July 8, 2005), wherein the Fifth District expressed
agreement with Judge Padovano’s dissent and his view “that confinement
under the Jimmy Ryce Act must rest on something more substantial than the
effect of a mechanical rule that goes into effect by the unilateral action of the
State.” Id. at 71 (quoting Mitchell, 848 So. 2d at 1212 (Padovano, J.,
dissenting)).

156
However, we conclude that simply because Jimmy Ryce Act proceedings
may not have been specifically contemplated or envisioned by this Court at
the time it adopted the rule, this does not necessarily mean that the automatic
stay provisions of the rule should not be applied. Rather, it is necessary to
look at the plain language and purpose of the rule, which is to initially
maintain the status quo pending the outcome of an appeal by the
government, except in criminal proceedings. In addition, we conclude that
the liberty interests and due process rights of the respondent are adequately
protected when the provisions for dissolution of the automatic stay are
considered.
We need only look to the plain language of the rule and statute to
determine that the provisions of rule 9.310(b)(2) and the language of the
Jimmy Ryce Act are clear as to the issue before us. In this regard, we agree
with the analysis of the First District:
Rule 9.310(b)(2) provides that “[t]he timely filing of a notice shall
automatically operate as a stay pending review, except in criminal cases,
when the state, any public officer in an official capacity, board, commission,
or other public body seeks review....” The statute and the case law
construing the Jimmy Ryce Act make it clear that the commitment
proceedings under the Jimmy Ryce Act are civil in nature. See, e.g.,
Westerheide v. State, 767 So. 2d 637, 648 (Fla. 5th DCA 2000) (holding that
the Jimmy Ryce Act is civil in nature and that confinement is for treatment
and the protection of the public, not punishment), approved by 831 So. 2d 93
(Fla.2002). Moreover, section 394.9155(1), Florida Statutes (2000), provides
that the Florida Rules of Civil Procedure apply to all civil commitment
proceedings for sexually violent predators unless otherwise specified. It has
also been observed that automatic stays are available only in civil cases and
that “[s]ubject to the exception for criminal cases, rule 9.310(b)(2) makes no
distinction as to the type of case in which a public party is entitled to an
automatic stay.” See Philip J. Padovano, Florida Appellate Practice § 12.5 at
184 (2003 ed.). Although, the automatic stay provision has been held
inapplicable to discretionary review proceedings, see, e.g., State,
Department of Health and Rehabilitative Services v. E.D.S. Federal
Corporation, 622 So. 2d 90 (Fla. 1st DCA 1993) and City of Miami v.
Arostegui, 616 So. 2d 1117 (Fla. 1st DCA 1993), in the instant case, the
State has the right to appeal the final order of dismissal. Accordingly,
because the commitment of sexually violent predators has been held to be

157
civil in nature and because rule 9.310(b)(2) applies to civil cases, we
conclude that rule 9.310(b)(2) is applicable to this appeal.
Mitchell, 848 So. 2d at 1210. We concur in this straightforward analysis,
although we also agree with many of the concerns expressed by Judge
Padovano.
[5] Initially, we note that it is now settled law that the statutes authorizing
civil commitment of sexually violent predators ( i.e., the Jimmy Ryce Act),
are civil. Kansas v. Hendricks, 521 U.S. 346, 365-66, 117 S.Ct. 2072, 138
L.Ed.2d 501 (1997); Westerheide v. State, 831 So. 2d 93, 100 (Fla.2002)
(stating that the Jimmy Ryce Act “was clearly intended to create a civil
commitment scheme” for those who are determined to be sexually violent
predators under the Act). Moreover, section 394.9155(1), Florida Statutes
(2000), states, “In all civil commitment proceedings for sexually violent
predators under this part, the following shall apply: (1) The Florida Rules of
Civil Procedure apply unless otherwise specified in this part.” Rule
9.310(b)(2) states:
Public Bodies; Public Officers: The timely filing of a notice shall
automatically operate as a stay pending review, except in criminal cases,
when the state, any public officer in an official capacity, board, commission,
or other public body seeks review; provided that an automatic stay shall
exist for 48 hours after the filing of the notice of appeal for public records
and public meeting cases. On motion, the lower tribunal or the court may
extend a stay, impose any lawful conditions, or vacate the stay.
Finally, rule 9.310(b)(2) does not expressly exempt civil commitment
proceedings or Jimmy Ryce Act cases, although, significantly, it does
exempt criminal cases. Therefore, because the Jimmy Ryce Act is civil, and
there is no exemption, we conclude rule 9.310(b)(2) and its automatic stay
provisions are applicable.
[6] However, when applying the automatic stay provisions to the Jimmy
Ryce Act, courts must remain mindful of the due process concerns when a
liberty interest is involved. As noted by Judge Padovano and as we said in
Goode, “[c]ivil commitment proceedings involve a serious deprivation of
liberty and, thus, such proceedings must comply with the due process
clauses of the Florida and United States Constitutions.” State v. Goode, 830
So. 2d 817, 825-26 (Fla.2002) (citing Addington v. Texas, 441 U.S. 418,
425, 99 S.Ct. 1804, 60 L.Ed.2d 323 (1979)).

158
Moreover, we do not disagree with the concerns of Judge Padovano and
the Fifth District that the continuing confinement of a respondent in Jimmy
Ryce proceedings merits special consideration in any evaluation of the
justification for a continuing stay once the respondent has prevailed in the
commitment proceedings. Indeed, the exemption of criminal proceedings
from the operation of the automatic stay proceedings was no doubt based
upon the obvious and important liberty interests involved in criminal
proceedings.FN1 The liberty interests affected in Ryce Act proceedings are
not dissimilar to the liberty interests involved in criminal proceedings,
although there are obviously critical distinctions.
FN1. We do not have before us, for example, a stay in favor of the State
where a jury has determined that the State has not proved its case on the
merits in a Ryce Act proceeding.
[7] Importantly, however, while rule 9.310(b)(2) is effectively automatic
in its initial application, a stay entered pursuant to the rule may be dissolved.
We believe that the provisions for the dissolution of the stay, where the
special circumstances of Ryce Act proceedings and the circumstances of
respondents’ continuing confinement can and should be considered, are
sufficient to alleviate the constitutional concerns obviously present in such a
scenario. We also believe Florida’s trial courts are capable of the sensitive
balancing required in such cases.
[8] In fact, of course, as noted above, the rule itself recognizes these due
process concerns by exempting criminal proceedings entirely from the
automatic stay provisions. This exemption is directly related to the serious
constitutional issues that would be implicated if the State were to be
automatically entitled to stay the effect of a decision adverse to the State in a
criminal proceeding. In a like manner, when liberty interests are directly at
stake in a proceeding under the Ryce Act, the trial court must take special
care to assess these interests along with the interests of the State in
determining whether detention should be continued. These interests were
specifically identified and addressed by Judge Padovano in his dissent:
I do not think that the automatic stay provision in rule 9.310(b)(2) can be
applied in an appeal from an order dismissing a petition for involuntary
commitment under the Jimmy Ryce Act, because the effect of the stay in
such a case would be to detain a person in custody without due process of
law. For this reason, I am unable to join in the majority’s decision to keep
the stay in place in the present case.

159
The power to detain an individual for involuntary civil commitment is
subject to certain well-defined constitutional limitations. As the Supreme
Court explained in Foucha v. Louisiana, 504 U.S. 71, 80, 112 S.Ct. 1780,
118 L.Ed.2d 437 (1992), the “freedom from bodily restraint has always been
at the core of the liberty protected by the Due Process Clause.” This view is
reflected in other Supreme Court opinions, as well. Earlier, the Court said,
“[i]t is clear that commitment for any purpose constitutes a significant
deprivation of liberty that requires due process protection.” Jones v. United
States, 463 U.S. 354, 361, 103 S.Ct. 3043, 77 L.Ed.2d 694 (1983) (quoting
Addington v. Texas, 441 U.S. 418, 425, 99 S.Ct. 1804, 60 L.Ed.2d 323
(1979)). These statements plainly illustrate that we are dealing here not only
with the applicability of a procedural rule, but also with an important issue
of constitutional law.
Statutes authorizing civil commitment of sexually violent predators have
been upheld primarily because they require a judicial finding that a person
suffers from a condition that makes him dangerous to others. For example,
in Kansas v. Hendricks, 521 U.S. 346, 357-358, 117 S.Ct. 2072, 138
L.Ed.2d 501 (1997), the Court concluded that the statute at issue did not
offend the Due Process Clause, because it “requires evidence of past
sexually violent behavior and a present mental condition that creates a
likelihood of such conduct in the future if the person is not incapacitated.”
The Florida Supreme Court has rejected a due process challenge to the
Jimmy Ryce Act for the same reason. See Westerheide v. State, 831 So. 2d
93, 106 (Fla.2002). The constitutional objections to commitment can be
overcome, but only if the government complies with the applicable
procedures and evidentiary standards, and if the court makes the findings of
dangerousness required by the statute.
In the present case there is no judicial finding that could be used to
support the appellee’s continued detention. The appellee has completed the
sentence he was serving when the petition for involuntary commitment was
filed. His detention may have been justified at one time by the trial court’s
finding of probable cause in the civil commitment proceeding, but that
finding has now been undermined by the court’s final order dismissing the
case. The appellee is now being held in custody solely on the authority of the
stay imposed by rule 9.310(b)(2).
This stay is not based on a finding of dangerousness, or upon any judicial
finding for that matter. Rather, the stay has been imposed automatically by

160
operation of law, merely because the state has chosen to appeal the dismissal
of its petition. I believe that confinement under the Jimmy Ryce Act must
rest on something much more substantial than the effect of a mechanical rule
that goes into effect by the unilateral action of the state.
The majority takes the view that the automatic stay effectively restores
the trial court’s earlier finding of probable cause. That is a logical way to
construe the effect of the rule, but it does not entirely resolve the
constitutional problem presented by this case. Section 394.916(1), Florida
Statutes (2002), provides that “[w]ithin thirty days after the determination of
probable cause, the court shall conduct a trial to determine whether the
person is a sexually violent predator.” The imposition of the automatic stay
would extend this time period indefinitely. Although the court has expedited
the appeal, that is not an adequate remedy for a person who is detained
without authority. This is a plenary appeal from a final order. Despite our
best efforts to expedite the decision, we could not ensure that the appellee
will not be held beyond the thirty-day time limit set by section 394.916(1).
The statutory time period for which a person can be held in custody
pending trial on a petition for involuntary commitment is not a mere
procedural requirement. Except in the limited circumstances listed in the
statute itself, the time period cannot be altered or extended. In State v.
Goode, 830 So. 2d 817, 823 (Fla.2002), the Florida Supreme Court held that
the thirty-day time limit in section 394.916(1) is mandatory. The court
explained that “based on the importance of the obvious liberty rights at
stake,” the Legislature must have intended that there should be “scrupulous
compliance” with the thirty-day limit. Id. at 826.
The Jimmy Ryce Act contains no provision addressing the rights of the
respondent during an appeal by the state. The Legislature may have
overlooked the possibility of an appeal by the state, but it is just as likely the
Legislature believed that a person who is exonerated in the trial court should
not be in custody at all. I am not certain of the Legislature’s intent or lack of
intent, but I am certain that the courts should not attempt to fill in the blank
by applying a mechanical rule of procedure that would allow for a detention
beyond the period of time authorized by statute. The constitutional validity
of the involuntary commitment process depends on the requirements of the
statute. It follows that the courts should not deviate from the statute by
extending the allowable time for detention.

161
Mitchell, 848 So. 2d at 1211-13 (Padovano, J., dissenting) (footnote
omitted).FN2 As noted by Judge Padovano and because of the constitutional
considerations implicit in the scheme, there are important protections
provided to the respondent under the Act and this Court has affirmed the
necessity of those protections. For example, section 394.916(1), Florida
Statutes (2000), provides that “[w]ithin thirty days after the determination of
probable cause, the court shall conduct a trial to determine whether the
person is a sexually violent predator.” We have previously held that there
shall be “scrupulous compliance” with the thirty-day time limit. Goode, 830
So. 2d at 826.FN3 Additionally, in State v. Kinder, 830 So. 2d 832 (Fla.2002),
this Court approved the district court’s holding “that the time provisions of
section 394.916(1) of the [Ryce Act] are mandatory unless a continuance is
granted for good cause.” Id. at 832.
FN2. While Judge Padovano asserts “there is no judicial finding that
could be used to support the appellee’s continued detention,” id. at 1212, he
subsequently acknowledges the trial court’s finding of probable cause that
the appellee is properly subject to detention under the Ryce Act.
FN3. However, this Court noted in Goode the time limit would be subject
to exceptions:
While we conclude that the time provisions regarding the holding of a
trial were meant to be mandatory, we do not believe that the thirty-day
period was intended as a rigid jurisdictional bar to further proceedings. In
addition to the provision for an adversarial probable cause determination,
section 394.916(2) provides that “[t]he trial may be continued upon the
request of either party and a showing of good cause, or by the court on its
own motion in the interests of justice, when the person will not be
substantially prejudiced.” By this language, the Legislature demonstrated
that there will be instances when the trial court will retain jurisdiction
beyond the thirty-day time period. In cases where the alleged sexually
violent predator will not be substantially prejudiced, a trial court is given
authority to grant a continuance when one of the parties shows good cause or
the court determines that the interests of justice so dictate. Id. at 828.
Importantly, in Osborne v. State, 907 So. 2d 505 (Fla.2005), this Court has
recently held, “[W]here a respondent has completed his criminal sentence
and is being detained awaiting a Ryce Act trial and the trial period has
exceeded thirty days without a continuance for good cause, the respondent’s
remedy is release from detention and a dismissal without prejudice of the

162
pending proceedings.” Id. at 509. Hence, we have already recognized an
instance where the State may be entitled to continue the proceedings, but the
respondent may be entitled to his freedom where the State has not
scrupulously complied with the Act’s provisions.
Significantly, as we expressed in Goode, the Legislature intended that
“the review process of potential sexual predators would be concluded while
the person was still in prison.” Goode, 830 So. 2d at 825. We also
recognized “that while the Legislature intended that the Ryce Act operate in
this way, there is evidence that in practice this is not occurring and that often
people are being detained for long periods after their scheduled release date
without being taken to trial.” Id. at 825 n. 7. We again emphasize that the
State should make every effort to initiate the commitment trial “well in
advance of the [detainee’s] date of release from prison[, so that] the due
process concerns of commitment beyond imprisonment would be
substantially alleviated.” Id. at 826.
However, while we agree with many of Mitchell’s and Judge Padovano’s
concerns, it is also important to note that when the State appeals the
dismissal of a petition seeking commitment, the State may sometimes be
entitled to a stay of the trial court’s decision until the dismissal order can be
reviewed. We note that other states also permit stays in some instances. In
California, for example, the State is allowed to seek writ review and a
temporary stay until the propriety of a dismissal order can be reviewed.
People v. Superior Court, 70 Cal.App.4th 558, 82 Cal.Rptr.2d 852, 854 n.5
(Cal.Ct.App.1999); People v. Superior Court (Howard ), 70 Cal.App.4th
136, 82 Cal.Rptr.2d 481, 487 (1999). Similarly, in Massachusetts, a stay
may be issued to prevent an individual’s release and to review a trial courts
determination that no probable cause existed to involuntarily commit an
individual as a sexually dangerous person. Commonwealth v. Blanchette, 54
Mass.App.Ct. 165, 764 N.E.2d 353, 355 (2002).
[9] Ordinarily, there are two principal considerations that courts must
take into account when deciding whether to vacate a stay: the likelihood of
irreparable harm if the stay is not granted and the likelihood of success on
the merits by the entity seeking to maintain the stay. Perez v. Perez, 769
So.2d 389, 391 n. 4 (Fla. 3d DCA 1999). In this case, we are mindful of the
potential problem with a stay allowing for a person to be detained beyond
completion of his or her sentence. However, as noted by the First District
majority, in this case the trial court expressly found that there was probable

163
cause to detain the respondent under the Ryce Act. Mitchell, 848 So. 2d at
1211. In addition, as with any new legislation affecting substantial
constitutional rights, there will be numerous instances where the meaning of
the statutory provisions and their constitutional implications will have to be
adjudicated before the law will be settled. Trial courts must have some
leeway to balance the competing interests while the issues are debated and
resolved. The present case presents a perfect example of this process. We
conclude that Florida’s provisions for a stay and the dissolution of such a
stay are adequate to balance the due process rights and liberty interests of the
respondent with the concerns of the State to protect the public.
CONCLUSION
We approve the First District’s decision and conclude that the State is
entitled to the benefit of the automatic stay provision of rule 9.310(b)(2) on
appeal in a civil commitment proceeding brought under the Jimmy Ryce
Act, subject to the discretion of the trial court to dissolve or modify the stay
based upon the important interests of the parties and the particular
circumstances of the case.
It is so ordered.
PARIENTE, C.J., and LEWIS, QUINCE, and CANTERO, JJ., concur.
WELLS, J., concurs in result only with an opinion, in which BELL, J.,
concurs.
WELLS, J., concurring in result only.
I concur that Florida Rule of Appellate Procedure 9.310(b)(2) applies to
an appeal in a civil commitment proceeding that is brought pursuant to part
V of chapter 394, Florida Statutes. I would affirm the decision of the First
District Court of Appeal in this case.
I do not join in the majority opinion because it is too broadly written,
covering issues which I do not believe are necessary in answering the
question that was certified in this case.
BELL, J., concurs.

The automatic stay provision in rule 9.310(b)(2) does not apply to a petition for
discretionary review. See Dep’t of Health & Rehab. Servs. v. E.D.S. Federal Corp., 622
So. 2d 90 (Fla. 1st DCA 1993). The proceeding in the appellate court must be an appeal.

164
E. Stay of Appellate Decisions

State v. Roberts
661 So. 2d 821 (Fla. 1995)
PER CURIAM.
On May 18, 1995, the First District Court of Appeal rendered its opinion
reversing Roberts’ conviction and remanding for a new trial. The court
issued its mandate on June 6, 1995. On June 16, 1995, the State timely filed
a notice to invoke the discretionary jurisdiction of this Court predicated upon
conflict of decisions. On June 20, 1995, the State filed in the First District
Court of Appeal a motion to recall mandate. On July 10, 1995, the First
District Court of Appeal denied the motion to recall mandate, specifically
stating that pursuant to State v. McKinnon, 540 So. 2d 111 (Fla.1989), a
party desiring stay of mandate during the pendency of a petition for review
in this Court must apply to this Court for the stay. The State has now filed a
motion with this Court seeking a stay and requiring the First District Court
of Appeal to withdraw its mandate pending disposition of the notice to
invoke the discretionary jurisdiction of this Court.
We have chosen to publish this order in order to clarify our opinion in
McKinnon. Our opinion in McKinnon contained language indicating that a
party desiring a stay of mandate during the pendency of a petition for review
in this Court must apply to this Court for a stay. However, the issue in that
case was not where the motion for stay should be filed. Rather, the holding
in that case was that the pendency of a petition for review in this Court did
not deprive the trial court of jurisdiction to resentence a defendant pursuant
to the district court’s mandate which had reversed and remanded the case for
resentencing.
While a motion for stay and to recall a mandate may be filed in this
Court, it may also be filed in the district court of appeal. The fact that a
notice to invoke the discretionary jurisdiction of this Court has already been
filed does not deprive the district court of appeal of jurisdiction to rule upon
the motion. This is explained in Florida Rule of Appellate Procedure
9.310(a), which reads as follows:
(a) Application. Except as provided by general law and in subdivision
(b) of this rule, a party seeking to stay a final or non-final order pending
review shall file a motion in the lower tribunal, which shall have continuing
jurisdiction, in its discretion, to grant, modify, or deny such relief. A stay

165
pending review may be conditioned on the posting of a good and sufficient
bond, other conditions, or both.
(Emphasis added.)
Generally speaking, this Court prefers that the motion for stay be filed in
the district court of appeal because at that stage of the case the district court
ordinarily will be better informed concerning the case and thereby better
able to predict the likelihood of this Court’s accepting jurisdiction.
Therefore, we recede from McKinnon to the extent that it suggests that the
filing of a notice to invoke discretionary jurisdiction precludes the district
court of appeal from entertaining a motion to stay or withdraw its mandate.
Because there is a reasonable possibility that this Court will accept
jurisdiction in this case and perhaps obviate the necessity for a new trial, we
hereby grant the State’s motion and direct the district court of appeal to
withdraw its mandate pending consideration of the notice to invoke this
Court’s discretionary jurisdiction.
GRIMES, C.J., and OVERTON, SHAW, KOGAN, HARDING, WELLS
and ANSTEAD, JJ., concur.

F. Review of Stay Orders

Florida Rules of Appellate Procedure

Rule 9.310. Stay Pending Review


***
(f) Review. Review of orders entered by lower tribunals under this rule
shall be by the court on motion.

An order establishing the conditions necessary to obtain a stay is subject to review by the
abuse of discretion standard. See Mariner Health Care of Nashville, Inc. v. Baker, 739
So. 2d 608 (Fla. 1st DCA 1999).

166
VIII. The Record

Florida Rules of Appellate Procedure

Rule 9.200. The Record


(a) Contents.
(1) Except as otherwise designated by the parties, the record shall consist
of the original documents, exhibits, and transcript(s) of proceedings, if any,
filed in the lower tribunal, except summonses, praecipes, subpoenas,
returns, notices of hearing or of taking deposition, depositions, other
discovery, and physical evidence. The record shall also include a progress
docket.
(2) In family law, juvenile dependency, and termination of parental rights
cases, and cases involving families and children in need of services, the
record shall include those items designated in subdivision (a)(1) except
that the clerk of the lower tribunal shall retain the original orders, reports
and recommendations of magistrates or hearing officers, and judgments
within the file of the lower tribunal and shall include copies thereof within
the record.
(3) Within 10 days of filing the notice of appeal, an appellant may direct
the clerk to include or exclude other documents or exhibits filed in the
lower tribunal. The directions shall be substantially in the form prescribed
by rule 9.900(g). If the clerk is directed to transmit less than the entire
record or a transcript of trial with less than all of the testimony, the
appellant shall serve with such direction a statement of the judicial acts to
be reviewed. Within 20 days of filing the notice, an appellee may direct the
clerk to include additional documents and exhibits.
(4) The parties may prepare a stipulated statement showing how the
issues to be presented arose and were decided in the lower tribunal,
attaching a copy of the order to be reviewed and as much of the record in
the lower tribunal as is necessary to a determination of the issues to be
presented. The parties shall advise the clerk of their intention to rely on a
stipulated statement in lieu of the record as early in advance of filing as
possible. The stipulated statement shall be filed by the parties and
transmitted to the court by the clerk of the lower tribunal within the time
prescribed for transmittal of the record.
(b) Transcript(s) of Proceedings.
(1) Within 10 days of filing the notice, the appellant shall designate those
portions of the proceedings not on file deemed necessary for transcription
and inclusion in the record. Within 20 days of filing the notice, an appellee

167
may designate additional portions of the proceedings. Copies of
designations shall be served on the court reporter. Costs of the original
and all copies of the transcript(s) so designated shall be borne initially by
the designating party, subject to appropriate taxation of costs as
prescribed by rule 9.400. At the time of the designation, unless other
satisfactory arrangements have been made, the designating party must
make a deposit of 1/2 of the estimated transcript costs, and must pay the
full balance of the fee on delivery of the completed transcript(s).
(2) Within 30 days of service of a designation, or within the additional time
provided for under subdivision (b)(3) of this rule, the court reporter shall
transcribe and file with the clerk of the lower tribunal the designated
proceedings and shall serve copies as requested in the designation. In
addition to the paper copies, the court reporter shall file with the clerk of
the lower tribunal and serve on the designated parties an electronic copy
of the designated proceedings in a format approved by the supreme court.
If a designating party directs the court reporter to furnish the transcript(s)
to fewer than all parties, that designating party shall serve a copy of the
designated transcript(s), in both electronic and paper form, on the parties
within 5 days of receipt from the court reporter. The transcript of the trial
shall be securely bound in consecutively numbered volumes not to exceed
200 pages each, and each page shall be numbered consecutively. Each
volume shall be prefaced by an index containing the names of the
witnesses, a list of all exhibits offered and introduced in evidence, and the
pages where each may be found.
(3) On service of a designation, the reporter shall acknowledge at the foot
of the designation the fact that it has been received and the date on which
the reporter expects to have the transcript(s) completed and shall transmit
the designation, so endorsed, to the parties and to the clerk of the
appellate court within 5 days of service. If the transcript(s) cannot be
completed within 30 days of service of the designation, the reporter shall
request such additional time as is reasonably necessary and shall state
the reasons therefor. If the reporter requests an extension of time, the
court shall allow the parties 5 days in which to object or agree. The
appellate court shall approve the request or take other appropriate action
and shall notify the reporter and the parties of the due date of the
transcript(s).
(4) If no report of the proceedings was made, or if the transcript is
unavailable, the appellant may prepare a statement of the evidence or
proceedings from the best available means, including the appellant's
recollection. The statement shall be served on the appellee, who may
serve objections or proposed amendments to it within 10 days of service.
Thereafter, the statement and any objections or proposed amendments
shall be submitted to the lower tribunal for settlement and approval. As

168
settled and approved, the statement shall be included by the clerk of the
lower tribunal in the record.
(c) Cross-Appeals. Within 20 days of filing the notice, a cross-appellant
may direct that additional documents, exhibits, or transcript(s) be included
in the record. If less than the entire record is designated, the cross-
appellant shall serve, with the directions, a statement of the judicial acts to
be reviewed. The cross-appellee shall have 10 days after such service to
direct further additions. The time for preparation and transmittal of the
record shall be extended by 10 days.
(d) Duties of Clerk; Preparation and Transmittal of Record.
(1) The clerk of the lower tribunal shall prepare the record as follows:
(A) The clerk of the lower tribunal shall not be required to verify and
shall not charge for the incorporation of any transcript(s) into the record.
The transcript of the trial shall be incorporated at the end of the record,
and shall not be renumbered by the clerk. The progress docket shall be
incorporated into the record immediately after the index.
(B) The remainder of the record, including all supplements and any
transcripts other than the transcript of the trial, shall be consecutively
numbered. The record shall be securely bound in consecutively numbered
volumes not to exceed 200 pages each. The cover sheet of each volume
shall contain the name of the lower tribunal and the style and number of
the case.
(2) The clerk of the lower tribunal shall prepare a complete index to the
record and shall attach a copy of the progress docket to the index.
(3) The clerk of the lower tribunal shall certify and transmit the record to
the court as prescribed by these rules; provided that if the parties stipulate
or the lower tribunal orders that the original record be retained, the clerk
shall prepare and transmit a certified copy.
(e) Duties of Appellant or Petitioner. The burden to ensure that the
record is prepared and transmitted in accordance with these rules shall be
on the petitioner or appellant. Any party may enforce the provisions of this
rule by motion.
(f) Correcting and Supplementing Record.
(1) If there is an error or omission in the record, the parties by stipulation,
the lower tribunal before the record is transmitted, or the court may correct
the record.
(2) If the court finds the record is incomplete, it shall direct a party to
supply the omitted parts of the record. No proceeding shall be determined,
because of an incomplete record, until an opportunity to supplement the

169
record has been given.
(g) Return of Record. In civil cases, the record shall be returned to the
lower tribunal after final disposition by the court.

A. Duty to Produce

Applegate v. Barnett Bank of Tallahassee


377 So. 2d 1150 (Fla. 1979)
ADKINS, Justice.
By petition for writ of certiorari we have for review the decision of the
district court of appeal in Barnett Bank of Tallahassee v. Applegate, --So. 2d
-- (Fla. 1st DCA 1978), which directly conflicts with Okun v. Stuart House
Condominium Association, Inc., 362 So. 2d 713 (Fla. 3d DCA 1978); Moore
v. Sky Realty, Inc., 339 So. 2d 299 (Fla. 3d DCA 1976); Steinhauer v.
Steinhauer, 336 So. 2d 665 (Fla. 4th DCA 1976); Buckalew v. Buckalew,
115 So. 2d 564 (Fla. 2d DCA 1959); and Bolick v. Sperry, 88 So. 2d 495
(Fla.1956). We have jurisdiction. Art. V, s 3(b)(3), Fla.Const.
On January 26, 1974, Albert V. and Phyllis J. Applegate entered into an
agreement with Lewis C. Schacht. Among other things, the agreement
provided for the installment sale of 550 head of cattle to Schacht. The
Applegates filed suit against Schacht on February 20, 1976. The Barnett
Bank of Tallahassee was joined as a defendant due to its December 28,
1975, security agreement with Schacht and related UCC financing statement.
In late April, 1976, after the filing of the suit, Schacht sold at least 140 of the
cattle and paid Barnett Bank the sale proceeds of $16,233.36 and $6,496.75.
After a nonjury trial without a reporter, the circuit court found that the
Applegates’ lien on the cattle was superior to Barnett Bank’s lien. The court
also found and ordered the proceeds of the April sales of the cattle to be held
in trust for the Applegates to the extent of any deficiency on the sum still
due them. Barnett Bank appealed but did not bring forward any substitute for
a trial transcript. See Fla.R.App.P. 3.6(h) and 9.200(b)(3).
The First District Court of Appeal ruled that the Applegates had lost any
priority to the proceeds of the sale of the cattle because their financing
statement did not cover proceeds from a sale of collateral and they failed to
perfect their security interest within ten days after the sales as required by
Section 679.306(3)(b), Florida Statutes (1977). The district court of appeal

170
concluded that “the (trial) court’s finding that a constructive trust arose by
operation of law is not supported by the facts.” The appellate court reversed
the imposition of a constructive trust. Barnett Bank of Tallahassee v.
Applegate at 7, 8.
[1] In appellate proceedings the decision of a trial court has the
presumption of correctness and the burden is on the appellant to demonstrate
error. The Applegates correctly point to the lack of a trial transcript or a
proper substitute as fatally flawing the appellate court’s ruling. The written
final judgment by the trial court could well be wrong in its reasoning, but the
decision of the trial court is primarily what matters, not the reasoning used.
[2] [3] Even when based on erroneous reasoning, a conclusion or
decision of a trial court will generally be affirmed if the evidence or an
alternative theory supports it. E. g., 3 Fla.Jur.2d s 296; In Re Yohn’s Estate,
238 So. 2d 290 (Fla.1970); Goodman v. Goodman, 204 So. 2d 21 (4th DCA
Fla. 1967); Escarra v. Winn Dixie Stores, Inc., 131 So. 2d 483 (Fla.1961).
However, a misconception by the trial judge of a controlling principle of law
can constitute grounds for reversal. E. g., Aronson v. Siquier, 318 So. 2d 452
(3d DCA Fla. 1975); Green v. Putnam, 93 So. 2d 378 (Fla.1957); Knight v.
City of Miami, 127 Fla. 585, 173 So. 801 (1937).
[4] The trial court’s imposition of a constructive trust could well be
supported by evidence adduced at trial but not stated in the judge’s order or
otherwise apparent in the incomplete record on appeal. The question raised
by Barnett Bank clearly involves underlying issues of fact. When there are
issues of fact the appellant necessarily asks the reviewing court to draw
conclusions about the evidence. Without a record of the trial proceedings,
the appellate court cannot properly resolve the underlying factual issues so
as to conclude that the trial court’s judgment is not supported by the
evidence or by an alternative theory. Without knowing the factual context,
neither can an appellate court reasonably conclude that the trial judge so
misconceived the law as to require reversal. The trial court should have been
affirmed because the record brought forward by the appellant is inadequate
to demonstrate reversible error. E. g., South Florida Apartment Association,
Inc. v. Dansyear, 347 So.2d 710 (3d DCA Fla. 1977); Strickland v. Lewis,
328 So. 2d 244 (1st DCA Fla. 1976); Troutman v. Couture, 98 Fla. 889, 124
So. 443 (1929). See also cases cited above for conflict.

171
The order of the district court of appeal is quashed and the cause
remanded for entry of an order affirming the final judgment of the trial
court.
It is so ordered.
OVERTON and ALDERMAN, JJ., and VANN, Associate Justice,
concur.
BOYD, J., dissents.
ENGLAND, Chief Justice, concurs specially: * * *

B. Time for Filing

Florida Rules of Appellate Procedure

Rule 9.110. Appeal Proceedings to Review Final Orders of Lower


Tribunals and Orders Granting New Trial in Jury and Non-Jury Cases
***
(e) Record. Within 50 days of filing the notice, the clerk shall prepare the
record prescribed by rule 9.200 and serve copies of the index on all
parties. Within 110 days of filing the notice, the clerk shall transmit the
record to the court.

Rule 9.140. Appeal Proceedings in Criminal Cases


***
(f) Record.
(1) Service.The clerk of the lower tribunal shall prepare and serve the
record prescribed by rule 9.200 within 50 days of the filing of the notice of
appeal.
(2) Transcripts.
(A) If a defendant's designation of a transcript of proceedings
requires expenditure of public funds, trial counsel for the defendant (in
conjunction with appellate counsel, if possible) shall serve, within 10 days
of filing the notice, a statement of judicial acts to be reviewed, and a
designation to the court reporter requiring preparation of only so much of
the proceedings as fairly supports the issue raised.
(B) Either party may file motions in the lower tribunal to reduce or

172
expand the transcripts.
(C) Except as permitted in subdivision (f)(2)(D) of this rule, the
parties shall designate the court reporter to file with the clerk of the lower
tribunal the original transcripts for the court and sufficient copies for the
state and all indigent defendants.
(D) Non-indigent defendants represented by counsel may
designate the court reporter to prepare only original transcripts. Counsel
adopting this procedure shall, within 5 days of receipt of the original
transcripts from the court reporter, file the original transcripts along with
securely bound copies for the state and all defendants. Counsel shall
serve notice of the use of this procedure on the attorney general (or the
state attorney in appeals to circuit court) and the clerk of the lower
tribunal. Counsel shall attach a certificate to each copy certifying that it is
an accurate and complete copy of the original transcript. When this
procedure is used, the clerk of the lower tribunal upon conclusion of the
appeal shall retain the original transcript for use as needed by the state in
any collateral proceedings and shall not destroy the transcripts without the
consent of the Office of the Attorney General.
(E) In state appeals, the state shall designate the court reporter to
prepare and file with the clerk of the lower tribunal the original transcripts
and sufficient copies for all separately represented defendants.
Alternatively, the state may elect to use the procedure specified in
subdivision (f)(2)(D) of this rule.
(F) The lower tribunal may by administrative order in publicly-
funded cases direct the clerk of the lower tribunal rather than the court
reporter to prepare the necessary copies of the original transcripts.
(3) Retention of Documents.Unless otherwise ordered by the court, the
clerk of the lower tribunal shall retain all original documents except the
original transcripts designated for appeal which shall be included in the
record transmitted to the court.
(4) Service of Copies.The clerk of the lower tribunal shall serve copies of
the record to the court, attorney general (or state attorney in appeals to
circuit court), and all counsel appointed to represent indigent defendants
on appeal. The clerk of the lower tribunal shall simultaneously serve
copies of the index to all non-indigent defendants and, upon their request,
copies of the record or portions thereof at the cost prescribed by law.
(5) Return of Record.Except in death penalty cases, the court shall return
the record to the lower tribunal after final disposition of the appeal.
(6) Supplemental Record for Motion to Correct Sentencing Error Pursuant
to Florida Rule of Criminal Procedure 3.800(b)(2).

173
(A) The clerk of circuit court shall automatically supplement the
appellate record with any motion pursuant to Florida Rule of Criminal
Procedure 3.800(b)(2), any response, any resulting order, and any
amended sentence. The clerk shall transmit the supplement to the
appellate court within 5 days of the filing of the order ruling on the motion.
If an order is not filed within 60 days from the filing of the motion, this time
shall run from the expiration of the 60 day period, and the clerk shall
supplement the record with the motion and a statement that no order was
timely filed.
(B) If any appellate counsel determines that a transcript of a
proceeding relating to such a motion is required to review the sentencing
issue, appellate counsel shall, within 5 days from the transmittal of the
supplement described in subdivision (A), designate those portions of the
proceedings not on file deemed necessary for transcription and inclusion
in the record. A copy of the designation shall be filed with the appellate
court. The procedure for this supplementation shall be in accordance with
this subdivision, except that counsel is not required to file a revised
statement of judicial acts to be reviewed, the court reporter shall deliver
the transcript within 15 days, and the clerk shall supplement the record
with the transcript within 5 days of its receipt.

C. Contents of Record

Florida Rules of Appellate Procedure

Rule 9.200. The Record


(a) Contents.
(1) Except as otherwise designated by the parties, the record shall
consist of the original documents, exhibits, and transcript(s) of
proceedings, if any, filed in the lower tribunal, except summonses,
praecipes, subpoenas, returns, notices of hearing or of taking deposition,
depositions, other discovery, and physical evidence. The record shall also
include a progress docket.
***

174
D. Statement of the Evidence

All-Brite Aluminum, Inc. v. Desrosiers


626 So. 2d 1020 (Fla. 2d DCA 1993)
PARKER, Acting Chief Judge.
All-Brite Aluminum, Inc., appeals the final judgment which failed to
award All-Brite its attorney’s fees and costs against Norman and Irene
Desrosiers in a foreclosure of a construction lien. We reverse, concluding
that All-Brite was the prevailing party and, thus, was entitled to an award of
attorney’s fees and costs.
[1] All-Brite filed a complaint against the Desrosiers for foreclosure of a
construction lien. The matter proceeded to trial. The record FN1 establishes
that the Desrosiers entered into a contract with a general contractor for
construction of a residence. * * *
FN1. A court reporter did not transcribe the final hearing; therefore, the
appellate record of the final hearing consists of the trial court’s Approval
and Settlement of Statement of Evidence Pursuant to Florida Rule of
Appellate Procedure 9.200(b)(4). Florida Rule of Appellate Procedure
9.200(b)(4) requires the appellant to prepare and submit a statement of
evidence or proceedings to the appellee, who can serve objections or
proposed amendments. Thereafter the appellant submits the statement and
any objections or amendments to the trial court for settlement and approval.
In this case All-Brite submitted a statement of the evidence. Thereafter the
Desrosiers submitted a statement of the evidence. The trial court struck
through two sentences in All-Brite’s statement of the evidence and then
approved both All-Brite’s and the Desrosiers’ statements of the evidence.
We conclude that the parties and the court substantially complied with Rule
9.200(b)(4).
[2] The issue on appeal is whether All-Brite was the prevailing party in
this litigation and thereby entitled to an award of attorney’s fees and costs.
FN2
* * * We hold that All-Brite was the prevailing party because the
Desrosiers did not tender the payment before All-Brite filed the complaint.
[ * * *]
Reversed and remanded.
ALTENBERND and BLUE, JJ., concur.

175
E. Transcript of Proceedings

Florida Rules of Appellate Procedure

Rule 9.200. The Record


***
(b) Transcript(s) of Proceedings.
(1) Within 10 days of filing the notice, the appellant shall designate
those portions of the proceedings not on file deemed necessary for
transcription and inclusion in the record. Within 20 days of filing the notice,
an appellee may designate additional portions of the proceedings. Copies
of designations shall be served on the court reporter. Costs of the original
and all copies of the transcript(s) so designated shall be borne initially by
the designating party, subject to appropriate taxation of costs as
prescribed by rule 9.400. At the time of the designation, unless other
satisfactory arrangements have been made, the designating party must
make a deposit of 1/2 of the estimated transcript costs, and must pay the
full balance of the fee on delivery of the completed transcript(s).
(2) Within 30 days of service of a designation, or within the
additional time provided for under subdivision (b)(3) of this rule, the court
reporter shall transcribe and file with the clerk of the lower tribunal the
designated proceedings and shall serve copies as requested in the
designation. In addition to the paper copies, the court reporter shall file
with the clerk of the lower tribunal and serve on the designated parties an
electronic copy of the designated proceedings in a format approved by the
supreme court. If a designating party directs the court reporter to furnish
the transcript(s) to fewer than all parties, that designating party shall serve
a copy of the designated transcript(s), in both electronic and paper form,
on the parties within 5 days of receipt from the court reporter. The
transcript of the trial shall be securely bound in consecutively numbered
volumes not to exceed 200 pages each, and each page shall be
numbered consecutively. Each volume shall be prefaced by an index
containing the names of the witnesses, a list of all exhibits offered and
introduced in evidence, and the pages where each may be found.
(3) On service of a designation, the reporter shall acknowledge at
the foot of the designation the fact that it has been received and the date
on which the reporter expects to have the transcript(s) completed and
shall transmit the designation, so endorsed, to the parties and to the clerk
of the appellate court within 5 days of service. If the transcript(s) cannot be
completed within 30 days of service of the designation, the reporter shall
request such additional time as is reasonably necessary and shall state

176
the reasons therefor. If the reporter requests an extension of time, the
court shall allow the parties 5 days in which to object or agree. The
appellate court shall approve the request or take other appropriate action
and shall notify the reporter and the parties of the due date of the
transcript(s).
(4) If no report of the proceedings was made, or if the transcript is
unavailable, the appellant may prepare a statement of the evidence or
proceedings from the best available means, including the appellant's
recollection. The statement shall be served on the appellee, who may
serve objections or proposed amendments to it within 10 days of service.
Thereafter, the statement and any objections or proposed amendments
shall be submitted to the lower tribunal for settlement and approval. As
settled and approved, the statement shall be included by the clerk of the
lower tribunal in the record.
***

F. Supplementing the Record

Florida Rules of Appellate Procedure

Rule 9.200. The Record


***
(f) Correcting and Supplementing Record.
***
(2) If the court finds the record is incomplete, it shall direct a party
to supply the omitted parts of the record. No proceeding shall be
determined, because of an incomplete record, until an opportunity to
supplement the record has been given.

Poteat v. Guardianship of Poteat


771 So. 2d 569 (Fla. 4th DCA 2000)
KLEIN, J.
These appeals question whether there was clear and convincing evidence
that an elderly woman, Willie Florence Poteat, is partially incapacitated and
whether the appointment of her granddaughter as a limited guardian was
improper because of conflicts of interest. We affirm.

177
[ * * *]
[3] We now address a motion to supplement record filed by Louis B.
Vocelle, Jr., counsel for Guy and Ann. On May 31, 2000, one day before
oral argument, Mr. Vocelle filed a motion to supplement the record in this
court to include a complaint, which his clients, Ann and Guy, filed against
Jerilyn and others, on May 26, 2000, in circuit court. The purpose of this
motion, as indicated by counsel in oral argument, was to demonstrate to us
that, if we did not reverse the appointment of Jerilyn as guardian so that a
non-family member could be appointed, endless litigation between family
members would be the result.
This motion constitutes a “flagrant violation” of Florida Rule of
Appellate Procedure 9.200(f), which authorizes the correction and
supplementation of the record. Thornber v. City of Fort Walton Beach, 534
So. 2d 754, 756 (Fla. 1st DCA 1988). As the Thornber court explained:
That an appellate court may not consider matters outside the record is so
elemental there is no excuse for an attorney to attempt to bring such matters
before the court. Id. at 755 and cases cited. The purpose of rule 9.200(f) is to
allow supplementation of the record submitted to the appellate court, with an
item which was considered FN1 by the trial court, but was omitted from the
record on appeal. To attempt to supplement the record on appeal with a
document created by counsel several days before oral argument is highly
unprofessional, and we accordingly strike the motion.
FN1. We use the term “considered,” because we recognize that
sometimes things such as depositions or memoranda of law are considered
by the court, but through inadvertence are not placed in the record prior to
the entry of final judgment.
Affirmed.
POLEN and HAZOURI, JJ., concur

Baron Transport and Claims Center v. Riley


510 So. 2d 1150 (Fla. 1st DCA 1987)
PER CURIAM.
After taking an appeal of a workers’ compensation order awarding
attorney’s fees, counsel for the employer/carrier filed directions for
preparation of the record that designated only the order appealed and an
earlier order which awarded certain benefits to the claimant (and which was

178
not appealed). Appellee moves this court for an order dismissing this appeal
for appellants’ failure to provide the court with a record that includes the
transcript of the hearing below. Alternatively, appellee asks the court to
order the appellants to supplement the record with those items enumerated in
Rule 4.180(a)(1), Florida Workers’ Compensation Rules of Procedure. We
directed the appellants to show cause why the motion should not be granted
and their response contends that because of the narrow issue raised in this
appeal, the record as designated is sufficient. Appellants suggest that they
have complied with Rule 4.180(b), which permits an appellant to designate
an abbreviated record when the issue(s) to be raised in the appeal can be
presented to the court with such a record. Appellants’ response concludes:
If this court finds supplementing of the record necessary, such
supplementing could be accomplished by utilizing the transcript in pending
appeal case BT-187 wherein Baron Transport and Claims Center are the
appellants and Teddy Paul Riley is the appellee. Additional supplementing
could be obtained by utilization of the prior record in appeal BI-401.
We deny the motion to dismiss or to compel supplementation, but we
also expressly reject appellants’ suggestion that this court examine other
files to build a record for this appeal. We recognize that Rule 4.180(d)
(which is similar to Rule 9.200(f)(2), Florida Rules of Appellate Procedure),
authorizes the court to order supplementation of the record on its own
motion or the motion of a party if the record is incomplete. While the
ultimate concern of the court is that cases be decided on the merits, it is not
our purpose to “[lead] the parties by the hand in an attempt to get an
adequate record.” Stack v. Lareau, 433 So. 2d 66, 68 (Fla.4th DCA 1983)
(quoting Kauffman v. Baker, 392 So. 2d 13, 15 (Fla. 4th DCA 1980)); see
also Rule 9.200(f), Florida Rules of Appellate Procedure, committee notes
(“A failure to supplement the record after notice by the court may be held
against the party at fault.”) We find that because appellants continue to
assert that the record is complete for appellate review (and, as the initial
brief has not yet been filed, we are unable to contradict that assertion),
appellee’s proper remedy is to argue for affirmance in the answer brief on
the basis of Applegate v. Barnett Bank, 377 So. 2d 1150 (Fla.1979) and
Clayton v. Clayton, 442 So. 2d 310 (Fla. 1st DCA 1983).
Appellants’ suggestion that we look to other files to supplement the
record is not well-taken and is at odds with the express practices of the court,
see Gulf Coast Home Health Services, Inc. v. Department of Health and

179
Rehabilitative Services, 503 So. 2d 415 (Fla. 1st DCA 1987). Moreover, the
file in BI-401 has been closed for a full year and the record has been
returned, see Rule 2.040(b)(5), Florida Rules of Judicial Administration. As
for Case No. BT-187, appellants have failed to explain what relationship, if
any, that appeal and the record therein have to the instant appeal. If all or
some of the record in BT-187 should be considered in disposing of this
appeal, appellants must either see that copies of the relevant documents are
made a part of this record or, in the alternative, move to consolidate the
appeals for purposes of the record.FN1
FN1. After this court’s consideration of appellee’s motion and
preparation of this opinion, appellants moved for consolidation of this case
with BT-187. Appellants offer to serve a single brief in the two appeals and
the motion represents that appellee has no objection. Accordingly, the
motion to consolidate is granted and the briefs of the parties may refer to and
rely on both records in the consolidated cases.
Motion denied.
JOANOS, WIGGINTON and ZEHMER, JJ., concur.

IX. Motions

Florida Rules of Appellate Procedure

Rule 9.300. Motions


(a) Contents of Motion; Response. Unless otherwise prescribed by
these rules, an application for an order or other relief available under
these rules shall be made by filing a motion therefor. The motion shall
state the grounds on which it is based, the relief sought, argument in
support thereof, and appropriate citations of authority. A motion for an
extension of time shall, and other motions if appropriate may, contain a
certificate that the movant's counsel has consulted opposing counsel and
that the movant's counsel is authorized to represent that opposing counsel
either has no objection or will promptly file an objection. A motion may be
accompanied by an appendix, which may include affidavits and other
appropriate supporting documents not contained in the record. A party
may serve 1 response to a motion within 10 days of service of the motion.
The court may shorten or extend the time for response to a motion.
(b) Effect on Proceedings. Except as prescribed by subdivision (d) of

180
this rule, service of a motion shall toll the time schedule of any proceeding
in the court until disposition of the motion. An order granting an extension
of time for any act shall automatically extend the time for all other acts that
bear a time relation to it. An order granting an extension of time for
preparation of the record, or the index to the record, or for filing of the
transcript of proceedings, shall extend automatically, for a like period, the
time for service of appellant's initial brief. A conformed copy of an order
extending time shall be transmitted forthwith to the clerk of the lower
tribunal until the record has been transmitted to the court.
(c) Emergency Relief; Notice. A party seeking emergency relief shall, if
practicable, give reasonable notice to all parties.
(d) Motions Not Tolling Time.
(1) Motions for post-trial release, rule 9.140(g).
(2) Motions for stay pending appeal, rule 9.310.
(3) Motions relating to oral argument, rule 9.320.
(4) Motions relating to joinder and substitution of parties, rule 9.360.
(5) Motions relating to amicus curiae, rule 9.370.
(6) Motions relating to attorneys' fees on appeal, rule 9.400.
(7) Motions relating to service, rule 9.420.
(8) Motions relating to admission or withdrawal of attorneys, rule
9.440.
(9) Motions relating to expediting the appeal.
(10) All motions filed in the supreme court, unless accompanied by
a separate request to toll time.

A. Time Limits

Florida Rules of Appellate Procedure

Rule 9.330. Rehearing; Clarification; Certification


(a) Time for Filing; Contents; Response. A motion for rehearing,
clarification, or certification may be filed within 15 days of an order or
within such other time set by the court. * * *

181
Rule 9.400. Costs and Attorneys' Fees
(a) Costs. Costs shall be taxed in favor of the prevailing party unless the
court orders otherwise. Taxable costs shall include
(1) fees for filing and service of process;
(2) charges for preparation of the record;
(3) bond premiums; and
(4) other costs permitted by law.Costs shall be taxed by the lower tribunal
on motion served within 30 days after issuance of the mandate.

B. Contents of Motions

Florida Rules of Appellate Procedure

Rule 9.300. Motions


(a) Contents of Motion; Response.Unless otherwise prescribed by these
rules, an application for an order or other relief available under these rules
shall be made by filing a motion therefor. The motion shall state the
grounds on which it is based, the relief sought, argument in support
thereof, and appropriate citations of authority. A motion for an extension of
time shall, and other motions if appropriate may, contain a certificate that
the movant's counsel has consulted opposing counsel and that the
movant's counsel is authorized to represent that opposing counsel either
has no objection or will promptly file an objection. A motion may be
accompanied by an appendix, which may include affidavits and other
appropriate supporting documents not contained in the record. A party
may serve 1 response to a motion within 10 days of service of the motion.
The court may shorten or extend the time for response to a motion.
***

Hilltop Developers, Inc. v. Masterpiece Homes, Inc.


455 So. 2d 1155 (Fla. 5th DCA 1984).
PER CURIAM.
This case was dismissed pursuant to stipulation of the parties. A matter
involving conduct of counsel will be considered here. On May 31, 1984,
appellant, by its attorney, Alan B. Robinson, moved for an extension of time
pursuant to Florida Rule of Appellate Procedure 9.300. As provided in the

182
rule, Robinson stated: “Counsel has consulted with Robert Kieffer, Esquire,
[appellee’s counsel] who has indicated he has no objection to a one-time
thirty (30) day extension.” Relying on such representation, this court, on
June 1, 1984, granted the extension without affording opposing counsel an
opportunity to reply. On June 4, 1984, following issuance of the order,
appellee’s co-counsel responded to the motion stating that Kieffer had not
consented to the extension.
[1] It is essential to the expeditious handling of motions under rule 9.300
that this court be able to rely upon the accuracy of representations of
counsel. The rule states in relevant part:
A motion for extension of time shall, and other motions where
appropriate may, contain a certificate by the movant’s counsel that he has
consulted opposing counsel and that he is authorized to represent that
opposing counsel either has no objection or will promptly file an objection.
(emphasis added)
Counsel within this district have generally respected and appreciated the
purpose and intent of this provision as is indicated by the fact that very few
instances have been called to our attention where opposing counsel denies
having authorized such a representation.
In an attempt to determine the accuracy of the representation here, an
order was issued to Alan B. Robinson to file a response. In his response,
Robinson stated that his file contained a note “no objection appeal
continuance 30 days, R. Kieffer.” In view of this conflict, the Honorable Lon
Cornelius, Circuit Judge, was appointed Commissioner and was instructed to
conduct a hearing for the purpose of taking testimony from the parties and
then to report his findings to this court. Judge Cornelius found that there was
no intentional act by either lawyer to try to mislead each other in any way,
but instead it was some type of misunderstanding. The statement by
Robinson as to the note in his file was not addressed.
[2] We have concluded that the allegation in the motion and the response
of Robinson do not rise to the level of contempt. It is clear, however, that he
was not authorized to state that Kieffer had agreed to the extension. It is also
apparent that Robinson did not have a note in his file signed by Robert
Kieffer, and while he did not so state, his claim was capable of this
interpretation and reflects less than the candor this court expects of attorneys
practicing before it.

183
Costs of the transcript at the hearing before Judge Cornelius are assessed
against Alan B. Robinson, Esquire.
DAUKSCH, FRANK D. UPCHURCH, Jr. and SHARP, JJ., concur.

C. Motions in the Lower Tribunal

Florida Rules of Appellate Procedure

Rule 9.600. Jurisdiction of Lower Tribunal Pending Review


(a) Concurrent Jurisdiction. Only the court may grant an extension of
time for any act required by these rules. Before the record is transmitted,
the lower tribunal shall have concurrent jurisdiction with the court to render
orders on any other procedural matter relating to the cause, subject to the
control of the court.
(b) Further Proceedings. If the jurisdiction of the lower tribunal has been
divested by an appeal from a final order, the court by order may permit the
lower tribunal to proceed with specifically stated matters during the
pendency of the appeal.
(c) Family Law Matters. In family law matters:
(1) The lower tribunal shall retain jurisdiction to enter and enforce
orders awarding separate maintenance, child support, alimony, attorneys'
fees and costs for services rendered in the lower tribunal, temporary
attorneys' fees and costs reasonably necessary to prosecute or defend an
appeal, or other awards necessary to protect the welfare and rights of any
party pending appeal.
(2) The receipt, payment, or transfer of funds or property under an
order in a family law matter shall not prejudice the rights of appeal of any
party. The lower tribunal shall have the jurisdiction to impose, modify, or
dissolve conditions upon the receipt or payment of such awards in order to
protect the interests of the parties during the appeal.
(3) Review of orders entered pursuant to this subdivision shall be
by motion filed in the court within 30 days of rendition.
(d) Criminal Cases. The lower tribunal shall retain jurisdiction to consider
motions pursuant to Florida Rules of Criminal Procedure 3.800(b)(2) and
in conjunction with post-trial release pursuant to rule 9.140(h).

184
D. Motions for Review

Florida Rules of Appellate Procedure

Rule 9.310. Stay Pending Review


***
(f) Review. Review of orders entered by lower tribunals under this rule
shall be by the court on motion.

Rule 9.400. Costs and Attorneys' Fees


***
(c) Review. Review of orders rendered by the lower tribunal under this
rule shall be by motion filed in the court within 30 days of rendition.

Rule 9.600. Jurisdiction of Lower Tribunal Pending Review


***
(c) Family Law Matters. In family law matters:
(1) The lower tribunal shall retain jurisdiction to enter and enforce orders
awarding separate maintenance, child support, alimony, attorneys' fees
and costs for services rendered in the lower tribunal, temporary attorneys'
fees and costs reasonably necessary to prosecute or defend an appeal, or
other awards necessary to protect the welfare and rights of any party
pending appeal.
(2) The receipt, payment, or transfer of funds or property under an order in
a family law matter shall not prejudice the rights of appeal of any party.
The lower tribunal shall have the jurisdiction to impose, modify, or dissolve
conditions upon the receipt or payment of such awards in order to protect
the interests of the parties during the appeal.
(3) Review of orders entered pursuant to this subdivision shall be by
motion filed in the court within 30 days of rendition.

185
E. Sanctions

Dubowitz v. Century Village East, Inc.


381 So. 2d 252 (Fla. 4th DCA 1979)
LETTS, Judge.
[1] The supplemental response to the Appellees Motion to Strike is
hereby stricken and the sua sponte striking thereof used as a springboard for
this order.
This Court is being deluged nowadays with a plethora of pleadings which
have no place in any appellate court and which are causing a distressing
waste of time. We are in truth suffering from acute motion sickness.
The introductory note to the 1978 version of the Florida Appellate Rules
concludes with the following paragraph:
It was the intent of the many persons involved in the drafting of these
revised rules to implement the public policy of Florida that appellate
procedures operate to protect rather than thwart the substantive legal rights
of the people by alleviating existing burdens on the judicial system, by
reducing the cost of appeals, by standardizing and expediting the appellate
procedures which have at times frustrated the cause of justice.
Notwithstanding, there is an ever growing tendency for attorneys to
ignore the spirit of the above and file unnecessary pleadings which we seek
by this order to discourage.
The supplemental response which provides the springboard for this order
is reproduced as follows:
SUPPLEMENTAL RESPONSE TO APPELLEES’ MOTION TO
STRIKE
In further response to Appellees’ continuing effort to narrowly constrain
the Attorney General’s conduct in this appeal, amicus submits the following
illustration of the proper judicial approach to such objections:
Image 1 (2.41" X 2.1") Available for Offline Print
The ridicule reproduced above, no doubt induced by a spat of motions
from other counsel, was only one of fifty-one motions and responses etc. all
filed in various cases before this Court on November 26th, 1979. We have
examined all fifty-one and have determined that over thirty of them ranged
from the unnecessary to the ludicrous.

186
Another irritating series involved a motion to dismiss because an
opposing brief was served on the movant one day late (the movant had
figured wrong, it was not late). This erroneous pleading spawned
successively a:
Response
Motion for Sanctions
Reply to the Response
Motion for Attorneys’ Fees
Reply to the Motion for Attorneys’ Fees
Furthermore, there are no end of pleadings which unashamedly provide,
as their only discernible purpose, a vehicle for the expression of venomous
ill will towards opposing counsel. We also are not blind to the high
incidence of frivolous motions which have as their obvious unstated purpose
(by reason of the moment in time chosen to file them) a tolling of the time
for filing briefs under Rule 9.300(b).
[2] The reader may well be disposed to remark: “So what, everyone has
problems what else is new.” Well, what is new is that we intend to start
imposing sanctions on those who file frivolous pleadings. The function of an
appellate court is to review the record created at the trial level and reconcile
it with the law, not provide a forum for a continuation of the nowadays
vastly exaggerated trial motion practice. In the overwhelming majority of
instances the extra pleadings filed do not assist our function and in fact
impede it inasmuch as every judge on this Court is required to spend part of
each working day giving attention to unnecessary pleadings as applicable
case files are trundled among the judges’ offices in search of the necessary
collegiate action.
[3] We do not seek an end to all motion practice for, in some instances, it
plays a proper role under the Rules. We do, however, demand that counsel
not turn what should be an extraordinary measure into a standard modus
operandi. We know of few experienced appellate practitioners who engage
in such excess. They know better.
In conclusion, we here stop short of issuing an order to show cause why
sanctions should not be imposed under Rule 9.410, not only because we
deem it unfair, without prior warning, to single out the Attorney General for
punishment, but also because we interpret his supplemental response to be a
form of protest with which we have some sympathy.

187
We ask the Bar to take note that fair warning has now been given and ask
further that this order be not itself misused to provide inspiration for
frivolous motions seeking sanctions against other allegedly frivolous
motions. We wish to make that determination for ourselves.
MOORE and GLICKSTEIN, JJ., concur.

Moral Majority, Inc. v. Broward County Chapter of Nat’l Organization for


Women, Inc.
606 So. 2d 630 (Fla. 4th DCA 1992)

PER CURIAM.
This is an appeal from an order denying attorney’s fees.
The Broward County Chapter of the National Organization for Women,
Inc., and others, filed a lawsuit against appellant, The Moral Majority, Inc.
(Moral Majority), and other defendants, alleging conspiracy, violations of
the Florida RICO Act, public nuisance, tortious interference with business
relationships, intentional infliction of emotional distress, false imprisonment,
defamation and trespass. The complaint sought damages and was
accompanied by a verified motion for a temporary restraining order.
Declining to enter a restraining order against appellant, Moral Majority,
the judge stated: “[n]ot one shred of evidence, in the form of testimony or
otherwise was presented concerning” Moral Majority. Subsequently, the trial
court granted appellant’s motion for final summary judgment. Thereafter
appellant filed a motion seeking attorney’s fees pursuant to sections 772.104
and 895.05(7), Florida Statutes (1991), and pursuant to section 57.105,
Florida Statutes (1991).
At the hearing on this motion the trial court made the following
observation:
There is not anything in this case. I mean, there is not one single scintilla
of anything that was done by Reverend Falwell or The Moral Majority in
this case that is not protected by the First Amendment.
Thereafter the trial court, without explanation, denied the motion for
attorney’s fees. We reverse.
The freedoms memorialized in and protected by the constitutions of the
United States and of the State of Florida do not, and should not, include a

188
“right” to institute baseless, harassing, time-consuming and expensive
litigation against a party. While any plaintiff has the power to initiate a
lawsuit, the right to do so is a limited one. The limits are, inter alia, those
prescribed and proscribed by the statutes relied on by appellant here. Those
limits were exceeded by appellees as exemplified by the trial court’s finding
there was not a scintilla of evidence to support their allegations against
appellant.
We reverse and remand with instructions to grant reasonable attorney’s
fees to appellant.
REVERSED AND REMANDED.
HERSEY and POLEN, JJ., and SCHWARTZ, ALAN R., Associate
Judge, concur.
ON POST-OPINION MOTIONS
ORDERED that appellees’ motion filed September 10, 1992, for
rehearing en banc is hereby denied; further,
ORDERED that appellees’ motion filed September 10, 1992, for
rehearing of trial court attorney’s fees is hereby denied; further,
ORDERED that appellees’ request filed September 21, 1992, for
additional certified question and/or for rehearing is hereby denied; further,
ORDERED that appellees’ amendment filed October 9, 1992, to motion
for rehearing is hereby denied; further,
ORDERED that appellant’s motion to strike request for certification and
for sanctions filed September 22, 1992, is hereby denied; further,
ORDERED that appellant’s motion filed September 30, 1992, to strike
request for additional certified question and/or for rehearing is hereby
denied; further,
ORDERED that appellant’s motion for attorney’s fees filed by Jeffrey A.
O'Keefe, counsel for the appellant, is hereby granted and remanded to the
trial court, and pursuant to Fla.R.App.P. 9.400(b), upon remand of this cause
the amount thereof shall be assessed by the trial court upon due notice and
hearing, subject to review by this court under Fla.R.App.P. 9.400(c); further,
ORDERED that appellant’s motion filed October 19, 1992, to strike and
for sanctions is hereby denied; further,
ORDERED the parties’ post-opinion motion practice in this case has
exceeded the proscriptions of Dubowitz v. Century Village East, 381 So.2d

189
252 (4th DCA 1979). The filing of any further motions by either party will
result in the imposition of sanctions.

X. Dismissal

Florida Rules of Appellate Procedure

Rule 9.350. Dismissal of Causes


***
(b) Voluntary Dismissal. A proceeding of an appellant or petitioner may
be dismissed before a decision on the merits by filing a notice of dismissal
with the clerk of the court without affecting the proceedings filed by joinder
or cross-appeal; provided that dismissal shall not be effective until 10 days
after filing the notice of appeal or until 10 days after the time prescribed by
rule 9.110(b), whichever is later.

A. Voluntary Dismissal

State v. Schopp
653 So. 2d 1016 (Fla. 1995)
KOGAN, Justice.
We have for review Schopp v. State, 641 So. 2d 141, (Fla. 4th DCA
1994), which presents the question of whether a trial court’s failure to
conduct adequate inquiry into alleged discovery violations under this Court’s
decision in Richardson v. State, 246 So. 2d 771 (Fla.1971), should continue
to be treated as per se reversible error.FN1 We have jurisdiction pursuant to
article V, section 3(b)(4) of the Florida Constitution.
FN1. The Fourth District Court of Appeal has certified the following
question as being of great public importance:
SHOULD THE PER SE RULE OF SMITH [ v. STATE, 500 So.2d 125
(FLA.1986) ] BE RECONSIDERED IN LIGHT OF THE PRINCIPLES
SET OUT IN [ STATE v.] DIGUILIO [, 491 So.2d 1129 (FLA.1986) ]?
Schopp v. State, 641 So.2d 141, 143 (Fla. 4th DCA 1994).
NOTICE OF VOLUNTARY DISMISSAL

190
[1] Before we address the certified question, we must address Schopp’s
contention that this Court lacks jurisdiction because he filed a notice of
voluntary dismissal in the district court prior to the final disposition of his
appeal. The decision under review was issued on July 6, 1994. Schopp filed
a timely motion for rehearing and the State filed a timely petition for review
in this Court. While the motion for rehearing was pending in the district
court, Schopp filed a notice of voluntary dismissal pursuant to Florida Rule
of Appellate Procedure 9.350(b). The State filed a motion to strike the
notice. On August 1, 1994, Schopp sought a writ of mandamus from this
Court compelling the district court to dismiss the appeal. On August 15, the
district court denied the motion for rehearing and granted the State’s motion
to strike the notice. Then, on September 1, Schopp filed a motion to dismiss
the State’s petition for review alleging that this Court lacks subject matter
jurisdiction because he filed his notice of voluntary dismissal before the
district court’s decision became final. This Court denied both the petition for
writ of mandamus and the motion to dismiss.
Florida Rule of Appellate Procedure 9.350(b) provides in pertinent part:
A proceeding of an appellant or petitioner may be dismissed before a
decision on the merits by filing a notice of dismissal with the clerk of the
court....
Schopp argues that since his notice of voluntary dismissal was filed
before the district court’s decision became final, his appeal had to be
dismissed as a matter of right; and therefore there is no decision for this
Court to review.
[2] [3] We agree with Schopp that his notice of dismissal was timely filed
under rule 9.350(b) because there was no “decision on the merits” until the
district court disposed of his motion for rehearing. See Fla.R.App.P.
9.020(g)(1) (a decision is not rendered for appellate purposes until
disposition of motion for rehearing); Cf. Haverley v. Clann, 196 So.2d 38
(Fla. 2d DCA 1967) (notice of dismissal due to settlement would have been
timely filed under rule providing for the filing of such notice “before a
decision on the merits” if notice had been filed prior to resolution of motion
for rehearing). However, the fact that the notice was timely filed does not
end our analysis. Even where a notice of voluntary dismissal is timely filed,
a reviewing court has discretion to retain jurisdiction and proceed with the
appeal. Cf. Ervin v. Capital Weekly Post, 97 So.2d 464 (Fla.1957) (this
Court retained jurisdiction to consider issue of public importance where

191
appellees sought dismissal prior to initial decision); Phibro Resources Corp.
v. Department of Environmental Regulation, 579 So.2d 118 (Fla. 1st DCA)
(district court retained jurisdiction where case had been settled while
motions for rehearing were pending and notices of voluntary dismissal had
been filed prior to decision on rehearing), cause dismissed, 592 So.2d 679
(Fla.1991). This is particularly true where, as here, the case presents a
question of public importance and substantial judicial labor has been
expended as evidenced by the issuance of an initial opinion. Accordingly, it
was within the district court’s discretion to retain jurisdiction and refuse to
withdraw its opinion certifying the question of public importance to this
Court.
CERTIFIED QUESTION
[4] Next, we turn to the certified question which arises in the following
context. [ * * * ] . . . we now recognize that there are cases, such as this,
where a reviewing court can say beyond a reasonable doubt that the defense
was not prejudiced by the underlying violation and thus the failure to make
adequate inquiry was harmless error. While this case is clearly the exception
rather than the rule, it illustrates that a per se reversal rule is not warranted in
this context. It also leads us to agree with the State that continued
application of the per se reversal rule to all Richardson violations would
have the effect of “elevating form over substance,” contrary to section
924.33. DiGuilio, 491 So.2d at 1135. [ * * *]
Because it is clear in this case that the State’s failure to include the
officer on its original witness list could not have materially hindered the
defense, the trial court’s failure to adequately inquire into what, if any,
corrective measures should have been taken was harmless beyond a
reasonable doubt. During discovery, defense counsel was given the police
report, which was consistent with the officer’s testimony at trial. The
challenged testimony was cumulative to other testimony, including that of
the defendant. And, even if the officer had been listed as a witness and the
defense had deposed him, there would have been nothing in the officer’s
testimony that could have supported a strategy different from that taken-to
admit that Schopp committed burglary but maintain that he was unarmed
and did not know that he was stealing a firearm. Moreover, because Schopp
effectively “won” his case, there is no reasonable possibility that a change in
trial tactics could have benefited him.

192
Accordingly, we quash the decision under review and remand for
proceedings consistent with this opinion.
It is so ordered.
GRIMES, C.J., and SHAW and WELLS, JJ., concur.
HARDING, J., dissents with an opinion, in which OVERTON, J.,
concurs.
ANSTEAD, J., recused.
HARDING, Justice, dissenting. [ * * *]

B. Settlement

Florida Rules of Appellate Procedure

Rule 9.350. Dismissal of Causes


(a) Dismissal of Causes When Settled. When any cause pending in the
court is settled before a decision on the merits, the parties shall
immediately notify the court by filing a signed stipulation for dismissal.

The appellate court may write an opinion even after the case has been settled, if the issue
is an important issue that is likely to recur. See Pleasures II Adult Video, Inc. v. City of
Sarasota, 833 So. 2d 185 (Fla. 2d DCA 2002).

C. Involuntary Dismissal

Gazil v. Gazil
343 So. 2d 595 (Fla. 1977)
ADKINS, Justice.
By petition for certiorari, we have for review a decision of the District
Court of Appeal, Fourth District (Gazil v. Gazil, 313 So. 2d 484, (opinion
filed January 30, 1975)), which allegedly conflicts with a prior decision of
this Court (Bronk v. Bronk, 46 Fla. 474, 35 So. 870 (1903)), on the same
point of law. Article V, Section 3(b)(3), Florida Constitution.

193
In March 1974, a final judgment of dissolution of marriage was entered,
and on April 1, 1974, petitioner timely filed a notice of appeal seeking to
challenge various financial rulings in the final judgment.
Subsequently, in May and September 1974, after the notice of appeal was
filed, the trial court entered supplemental orders awarding temporary
alimony pending appeal and court costs and attorneys’ fees to respondent. In
November 1974 and January 1975, petitioner was adjudged by the trial
judge to be in contempt for failure to pay costs, attorneys’ fees and
temporary alimony under the supplemental orders. Respondent filed a
motion to dismiss the appeal on January 27, 1975, setting out these matters
and suggesting that petitioner was a fugitive from justice. The cause was
argued on the merits before the appellate court on January 28, 1975, at
which time the court also heard argument on respondent’s motion. The order
of dismissal which petitioner now attacks was entered on January 30, 1975.
The Constitution of Florida creates the right to appeal. As said in Willey
v. W. J. Hoggson Corporation, 89 Fla. 446, 105 So. 126 (1925):
‘While the Constitution provides that ‘the Supreme Court shall have
appellate jurisdiction in all cases at law and in equity originating in circuit
courts,’ the statutes regulate the appellate processes and procedure and the
stages in a cause at which and the time within which the appropriate
appellate process and procedure may be utilized; but Such statutory
regulations and the appellate procedure had under the regulations cannot
legally be used to impair the right to invoke the appellate jurisdiction
conferred upon the Supreme Court by the Constitution or to obstruct the
powers of the court to make its appellate jurisdiction effective.’ (Emphasis
supplied.) At p. 128.
The question before the Court is whether an appeal can be dismissed for
the appellant’s disobedience of an order of the trial court.
In an Annotation, 49 A.L.R.2d 1425 (1956), the following appears:
‘In contrast with the relative clarity of the law with respect to the
permissibility of the dismissal of an appeal where the appellant has
disobeyed an order of the appellate court, there is considerable judicial
disagreement as to whether an appeal can be dismissed for the appellant’s
disobedience of an order of the trial court. . . . the reader will note that there
are not only interjurisdiction conflicts, but also-in California, Florida,
Illinois-apparent intrajurisdictional conflicts in the decisions.’ At 1429.

194
‘The question whether an appellate court may dismiss the appeal for the
appellant’s failure to obey a trial court’s order has been before the Florida
courts in two cases, the results in which are diametrically opposed.’ At 1438.
The Florida cases referred to in the Annotation are Bronk v. Bronk,
supra, and Palmer v. Palmer, 28 Fla. 295, 9 So. 657 (1891). The annotator
points out that the opinion in Bronk, supra, contains no reference to Palmer,
and the inconsistency is at least superficial.
In Bronk writs of Ne exeat and injunction were issued against a
defendant who then escaped from custody and finally remained beyond the
court’s jurisdiction, so that the trial court was powerless to enforce the writs.
The appellate court refused to hear an appeal on the question of granting the
Ne exeat writ, saying:
‘The appellant having, since he took this appeal, voluntarily placed
himself in contempt of the orders of the court below, and having gone
beyond the jurisdiction of this court, so that no order or decree it might make
in the premises could be personally enforced against him, we are confronted
with the question whether he has a right to a hearing on questions based on
the injunction and ne exeat orders. Doubtless the general rule is that a party
is not deprived of any strict legal right to be heard by placing himself in
contempt of the court, especially if there be other means available by which
the court may enforce its orders. . . . Governed by these views, we decline to
entertain and hear the assignments of error of John P. Bronk numbered 1, 2,
3, 6 and 7.’ (Emphasis supplied.) 35 So. 870, at 871.
The assignments of error referred to in the opinion related to the writ of
Ne exeat. The Court did not refuse to entertain assignment of error number
4, relating to the granting of the temporary injunction, and assignment of
error number 5, relating to an order on the pleadings. In other words, the
Court held that the entire appeal should not be dismissed.
In Bronk, supra, the Court held that a party in contempt is not deprived of
his right to be heard, if there are other means available to enforce the Court’s
order. Where one absconds from the jurisdiction, of course, there is no way
for the Court to enforce its order, and an appeal could properly be dismissed.
But even in Bronk the Court retained jurisdiction of the case so that other
matters unrelated to the contempt could be considered.
In Morris v. Rabara, 145 So. 2d 265 (Fla.2d DCA 1962), it appeared that
the appellant had disobeyed an order of the trial court. The appellate court in
granting relief said:

195
‘The motion of the appellee will be granted, and an order will be entered
that this appeal be dismissed unless it shall be made to appear to this court,
on or before thirty (30) days from the date of the filing of this opinion, that
the appellant has either purged himself of his contempt or is in the custody
of the Sheriff of Palm Beach County.’ At 267.
Durham v. Durham, 297 So. 2d 857 (Fla.4th DCA 1974), involved a
situation where the appellant had been adjudged in contempt for failure to
comply with an order of the trial court. In granting the motion to dismiss, the
court said:
‘Accordingly, the motion to dismiss will be granted and an order will be
entered that this appeal be dismissed unless it shall be made to appear to this
court, on or before 15 days from the date of the filing of this opinion, that
the appellant has either purged himself of contempt or is in the custody of
the Sheriff of Orange County.’ At 858.
[1] From these decisions it appears the rule in Florida is as follows:
Where the appellant has disobeyed an order of the trial court, the appellate
court may, in its discretion, either entertain or dismiss an appeal. However,
where a dismissal is ordered it is mandatory that the disobedient appellant
must be given a period of grace, prior to the effective date of the dismissal,
in which to comply with the disobeyed order.
[2] The order of the District Court of Appeal in the case Sub judice
conflicts with these decisions in that petitioner was not given an opportunity
to purge himself so that he could be heard on the merits of the appeal.
[3] If an appellant husband absconds, the situation would be quite
different and there would be no need to allow a period of time within which
he could purge himself and be heard on appeal. If the contempt is for
nonpayment of money, he should be given an opportunity to purge himself
so that he could be heard on the merits of the case.
The decision of the District Court of Appeal is quashed and the cause
remanded for further proceeding in accordance with the views expressed in
this opinion.
It is so ordered.
OVERTON, C.J., and SUNDBERG and HATCHETT, JJ., concur.
ROBERTS (Retired), J., concurs specially with an opinion, with which
ADKINS and HATCHETT, JJ., concur.
BOYD, J., concurs in part and dissents in part with an opinion.

196
ENGLAND, J., dissents.
ROBERTS, Justice (Retired), concurring specially. * * *
ADKINS and HATCHETT, JJ., concur.
BOYD, Justice, concurring in part and dissenting in part.

Griffis v. State
759 So. 2d 668 (Fla. 2000).
SHAW, J.
We have for review Griffis v. State, 703 So. 2d 522 (Fla. 1st DCA 1997),
wherein the district court certified the following question:
Should the holding in State v. Gurican, 576 So. 2d 709 (Fla.1991), be re-
evaluated in light of Ortega-Rodriguez v. United States, 507 U.S. 234, 113
S.Ct. 1199, 122 L.Ed.2d 581 (1993)?
Griffis, 703 So. 2d at 523. We have jurisdiction. See art. V, § 3(b)(4), Fla.
Const. We answer in the affirmative and quash Griffis.
Griffis was charged in 1989 with committing multiple counts of sexual
battery and lascivious assault on a five-year old male in 1988. Griffis
absconded in 1990 following jury selection and was tried in absentia, and
the jury found him guilty as charged. He subsequently was arrested in
Virginia in 1996 and was returned to Florida. He was adjudicated guilty on
June 5, 1996, and was sentenced to concurrent life terms with twenty-five
year minimum mandatory sentences. The district court dismissed his appeal
on December 30, 1997, pursuant to State v. Gurican, 576 So. 2d 709
(Fla.1991), wherein this Court held that “[the] appellate courts of this state
shall dismiss the appeal of a convicted defendant not yet sentenced who
flees the jurisdiction before filing a notice of appeal.” Id. at 712.
The State contends that Gurican was correctly decided and that the
present district court decision should be approved. The State’s position,
while attractive at first blush, is contrary to United States Supreme Court
precedent.
The defendant in Gurican absconded after the jury returned its verdict
but before the court adjudicated her guilty. She voluntarily returned to the
jurisdiction four years later, and the court adjudicated her guilty and
sentenced her. She subsequently filed an appeal. The district court denied the

197
State’s motion to dismiss the appeal, and this Court quashed that decision,
ruling as follows:
[W]here the convicted defendant escapes and fails to appear for
sentencing, we advise trial courts to proceed in absentia and render their
final judgments adjudicating the defendant guilty. Thus, the thirty-day
period for filing an appeal will commence running unless it is tolled until the
court disposes of any authorized and timely post-trial motion.... If the
defendant fails to return and timely file an appeal of the conviction, the
appellate court shall dismiss any later appeal unless the defendant can
establish that the escape or failure to appear was legally justified.
Gurican, 576 So. 2d at 712. Thus, under the analysis in Gurican, the line
of demarcation governing appellate dismissal is whether the trial has begun,
for if a defendant absconds after that point, the process is automatic: The
court will proceed with the trial in absentia and render judgment, and the
thirty day appeal period will commence.
In the federal system, on the other hand, the traditional line of
demarcation governing appellate dismissal is more logical: It is not whether
the trial has begun, but whether the appellate process has begun. The United
States Supreme Court in Ortega-Rodriguez v. United States, 507 U.S. 234,
240, 113 S.Ct. 1199, 122 L.Ed.2d 581 (1993), addressed the issue posed in
Gurican and reached a contrary result. The Court in Ortega-Rodriguez noted
preliminarily that it is well-settled in the federal system that when a
defendant absconds after the appellate process has begun the appellate court
may dismiss the appeal. This long-standing rule is based on several
common-sense considerations: enforceability, efficiency, unentitlement, and
deterrence.FN1
FN1. The United States Supreme Court explained: (1) There is no point
in continuing with the appeal if the appellate judgment will be
unenforceable, i.e., if the defendant will not be present to submit to the
judgment; (2) the dismissal rule promotes an orderly, efficient appellate
process by allowing the court to dispose of the matter promptly; (3) a
defendant who flouts the appellate process is not entitled to pursue an
appeal; and (4) the dismissal rule deters escapes. See Ortega-Rodriguez, 507
U.S. at 240-42, 113 S.Ct. 1199.
[1] [2] Where the defendant absconds and returns before entering the
appellate process, on the other hand, the above considerations are attenuated:

198
[T]he justifications we have advanced for allowing appellate courts to
dismiss pending fugitive appeals all assume some connection between a
defendant’s fugitive status and the appellate process, sufficient to make an
appellate sanction a reasonable response. These justifications are necessarily
attenuated when applied to a case in which both flight and recapture occur
while the case is pending before the [trial] court, so that a defendant’s
fugitive status at no time coincides with his appeal.
Ortega-Rodriguez, 507 U.S. at 244, 113 S.Ct. 1199 (footnote omitted)
(emphasis added). Specifically, the above rationales of enforceability,
efficiency, and unentitlement are not dispositive.FN2 The deterrence
rationale, too, is inapplicable, for the trial court is equipped with a variety of
sanctions that will deter this conduct.FN3
FN2. See Ortega-Rodriguez, 507 U.S. at 244, 113 S.Ct. 1199 (“A
defendant returned to custody before he invokes the appellate process
presents no risk of unenforceability; he is within control of the appellate
court throughout the period of appeal and issuance of judgment.”); id. at
245, 113 S.Ct. 1199 (noting that the court that is disrupted by a defendant’s
fugitivity during trial is the trial court, not the appellate court); id.
(explaining that a defendant’s fugitivity during trial flouts the dignity and
authority of the trial court and thus disentitles the defendant to call upon the
resources of that court, not the appellate court).
FN3. The Court explained why the deterrence rationale is inapplicable
when the defendant absconds before entering the appellate process:
Finally, [the] deterrence rationale offers little support for the [dismissal]
rule. Once jurisdiction has vested in the appellate court ... then any deterrent
to escape must flow from the appellate consequences, and dismissal may be
an appropriate sanction by which to deter. Until that time, however, the
[trial] court is quite capable of defending its own jurisdiction. While a case
is pending before the [trial] court, flight can be deterred with the threat of a
wide range of penalties available to the [trial] court judge.
Moreover, should this deterrent prove ineffective, and a defendant flee
while his case is before a [trial] court, the [trial] court is well situated to
impose an appropriate punishment. While an appellate court has access only
to the blunderbuss of dismissal, the [trial] court can tailor a more finely
calibrated response. Most obviously, because flight is a separate offense
punishable under the [federal statutes], the [trial] court can impose a separate

199
sentence that adequately vindicates the public interest in deterring escape
and safeguards the dignity of the court....
Indeed ... punishment by appellate dismissal introduces an element of
arbitrariness and irrationality into sentencing for escape. Use of the dismissal
sanction as, in practical effect, a second punishment for a defendant’s flight
is almost certain to produce the kind of disparity in sentencing that the ...
Sentencing Guidelines were intended to eliminate.
Ortega-Rodriguez, 507 U.S. at 247-48, 113 S.Ct. 1199 (citations and
footnotes omitted).
The Court in Ortega-Rodriguez concluded that, absent some detrimental
connection between the defendant’s fugitive status and the appellate process,
dismissal of an appeal is generally improper where a defendant absconds and
returns prior to filing the appeal:
Accordingly, we conclude that while dismissal of an appeal pending
while the defendant is a fugitive may serve substantial interests, the same
interests do not support a rule of dismissal for all appeals filed by former
fugitives, returned to custody before invocation of the appellate system.
Absent some connection between a defendant’s fugitive status and his
appeal as provided when a defendant is at large during “the ongoing
appellate process,” the justifications advanced for dismissal of fugitives’
pending appeals generally will not apply.
Id. at 249, 113 S.Ct. 1199 (citation omitted).FN4 The offensive conduct is
best sanctioned by the trial court, not the appellate court. FN5 The Court
remanded the case to the lower appellate court to determine whether there
was a sufficiently detrimental nexus between Ortega-Rodriguez’s fugitive
status and his appeal to warrant appellate dismissal.
FN4. The Court noted that at times the connection between the
defendant’s absconding and the appellate process may be sufficiently
detrimental to warrant dismissal of the appeal:
We do not ignore the possibility that some actions by a defendant, though
they occur while his case is before the [trial] court, might have an impact on
the appellate process sufficient to warrant an appellate sanction. For that
reason, we do not hold that a court of appeals is entirely without authority to
dismiss an appeal because of fugitive status predating the appeal. For
example ... a long escape, even if ended before sentencing and appeal, may
so delay the onset of appellate proceedings that the Government would be
prejudiced in locating witnesses and presenting evidence at retrial after a

200
successful appeal. We recognize that this problem might, in some instances,
make dismissal an appropriate response.
Ortega-Rodriguez, 507 U.S. at 249, 113 S.Ct. 1199 (citations omitted).
FN5. The Court explained as follows:
In short, when a defendant’s flight and recapture occur before appeal, the
defendant’s former fugitive status may well lack the kind of connection to
the appellate process that would justify an appellate sanction of dismissal. In
such cases, fugitivity while a case is pending before a [trial] court, like other
contempts of court, is best sanctioned by the [trial] court itself.
Id. at 251, 113 S.Ct. 1199 (emphasis added).
The question posed in the present case is whether this Court should
recede from Gurican in light of Ortega-Rodriguez. We answer in the
affirmative for several reasons. First, the Florida Constitution, unlike its
federal counterpart, contains an express right of appeal,FN6 and logic dictates
that this state constitutional right should receive at least the same level of
protection as the federal statutory right.FN7 Second, Ortega-Rodriguez is not
inconsistent with our established rules of procedure in Florida. And finally,
as explained below, Ortega-Rodriguez comports with the statutory scheme
in Florida.
FN6. See Art. V, § 4(b)(1), Fla. Const. (“District courts of appeal shall
have jurisdiction to hear appeals, that may be taken as a matter of right, from
final judgments or orders of trial courts....”); Amendments to the Rules of
Appellate Procedure, 685 So. 2d 773, 774 (Fla.1996) (“[W]e construe the
language of article V, section 4(b) as a constitutional protection of the right
to appeal.”).
FN7. See generally Traylor v. State, 596 So. 2d 957, 962 (Fla.1992) (“In
any given state, the [federal law] thus represents the floor for basic
freedoms; the state constitution, the ceiling.”).
[3] In the present case, the fact that Griffis absconded during trial several
years earlier is insufficient under Ortega-Rodriguez to warrant automatic
dismissal of his appeal. Griffis absconded and returned prior to filing his
notice of appeal, and the offense thus was committed while he still was
within the jurisdiction of the trial court, not the appellate court. Florida’s
trial courts have at their disposal a variety of sanctions with which to punish
this affront to their authority. A defendant can be charged with and
prosecuted for (where appropriate) escape FN8 or failure to appear, FN9 or the

201
court can hold the defendant in contempt FN10 or assess status points under
the sentencing guidelines FN11 or the Criminal Punishment Code.FN12 The trial
court in the present case thus had before it a number of options.
FN8. See § 944.40, Fla. Stat. (1997).
FN9. See § 843.15, Fla. Stat. (1997).
FN10. See §§ 38.22, 900.04 Fla. Stat. (1997).
FN11. See § 921.0011, Fla. Stat. (1997).
FN12. See § 921.0021, Fla. Stat. (1997).
The district court, on the other hand, pursuant to the reasoning of Ortega-
Rodriguez, should have dismissed Griffis’s appeal only if there had been an
affront to its authority-i.e., only if there was a sufficiently detrimental
connection between Griffis’ fugitive status and the appellate process. For
instance, if it had been shown by the State that during his fugitivity crucial
evidence grew stale or key witnesses became unavailable, then the State’s
burden at retrial (i.e., its obligation to establish guilt beyond a reasonable
doubt) would have been unduly prejudiced and the district court could have
dismissed the appeal. Instead of making this determination, the district court
felt compelled under Gurican to summarily dismiss the appeal.
[4] [5] In sum, it is well settled that when a defendant absconds after
filing an appeal, the appellate court has the authority to dismiss the appeal.
FN13
However, where the defendant absconds and returns before filing an
appeal, trial courts are better suited to address the matter. The latter
misconduct is committed within the jurisdiction of the trial court, and the
Florida Legislature has armed that court with a series of statutory sanctions
(which does not include appellate dismissal). The court can fine-tune a
response to fit the circumstances of each case. The district court, on the other
hand, has only a single big gun in its arsenal-i.e., appellate dismissal-and this
court-made blunderbuss should be fired only when the defendant’s
misconduct has unduly prejudiced the State.FN14
FN13. See, e.g., Bretti v. Wainwright, 225 So. 2d 516 (Fla.1969),
language expunged, 255 So. 2d 266 (Fla.1971).
FN14. Appellate dismissal should be imposed rarely, for an element of
arbitrariness is injected into the criminal justice system whenever the
appellate safety net is withdrawn. See Ortega-Rodriguez, 507 U.S. at 248,
113 S.Ct. 1199 (“Indeed ... punishment by appellate dismissal introduces an
element of arbitrariness and irrationality into sentencing for escape.”); see

202
also Estelle v. Dorrough, 420 U.S. 534, 544, 95 S.Ct. 1173, 43 L.Ed.2d 377
(1975) (Stewart, J., dissenting) (“If an escaped felon has been convicted in
violation of law, the loss of his right to appeal results in his serving a
sentence that under law was erroneously imposed. If, on the other hand, his
trial was free of reversible error, the loss of his right to appeal results in no
punishment at all. And those whose appeals would have been reversed if
their appeals had not been dismissed serve totally disparate sentences,
dependent not upon the circumstances of their escape, but upon whatever
sentences may have been meted out under their invalid convictions.”).
We cannot fault the district court for following our holding in Gurican,
but we now recede from that decision in light of Ortega-Rodriguez. We
answer the certified question in the affirmative, quash the district court
decision below, and remand this case so the district court can decide if
Griffis’ fugitivity has unduly prejudiced the State.
It is so ordered.
HARDING, C.J., and WELLS, ANSTEAD, PARIENTE and LEWIS, JJ.,
concur.
QUINCE, J., concurs in result only.

XI. Briefs

Florida Rules of Appellate Procedure

Rule 9.210. Briefs


(a) Generally. In addition to briefs on jurisdiction under rule 9.120(d), the
only briefs permitted to be filed by the parties in any one proceeding are
the initial brief, the answer brief, a reply brief, and a cross-reply brief. All
briefs required by these rules shall be prepared as follows:
(1) Briefs shall be printed, typewritten, or duplicated on opaque,
white, unglossed 8 1/2-by-11 inch paper.
(2) The lettering in briefs shall be black and in distinct type, double-
spaced, with margins no less than 1 inch. Lettering in script or type made
in imitation of handwriting shall not be permitted. Footnotes and quotations
may be single spaced and shall be in the same size type, with the same
spacing between characters, as the text. Computer-generated briefs shall
be submitted in either Times New Roman 14-point font or Courier New 12-

203
point font. All computer-generated briefs shall contain a certificate of
compliance signed by counsel, or the party if unrepresented, certifying that
the brief complies with the font requirements of this rule. The certificate of
compliance shall be contained in the brief immediately following the
certificate of service.
(3) Briefs shall be securely bound in book form and fastened along
the left side in a manner that will allow them to lie flat when opened or be
securely stapled in the upper left corner. Headings and subheadings shall
be at least as large as the brief text and may be single spaced.
(4) The cover sheet of each brief shall state the name of the court,
the style of the cause, including the case number if assigned, the lower
tribunal, the party on whose behalf the brief is filed, the type of brief, and
the name and address of the attorney filing the brief.
(5) The initial and answer briefs shall not exceed 50 pages in
length, provided that if a cross-appeal has been filed, the answer
brief/initial brief on cross-appeal shall not exceed 85 pages. Reply briefs
shall not exceed 15 pages in length; provided that if a cross-appeal has
been filed, the reply brief shall not exceed 50 pages, not more than 15 of
which shall be devoted to argument replying to the answer portion of the
appellee/cross-appellant's brief. Cross-reply briefs shall not exceed 15
pages. Briefs on jurisdiction shall not exceed 10 pages. The tables of
contents and cita-tions of authorities, and certificates of service and
compliance, shall be excluded from the computation. Longer briefs may be
permitted by the court.
(b) Contents of Initial Brief. The initial brief shall contain the following, in
order:
(1) A table of contents listing the issues presented for review, with
references to pages.
(2) A table of citations with cases listed alphabetically, statutes and
other authorities, and the pages of the brief on which each citation
appears. See rule 9.800 for a uniform citation system.
(3) A statement of the case and of the facts, which shall include the
nature of the case, the course of the proceedings, and the disposition in
the lower tribunal. References to the appropriate volume and pages of the
record or transcript shall be made.
(4) A summary of argument, suitably paragraphed, condensing
succinctly, accurately, and clearly the argument actually made in the body
of the brief. It should not be a mere repetition of the headings under which
the argument is arranged. It should seldom exceed 2 and never 5 pages.
(5) Argument with regard to each issue including the applicable

204
appellate standard of review.
(6) A conclusion, of not more than 1 page, setting forth the precise
relief sought.
(c) Contents of Answer Brief. The answer brief shall be prepared in the
same manner as the initial brief; provided that the statement of the case
and of the facts may be omitted. If a cross-appeal has been filed, the
answer brief shall include the issues in the cross-appeal that are
presented for review, and argument in support of those issues.
(d) Contents of Reply Brief. The reply brief shall contain argument in
response and rebuttal to argument presented in the answer brief.
(e) Contents of Cross-Reply Brief. The cross-reply brief is limited to
rebuttal of argument of the cross-appellee.
(f) Times for Service of Briefs. The times for serving jurisdiction and
initial briefs are prescribed by rules 9.110, 9.120, 9.130, and 9.140. Unless
otherwise required, the answer brief shall be served within 20 days after
service of the initial brief; the reply brief, if any, shall be served within 20
days after service of the answer brief; and the cross-reply brief, if any,
shall be served within 20 days thereafter.
(g) Filing with Courts. The filing requirements of the courts are as
follows:
(1) Circuit Courts.Original and 1 copy.
(2) District Courts of Appeal.Original and 3 copies.
(3) Supreme Court.Original and 7 copies; except that 5 copies only
shall accompany the original jurisdictional briefs prescribed in rule
9.120(d).
(h) Citations. Counsel are requested to use the uniform citation system
prescribed by rule 9.800.

The point of an appellate brief is to present arguments in support of the points on appeal,
and without further elucidation on the arguments, this court may not engage in
meaningful appellate review. See Florida Hometown Democracy, Inc. v. Cobb, 953 So.
2d 666 (Fla. 1st DCA 2007).

205
A. Duty to File

Airmark Engines, Inc. v. Airpark Aviation, Inc.


599 So. 2d 1032 (Fla. 4th DCA 1992)
PER CURIAM.
AFFIRMED.
POLEN, J., concurs.
LETTS, J., concurs specially with opinion.
FARMER, J., dissents with opinion.
LETTS, Judge, specially concurring.
I concur specially to comment on the dissent. I have never before seen
nor heard that the failure to file a brief might be a confession of error. Were
this so, every answer brief unaccompanied by a reply brief would result in
invariable affirmance. Surely the appellant must demonstrate error.
FARMER, Judge, dissenting.
A defendant’s excuse for not responding timely to a validly served
complaint must be established by proof, not by unsworn argument. Bil-Jax
Inc. v. Williamson, 497 So.2d 1350 (Fla. 4th DCA 1986); Gibraltar Service
Corp. v. Lone and Associates Inc., 488 So. 2d 582 (Fla. 4th DCA 1986);
C.E. Peters Landclearing Inc. v. Gossington, 487 So. 2d 319 (Fla. 4th
DCA), rev. denied, 496 So. 2d 142 (Fla.1986); Hall v. Byington, 421 So. 2d
817 (Fla. 4th DCA 1982); and Yu v. Weaver, 364 So. 2d 539 (Fla. 4th DCA
1978). The usual form of this proof is an affidavit, but testimony at the
hearing will also do.
Here, there was neither. I reach this conclusion because appellant says
there was no testimony at the hearing on appellee’s motion to vacate the
default final judgment, and the motion was unsworn. Appellee did not file
any brief in this appeal, a failure which I take to be a concession of the
accuracy of appellant’s account of the proceedings below.
A reversal is unavoidable.

206
B. Form

Florida Rules of Appellate Procedure

Rule 9.210. Briefs


(a) Generally. In addition to briefs on jurisdiction under rule 9.120(d), the
only briefs permitted to be filed by the parties in any one proceeding are
the initial brief, the answer brief, a reply brief, and a cross-reply brief. All
briefs required by these rules shall be prepared as follows:
(1) Briefs shall be printed, typewritten, or duplicated on opaque,
white, unglossed 8 1/2-by-11 inch paper.
(2) The lettering in briefs shall be black and in distinct type, double-
spaced, with margins no less than 1 inch. Lettering in script or type made
in imitation of handwriting shall not be permitted. Footnotes and quotations
may be single spaced and shall be in the same size type, with the same
spacing between characters, as the text. Computer-generated briefs shall
be submitted in either Times New Roman 14-point font or Courier New 12-
point font. All computer-generated briefs shall contain a certificate of
compliance signed by counsel, or the party if unrepresented, certifying that
the brief complies with the font requirements of this rule. The certificate of
compliance shall be contained in the brief immediately following the
certificate of service.
(3) Briefs shall be securely bound in book form and fastened along
the left side in a manner that will allow them to lie flat when opened or be
securely stapled in the upper left corner. Headings and subheadings shall
be at least as large as the brief text and may be single spaced.
(4) The cover sheet of each brief shall state the name of the court,
the style of the cause, including the case number if assigned, the lower
tribunal, the party on whose behalf the brief is filed, the type of brief, and
the name and address of the attorney filing the brief.
(5) The initial and answer briefs shall not exceed 50 pages in
length, provided that if a cross-appeal has been filed, the answer
brief/initial brief on cross-appeal shall not exceed 85 pages. Reply briefs
shall not exceed 15 pages in length; provided that if a cross-appeal has
been filed, the reply brief shall not exceed 50 pages, not more than 15 of
which shall be devoted to argument replying to the answer portion of the
appellee/cross-appellant's brief. Cross-reply briefs shall not exceed 15
pages. Briefs on jurisdiction shall not exceed 10 pages. The tables of
contents and cita-tions of authorities, and certificates of service and
compliance, shall be excluded from the computation. Longer briefs may be

207
permitted by the court.
***

C. Time for Service

Florida Rules of Appellate Procedure

Rule 9.110. Appeal Proceedings to Review Final Orders of Lower


Tribunals and Orders Granting New Trial in Jury and Non-Jury Cases
***
(f) Briefs. Appellant's initial brief shall be served within 70 days of filing
the notice. Additional briefs shall be served as prescribed by rule 9.210.
***

Rule 9.130. Proceedings to Review Non-Final Orders and Specified


Final Orders
***
(e) Briefs. Appellant's initial brief, accompanied by an appendix as
prescribed by rule 9.220, shall be served within 15 days of filing the notice.
Additional briefs shall be served as prescribed by rule 9.210
***

Rule 9.140. Appeal Proceedings in Criminal Cases


***
(g) Briefs. Initial briefs shall be served within 30 days of service of the
record or designation of appointed counsel, whichever is later. Additional
briefs shall be served as prescribed by rule 9.210.
***

Rule 9.120. Discretionary Proceedings to Review Decisions of


District Courts of Appeal
***
(d) Briefs on Jurisdiction. Petitioner's brief, limited solely to the issue of
the supreme court's jurisdiction and accompanied by an appendix

208
containing only a conformed copy of the decision of the district court of
appeal, shall be served within 10 days of filing the notice. Respondent's
brief on jurisdiction shall be served within 20 days after service of
petitioner's brief. Formal requirements for both briefs are specified in rule
9.210. No reply brief shall be permitted. If jurisdiction is invoked under rule
9.030(a)(2)(A)(v) (certifications of questions of great public importance by
the district courts to the supreme court), no briefs on jurisdiction shall be
filed.
***
(f) Briefs on Merits. Within 20 days of rendition of the order accepting or
postponing decision on jurisdiction, the petitioner shall serve the initial
brief on the merits, accompanied by an appendix that must include a
conformed copy of the decision of the district court of appeal.
***

Rule 9.210. Briefs


***
(f) Times for Service of Briefs. The times for serving jurisdiction and
initial briefs are prescribed by rules 9.110, 9.120, 9.130, and 9.140. Unless
otherwise required, the answer brief shall be served within 20 days after
service of the initial brief; the reply brief, if any, shall be served within 20
days after service of the answer brief; and the cross-reply brief, if any,
shall be served within 20 days thereafter.
***

D. Appendix

Florida Rules of Appellate Procedure

Rule 9.220. Appendix


(a) Purpose.The purpose of an appendix is to permit the parties to prepare
and transmit copies of those portions of the record deemed necessary to
an understanding of the issues presented. It may be served with any
petition, brief, motion, response, or reply but shall be served as otherwise
required by these rules. In any proceeding in which an appendix is
required, if the court finds that the appendix is incomplete, it shall direct a
party to supply the omitted parts of the appendix. No proceeding shall be

209
determined until an opportunity to supplement the appendix has been
given.
(b) Contents.The appendix shall contain an index and a conformed copy
of the opinion or order to be reviewed and may contain any other portions
of the record and other authorities. It shall be separately bound or
separated from the petition, brief, motion, response, or reply by a divider
and appropriate tab. Asterisks should be used to indicate omissions in
documents or testimony of witnesses. If the appendix includes documents
filed before January 1991 on paper measuring 8 ½ by 14 inches, the
documents should be reduced in copying to 8 ½ by 11 inches, if
practicable. If impracticable, the appendix may measure 8 ½ by 14 inches,
but it should be bound separately from the document that it accompanies.

E. Notice of Supplemental Authority

Florida Rules of Appellate Procedure

Rule 9.225. Notice of Supplemental Authority


Notices of supplemental authority may be filed with the court before a
decision has been rendered to call attention to decisions, rules, statutes,
or other authorities that are significant to the issues raised and that have
been discovered after the last brief served in the cause. The notice may
identify briefly the points argued on appeal to which the supplemental
authorities are pertinent, but shall not contain argument. Copies of the
supplemental authorities shall be attached to the notice.

Ogden Allied Services v. Panesso


619 So. 2d 1023 (Fla. 1st DCA 1993)
PER CURIAM.
This workers’ compensation appeal involves only one issue-whether the
judge of compensation claims erred when, because of a perceived discovery
violation, he refused to permit the employer and servicing agent to present
surveillance evidence. Briefs dealing exhaustively with the issue had been
filed by August of last year. Supplemental briefs were filed in December of
last year and January of this year. In due course, the parties were notified of
the date for oral argument.

210
On the afternoon preceding the day set for oral argument, appellee filed
and served a notice of supplemental authority (his second). The notice had
attached to it copies of twenty-two cases, totaling 125 pages. The cases are,
at best, peripherally related to the issue presented on appeal. All of the cases
had been decided before appellee’s answer brief was filed. Appellee’s notice
of supplemental authority prompted appellants to respond with their own, on
the morning of oral argument. When asked at oral argument why the
“supplemental authorities” had not been cited in his briefs, counsel for
appellee had no satisfactory explanation. Of late, this scenario has been
occurring with disturbing frequency.
We believe such filings to be a misuse of Florida Rule of Appellate
Procedure 9.210(g) and Florida Rule of Workers’ Compensation Procedure
4.225. Those rules are intended to permit a litigant to bring to the court’s
attention cases of real significance to the issues raised which were not cited
in the briefs, either because they were not decided until after the briefs had
been filed; or because, through inadvertence, they were not discovered
earlier. They are not intended to permit a litigant to submit what amounts to
an additional brief, under the guise of “supplemental authorities”; or to
ambush an opponent by deliberately withholding significant case citations
until just before oral argument.
We simply do not have time, on the eve of oral argument, to read
numerous cases and then to attempt to divine why it is that the party
submitting them believes they are relevant to the issues raised. Moreover,
permitting such a practice places the opposing party at a disadvantage. He or
she must divert attention from preparation for the argument to read the
submission and determine what type of response, if any, is appropriate. Not
infrequently, the opposing party is forced at oral argument to request an
opportunity to respond in writing to such a submission. Although fairness
mandates that such a request be granted, the results include increased cost to
the litigants, a waste of oral argument time and delay in resolving the appeal.
Accordingly, on our own motion, we strike both the latest notice of
supplemental authority filed by appellee and appellants’ notice filed in
response. We publish this order to place the bar on notice that, in the future,
we shall treat similar abuses of rule 9.210(g) or rule 4.225 in the same way.
ERVIN, ZEHMER and WEBSTER, JJ., concur.

211
Sarasota County v. Ex
645 So. 2d 7 (Fla. 2d DCA 1994)
ALTENBERND, Judge.
Sarasota County appeals a final judgment in an action filed by Gene and
Elaine Ex in September 1991. The Exes sought monetary compensation for
land they had deeded to the county in August 1983. We conclude the statute
of limitations for this action expired before it was filed. Although the Exes
describe their action as a claim for inverse condemnation arising out of the
county’s unconstitutional demand for a dedication of their land, they are
actually seeking additional consideration for a valid deed they granted to the
county more than eight years ago.
I. A WORD ABOUT MOTIONS
Before reaching the merits of this case, we comment upon a problem that
frequently occurs in appellate motion practice. This case is merely a good
example of the problem. There is a tendency for motions to proliferate
because lawyers simply will not permit an adversary to have the last word.
During 1993, this court received approximately 8500 motions, excluding
motions for rehearing and rehearing en banc. Many, if not most, of those
motions were filed in good faith and in an effort to facilitate the timely
resolution of the relevant appeal. We do, however, receive unnecessary or
inappropriate motions in many cases. Occasionally, we deny such a motion
with a citation to Dubowitz v. Century Village East, 381 So.2d 252, 253 (Fla.
4th DCA 1979), as a reminder that this court, as well as the Fourth District,
suffers from “acute motion sickness.”
This case involves a more severe problem. It demonstrates the all-too-
common process by which one questionable filing results in a volley of
unnecessary motions, responses, replies, answers, etc. Lawyers need to
realize that appellate motion practice is not a game of ping-pong in which
the last lawyer to serve wins.
During oral argument, the Exes’ attorney received numerous questions
from the court. The attorney was legitimately concerned that the issues
might not be briefed as thoroughly as the court desired. Accordingly, he
filed two notices of supplemental authority a few days after the oral
argument. The notices did not further argue the case, but cited a few cases
and identified the issues discussed in those cases. Although it would have
been better if these cases had been cited prior to oral argument, the notices

212
were clearly filed in good faith and were not abusive under Florida Rule of
Appellate Procedure 9.210(g). Cf. Ogden Allied Servs. v. Panesso, 619 So.2d
1023 (Fla. 1st DCA 1993) (notice of twenty-two supplemental authorities on
afternoon before oral argument is abuse of rule).
These notices generated the following filings:
1. The county’s motion to strike appellees’ two notices of supplemental
authority.
2. The Exes’ reply to motion to strike appellees’ two notices of
supplemental authority.
3. The county’s motion to strike reply to appellees’ notices of
supplemental authority.
4. The Exes’ response to motion to strike reply to appellees’ notices of
supplemental authority.
None of these filings was necessary or helpful to this court. Rather than
facilitate this court’s operation, they slowed the operation and caused
needless administrative labor. These motions undoubtedly have cost clients
and taxpayers unnecessary dollars.
We do not assess fault for these filings on one party or the other, and we
impose no sanction. We simply remind attorneys, that, as officers of the
court, they must exercise restraint when filing motions. In most cases,
motions to strike motions and other similar pleadings are simply
unauthorized responses that demonstrate an attorney’s lack of self-discipline.
II. STATUTE OF LIMITATIONS CONCERNING A CLAIM FOR
ADDITIONAL COMPENSATION ON A VALID DEED
[* * *]
We do not need to decide whether this action is governed by a four-year
limitations period under section 95.11(3), as urged by the county. The
longest possible statute of limitations for this case is seven years from the
date the Exes last possessed this property. § 95.14, Fla.Stat. (1991). This
statute, which governs actions founded on title to real property, or to rents or
service from it, would bar the Exes’ action, filed more than eight years after
they transferred title to the county.
Reversed and remanded.
CAMPBELL, A.C.J., and FULMER, J., concur.

213
F. Sanctions

Florida Rules of Appellate Procedure

Rule 9.410. Sanctions


After 10 days' notice, on its own motion, the court may impose sanctions
for any violation of these rules, or for the filing of any proceeding, motion,
brief, or other paper that is frivolous or in bad faith. Such sanctions may
include reprimand, contempt, striking of briefs or pleadings, dismissal of
proceedings, costs, attorneys' fees, or other sanctions.

Island Harbor Beach Club, Ltd. v. Dep’t of Natural Resources


471 So. 2d 1380 (Fla. 1st DCA 1985)
The record on appeal in this case contains seventeen (17) volumes of
pleadings and transcripts and two (2) boxes of exhibits. Appellants Island
Harbor Beach Club, Ltd., et al., have filed an initial brief containing a
thirteen (13) page statement of the case and facts, with only seven (7)
references to the record on appeal. There are many unsupported factual
assertions and extensive improper legal arguments contained in the
statement of facts. The brief does not, therefore, comply with either the letter
or spirit of rule 9.210(b)(3), Florida Rules of Appellate Procedure. This has
caused an unnecessary and undue delay in the handling of this case by the
court.
The initial brief of appellants Island Harbor Beach Club, Ltd., et al., is
hereby stricken, sua sponte, with leave to file an amended brief fully
complying with the appellate rules no later than ten (10) days from the date
hereof. A copy of such brief shall be promptly served on all counsel of
record.
This order shall be published in the official reporter for the information
of members of the bar practicing in this court.

214
G. Types of Briefs

Florida Rules of Appellate Procedure

Rule 9.210. Briefs


***
(b) Contents of Initial Brief. The initial brief shall contain the following, in
order:
(1) A table of contents listing the issues presented for review, with
references to pages.
(2) A table of citations with cases listed alphabetically, statutes and
other authorities, and the pages of the brief on which each citation
appears. See rule 9.800 for a uniform citation system.
(3) A statement of the case and of the facts, which shall include the
nature of the case, the course of the proceedings, and the disposition in
the lower tribunal. References to the appropriate volume and pages of the
record or transcript shall be made.
(4) A summary of argument, suitably paragraphed, condensing
succinctly, accurately, and clearly the argument actually made in the body
of the brief. It should not be a mere repetition of the headings under which
the argument is arranged. It should seldom exceed 2 and never 5 pages.
(5) Argument with regard to each issue including the applicable
appellate standard of review.
(6) A conclusion, of not more than 1 page, setting forth the precise
relief sought.
(c) Contents of Answer Brief. The answer brief shall be prepared in the
same manner as the initial brief; provided that the statement of the case
and of the facts may be omitted. If a cross-appeal has been filed, the
answer brief shall include the issues in the cross-appeal that are
presented for review, and argument in support of those issues.
(d) Contents of Reply Brief. The reply brief shall contain argument in
response and rebuttal to argument presented in the answer brief.
(e) Contents of Cross-Reply Brief. The cross-reply brief is limited to
rebuttal of argument of the cross-appellee.
***

215
Rule 9.120. Discretionary Proceedings to Review Decisions of
District Courts of Appeal
***
(d) Briefs on Jurisdiction. Petitioner's brief, limited solely to the issue of
the supreme court's jurisdiction and accompanied by an appendix
containing only a conformed copy of the decision of the district court of
appeal, shall be served within 10 days of filing the notice. Respondent's
brief on jurisdiction shall be served within 20 days after service of
petitioner's brief. Formal requirements for both briefs are specified in rule
9.210. No reply brief shall be permitted. If jurisdiction is invoked under rule
9.030(a)(2)(A)(v) (certifications of questions of great public importance by
the district courts to the supreme court), no briefs on jurisdiction shall be
filed.
***
Rule 9.370. Amicus Curiae
(a) When Permitted. An amicus curiae may file a brief only by leave of
court. A motion for leave to file must state the movant's interest, the
particular issue to be addressed, how the movant can assist the court in
the disposition of the case, and whether all parties consent to the filing of
the brief.
(b) Contents and Form. An amicus brief must comply with Rule 9.210(b)
but shall omit a statement of the case and facts and may not exceed 20
pages. The cover must identify the party or parties supported. An amicus
brief must include a concise statement of the identity of the amicus curiae
and its interest in the case.
(c) Time for Service. An amicus curiae must serve its brief no later than 5
days after the first brief, petition, or response of the party being supported
is served. An amicus curiae that does not support either party must serve
its brief no later than 5 days after the initial brief or petition is served. A
court may grant leave for later service, specifying the time within which an
opposing party may respond. The service of an amicus curiae brief does
not alter or extend the briefing deadlines for the parties. An amicus curiae
may not file a reply brief.
(d) Notice of Intent to File Amicus Brief in Supreme Court. When a
party has invoked the discretionary jurisdiction of the supreme court, an
amicus curiae may file a notice with the court indicating its intent to seek
leave to file an amicus brief on the merits should the court accept
jurisdiction. The notice shall state briefly why the case is of interest to the
amicus curiae, but shall not contain argument. The body of the notice shall
not exceed one page.

216
XII. Oral Argument

Florida Rules of Appellate Procedure

Rule 9.320. Oral Argument


Oral argument may be permitted in any proceeding. A request for oral
argument shall be a separate document served by a party not later than
the time the last brief of that party is due. Each side will be allowed 20
minutes for oral argument, except in capital cases in which each side will
be allowed 30 minutes. On its own motion or that of a party, the court may
require, limit, expand, or dispense with oral argument.

A. Request for Oral Argument

Noel Enters., Inc. v. Smitz


490 So. 2d 95 (Fla. 5th DCA 1986)

PER CURIAM. Affirmed.


DAUKSCH, UPCHURCH and SHARP, JJ., concur.
ON MOTION FOR REHEARING
UPCHURCH, Judge.
In its motion for rehearing, appellant states that it requested oral
argument at the time of service of its initial brief. The matter was decided
without oral argument and without an order being entered denying oral
argument. We note that counsel for appellant simply requested oral
argument following the conclusion of the brief. He did not comply with
Florida Rule of Appellate Procedure 9.320 which states: “A request for oral
argument shall be a separate document....” (emphasis added).
While our failure to grant the request for oral argument may seem
arbitrary to counsel, it is not, as an explanation of our procedure
demonstrates. The ministerial acts of filing and docketing briefs, motions
and other documents are handled by clerical personnel whose interest in the
progress of a case is generally limited to seeing that the briefs and motions
are properly filed and docketed. While often they do find a request for oral
argument contained in a brief and place the case on the oral argument

217
docket, that is probably the exception rather than the rule. As stated in
Haines v. State, 113 So. 2d 601 (Fla.2d DCA 1959):
To require a clerk to search a brief or a bound record to ascertain whether
an application for oral argument had been made would impose upon him and
his staff an unnecessarily burdensome duty. 113 So. 2d at 602. See also
Quince v. State, 91 So. 2d 632 (Fla.1956).
In contrast, if a request for oral argument is made by separate document
as required by rule 9.320, it is independently docketed, and, except in the
very rare instance where oral argument is dispensed with by order of the
court, the case is placed upon the oral argument calendar. If oral argument is
not requested or, as in this situation, improperly requested and therefore
overlooked, the case is handled by assignment to a panel of judges for
conference without oral argument.
REHEARING DENIED.
DAUKSCH and SHARP, JJ., concur.

XIII. Scope of Review

A. Nature of the Proceeding

1. Final Orders

Murphy White Dairy, Inc. v. Simmons


405 So. 2d 298 (Fla. 4th DCA 1981)
PER CURIAM.
In 1977, Murphy White Dairy, Inc., brought a multi-count complaint
containing assorted claims arising out of an alleged oral agreement made by
the Simmonses to sell certain real property to the Dairy. In August 1978, a
summary judgment was entered in favor of the Simmonses on one count of
the complaint which sought specific performance, and the Dairy appealed.
On October 24, 1979, we dismissed the Dairy’s appeal on the authority of
Mendez v. West Flagler Family Association, Inc., 303 So. 2d 1 (Fla.1974).
The next day, October 25, 1979, our order of dismissal was filed in the trial
court.
[1] [2] Some eight months later, in July 1980, the Simmonses, asserting
that no record activity had occurred for a period of one year, moved to

218
dismiss the Dairy’s action for lack of prosecution under Florida Rule of
Civil Procedure 1.420(e). [FN2] The trial court dismissed the cause, and the
Dairy seeks review of that order.[FN3] We reverse.
FN2. Florida Rule of Civil Procedure 1.420(e) provides, in pertinent
part:“Failure to Prosecute. All actions in which it appears on the face of the
record that no activity by filing of pleadings, order of court or otherwise has
occurred for a period of one year shall be dismissed by the court on its own
motion or on the motion of any interested person, ...”
FN3. A dismissal for lack of prosecution is a final order for purposes of
appeal, but not final-that is, without prejudice-so as to be res judicata. Bair v.
Palm Beach Newspapers, Inc., 387 So. 2d 517 (Fla. 4th DCA 1980); Gibbs
v. Trudeau, 283 So. 2d 889 (Fla. 1st DCA 1973). Moreover, the order
appealed from, although titled “Order Granting Motion to Dismiss, etc.,”
actually dismisses the cause so as to give the order requisite finality for
appeal purposes. Compare, e. g., Shupack v. Allstate Insurance Company,
356 So. 2d 1298 (Fla. 3d DCA 1978).
[3] We reject the Simmonses’ contention that the filing in the trial court
of our order dismissing the Dairy’s appeal, clearly record activity, see Ortiz
v. Biscayne Medical Center, Inc., 385 So. 2d 1146 (Fla. 3d DCA 1980), is
not as required, see Bair v. Palm Beach Newspapers, Inc., 387 So. 2d 517
(Fla. 4th DCA 1980), record activity directed towards disposition of the case
and is thus insufficient to preclude dismissal. The Dairy’s act of appealing
the summary judgment entered against it on the count seeking specific
performance was not merely a passive act on its part which had no tendency
to move the case to its conclusion. Simply because the Dairy’s appeal,
viewed retrospectively, did not succeed in accomplishing that end does not
mean that it was not genuinely calculated to bring the suit to finality.[FN4]
See, e. g., Ortiz v. Biscayne Medical Center, Inc., supra (filing in trial court
of appellate court order denying plaintiff’s petition for common law
certiorari is record activity precluding dismissal). Indeed, even where record
activity calculated to advance the cause is negated by a later act or omission
which tends to delay the cause, the initial record activity is still considered
sufficient to preclude dismissal under the Rule. See, e. g., Harris v. Winn-
Dixie Stores, Inc., 378 So. 2d 90 (Fla. 1st DCA 1979) (record activity shown
by defendant’s filing of notice of taking deposition of co-defendant is not
vitiated by later cancellation of deposition); Eastern Elevator, Inc. v. Page,
263 So. 2d 218 (Fla.1972) (record activity shown by filing of written

219
interrogatories to plaintiff is not vitiated by plaintiff’s failure to answer).
[FN5]
FN4. It certainly cannot be said that an unsuccessful appeal by a party
faced with the Mendez finality-nonfinality dilemma is frivolous and
designed to delay rather than move the case towards prosecution. As
Mendez notes, had we concluded that the specific performance claim was
distinct and separate so as to permit the Dairy’s appeal, and reversed the
summary judgment, a trial on all claims could be had together. It is better to
be safe than sorry under Mendez, and an unsuccessful appeal under Mendez
cannot be condemned as dilatory or not a good faith effort to bring the case
to a conclusion.
FN5. It appears that the one type of record activity that is not facially
sufficient is the withdrawal of counsel. The progenitor of this line of cases is
Gulf Appliance Distributors, Inc. v. Long, 53 So. 2d 706 (Fla.1951), which
held that a court order approving a stipulation for withdrawal of defendant’s
counsel was not “ ‘action’ in the prosecution of a cause within the meaning
of s 45.19, Fla.Stat. (1949),” the predecessor to Florida Rule of Civil
Procedure 1.420. While that case involved change of defense counsel, it was
followed in cases involving withdrawal of plaintiff’s counsel, see, e. g., St.
Anne Airways Corp. v. Larotonda, 308 So. 2d 129 (Fla. 3d DCA 1975);
Florida Power & Light Company v. Gilman, 280 So. 2d 15 (Fla. 3d DCA
1973), an action that might very well be calculated to move the case along. It
is clear, however, that despite Gulf Appliances’ insistence on “some active
measure taken by plaintiff,” record activity solely by a defendant is
sufficient to preclude dismissal. See Musselman Steel Fabricators, Inc. v.
Radziwon, 263 So. 2d 221 (Fla.1972) (filing by court reporter of depositions
taken by defendant).
[4] The Dairy also challenges the propriety of the trial court’s entry of
summary judgment on the specific performance count. Of course, were the
Dairy’s appeal taken solely from the summary judgment, we would be
without jurisdiction to entertain it, Agriesti v. Clevetrust Realty Investors,
381 So. 2d 753 (Fla. 4th DCA 1980), and would dismiss it even as we did in
1979. But here our jurisdiction has been properly invoked, because the
Dairy’s appeal is from an order dismissing for lack of prosecution. See n. 3,
supra. Since we have jurisdiction to entertain an appeal from that order, we
“may review any ruling or matter occurring prior to filing of the notice,”
here, the summary judgment. Fla.R.App.P. 9.110(h) (emphasis supplied).

220
Thus, while we are authorized by Rule 9.110(h) to reach this secondary
issue, we are not required to do so.
[5] The design of Rule 9.110(h) was to enable appellate courts to
consider more than the order appealed so as to bring the matter to an end
then and there. For example, while formerly we could not decide a party’s
entitlement to a directed verdict where the opposing party appealed from the
grant of a new trial, we now can decide the potentially dispositive directed
verdict issue. See Bowen v. Willard, 340 So. 2d 110 (Fla.1976). In the
present case, however, no matter what we decide about the propriety of the
summary judgment on the specific performance count, the case must be
returned for trial, and appeal will or will not follow thereafter depending not
on our decision, but on the outcome of the trial of the interrelated claims.
The issue which we are urged to review may very well be mooted or
controlled by further developments in the case, the very reasons why an
interrelated claim is not reviewable in the first instance. See Mendez v. West
Flagler Family Association, Inc., supra. We do not consider it an appropriate
use of judicial time for us to address this further issue.[FN6]
FN6. Had we affirmed the dismissal for failure to prosecute, then the
summary judgment on the specific performance count would become ripe
for review, since a ruling thereon could be dispositive.
REVERSED AND REMANDED for further proceedings.
DOWNEY and GLICKSTEIN, JJ., and PEARSON, DANIEL S.,
Associate Judge, concur.

2. Nonfinal Orders

RD & G Leasing, Inc. v. Stebnicki


626 So. 2d 1002 (Fla. 3d DCA 1993)
Before HUBBART, BASKIN and COPE, JJ.
COPE, Judge.
[1] RD & G Leasing, Inc., appeals an order denying its motion to dismiss
the complaint for failure of plaintiff-appellee Richard N. Stebnicki to
accomplish service of process within the 120 days required by Florida Rule
of Civil Procedure 1.070(i) (1993).FN* As the order is a nonappealable non-
final order, the appeal is dismissed on authority of Rosenthal v. Watkins, 623
So. 2d 855 (Fla. 3d DCA 1993); Macke Laundry Services, Inc. v. Saintil, 568

221
So. 2d 541 (Fla. 4th DCA 1990); DCA of Hialeah, Inc. v. Lago Grande One
Condominium Ass’n, Inc., 559 So. 2d 1178 (Fla. 3d DCA 1990); Cole v.
Posada, 555 So. 2d 367 (Fla. 3d DCA 1989). Certiorari is denied on
authority of Macke, 568 So. 2d at 542, because there is an adequate remedy
by appeal at the conclusion of the case.
FN* Prior to January 1, 1993, the rule was designated Rule 1.070(j). See
In re Amendments to Florida Rules of Civil Procedure, 604 So. 2d 1110,
1117 (Fla.1992).An earlier appeal between these parties is reported as
Stebnicki v. Wolfson, 584 So. 2d 177 (Fla. 3d DCA 1991).
[2] [3] [4] [5] [6] RD & G attempts to distinguish the foregoing
authorities, based on the fortuity that the written order denying the motion to
dismiss also contained a ruling which quashed service of process. It is, of
course, true that an order quashing service of process is an appealable non-
final order. See Fla.R.App.P. 9.130(a)(3)(C)(i); Far Out Music, Inc. v.
Jordan, 438 So. 2d 912, 913 (Fla. 3d DCA 1983). RD & G reasons that if
the written order contains one ruling which is subject to interlocutory appeal
under Rule 9.130, then any other ruling which is contained in the same
written order “tags along” and is reviewable on interlocutory appeal. That is
not so. To begin with, RD & G procured the ruling quashing service. Neither
RD & G nor any other party to this case has appealed the ruling quashing
service. More important, only the matters set forth in Rule 9.130 have been
determined to justify an interlocutory appeal as a matter of right. An order
denying a motion to dismiss under present rule 1.070(i) is not such a matter
and may not be the subject of interlocutory appeal. See Northcutt v. Pathway
Financial, 555 So. 2d 368, 369 (Fla. 3d DCA 1989), review denied, 563 So.
2d 633 (Fla.1990). Orders which are not enumerated in Rule 9.130 must
await review at the end of the case, unless they meet the standard for
certiorari or another extraordinary writ. As already stated, the standards for
review by certiorari, the only arguably available writ, have not been satisfied
in this case.
In support of its analysis, RD & G relies on Austin v. Gaylord, 603 So.2d
66 (Fla. 1st DCA 1992). That case, however, by its terms involves the
jurisdictional effects of section 768.28, Florida Statutes (1989), a
consideration which is entirely absent in the present case. Austin is readily
distinguished from the case now before us.
RD & G’s reliance on Morales v. Sperry Rand Corp., 601 So. 2d 538
(Fla.1992), is also misplaced. That decision did not address the question of

222
appealability of a non-final order denying a motion to dismiss under Rule
1.070(i).
Appeal dismissed.

3. Discretionary Review

Savoie v. State
422 So. 2d 308 (Fla. 1982)
OVERTON, Justice.
This is a petition to review the decision of the Fifth District Court of
Appeal reported as Savoie v. State, 401 So. 2d 1138 (Fla. 5th DCA 1981).
We find conflict, as acknowledged by the district court in its opinion, with
T.C. v. State, 336 So. 2d 17 (Fla. 3d DCA 1976), and Davis v. State, 226 So.
2d 257 (Fla. 2d DCA 1969). We have jurisdiction. Art. V, § 3(b)(3), Fla.
Const.
[1] The issue before us arose from a denial of a motion to suppress made
during trial. Although the trial judge heard the motion on the merits, he
denied it both on the merits and on the ground of waiver, finding waiver
because the motion was made during trial and was, therefore, not timely
under the provisions of Florida Rule of Criminal Procedure 3.190(h)(4). On
appeal, the district court affirmed the waiver ruling and refused to consider
the denial on the merits. Under the circumstances of this case, we reject the
district court’s holding that waiver was the proper basis for denying the
motion to suppress. The trial judge considered the motion on the merits, and
we find that this renders the waiver issue moot. We further find that the trial
judge correctly denied the motion to suppress on the merits. We have
addressed this issue because, once we accept jurisdiction over a cause in
order to resolve a legal issue in conflict, we may, in our discretion, consider
other issues properly raised and argued before this Court. We, therefore,
approve the result of the district court’s decision.
We must detail the mostly uncontroverted facts in the record in order to
properly address the issues in this cause. Petitioner, Armond Savoie, was
charged by information with having committed bribery by corruptly giving
the sum of five thousand dollars to Officer Stewart of the City of Winter
Park police force with the intent to influence the performance of that officer
by requesting that he destroy or dispose of evidence in two pending criminal

223
cases. According to Officer Stewart’s testimony, he met with Savoie in a
restaurant in Clermont, Florida, at which time Savoie asked if he could
dispose of the criminal charges pending against one James Savage, and, if
so, how much this service would cost. Officer Stewart, who was an
investigator in the Savage case, prepared simulated evidence to sell to
Savoie. This simulated evidence consisted of fabricated video tapes and two
envelopes containing money. Officer Stewart put police evidence tape on all
of these items and marked them as evidence. He then contacted Savoie and
arranged to meet him at a motel in Kissimmee, Florida. Several law
enforcement officers were stationed in the room adjoining that in which
Officer Stewart and Savoie were meeting.
Officer Stewart testified that, when he gave Savoie the simulated
evidence, Savoie in turn gave him fifty $100 bills and that Savoie placed the
simulated evidence in an attaché case, which he locked, and left the room
with the case. A short distance outside the room, Savoie was placed under
arrest by one of the accompanying police officers; Savoie was carrying the
attaché case at the time of his arrest. The officer searched Savoie,
handcuffed him, and took control of the attaché case. During the search, the
officer found the key to the attaché case and immediately opened the case at
the scene of the arrest. The search was conducted without a warrant and
without Savoie’s consent.
Savoie did not file a pre-trial motion to suppress the contents of the
attaché case, and the trial commenced on May 19, 1980. On May 20, during
the testimony of the officer who made the arrest, the prosecution attempted
to have him identify the contents of the attaché case. Savoie’s counsel
objected, asserting that Savoie’s constitutional rights had been violated. The
trial judge excused the jury and heard the testimony of the officer as to the
circumstances of the arrest and preliminary legal argument by counsel as to
the validity of the search and seizure. A recess was then taken so that
counsel and the court could research and examine relevant cases. It was only
after the trial court had heard all the testimony from the arresting officer and
full argument by both counsel on the merits of the motion to suppress that
the state objected to the motion because it was not timely filed. The state
asserted that Savoie had waived the right to make the motion to suppress by
failing to make the motion before trial, as required by rule 3.190(h)(4).
During the discussion which ensued on the waiver issue, the trial judge
expressed concern that the state could not immediately appeal an adverse

224
ruling on the motion as it could have done had a pre-trial motion to suppress
been granted. Counsel for Savoie contested the waiver claim by contending
that the discovery information furnished by the state did not adequately
disclose that the contents of the attaché case were to be submitted as
evidence.
The trial judge, in ruling on the motion, stated: “On the merits ... I am
going to deny the Motion to Suppress.” The trial judge ruled alternatively,
stating, “I think it has been waived.” With regard to the motion’s merits, the
trial judge expressly relied on United States v. Chadwick, 433 U.S. 1, 97
S.Ct. 2476, 53 L.Ed.2d 538 (1977), in finding that the search in this case was
reasonable because it was made incident to a valid arrest and was not
rendered invalid by the fact that the police did not believe that Savoie had
access to a weapon in the attaché case or was about to destroy evidence
which might be contained in the case.
The district court affirmed the trial court’s denial of the motion on the
basis of waiver, reasoning that:
Because Savoie failed to move to suppress prior to the trial as required by
the Rule when he was aware of the grounds for the motion and had ample
opportunity to make the motion, we hold that he failed to preserve for appeal
any error in the admission of the evidence.
Savoie, 401 So. 2d at 1139. It did not address the trial court’s ruling on
the merits.
Waiver under Rule 3.190
The first issue we must address is whether Savoie’s failure to make a pre-
trial motion resulted in an absolute waiver of his right to contest the validity
of the search and seizure of the contents of his attaché case and the
admission of these contents into evidence. The state contends that petitioner
is not entitled to a hearing on the merits of the motion to suppress and that a
waiver is the only appropriate sanction for enforcing the provisions of rule
3.190(h)(4).
Rule 3.190(h) concerns motions to suppress evidence obtained in an
unlawful search and seizure. The relevant portions of the rule read as
follows:
(2) Contents of Motion. Every motion to suppress evidence shall clearly
state the particular evidence sought to be suppressed, the reasons for

225
suppression and a general statement of the facts on which the motion is
based.
(3) Hearing. Before hearing evidence, the court shall determine if the
motion is legally sufficient. If it is not, the motion shall be denied. If the
court hears the motion on its merits, the defendant shall present evidence
supporting his position and the State may offer rebuttal evidence.
(4) Time for Filing. The motion to suppress shall be made before trial
unless opportunity therefor did not exist or the defendant was not aware of
the grounds for the motion, but the court may entertain the motion or an
appropriate objection at the trial.
(Emphasis added.) This rule is designed to promote the orderly process of
trial by avoiding the problems and delay caused when the trial judge must
interrupt trial, remove the jury from the courtroom, and hear argument on a
motion to suppress that could easily have been disposed of before trial.
Davis v. State, 226 So. 2d 257 (Fla. 2d DCA 1969). Also, when the rule is
complied with, the state is afforded an opportunity to appeal the ruling of a
trial judge in the event the evidence is suppressed; when the judge rules at
trial to suppress evidence, the state is foreclosed from appealing that
decision. See Fla.R.App.P. 9.140(c)(1)(B).
Rule 3.190(h)(4) does not, however, require that all motions to suppress
be heard before trial. Rather, the rule expressly grants the trial judge
discretionary authority to entertain either a motion to suppress or an
objection to the introduction of certain evidence made during the course of
the trial. This discretionary authority is necessary in order to avoid the sixth
amendment ramifications which might result from the application of an
absolute waiver rule against a defendant whose counsel failed to comply
with the requirements of rule 3.190(h). Likewise, the rule does not affect the
inherent power of the trial court to reconsider, while the court has
jurisdiction of the case and upon appropriate motion or objection by either
counsel, a ruling previously made on a motion to suppress.
[2] We find that in the instant case the trial judge exercised his discretion
to entertain the motion to suppress made during trial. He heard the proffer of
testimony and the arguments of counsel on the validity of the search and
seizure, and he ruled on the merits of the motion. In our opinion, the issue of
waiver became moot because the hearing was held and a ruling made on the
motion’s merits. We fully agree with the statement in Davis that, if the judge
decides to entertain this motion it is then his decision that results in

226
interruption of the trial, and once the trial has been interrupted the reason for
the old rule vanishes, whether the judge subsequently grants or denies the
motion.... [W]hen the trial judge proceeds to exercise his discretion to
entertain the motion he has, in a sense, rejected defendant’s waiver of his
right to contest this issue. 226 So. 2d at 259.
[3] [4] In exercising the discretionary authority granted by the rule to
decide whether to hear a motion to suppress made during the course of a
trial, the judge must balance the rights of the defendant to due process and
effective assistance of counsel with the rights of the state to have an
opportunity to appeal an adverse ruling on the motion to suppress.FN1
However, once the decision is made to hear the motion on the merits, we
find the issue of waiver is no longer before the court. We conclude,
therefore, that, in the circumstances of this case, once the trial judge
interrupted the trial and conducted a hearing on the merits of the motion to
suppress, waiver was no longer a proper ground for denying the motion.
FN1. The state could retain the right to appeal if the defendant consented,
prior to the hearing on the motion, to a mistrial in the event the trial court
suppresses the evidence.
Jurisdiction of this Court to Consider the Merits of the Motion to
Suppress
[5] [6] We agree with the acknowledgment in the district court’s opinion
that its decision conflicts on the waiver issue with Davis and T.C. v. State,
336 So. 2d 17 (Fla. 3d DCA 1976). We have jurisdiction, and, once this
Court has jurisdiction of a cause, it has jurisdiction to consider all issues
appropriately raised in the appellate process, as though the case had
originally come to this Court on appeal. This authority to consider issues
other than those upon which jurisdiction is based is discretionary with this
Court and should be exercised only when these other issues have been
properly briefed and argued and are dispositive of the case. In Zirin v.
Charles Pfizer & Co., 128 So. 2d 594, 596 (Fla.1961), Justice Drew
explained the reasons why, once it has jurisdiction, this Court should
exercise its discretion and dispose of the entire cause when the issues are
properly before it:
Needless steps in litigation should be avoided wherever possible and
courts should always bear in mind the almost universal command of
constitutions that justice should be administered without “sale, denial or
delay.” Piecemeal determination of a cause by our appellate court should be

227
avoided and when a case is properly lodged here there is no reason why it
should not then be terminated here.... “[m]oreover, the efficient and speedy
administration of justice is ... promoted” by doing so.
We have concluded that, based upon these principles, we can
appropriately address the correctness of the trial judge’s denial of the motion
to suppress on its merits. Both parties have fully briefed and argued the issue
before this Court, and our action in resolving this question will avoid a
piecemeal determination of the case.
The Merits of the Motion to Suppress
At the outset it should be noted that Savoie concedes the validity of his
custodial arrest. The question to be determined is whether the search of the
contents of the attaché case Savoie was carrying at the time of his arrest was
a reasonable search incident to a lawful arrest.
[***]
We, therefore, conclude that the trial judge was correct in denying the
motion to suppress the contents of Savoie’s attaché case.
For the reasons expressed, the judgment and sentence, as affirmed by the
district court, are approved, but the reasoning contained in the district court’s
opinion, based as it is upon a waiver theory, is disapproved.
It is so ordered.
ALDERMAN, C.J., and BOYD, McDONALD and EHRLICH, JJ.,
concur.
ADKINS, J., concurs in result only.

B. Decisions

The scope of review is limited to those decisions of the lower tribunal that are at issue in
the appellate proceeding.
The appellate court may not review the correctness of a decision made in another case.
See Weignberg v. Bort, 961 So. 2d 1017 (Fla. 4th DCA 2007).

228
C. Issues

Pagan v. Sarasota County Public Hosp. Bd.


884 So. 2d 257 (Fla. 2d DCA 2004)
ALTENBERND, Chief Judge.
Valerie and Jami Pagan appeal a summary final judgment that was
entered in an unusual action for declaratory relief filed by Sarasota County
Public Hospital Board (Hospital Board) and SMH Physician Services, Inc.,
d/b/a First Physicians Group of Sarasota (First Physicians Group). The
Hospital Board and First Physicians Group sought a legal ruling that First
Physicians Group and the doctors who practiced medicine through this
nonprofit corporation were entitled to all the benefits of sovereign immunity.
See § 768.28, Fla. Stat. (2001). The trial court’s judgment so rules as to the
Pagans’ claim. We conclude that the Pagans have not demonstrated a basis
that entitles them to receive a reversal of this judgment.
***
[1] In light of the limited arguments made below and thus preserved for
appellate review, and in light of the existence of case law that supports the
trial court’s conclusions based on those arguments, the Pagans have not
demonstrated a basis for this court to reverse the final summary judgment in
this case. See, e.g., Betterson, 648 So. 2d 778. We therefore affirm the final
summary judgment as to these parties.
We recognize that the language of the trial court’s orders was
substantially broader than was necessary to resolve the dispute between First
Physicians Group and one or two of its doctors, on one side, and the Pagans,
on the other side. The language could be misread to have binding effect on
the intervenors or other nonparties. However, the trial court clearly did not
intend this, and section 86.091 does not permit it. See also Fasig v. Fla.
Soc’y of Pathologists, 769 So. 2d 1151, 1154 (Fla. 5th DCA 2000). Dr.
Caballero affirmatively opted out of this ruling, and the great majority of
doctors at First Physicians Group have neither been involved in these
proceedings nor asserted a claim of sovereign immunity to limit their own
liability to claims of wanton and willful misconduct. The trial court’s ruling
is restricted to the claim between the Pagans and Dr. Michael M. Shroder
and First Physicians Group as Dr. Shroder’s employer.
[2] It appears that the Hospital Board, First Physicians Group, and the
two physicians who remain in this litigation are seeking an appellate court

229
opinion holding, as a matter of law, that First Physicians Group and its
employees are entitled to sovereign immunity in any medical malpractice
action. It is a long-standing rule of appellate jurisprudence that the appellate
court should not undertake to resolve issues which, though of interest to the
bench and bar, are not dispositive of the particular case before the court.
Marion County Hosp. Dist. v. Akins, 435 So. 2d 272, 273 (Fla. 1st DCA
1983). Accordingly, while we affirm the trial court’s ruling as it affects these
parties on the basis of this limited record and based on the specific issues
preserved for review by this court, the trial court was not authorized to make
a broader legal holding, nor does this appellate court have a record or
adequate arguments upon which it could announce any broader rule of law.
Affirmed.
SILBERMAN, J., Concurs.
CANADY, J., Concurs in result only.
CANADY, Judge, Concurring specially. [* * *]

D. Arguments – The Tipsy Coachman Rule

Robertson v. State
829 So. 2d 901 (Fla. 2002)
PARIENTE, J.
We have for review Robertson v. State, 780 So. 2d 106 (Fla. 3d DCA
2001) (en banc), a decision from the Third District Court of Appeal that
misapplies this Court’s holding in Dade County School Board v. Radio
Station WQBA, 731 So. 2d 638, 644 (Fla.1999), and is in conflict with the
opinion of the First District Court of Appeal in State Dept. of Revenue ex rel.
Rochell v. Morris, 736 So. 2d 41, 42 (Fla. 1st DCA 1999), regarding when
an appellate court may uphold a lower court ruling on an alternative ground
not considered by the lower court. The Third District’s decision also
misapplies this Court’s opinions in Jordan v. State, 107 Fla. 333, 144 So.
669, 669-70 (1932), and Foy v. State, 115 Fla. 245, 155 So. 657, 658 (1934),
regarding the permissible scope of impeachment of a testifying defendant.
Based on the conflict created by these misapplications, we have jurisdiction
under article V, section 3(b)(3), of the Florida Constitution. See Florida
Dep’t of Transp. v. Juliano, 801 So.2d 101, 103 (Fla.2001) (citing Vest v.

230
Travelers Ins. Co., 753 So. 2d 1270, 1272 (Fla.2000)). For the reasons that
follow, we quash the Third District’s decision and remand for proceedings
consistent with this opinion.
BACKGROUND
On September 16, 1996, Officer Dominguez was dispatched to an
apartment to investigate a shooting. In the bedroom of the apartment shared
by Robertson and the victim, Maria Nelson, Officer Dominguez observed
Nelson lying on the bed. Nelson was conscious and breathing, but was not
communicating. On the floor, about five feet from the bed, was a .40 caliber
Ruger handgun with a bullet clip next to it. Officer Dominguez found an
entrance wound in Nelson’s chest and an exit wound in the middle of her
back. Nelson ultimately died from the gunshot wound, and Robertson was
charged with second-degree murder. Robertson’s theory of defense was that
the gun accidentally misfired while he was trying to clean it. The State’s
theory of the case was that Robertson intentionally shot Nelson during a
domestic dispute.
At trial, the defense called Robertson to testify on his own behalf, and
during the State’s cross-examination of Robertson the following exchange
took place:
[STATE] Q: In fact, you are familiar with large assault rifles, weren’t
[sic] you?
[DEFENDANT] A: Several models, yes sir.
Q: In fact, you purchased an AK-47, didn’t you?
Defense counsel immediately objected on the basis that the question was
“irrelevant” and “outside the scope of direct.” The trial court overruled the
objection and the State thereafter inquired:
Q: Isn’t that correct, Mr. Robertson?
A: Yes, right after Hurricane Andrew I did.
Q: And in fact, isn’t it a fact that you have threatened people with assault
rifles before?
Defense counsel again objected on the ground that this question was
“totally outside the scope” of direct. The trial court again overruled the
objection and the following exchange ensued:
[DEFENDANT] A: No.

231
[STATE] Q: You have never threatened anyone close to you with an AK-
47, Mr. Robertson?
A: I have never threatened anybody close to me with a weapon, anybody
period, with a weapon, sir.
The State argued that based on Robertson’s negative response to these
questions, Robertson’s ex-wife should be allowed to testify on rebuttal that
Robertson had threatened her with an AK-47 six years earlier. The trial
court, over defense objection, concluded that in light of Robertson’s answers
to the State’s questions during its cross-examination, the rebuttal testimony
constituted proper impeachment. Robertson’s ex-wife thereafter testified to
an alleged incident in which she claimed Robertson threatened her with an
AK-47. She testified that Robertson became enraged and pointed the AK-47
at her as she was running away, causing her to fear that she would be shot in
the back. The incident was neither reported to the police nor investigated.
In closing argument, the State emphasized the significance of the rebuttal
testimony:
Now you have to figure in the rest of your story, the anger, the fighting
leading up to November 17th, the confrontation that was going to take place
that night, her incredible fear of this man as told to Shirley Baumgartner, his
anger.
For God sake, you saw his ex-wife on this witness stand because he lied
to you on cross-examination about owning an AK-47 which is a huge assault
rifle and threatening somebody close to you.
I asked him that question, and, boy, I gave him the opportunity. He could
have said, yeah, there was this situation with my wife and, you know, I
really didn’t mean it, but no, I would never do that. I don't threaten anybody
with guns, any type of guns, much less an AK-47.
So, Mrs. Robertson comes in here and tells you about a situation right
after the hurricane where they were getting some building supplies and this
defendant becomes enraged because mom and daughter can’t help him bring
in some heavy supplies from the car and what does he do? I am going to
teach you a lesson. He goes back into the hallway, grabs the AK-47, slams a
magazine, a clip into it and points it at her as she is running out the door and
she thinks she is going to get shot in the back.
That gives you a little insight into what you are dealing with here.
(Emphasis supplied.)

232
The jury returned a verdict finding Robertson guilty of second-degree
murder. Robertson appealed his conviction and sentence of life
imprisonment, claiming as error the trial court’s first permitting questions on
cross-examination regarding prior alleged threats with the AK-47, and
second allowing as impeachment Robertson’s ex-wife’s rebuttal testimony
regarding the prior threat with the AK-47. The Third District initially
reversed the trial court’s decision, holding both that the prosecutor’s
questions regarding the alleged prior crime were improper and that
Robertson’s ex-wife’s testimony was improper impeachment. See Robertson
v. State, 780 So. 2d 94, 96 (Fla. 3d DCA 2000). However, the Third District
reviewed the case en banc and, on rehearing, affirmed the trial court’s
admission of Robertson’s ex-wife’s testimony. See Robertson, 780 So. 2d at
113. The majority held that the evidence was admissible as WilliamsFN1 rule
evidence, see Robertson, 780 So. 2d at 113, a determination never made by
the trial court and never asserted by the State on appeal.
FN1. Williams v. State, 110 So. 2d 654 (Fla.1959).
The Third District’s plurality opinion, in which four judges concurred,
stated that the evidence was admissible as both impeachment and Williams
rule evidence. See id. The remaining four judges dissented, explaining why
Robertson’s ex-wife’s testimony was not admissible either as impeachment
or under the Williams rule. See id. at 123-24 (Sorondo, J., dissenting).
MISAPPLICATION OF THE TIPSY COACHMAN DOCTRINE
The first issue we address in this case is whether the Third District
majority, in concluding that the evidence that Robertson threatened his ex-
wife with an AK-47 constituted permissible Williams rule evidence, erred in
affirming the trial court on grounds neither raised by the State nor
considered by the trial court. The State concedes that at no time, either
before trial or on appeal, did it argue that the evidence that Robertson
threatened his ex-wife with an AK-47 six years earlier was admissible under
the Williams rule. The State’s sole argument was that Robertson’s ex-wife’s
testimony constituted proper impeachment because of the answers that
Robertson gave in response to the questions the State asked him on cross-
examination.
[1] We start with the proposition that “[g]enerally, if a claim is not raised
in the trial court, it will not be considered on appeal.” Radio Station WQBA,
731 So. 2d at 644. However, notwithstanding this principle “[i]n some
circumstances, even though a trial court’s ruling is based on improper

233
reasoning, the ruling will be upheld if there is any theory or principle of law
in the record which would support the ruling.” Id. (emphasis supplied).
[2] This longstanding principle of appellate law, sometimes referred to as
the “tipsy coachman” doctrine, allows an appellate court to affirm a trial
court that “reaches the right result, but for the wrong reasons” so long as
“there is any basis which would support the judgment in the record.” Id. at
644-45.FN2 As we explained in Radio Station WQBA, this Court has adhered
to this principle on many other occasions. See id. (citing Applegate v.
Barnett Bank, 377 So. 2d 1150, 1152 (Fla.1979) (“The written final
judgment by the trial court could well be wrong in its reasoning, but the
decision of the trial court is primarily what matters, not the reasoning used.
Even when based on erroneous reasoning, a conclusion or decision of a trial
court will generally be affirmed if the evidence or an alternative theory
supports it.”)); see also Muhammad v. State, 782 So. 2d 343, 359 (Fla.2001)
(“[T]he trial court’s ruling on an evidentiary matter will be affirmed even if
the trial court ruled for the wrong reasons, as long as the evidence or an
alternative theory supports the ruling.”).
FN2. In Home Depot U.S.A. Co. v. Taylor, 676 So. 2d 479, 480 (Fla. 5th
DCA 1996), the Fifth District explained the derivation of the “tipsy
coachman rule” as follows:
There is a rule used by courts of appeal which is called the “tipsy
coachman” rule. It comes to us from Georgia, Lee v. Porter, 63 Ga. 345, by
the way of our supreme court in Carraway v. Armour & Co., 156 So. 2d 494
(Fla.1963).
The pupil of impulse, it fore’d [sic] him along,
His conduct still right, with his argument wrong;
Still aiming at honour, yet fearing to roam,
The coachman was tipsy, the chariot drove home;....
[3] The key to the application of this doctrine of appellate efficiency is
that there must have been support for the alternative theory or principle of
law in the record before the trial court. Thus, following Radio Station
WQBA, the First District in Morris, 736 So. 2d at 42, correctly refused to
affirm on an alternative ground not argued to the trial court, reasoning that
“the record does not reflect an evidentiary basis sufficient to permit us to
make a determination as to whether these defenses are applicable in the
instant case.” (Emphasis supplied.)

234
[4] In the present case, the Third District improperly affirmed the
admission of Robertson’s ex-wife’s testimony on the alternative Williams
rule ground because there was no evidence in the record to support the Third
District’s holding that the previous threat was admissible under the Williams
rule. Regarding the admissibility of Williams rule evidence, we have
explained:
Similar fact evidence that the defendant committed a collateral offense is
inherently prejudicial. Introduction of such evidence creates the risk that a
conviction will be based on the defendant’s bad character or propensity to
commit crimes, rather than on proof that he committed the charged offense.
Heuring v. State, 513 So. 2d 122, 124 (Fla.1987).
In affirming the trial court in this case, the Third District found the
“evidence admissible as relevant under Section 90.404(2)(a), Florida
Statutes (1997).” Robertson, 780 So. 2d at 110. Section 90.404(2)(a) is the
codification of the Williams rule and provides:
Similar fact evidence of other crimes, wrongs, or acts is admissible when
relevant to prove a material fact in issue, such as proof of motive,
opportunity, intent, preparation, plan, knowledge, identity, or absence of
mistake or accident, but it is inadmissible when the evidence is relevant
solely to prove bad character or propensity.
Section 90.404(2)(b)1 also requires that the State file a notice of intent to
rely on evidence of a defendant’s prior misconduct:
When the state in a criminal action intends to offer evidence of other
criminal offenses under paragraph (a), no fewer than 10 days before trial, the
state shall furnish to the accused a written statement of the acts or offenses it
intends to offer, describing them with the particularity required of an
indictment or information. No notice is required for evidence of offenses
used for impeachment or on rebuttal.
[5] [6] [7] The requirement of pretrial notice allows the trial court to
make an initial determination of whether the Williams rule evidence is
admissible. Before admitting Williams rule evidence, it is incumbent upon
the trial court to make multiple determinations, including whether the
defendant committed the prior crime,FN3 whether the prior crime meets the
similarity requirements necessary to be relevant as set forth in our prior case
law,FN4 whether the prior crime is too remote so as to diminish its
relevance,FN5 and finally, whether the prejudicial effect of the prior crime
substantially outweighs its probative value.FN6 As the above-mentioned

235
requirements suggest, the determination of whether evidence properly may
be admitted as Williams rule evidence is a highly individualized, factually
intensive inquiry. Moreover, even if the evidence is admitted pursuant to the
Williams rule, a cautionary instruction is required when requested pursuant
to section 90.404(2)(b)(2) so that the jury is specifically apprised of the
limited purpose for which the jury is to consider the evidence:
FN3. See Acevedo v. State, 787 So. 2d 127, 128-29 (Fla. 3d DCA 2001);
Smith v. State, 743 So. 2d 141, 143 (Fla. 4th DCA 1999); Smith v. State, 700
So. 2d 446, 447 (Fla. 1st DCA 1997); Audano v. State, 641 So. 2d 1356,
1359-60 (Fla. 2d DCA 1994); Malcolm v. State, 415 So. 2d 891, 892 (Fla. 3d
DCA 1982); Chapman v. State, 417 So. 2d 1028, 1031 (Fla. 3d DCA 1982);
State v. Norris, 168 So. 2d 541, 543 (Fla.1964).
FN4. See Chandler v. State, 702 So. 2d 186, 194 n. 6 (Fla.1997); Heuring
v. State, 513 So. 2d 122, 124 (Fla.1987).
FN5. See Duffey v. State, 741 So. 2d 1192, 1197 (Fla. 4th DCA 1999)
(“Under section 90.404(2)(a), the remoteness of a prior crime is one aspect
of its relevance, its tendency to prove or disprove a material fact in issue.”);
see also Heuring, 513 So.2d at 123 (when faced with claim that prior crimes
are too remote to be relevant, the trial court “must consider not the passage
of time alone, but the effect of the passage of time on the evidence”).
FN6. See § 90.403, Fla. Stat. (1997); see also Williamson v. State, 681
So. 2d 688, 696 (Fla.1996).
When the evidence is admitted, the court shall, if requested, charge the
jury on the limited purpose for which the evidence is received and is to be
considered. After the close of the evidence, the jury shall be instructed on
the limited purpose for which the evidence was received and that the
defendant cannot be convicted for a charge not included in the indictment or
information.
[8] Because the State never filed a notice of intent pursuant to section
90.404(2)(b) and because the State never indicated it intended to introduce
this evidence as Williams rule evidence, the admissibility of the evidence in
question was never litigated within the parameters of section 90.404(2)(a),
Florida Statutes (1997). Because the matter was not at issue, defendant did
not have an opportunity to present evidence or arguments against the
admissibility of this evidence under the Williams rule.
Robertson, 780 So. 2d at 118 (Sorondo, J., dissenting). Thus, the record
as it existed was insufficient for the Third District to determine whether the

236
evidence was properly admitted under the Williams rule. Because the matter
was never litigated on the basis of the Williams rule, the trial court never
made determinations as to whether clear and convincing evidence existed
that Robertson committed the prior crime, whether the prior crime was
substantially similar, or whether the prior crime was too remote so as to
minimize any potential relevance. Further, as pointed out by this Court in
Heuring, 513 So. 2d at 123-24, where the evidence is being offered to prove
absence of mistake or accident, the fact-intensive inquiry must include an
analysis of the crime’s remoteness as well as the absence of any intervening
similar acts. Regarding remoteness, Judge Sorondo stated in his dissent:
In the present case, the record simply does not permit a full examination
of the factors necessary to determine whether defendant’s prior misconduct
is too remote to be relevant. Assuming for the sake of argument that the
introduction of a prior threat of violence against a different victim is
admissible at all, it would be reasonable and relevant to explore whether
there were any domestic relationships between defendant’s previous
marriage and his marriage to the victim in this case. The existence of
intervening relationships without any threats of violence would certainly
raise the possibility considered by the Supreme Court in Heuring that the
prior misconduct “is no longer characteristic of the defendant.”
Robertson, 780 So. 2d at 120 (Sorondo, J., dissenting) (quoting Heuring,
513 So. 2d at 124).
Moreover, because no notice was provided and because the State never
attempted to seek the admission of this evidence on the basis of the Williams
rule, Robertson never received an opportunity to present evidence or make
argument as to why the incident involving his ex-wife should not have been
admitted under the Williams rule. In short, the record did not permit the
Third District to affirm the trial court’s admission of collateral crime
evidence as Williams rule evidence. Thus, in so doing, the Third District
improperly relied upon the “tipsy coachman” doctrine to affirm the trial
court’s admission of this evidence.
[9] Finally, even if the record were sufficiently developed to allow
affirmance on this alternate ground, we conclude as a matter of law that the
Third District erred in holding that the testimony regarding a prior threat six
years earlier against a different victim and involving a different weapon was
admissible under the Williams rule. The Third District concluded that this
evidence was admissible because “the critical issue at trial was whether the

237
defendant accidentally shot” the victim. Robertson, 780 So. 2d at 110. Thus,
the Third District reasoned, and the State now contends, that the evidence of
the prior threat was admissible to prove “absence of mistake or accident,”
which is one of the permissible purposes for the admission of “similar fact
evidence” under the Williams rule, as codified in section 90.404(2)(a).
[10] [11] The State concedes, and we agree, that substantial similarity of
crimes is a requirement when the evidence is sought to be admitted for the
specific purpose of establishing absence of mistake or accident. See Duffey
v. State, 741 So. 2d 1192, 1196-97 (Fla. 4th DCA 1999). As we have
explained,
[t]o minimize the risk of a wrongful conviction, the similar fact evidence
must meet a strict standard of relevance. The charged and collateral offenses
must be not only strikingly similar, but they must also share some unique
characteristic or combination of characteristics which sets them apart from
other offenses.
Heuring, 513 So. 2d at 124. In this case, the crime with which Robertson
was charged was the completed offense of murder against his girlfriend
utilizing a handgun. The prior offense, assuming it occurred, involved a
threat of violence against Robertson’s former wife, involving an assault rifle.
Neither the crimes, the weapons, nor the victims are similar. Indeed, we
have been unable to find, and the parties have not cited, any cases in Florida
where a prior threat against a different victim was admitted under the
Williams rule to prove the absence of mistake or accident of the present
offense. The out-of-state cases cited by the parties and cited in the Third
District’s majority opinion and dissent are illuminating in that where the
prior acts involved different victims and different offenses, the prior criminal
acts were uniformly excluded.FN7 The cases relied on by the majority
involved either prior crimes against the same victim as the charged
offense,FN8 or the charged offenses and the prior offenses involved similar
completed crimes of violence.FN9 As stated by Judge Sorondo:
The defendant in this case was charged with the completed, violent
offense of second degree murder. The prior, alleged misconduct was an
offense that threatened violence. Although such a prior threat against the
victim in this case, if not too remote, may have been admissible to show
intent and the absence of mistake or accident, the same cannot be said of a
threat against another. Reducing these events to their simplest and most
common form-if the defendant had been charged with aggravated battery by

238
punching his wife in the nose and breaking it, and he defended by claiming
that he accidentally struck her while flailing his arms during an argument,
under the majority’s reasoning, evidence that the defendant had threatened
to punch a former wife in the nose six years earlier would be admissible to
rebut his claim of accident. As morally satisfying as this type of evidence
may be, in my judgment, such evidence serves only to establish defendant’s
propensity to commit violent crimes.
Robertson, 780 So. 2d at 121-22 (Sorondo, J., dissenting) (footnotes
omitted). Accordingly, the Third District erred in applying the “tipsy
coachman” doctrine because support for affirming the trial court on the
alternative Williams rule basis was absent from the record. Furthermore, the
record affirmatively shows that it would have been error for the trial court to
have allowed the State to present, as Williams rule evidence, an alleged
threat six years earlier involving a different victim and a different weapon to
prove absence of mistake or accident.
IMPEACHMENT
We next turn to the issue of whether the evidence was properly
admissible on the basis argued at trial by the State, and actually ruled on by
the trial court-as impeachment evidence. Because the trial court and a
plurality of the Third District relied upon impeachment as a basis for
admitting the evidence that Robertson allegedly threatened his ex-wife with
an AK-47, we deem it necessary to also explain why the evidence was
impermissible impeachment under the facts of this case.
We begin with the interplay between Williams rule evidence and the rule
regarding the impeachment of witnesses. As we have stated, section
90.404(2)(a) codifies Williams, and excludes prior crime evidence that is not
relevant, and thus offered only to prove propensity or bad character.
However, section 90.404(1), provides:
CHARACTER EVIDENCE GENERALLY.-Evidence of a person’s
character or a trait of character is inadmissible to prove action in conformity
with it on a particular occasion, except:
(a) Character of accused.-Evidence of a pertinent trait of character
offered by an accused, or by the prosecution to rebut the trait.
(b) Character of victim-
....

239
(c) Character of witness.-Evidence of the character of a witness, as
provided in ss. 90.608-90.610.
(Emphasis supplied.) The only section referred to within section
90.404(1)(c) that is pertinent in this case is section 90.608(5), Florida
Statutes (1997), which provides:
Any party, including the party calling the witness, may attack the
credibility of a witness by:
....
(5) Proof by other witnesses that material facts are not as testified to by
the witness being impeached.
[12] Thus, even if the prior crime evidence is not relevant under section
90.404(2)(a), a testifying defendant may nonetheless open the door to the
prior crime evidence by (1) offering a trait of the defendant’s good character,
see § 90.404(1)(a) (character of accused), or (2) inaccurately testifying to
material facts, see § 90.404(1)(c) (character of witness), § 90.608(5)
(contradiction on relevant facts).
[13] The Third District plurality reasoned in this case that by testifying
on cross-examination that he had never threatened anyone with a gun,
Robertson opened the door to questioning about his prior threats towards his
ex-wife with the AK-47. See Robertson, 780 So. 2d at 109. We disagree.
With regard to the impeachment of a defendant by showing bad
character, this Court stated seventy years ago that the State may do so only if
the defense opens the door to such impeachment by showing the defendant’s
good character:
The character of a person accused of crime is not a fact in issue, and the
state cannot, for the purpose of inducing belief in his guilt, introduce
evidence tending to show his bad character or reputation, unless the accused,
conceiving that his case will be strengthened by proof of good character,
opens the door to proof by the prosecution that his character in fact is bad.
This salutary rule is not permitted to be violated by the state, even when the
defendant offers himself as a witness. State v. Beckner, [194 Mo. 281,] 91
S.W. 892. And this is true, although a defendant offering himself as a
witness is subject to impeachment just like any other witness so far as his
status and character as a witness, not as a defendant on trial, is concerned.
See Cross v. State, [96 Fla. 768,] 119 So. 380.

240
Jordan v. State, 107 Fla. 333, 144 So. 669, 669-70 (1932) (emphasis
supplied) (parallel citations omitted). Two years later, this Court further
explained that the State cannot open the door itself by asking impermissible
questions on cross-examination:
[T]he prosecution in a criminal case cannot call witnesses to impeach the
character of the defendant, unless the defendant puts it in issue. Nor can the
prosecution accomplish the same forbidden end by indirection through
pursuing a method of questioning defendant and his witness on cross
examination that is principally designed, by means of innuendo and
suggestions of general criminality on accused’s part, to lead the jury to
believe that the accused should be found guilty of the particular crime
charged, because of his being suspected or accused of other offenses, or
because of his connections or association with other accused persons under
indictment for different crimes not constituting part of the charge on trial.
Foy v. State, 115 Fla. 245, 155 So. 657, 658 (1934) (emphasis supplied)
(citations omitted).
By asking Robertson whether he had ever threatened someone with an
AK-47, the State violated the principles of Jordan and Foy. In the present
case, Robertson did not offer a trait of his good character. Thus, even though
Robertson offered himself as a witness, and consequently was “subject to
impeachment just like any other witness,” Jordan, 144 So. at 670, Robertson
did not open the door to a rebuttal of any character trait through evidence of
an alleged prior crime. See § 90.404(1)(a); Foy, 155 So. at 658; Jordan, 144
So. at 669-70.
[14] Moreover, regarding a defendant’s testifying inaccurately to material
facts, this Court has stated:
When the direct examination opens a general subject, the cross-
examination may go into any phase, and may not be restricted to mere parts
... or to the specific facts developed by the direct examination. Cross-
examination should always be allowed relative to the details of an event or
transaction a portion only of which has been testified to on direct
examination. As has been stated, cross-examination is not confined to the
identical details testified to in chief, but extends to its entire subject matter,
and to all matters that may modify, supplement, contradict, rebut, or make
clearer the facts testified to in chief ....
Geralds v. State, 674 So. 2d 96, 99 (Fla.1996) (emphasis supplied)
(quoting Coco v. State, 62 So. 2d 892, 895 (Fla.1953)). Thus, to open the

241
door, “the defense must first offer misleading testimony or make a specific
factual assertion which the state has the right to correct so that the jury will
not be misled.” Bozeman v. State, 698 So. 2d 629, 630 (Fla. 4th DCA 1997);
see also Mosley v. State, 739 So. 2d 672, 676 (Fla. 4th DCA 1999)
(explaining that the “opening the door” concept allows the cross-
examination to reveal the whole story of a transaction only partly explained
in direct examination).
In this case, the prosecutor’s inquiry on cross-examination regarding
Robertson’s general experience with handling firearms was proper because
Robertson’s knowledge of firearms would tend to negate his theory of
defense that the gun fired by accident. However, whether Robertson
threatened (without shooting) someone in the past with an assault rifle does
not “modify, supplement, contradict, rebut, or make clearer” either
Robertson’s defense that his actual shooting in this case was unintentional,
or his testimony that he was not very familiar with the particular gun, a .40
caliber handgun that fired the shot that killed Nelson. See Geralds, 674 So.
2d at 99.FN10 Thus, Robertson did not open the door to evidence that he
allegedly threatened his ex-wife six years before with an assault rifle by
testifying inaccurately to material facts. See § 90.404(1)(c); § 90.608(5);
Geralds, 674 So. 2d at 99.
FN10. Robertson’s testimony that he was not familiar with the particular
gun in this case is not the same as if he testified that he was not familiar with
any guns at all. Cf. Davis v. State, 216 So. 2d 87, 89 (Fla. 2d DCA 1968)
(witness’s testimony on direct that he never owned any firearm opened the
door to impeachment on cross examination by the witness’s past charge of
exhibition of a dangerous weapon).
[15] Despite the fact that Robertson did not open the door, either by
offering a trait of his good character or by inaccurately testifying to material
facts, the State improperly attacked Robertson’s credibility on cross-
examination by directly asking whether Robertson had threatened anyone
with an assault rifle before the incident involved in this case. Thus, the
State’s argument that Robertson opened the door to questioning about the
prior incident where he had threatened his ex-wife with a gun is sequentially
inaccurate. The State cannot ask a series of impermissible questions
concerning prior acts of misconduct on cross-examination, and then claim
that the defendant opened the door by answering the impermissible
questions.

242
[16] [17] It is critical for the courts to enforce this restriction on
impeachment because the failure to do so would allow the State to
circumvent the procedural requirement of section 90.404(2)(b)(1), which
requires pretrial notice to the defendant when the State intends to introduce
evidence of prior crimes. Because the State could not introduce the evidence
of Robertson’s alleged prior threat to his ex-wife as Williams rule evidence,
the State cannot rely on the law of impeachment to introduce the same
evidence through the back door by asking an impermissible question
regarding an alleged prior crime.
[18] Finally, we address whether the admission of this collateral-crime
evidence constitutes reversible error. This Court has held that the erroneous
admission of irrelevant collateral crimes evidence “is presumed harmful
error because of the danger that a jury will take the bad character or
propensity to crime thus demonstrated as evidence of guilt of the crime
charged.” Castro v. State, 547 So. 2d 111, 115 (Fla.1989) (quoting Straight
v. State, 397 So. 2d 903, 908 (Fla.1981)); accord Czubak v. State, 570 So. 2d
925, 928 (Fla.1990) (holding collateral crime evidence that defendant was an
escaped convict was presumptively harmful). In this case, given the highly
inflammatory nature of Robertson’s ex-wife’s testimony and the emphasis
placed on it by the prosecutor in closing argument, the State is unable to
establish beyond a reasonable doubt and we are unable to conclude that the
admission of this collateral-crime evidence constituted harmless error.
Accordingly, we quash the decision of the Third District and remand for a
new trial consistent with this opinion.
It is so ordered.
ANSTEAD, C.J., SHAW, and QUINCE, JJ., and HARDING, Senior
Justice, concur.
LEWIS, J., concurs in result only.
WELLS, J., dissents with an opinion.
WELLS, J., dissenting. [* * *]

243
XIV. Standards of Review

A. Decisions of Law

State v. Slaughter
574 So. 2d 218 (Fla. 1st DCA 1991)
JOANOS, Judge.
The state appeals an order dismissing with prejudice an information
charging that appellee unlawfully threatened unlawful harm to a public
servant, to wit: a circuit judge, with the intent to influence said public
servant in the performance of his public duty in violation of section 838.021,
Florida Statutes. The issue presented is whether section 838.021(3)(b),
Florida Statutes (1989), is unconstitutional because it impermissibly creates
a class of protected governmental servants who are protected from threats
which have no relation to their official duties. We conclude that the statute is
not unconstitutional, and reverse the ruling of the trial court.
[1] The challenged statute, section 838.021, Florida Statutes (1989),
provides:
838.021 Corruption by threat against public servant.-
(1) Whoever unlawfully harms or threatens unlawful harm to any public
servant, to his immediate family, or to any other person with whose welfare
he is interested, with the intent or purpose:
(a) To influence the performance of any act or omission which the person
believes to be, or the public servant represents as being, within the official
discretion of the public servant, in violation of a public duty, or in
performance of a public duty.
(b) To cause or induce him to use or exert, or procure the use or exertion
of, any influence upon or with any other public servant regarding any act or
omission which the person believes to be, or the public servant represents as
being, within the official discretion of the public servant, in violation of a
public duty, or in performance of a public duty.
(2) Prosecution under this section shall not require any allegation or
proof that the public servant ultimately sought to be unlawfully influenced
was qualified to act in the desired way, that he had assumed office, that the
matter was properly pending before him or might by law properly be

244
brought before him, that he possessed jurisdiction over the matter, or that his
official action was necessary to achieve the person’s purpose.
(3)(a) Whoever unlawfully harms any public servant or any other person
with whose welfare he is interested shall be guilty of a felony of the second
degree, punishable as provided in s. 775.082, s. 775.083, or s. 775.084.
(b) Whoever threatens unlawful harm to any public servant or to any
other person with whose welfare he is interested shall be guilty of a felony
of the third degree, punishable as provided in s. 775.082, s. 775.083, or s.
775.084.
[2] At the outset, an exception to the rule that a trial court’s judgment is
presumptively valid occurs when an appellate court is called upon to pass
upon a statute which the trial court has declared unconstitutional. In such
circumstances, the statute, rather than the trial court’s ruling, is favored with
a presumption of validity. In Re Estate of Caldwell, 247 So. 2d 1, 3
(Fla.1971). That is, all reasonable doubts as to the validity of statutes under
the Constitution are to be resolved in favor of constitutionality. Id. Accord
State, Department of Legal Affairs v. Sanford-Orlando Kennel Club, Inc.,
411 So. 2d 1012, 1015 (Fla. 5th DCA 1982), reversed on other grounds, 434
So. 2d 879 (Fla.1983).
In construing statutes, legislative intent is the key. “The primary rule of
construction is to ascertain and give effect to that intent.” City of Boca Raton
v. Gidman, 440 So. 2d 1277, 1281 (Fla.1983); Parker v. State, 406 So. 2d
1089, 1092 (Fla.1981). One indicator of legislative intent is the title of the
statute. Parker, 406 So. 2d at 1092. If the language of a statute is ambiguous,
the court must determine legislative intent from a consideration of the act as
a whole. Id. Further, when it is reasonably possible and consistent with
legislative intent to do so, the court must give preference to a construction
which would give effect to the statute over a construction which would
defeat it. Schultz v. State, 361 So. 2d 416, 419 (Fla.1978).
[3] The test to be used in determining whether a statutory classification
satisfies the Equal Protection Clause is whether the classification rests on
some difference bearing a reasonable relation to the object of the legislation.
Soverino v. State, 356 So. 2d 269 (Fla.1978). Soverino concerned an equal
protection challenge to section 784.07, which enhances the sanction for
battery when the battery is upon a law enforcement officer or firefighter
rather than upon a person not a member of that class. In Soverino, the court
found that section 784.07 is constitutional, since it reclassified the offense

245
only if the law enforcement officer or firefighter is engaged in the lawful
performance of his duties. In this vein, the statute was deemed reasonably
related to the legislative purpose of affording greater protection to the officer
or firefighter when the officer or firefighter was engaged in protecting the
public welfare.
In the instant case, the trial court found that section 838.021(3)(b) is in
violation of the Equal Protection Clause, based on the view that the statute
does not relate the proscribed threats to a public servant to the performance
of any lawful governmental duty. We disagree. The order under review
indicates that the trial court’s analysis encompassed comparisons and
analogies to statutes having a similar purpose, but the trial court did not
resort to general rules of statutory construction. Application of principles of
statutory construction to section 838.021 indicates that, contrary to the trial
court’s view, the statute properly relates the proscribed threats to public
servants to an intent to influence or affect the performance of the public
servant’s duties. See, generally, Smith v. State, 532 So. 2d 50, 52 (Fla. 2d
DCA 1988).
Although the manner in which the statute is phrased and punctuated
impairs its clarity, legislative intent can be gleaned from a reading of the
statute as a whole, particularly by considering subsection (1) together with
subsection (3). Subsection (1) defines the proscribed offense. Subsection (3)
sets forth the penalties for violation of subsection (1), and “is designed to
protect public officials in the discharge of their duties without unlawful
intimidation.” Smith, 532 So. 2d at 52. When read together, these provisions
evince legislative intent to protect public servants from coercion or unlawful
influence in the performance of a discretionary public duty. See Bragg v.
State, 475 So. 2d 1255 (Fla. 5th DCA 1985). See generally Pangano v. State,
387 So. 2d 349, 350 (Fla.1980). In construing the statutory provision here
under review, the fifth district implicitly found that the statute properly
related the proscribed threats to the performance of a public duty when it
stated: “The elements of the crime require proof of threat of unlawful harm
to a public servant to influence the person threatened to do, or not do, some
discretionary act.” Bragg, 475 So. 2d at 1257. See also Smith, 532 So. 2d at
52. Because the statute reasonably relates the proscription against harm or
threat of harm to a public official to that official’s performance of his public
duties, it does not violate equal protection principles.

246
Accordingly, the order dismissing the information filed against appellee
with prejudice is reversed, and the matter is remanded for further
proceedings.
BARFIELD and WOLF, JJ., concur.

Virginia Ins. Reciprocal v. Walker


765 So. 2d 229 (Fla. 1st DCA 2000)
PADOVANO, J.
[1] The plaintiff, Virginia Insurance Reciprocal, appeals a final summary
judgment on its claim for contribution against the defendants, Dr. Frank C.
Walker and his professional association. The trial court determined that the
plaintiff’s compliance with the medical malpractice presuit screening
procedure was unnecessary and therefore ineffective to toll the running of
the statute of limitations for contribution claims. We conclude that the
medical malpractice presuit screening requirements apply to an action for
contribution based on the alleged medical negligence of a joint tortfeasor.
Although such an action is one that is asserted by a health care provider and
not by the injured party, it is, nonetheless, an action arising out of the
rendering or failure to render medical care. Because the plaintiff’s complaint
was filed within the applicable time limit as tolled, the summary judgment
for the defendants must be reversed.
[***]
We begin with the standard of review of summary judgments, which has
two steps. First, the appellate court must determine whether there was a
genuine issue of material fact on the claim or defense adjudicated by
summary judgment. In deciding this issue, the court must view every
inference in favor of the party against whom the summary judgment was
rendered. See Wills v. Sears, Roebuck & Co., 351 So.2d 29 (Fla.1977); Smith
v. Perry, 635 So. 2d 1019 (Fla. 1st DCA 1994); Williams v. Bevis, 509 So.
2d 1304 (Fla. 1st DCA 1987). The second question is a pure question of law.
Assuming there is no dispute of fact, the appellate court must determine
whether the trial court was correct in holding that the moving party was
entitled to a judgment as a matter of law. See Menendez v. Palms West
Condominium Ass’n, Inc., 736 So. 2d 58 (Fla. 1st DCA 1999). Because the
motion for summary judgment in this case turns on the application of a point

247
of law concerning the statute of limitations, we review the decision of the
trial court by the de novo standard of review.
[***]
For these reasons, we hold that the time for filing a suit for contribution
based on a claim of medical malpractice is tolled by compliance with the
statutory presuit screening requirements, and we certify that our decision is
in conflict with the decision of the Fourth District Court of Appeal in
Wendel. The complaint in the present case was filed within the applicable
time period as tolled. Therefore, the complaint is not barred by the statute of
limitations, and the defendants are not entitled to a summary judgment on
that ground.
Reversed.
DAVIS and BENTON, JJ., Concur.

Ornelas v. United States


517 U.S. 690, 116 S. Ct. 1657, 134 L. Ed.2d 911 (1996)
Chief Justice REHNQUIST delivered the opinion of the Court.
Petitioners each pleaded guilty to possession of cocaine with intent to
distribute. They reserved their right to appeal the District Court’s denial of
their motion to suppress the cocaine found in their car. The District Court
had found reasonable suspicion to stop and question petitioners as they
entered their car, and probable cause to remove one of the interior panels
where a package containing two kilograms of cocaine was found. The Court
of Appeals opined that the findings of reasonable suspicion to stop, and
probable cause to search, should be reviewed “deferentially,” and “for clear
error.” We hold that the ultimate questions of reasonable suspicion and
probable cause to make a warrantless search should be reviewed de novo.
The facts are not disputed. In the early morning of a December day in
1992, Detective Michael Pautz, a 20-year veteran of the Milwaukee County
Sheriff’s Department with 2 years specializing in drug enforcement, was
conducting drug-interdiction surveillance in downtown Milwaukee. Pautz
noticed a 1981 two-door Oldsmobile with California license plates in a
motel parking lot. The car attracted Pautz’s attention for two reasons:
because older model, two-door General Motors cars are a favorite with drug
couriers because it is easy to hide things in them; and because California is a

248
“source State” for drugs. Detective Pautz radioed his dispatcher to inquire
about the car’s registration. The dispatcher informed Pautz that the owner
was either Miguel Ledesma Ornelas or Miguel Ornelas Ledesma from San
Jose, California; Pautz was unsure which name the dispatcher gave.
Detective Pautz checked the motel registry and learned that an Ismael
Ornelas accompanied by a second man had registered at 4 a.m., without
reservations.
Pautz called for his partner, Donald Hurrle, a detective with
approximately 25 years of law enforcement experience, assigned for the past
6 years to the drug enforcement unit. When Hurrle arrived at the scene, the
officers contacted the local office of the Drug Enforcement Administration
(DEA) and asked DEA personnel to run the names Miguel Ledesma Ornelas
and Ismael Ornelas through the Narcotics and Dangerous Drugs Information
System (NADDIS), a federal database of known and suspected drug
traffickers. Both names appeared in NADDIS. The NADDIS report
identified Miguel Ledesma Ornelas as a heroin dealer from El Centro,
California, and Ismael Ornelas, Jr., as a cocaine dealer from Tucson,
Arizona. The officers then summoned Deputy Luedke and the department’s
drug-sniffing dog, Merlin. Upon their arrival, Detective Pautz left for
another assignment. Detective Hurrle informed Luedke of what they knew
and together they waited.
Sometime later, petitioners emerged from the motel and got into the
Oldsmobile. Detective Hurrle approached the car, identified himself as a
police officer, and inquired whether they had any illegal drugs or
contraband. Petitioners answered “No.” Hurrle then asked for identification
and was given two California driver’s licenses bearing the names Saul
Ornelas and Ismael Ornelas. Hurrle asked them if he could search the car
and petitioners consented. The men appeared calm, but Ismael was shaking
somewhat. Deputy Luedke, who over the past nine years had searched
approximately 2,000 cars for narcotics, searched the Oldsmobile’s interior.
He noticed that a panel above the right rear passenger armrest felt somewhat
loose and suspected that the panel might have been removed and contraband
hidden inside. Luedke would testify later that a screw in the doorjam
adjacent to the loose panel was rusty, which to him meant that the screw had
been removed at some time. Luedke dismantled the panel and discovered
two kilograms of cocaine. Petitioners were arrested.

249
[1] [2] Petitioners filed pretrial motions to suppress, alleging that the
police officers violated their Fourth Amendment rights when the officers
detained them in the parking lot and when Deputy Luedke searched inside
the panel without a warrant.FN1 The Government conceded in the court below
that when the officers approached petitioners in the parking lot, a reasonable
person would not have felt free to leave, so the encounter was an
investigatory stop. See 16 F.3d 714, 716 (C.A.7 1994). An investigatory stop
is permissible under the Fourth Amendment if supported by reasonable
suspicion, Terry v. Ohio, 392 U.S. 1, 88 S.Ct. 1868, 20 L.Ed.2d 889 (1968),
and a warrantless search of a car is valid if based on probable cause,
California v. Acevedo, 500 U.S. 565, 569-570, 111 S.Ct. 1982, 1985-1986,
114 L.Ed.2d 619 (1991).
After conducting an evidentiary hearing, the Magistrate Judge concluded
that the circumstances gave the officers reasonable suspicion, but not
probable cause. The Magistrate found, as a finding of fact, that there was no
rust on the screw and hence concluded that Deputy Luedke had an
insufficient basis to conclude that drugs would be found within the panel.
The Magistrate nonetheless recommended that the District Court deny the
suppression motions because he thought, given the presence of the drug-
sniffing dog, that the officers would have found the cocaine by lawful means
eventually and therefore the drugs were admissible under the inevitable
discovery doctrine. See Nix v. Williams, 467 U.S. 431, 104 S.Ct. 2501, 81
L.Ed.2d 377 (1984).
The District Court adopted the Magistrate’s recommendation with respect
to reasonable suspicion, but not its reasoning as to probable cause. The
District Court thought that the model, age, and source-State origin of the car,
and the fact that two men traveling together checked into a motel at 4
o’clock in the morning without reservations, formed a drug-courier profile
and that this profile together with the NADDIS reports gave rise to
reasonable suspicion of drug-trafficking activity; in the court’s view,
reasonable suspicion became probable cause when Deputy Luedke found the
loose panel. Accordingly, the court ruled that the cocaine need not be
excluded.FN2 * * *
[3] The Court of Appeals reviewed deferentially the District Court’s
determinations of reasonable suspicion and probable cause; it would reverse
only upon a finding of “clear error.” FN3 * * * 16 F.3d, at 719. The court
found no clear error in the reasonable-suspicion analysis and affirmed that

250
determination. Ibid. With respect to the probable-cause finding, however, the
court remanded the case for a determination on whether Luedke was credible
when testifying about the loose panel. Id., at 721-722.
On remand, the Magistrate Judge expressly found the testimony credible.
The District Court accepted the finding, and once again ruled that probable
cause supported the search. The Seventh Circuit held that determination not
clearly erroneous. Judgt. order reported at 52 F.3d 328 (1995).
We granted certiorari to resolve the conflict among the Circuits over the
applicable standard of appellate review. 516 U.S. 963, 116 S.Ct. 417, 133
L.Ed.2d 334 (1996).FN4 * * *
The United States, in accord with petitioners, contends that a de novo
standard of review should apply to determinations of probable cause and
reasonable suspicion. We therefore invited Peter D. Isakoff to brief and
argue this case as amicus curiae in support of the judgment below. 516 U.S.
1008, 116 S.Ct. 562, 133 L.Ed.2d 448 (1995). Mr. Isakoff accepted the
appointment and has well fulfilled his assigned responsibility.
[4] Articulating precisely what “reasonable suspicion” and “probable
cause” mean is not possible. They are commonsense, nontechnical
conceptions that deal with “ ‘the factual and practical considerations of
everyday life on which reasonable and prudent men, not legal technicians,
act.’ ” Illinois v. Gates, 462 U.S. 213, 231, 103 S.Ct. 2317, 2328, 76 L.Ed.2d
527 (1983) (quoting Brinegar v. United States, 338 U.S. 160, 175, 69 S.Ct.
1302, 1311, 93 L.Ed. 1879 (1949)); see United States v. Sokolow, 490 U.S.
1, 7-8, 109 S.Ct. 1581, 1585-1586, 104 L.Ed.2d 1 (1989). As such, the
standards are “not readily, or even usefully, reduced to a neat set of legal
rules.” Gates, supra, at 232, 103 S.Ct., at 2329. We have described
reasonable suspicion simply as “a particularized and objective basis” for
suspecting the person stopped of criminal activity, United States v. Cortez,
449 U.S. 411, 417-418, 101 S.Ct. 690, 694-695, 66 L.Ed.2d 621 (1981), and
probable cause to search as existing where the known facts and
circumstances are sufficient to warrant a man of reasonable prudence in the
belief that contraband or evidence of a crime will be found, see Brinegar,
supra, at 175-176, 69 S.Ct., at 1310-1311; Gates, supra, at 238, 103 S.Ct., at
2332. We have cautioned that these two legal principles are not “finely-
tuned standards,” comparable to the standards of proof beyond a reasonable
doubt or of proof by a preponderance of the evidence. Gates, supra, at 235,
103 S.Ct., at 2330-2331. They are instead fluid concepts that take their

251
substantive content from the particular contexts in which the standards are
being assessed. Gates, supra, at 232, 103 S.Ct., at 2329; Brinegar, supra, at
175, 69 S.Ct., at 1310 (“The standard of proof [for probable cause] is ...
correlative to what must be proved”); Ker v. California, 374 U.S. 23, 33, 83
S.Ct. 1623, 1630, 10 L.Ed.2d 726 (1963) (“This Cour[t] [has a] long-
established recognition that standards of reasonableness under the Fourth
Amendment are not susceptible of Procrustean application”; “[e]ach case is
to be decided on its own facts and circumstances” (internal quotation marks
omitted)); Terry v. Ohio, 392 U.S., at 29, 88 S.Ct., at 1884 (the limitations
imposed by the Fourth Amendment “will have to be developed in the
concrete factual circumstances of individual cases”).
[5] The principal components of a determination of reasonable suspicion
or probable cause will be the events which occurred leading up to the stop or
search, and then the decision whether these historical facts, viewed from the
standpoint of an objectively reasonable police officer, amount to reasonable
suspicion or to probable cause. The first part of the analysis involves only a
determination of historical facts, but the second is a mixed question of law
and fact: “[T]he historical facts are admitted or established, the rule of law is
undisputed, and the issue is whether the facts satisfy the [relevant] statutory
[or constitutional] standard, or to put it another way, whether the rule of law
as applied to the established facts is or is not violated.” Pullman-Standard v.
Swint, 456 U.S. 273, 289, n. 19, 102 S.Ct. 1781, 1791, n. 19, 72 L.Ed.2d 66
(1982).
[6] We think independent appellate review of these ultimate
determinations of reasonable suspicion and probable cause is consistent with
the position we have taken in past cases. We have never, when reviewing a
probable-cause or reasonable-suspicion determination ourselves, expressly
deferred to the trial court’s determination. See, e.g., Brinegar, supra
(rejecting District Court’s conclusion that the police lacked probable cause);
Alabama v. White, 496 U.S. 325, 110 S.Ct. 2412, 110 L.Ed.2d 301 (1990)
(conducting independent review and finding reasonable suspicion). A policy
of sweeping deference would permit, “[i]n the absence of any significant
difference in the facts,” “the Fourth Amendment’s incidence [to] tur[n] on
whether different trial judges draw general conclusions that the facts are
sufficient or insufficient to constitute probable cause.” Brinegar, supra, at
171, 69 S.Ct., at 1308. Such varied results would be inconsistent with the

252
idea of a unitary system of law. This, if a matter-of-course, would be
unacceptable.
In addition, the legal rules for probable cause and reasonable suspicion
acquire content only through application. Independent review is therefore
necessary if appellate courts are to maintain control of, and to clarify, the
legal principles. See Miller v. Fenton, 474 U.S. 104, 114, 106 S.Ct. 445,
451, 88 L.Ed.2d 405 (1985) (where the “relevant legal principle can be given
meaning only through its application to the particular circumstances of a
case, the Court has been reluctant to give the trier of fact’s conclusions
presumptive force and, in so doing, strip a federal appellate court of its
primary function as an expositor of law”).
Finally, de novo review tends to unify precedent and will come closer to
providing law enforcement officers with a defined “ ‘set of rules which, in
most instances, makes it possible to reach a correct determination
beforehand as to whether an invasion of privacy is justified in the interest of
law enforcement.’ ” New York v. Belton, 453 U.S. 454, 458, 101 S.Ct. 2860,
2863, 69 L.Ed.2d 768 (1981); see also Thompson v. Keohane, 516 U.S. 99,
115, 116 S.Ct. 457, 467, 133 L.Ed.2d 383 (1995) (“[T]he law declaration
aspect of independent review potentially may guide police, unify precedent,
and stabilize the law,” and those effects “serve legitimate law enforcement
interests”).
It is true that because the mosaic which is analyzed for a reasonable-
suspicion or probable-cause inquiry is multi-faceted, “one determination will
seldom be a useful ‘precedent’ for another,” Gates, supra, at 238, n. 11, 103
S.Ct., at 2332, n. 11. But there are exceptions. For instance, the
circumstances in Brinegar, supra, and Carroll v. United States, 267 U.S.
132, 45 S.Ct. 280, 69 L.Ed. 543 (1925), were so alike that we concluded that
reversing the Court of Appeals’ decision in Brinegar was necessary to be
faithful to Carroll. Brinegar, supra, at 178, 69 S.Ct., at 1312 (“Nor ... can
we find in the present facts any substantial basis for distinguishing this case
from the Carroll case”). We likewise recognized the similarity of facts in
United States v. Sokolow, supra, and Florida v. Royer, 460 U.S. 491, 103
S.Ct. 1319, 75 L.Ed.2d 229 (1983) (in both cases, the defendant traveled
under an assumed name; paid for an airline ticket in cash with a number of
small bills; traveled from Miami, a source city for illicit drugs; and appeared
nervous in the airport). The same was true both in United States v. Ross, 456
U.S. 798, 102 S.Ct. 2157, 72 L.Ed.2d 572 (1982), and California v.

253
Acevedo, 500 U.S. 565, 111 S.Ct. 1982, 114 L.Ed.2d 619 (1991), see id., at
572, 111 S.Ct., at 1987 (“The facts in this case closely resemble the facts in
Ross ”); and in United States v. Mendenhall, 446 U.S. 544, 100 S.Ct. 1870,
64 L.Ed.2d 497 (1980), and Reid v. Georgia, 448 U.S. 438, 100 S.Ct. 2752,
65 L.Ed.2d 890 (1980), see id., at 443, 100 S.Ct., at 2755 (Powell, J.,
concurring) (“facts [in Mendenhall ] [are] remarkably similar to those in the
present case”). And even where one case may not squarely control another
one, the two decisions when viewed together may usefully add to the body
of law on the subject.
[7] The Court of Appeals, in adopting its deferential standard of review
here, reasoned that de novo review for warrantless searches would be
inconsistent with the “ ‘great deference’ ” paid when reviewing a decision to
issue a warrant, see Illinois v. Gates, 462 U.S. 213, 103 S.Ct. 2317, 76
L.Ed.2d 527 (1983). See United States v. Spears, 965 F.2d 262, 269-271
(C.A.7 1992). We cannot agree. The Fourth Amendment demonstrates a
“strong preference for searches conducted pursuant to a warrant,” Gates,
supra, at 236, 103 S.Ct., at 2331, and the police are more likely to use the
warrant process if the scrutiny applied to a magistrate’s probable-cause
determination to issue a warrant is less than that for warrantless searches.
Were we to eliminate this distinction, we would eliminate the incentive.
We therefore hold that as a general matter determinations of reasonable
suspicion and probable cause should be reviewed de novo on appeal. Having
said this, we hasten to point out that a reviewing court should take care both
to review findings of historical fact only for clear error and to give due
weight to inferences drawn from those facts by resident judges and local law
enforcement officers.
A trial judge views the facts of a particular case in light of the distinctive
features and events of the community; likewise, a police officer views the
facts through the lens of his police experience and expertise. The
background facts provide a context for the historical facts, and when seen
together yield inferences that deserve deference. For example, what may not
amount to reasonable suspicion at a motel located alongside a
transcontinental highway at the height of the summer tourist season may rise
to that level in December in Milwaukee. That city is unlikely to have been
an overnight stop selected at the last minute by a traveler coming from
California to points east. The 85-mile width of Lake Michigan blocks any
further eastward progress. And while the city’s salubrious summer climate

254
and seasonal attractions bring many tourists at that time of year, the same is
not true in December. Milwaukee’s average daily high temperature in that
month is 31 degrees and its average daily low is 17 degrees; the percentage
of possible sunshine is only 38 percent. It is a reasonable inference that a
Californian stopping in Milwaukee in December is either there to transact
business or to visit family or friends. The background facts, though rarely
the subject of explicit findings, inform the judge’s assessment of the
historical facts.
In a similar vein, our cases have recognized that a police officer may
draw inferences based on his own experience in deciding whether probable
cause exists. See, e.g., United States v. Ortiz, 422 U.S. 891, 897, 95 S.Ct.
2585, 2589, 45 L.Ed.2d 623 (1975). To a layman the sort of loose panel
below the back seat armrest in the automobile involved in this case may
suggest only wear and tear, but to Officer Luedke, who had searched
roughly 2,000 cars for narcotics, it suggested that drugs may be secreted
inside the panel. An appeals court should give due weight to a trial court’s
finding that the officer was credible and the inference was reasonable.
We vacate the judgments and remand the case to the Court of Appeals to
review de novo the District Court’s determinations that the officer had
reasonable suspicion and probable cause in this case.
It is so ordered.
Justice SCALIA, dissenting. * * *

B. Decisions of Fact

Shaw v. Shaw
334 So. 2d 13 (Fla. 1976)
SUNDBERG, Justice.
This is a petition for writ of certiorari to review a decision of the Third
District Court of Appeal reported at 314 So.2d 205, which is asserted to be
in conflict with Westerman v. Shell’s City, Inc., 265 So.2d 43 (Fla.1972), as
well as similar cases which announce the proposition that an appellate court
may not substitute its judgment for that of the trial court by re-evaluating the
evidence in the cause.[FN1] Jurisdiction vests in this Court pursuant to
Article V, Section 3(b)(3), Florida Constitution.

255
FN1. See, e.g., Pope v. O’Brien, 213 So. 2d 620 (1st D.C.A.Fla.1968);
Cole v. Cole, 130 So. 2d 126 (1st D.C.A.Fla.1961); and Smith v. State, 118
So. 2d 257 (2d D.C.A.Fla.1960).
In proceedings for dissolution of marriage between petitioner-husband
and respondent-wife, in which respondent was the moving party, the trial
judge on July 15, 1974, entered an amended final judgment (i) finding that
the marriage was irretrievably broken and thereby dissolving the bonds of
marriage between the parties; (ii) finding that wife, ‘although she is in her
49th year and has been married 20 years and has minor children, is capable
of supporting herself’ and, based upon the financial affidavits of the parties,
exhibits admitted into evidence, and the testimony, that the husband could
not sustain both households, therefore ordering that the husband pay to the
wife rehabilitation alimony in the amount of $50 per week for a period of
one year commencing July 1, 1974; (iii) adjudicating that certain savings
accounts of the parties belonged to both parties equally and ordering that the
accounts be equalized based upon balances as of the date of the final
hearing; (iv) granting custody of the two minor children of the parties to the
wife with rights of visitation in the husband and ordering the husband to pay
to the wife the sum of $30 per week per child until said child reaches
majority, marries or becomes self-supporting; (v) granting use and
occupancy of the marital home to the wife until the children move out or
until she remarries or the children reach their majority provided that the wife
should make all payments on the home place, including mortgage payments,
taxes, insurance, utilities, and all repairs and maintenance with the
stipulation that the home place together with its contents belong to the
parties as tenants in common; (vi) providing that all future medical and
dental expenses of the children were to be divided equally between the
husband and wife; (vii) providing for transfer by the husband to the wife of
title to an automobile in the possession of the wife; and (viii) reserving
jurisdiction for costs and assessment, if any, of attorneys’ fees, but not
retaining jurisdiction for any other purposes.
The evidence at the final hearing upon which the amended final judgment
is based was essentially as follows: The 49-year-old wife has a high school
education, plus one year of business college. She has also attended court
reporting school and has engaged in sales work for less than a year. She has
had 16 years of experience in secretarial work, in addition to three years’

256
experience as a legal secretary, plus 10 to 12 years of legal secretarial
experience working intermittently for the husband.
The husband is an attorney at law practicing in Miami, Florida. Although
his 1973 tax return reflects an adjusted gross income in excess of $30,500,
the evidence reflected that, over the past five years, he has realized an
average weekly net income of $247.25. As testified by the wife, the parties
have lived modestly and ‘things have been tight from time to time.’ The
home of the parties is modest and has never been fully painted. The husband
drives a 1971 Volkswagen, and the wife drives a 1968 Ford. Title to the
home place was held by the husband and wife as a tenancy by the entireties.
The marriage produced four children, one of whom is deceased. The
oldest child is a male, 19 years of age, and is self-supporting. The remaining
two children are daughters, Leslie, age 16, and Julie, age 8.
The marital home was purchased entirely with funds of the husband. The
present mortgage payments on the home are $85 per month. The only other
assets of the parties were a savings account in the amount of $10,450 held in
the name of the husband and a savings account in the sum of $4,500 held in
joint name of husband and wife. At the time of commencement of the suit
for dissolution of the marriage, the wife withdrew $4,300 from the joint
savings account, $1,000 of which she paid to her attorney for his
representation in the proceedings.
An employment expert testified at the final hearing. Based upon
information related to him concerning the wife’s total background, including
her education, work experience, physical complaints (which were
unsubstantiated by medical testimony), age and current activities, he testified
that the wife is employable, that there is work available for her, and that she
could earn up to $175 per week. There was also evidence adduced from the
wife that on occasions after the separation she attended night art classes at
which time the older daughter babysat for the younger.
In reviewing the trial court’s amended final judgment in light of the
record, the district court concluded that there was no abuse of discretion
except (i) in failure of the trial court to reserve jurisdiction so that it might
reconsider an extension of alimony in the light of changed circumstances
because the wife testified that she was limited in seeking employment due to
her responsibilities in caring for the children and also because of a physical
problem with her arm; (ii) in adjudicating the wife solely responsible for the
mortgage payments, taxes and insurance upon the marital home; and (iii) in

257
requiring the wife to pay one-half of all future medical and dental expenses
of the minor children. Accordingly, the district court of appeal modified the
amended final judgment to provide for retention of jurisdiction for purposes
of entertaining a petition for modification to continue the payments of
alimony to her; to require that the husband be solely responsible for all
future medical and dental expenses of the minor children; and to require the
husband to pay the mortgage payments, taxes and insurance upon the marital
home with the proviso that upon the eventual sale thereof the husband shall
be entitled to a credit for one-half of such payments.
[1] [2] [3] [4] [5] We concur with the husband that the decision of the
district court of appeal in the instant case conflicts with the principles of law
enunciated in Westerman v. Shell’s City, Inc., supra. It is clear that the
function of the trial court is to evaluate and weigh the testimony and
evidence based upon its observation of the bearing, demeanor and credibility
of the witnesses appearing in the cause. It is not the function of the appellate
court to substitute its judgment for that of the trial court through re-
evaluation of the testimony and evidence from the record on appeal before it.
The test, as pointed out in Westerman, supra, is whether the judgment of the
trial court is supported by competent evidence. Subject to the appellate
court’s right to reject ‘inherently incredible and improbable testimony or
evidence,’[FN2] * * * it is not the prerogative of an appellate court, upon a
de novo consideration of the record, to substitute its judgment for that of the
trial court. The district court in the case at bar found it to be an abuse of the
trial court’s discretion not to reserve jurisdiction so that the court might
reconsider an extension of alimony in the light of changed circumstances.
The district court of appeal ‘deem(ed) it prudent’ to make such a reservation
in that the wife testified she was limited from seeking employment because
of her responsibilities in caring for the children and also because of a
physical problem with her arm. As indicated in Elkins v. Elkins, 287 So.2d
119, 120 (3d D.C.A.Fla.1973), cited by the district court of appeal, ‘A
chancellor is not required as a matter of law to reserve jurisdiction to award
periodic alimony in the future, rather it is a matter within his discretion.’ It
appears from the record that the only evidence of the wife’s physical
problem was her own uncorroborated testimony. It further appears that the
wife was content to leave the 8-year-old child in the charge of her 16-year-
old sister during nighttime periods when she attended art classes. The trial
judge was in a superior position to assess the significance of the wife’s
asserted physical impairment due to her ability to observe the demeanor of

258
the wife while on the stand. She apparently concluded that neither the
asserted physical disability nor responsibility to care for the minor children
of the parties would be an impediment to the wife’s seeking employment
where there was uncontradicted testimony that such employment was
available to her. Hence, her conclusion that reservation of jurisdiction to
entertain petitions for modification of the alimony award was unnecessary
cannot be said to be unsupported by the evidence nor an abuse of discretion.
Although the appellate court concluded that it would be ‘prudent’ in its
judgment to include such a reservation of jurisdiction, it was not error for the
trial court to fail to do so. Goldfarb v. Robertson, 82 So. 2d 504, 506
(Fla.1955).
[6] With respect to mortgage payments, taxes and insurance upon the
marital home and payment of future medical and dental expenses of the
minor children, the record reflects that the trial judge predicated her decision
upon the relative financial resources and earning capacity of the parties in
light of the disposition of the parties’ assets, including the marital home. The
district court of appeal apparently placed substantial weight upon the fact
that the husband’s 1973 tax return disclosed an adjusted gross income in
excess of $30,500. However, the evidence also disclosed that the husband,
for the preceding five years, realized an average weekly net income of
$247.25. Although the district court of appeal, and even this Court, might
honestly strike the financial balance and division of assets between the
parties in a different fashion, we do not deem it error or an abuse of
discretion for the trial court to arrive at the result it reached.
Accordingly, the petition for writ of certiorari is granted, and the decision
of the Third District Court of Appeal is quashed, with instructions to remand
to the trial court for reinstatement of the amended final judgment.
It is so ordered.
OVERTON, C.J., and ROBERTS, ADKINS and ENGLAND, JJ.,
concur.
BOYD and HATCHETT, JJ., dissent.

259
C. Discretionary Decisions

Canakaris v. Canakaris
382 So. 2d 1197 (Fla. 1980)
OVERTON, Justice.
This is a petition for writ of certiorari to review a decision of the First
District Court of Appeal reported at 356 So. 2d 858 (Fla. 1st DCA 1978).
The district court reversed the trial court’s award to the wife of the
husband’s undivided one-half interest in their marital home as lump sum
alimony, the award of permanent periodic alimony, and attorney’s fees. We
find conflict with Yandell v. Yandell, 39 So. 2d 554 (Fla.1949); Brown v.
Brown, 300 So. 2d 719 (Fla. 1st DCA 1974); and Calligarich v. Calligarich,
256 So. 2d 60 (Fla. 4th DCA 1971). We have jurisdiction.[FN1] We reverse
the district court and reinstate the judgment of the trial court.
FN1. Art. V, s 3(b)(3), Fla.Const.
This dissolution proceeding terminated a thirty-three-year marriage, the
parties having married when the wife was seventeen and the husband
twenty-one. The wife assisted the husband through college and worked in
his first medical office and in the hospital started by her husband. This
proceeding commenced in 1963 upon a complaint filed by the wife for
separate maintenance on grounds of adultery and extreme cruelty. A
temporary support order was entered in 1963, with which the parties
complied until the final judgment for dissolution was entered in December,
1976. The issues in this cause concern the disposition of wealth accumulated
during the marriage and the determination of alimony for the wife.
The financial statements of the parties reflected a significant
accumulation of wealth during the marriage. The husband, a medical doctor,
operated a hospital located on property owned by the parties. His admitted
net worth was $3,749,930. A substantial portion of these assets had been
acquired prior to the 1963 separation proceedings. The wife’s net worth
prior to the dissolution totaled no more than $292,000, and this amount is
entirely from her share of jointly held properties, except for a $15,000
inheritance. These jointly held properties were acquired during the marital
partnership and were not the result of any premarital or independent source,
such as a gift or inheritance. The annual income for the husband from 1971
through 1975 was never less than $130,000, and in 1975 his income was
$147,000. During this period the wife’s annual income was approximately

260
$1,000. Since their separation, the husband has acquired another home
consisting of eighty acres valued at $430,000.
In the final judgment of dissolution, the wife was awarded lump sum
alimony consisting of $50,000 in cash and the husband’s interest in their
jointly owned residence in which she was living; the residence was valued at
$75,000. The wife further received the sum of $500 per week permanent
periodic alimony and the automobile then in her possession. She retained her
undivided one-half interest in the Bunnell General Hospital real estate,
which was held by the parties as a tenancy by the entirety. Apart from
periodic alimony, the wife received property and assets worth approximately
$385,000, a majority of which consisted of her undivided one-half interest in
the hospital real estate. The final judgment also directed the husband to pay
the wife’s attorney’s fees in an amount to be set at a future hearing.
The district court reversed this final judgment, holding the award of the
marital home as lump sum alimony improper because “(a) review of the
record reveals no special equity of the wife in the marital home.” 356 So. 2d
at 860. The court cited as authority Cann v. Cann, 334 So. 2d 325 (Fla. 1st
DCA 1976), which held that lump sum alimony should be awarded only
where special equities require it or make it possible. The district court also
found insufficient evidence indicating needs of the wife which would
warrant the periodic alimony award of $500 per week. It remanded the
periodic alimony award to the trial court for the limited purpose of
“determining, based upon evidence, the needs of the wife.” 356 So. 2d at
860. Finally, the district court concluded that the award of attorney’s fees
was improper because the wife had the ability to pay for the services of her
attorney. We reject each of these holdings by the district court.
[1] Section 61.08, Florida Statutes, authorizes the trial judge to: grant
alimony to either party, which alimony may be rehabilitative or permanent
in nature. In any award of alimony, the court may order periodic payments
or payments in lump sum or both. . . . In determining a proper award of
alimony, the court may consider any factor necessary to do equity and
justice between the parties. (Emphasis supplied.)
In considering the appropriate criteria for the award of the different types
of alimony, it is important that appellate courts avoid establishing inflexible
rules that make the achievement of equity between the parties difficult, if not
impossible.

261
The issues presented in this cause require an examination of criteria
constituting “lump sum” and “permanent periodic alimony”; both are
important elements in the determination of property disposition and support
requirements in a dissolution proceeding. The related elements of “special
equity” and “exclusive possession of property” are discussed in the
accompanying case, Duncan v. Duncan, 379 So. 2d 949 (Fla.1980). We
recognize that the decisions in this subject area, both of this Court and of the
district courts of appeal, are not reconcilable. It is our intent that these two
opinions, to the extent possible, will bring some stability to this area of the
law.
Lump Sum Alimony
The district court in the instant decision held that the award of the
husband’s one-half interest to the wife as lump sum alimony was
inappropriate because she had no “special equity” in the marital home. The
term “special equity” has regrettably been used by this Court and the district
courts of appeal to justify both (1) a nonalimony property interest, and (2) an
award of lump sum alimony. The use of the term “special equity” to identify
facts which allow an award of lump sum alimony is in fact a misnomer and
has caused confusion. To eliminate this confusion, it is necessary to
distinguish the purposes for which the two types of “special equity” have
been used.
[2] The term “special equity” was created to describe a vested interest in
property brought into the marriage or acquired during the marriage because
of contribution of services or funds over and above normal marital duties.
Eakin v. Eakin, 99 So. 2d 854 (Fla.1958); Heath v. Heath, 103 Fla. 1071,
138 So. 796 (1932). This vested interest is not alimony. The “special equity”
doctrine was developed to avoid the inequities of the existing statutory
provision which denied alimony to an adulterous wife despite her special
contribution of services or funds over and above normal marital duties.
Although the statutory prohibitions underlying the formulation of the special
equity doctrine no longer exist, this vested interest aspect of the doctrine
remains a viable part of our case law.
The term “special equity” has also been used to justify an award of lump
sum alimony. When employed in this context, it concerns only whether the
equities of the case justify a lump sum award. The property interest or lien
concept of “special equity” is entirely distinct from the determination of
parties’ equities in a lump sum alimony award. The term “special equity”

262
should not be used when considering lump sum alimony; rather, it should be
used only when analyzing a vested property interest of a spouse. See Ball v.
Ball, 335 So. 2d 5 (Fla.1976); Eakin v. Eakin; Heath v. Heath.
[3] [4] [5] In granting lump sum alimony, the trial court should be
guided by all relevant circumstances to ensure “equity and justice between
the parties.” s 61.08, Fla.Stat. This Court recognized in Yandell v. Yandell
the broad scope of the trial judge’s discretion in granting lump sum alimony,
consistent with the statutory mandate. The “special equities” referred to in
Yandell concern only the general equities of the case. We stated in Yandell :
(O)rdinarily . . . a lump award should be made only in those instances
where some special equities might require it or make it advisable; for
instance, where the wife may have brought to the marriage, or assisted her
husband in accumulating, property. . . . There may be other situations which
might justify or possibly require a lump sum award . . . . A lump sum
allowance of permanent alimony is not “fit, equitable and just” unless the
husband is in a position to make payment of the sum so granted over and
above the requirements attendant upon the maintenance of his business or
employment . . . .
(W)here the husband’s financial condition is such as to enable him to
respond to a gross allowance of alimony without jeopardy to his business,
profession or employment it may be the better solution to grant such
allowance when all other equitable considerations justify it . . . .
39 So. 2d at 556-57 (citations omitted) (emphasis supplied). Yandell
clearly does not limit the use of lump sum alimony to instances of support or
vested property interests. A judge may award lump sum alimony to ensure
an equitable distribution of property acquired during the marriage, provided
the evidence reflects (1) a justification for such lump sum payment and (2)
financial ability of the other spouse to make such payment without
substantially endangering his or her economic status. Brown v. Brown. In
our opinion, the award of the marital home as lump sum alimony may be
coupled with other lump sum alimony or permanent periodic alimony
awards if justified by the evidence. The collective use of such alimony
awards has been approved recently by this Court in McDonald v. McDonald,
368 So. 2d 1283 (Fla.1979), and by the Third District Court of Appeal. See
Hyatt v. Hyatt, 315 So. 2d 11 (Fla. 3d DCA 1975); Vandervoort v.
Vandervoort, 300 So. 2d 694 (Fla. 3d DCA 1974).

263
[6] Although the award of lump sum alimony is not dependent upon a
finding of a prior vested right, there does arise upon the entry of a final
judgment of a lump sum award a vested right which is neither terminable
upon a spouse’s remarriage or death nor subject to modification. It may
consist of real or personal property, or may be a monetary award payable in
installments. Jurisdiction may be expressly retained, however, to terminate
lump sum alimony installment payments upon a spouse’s remarriage or
death when the parties agree to such a provision in a property settlement
agreement. Further, jurisdiction may be retained to enter periodic alimony if
found necessary after such termination of lump sum alimony installment
payments. See Hyatt v. Hyatt; Langston v. Langston, 257 So. 2d 625 (Fla.3d
DCA 1972).
Permanent Periodic Alimony
[7] Permanent periodic alimony is used to provide the needs and the
necessities of life to a former spouse as they have been established by the
marriage of the parties. The two primary elements to be considered when
determining permanent periodic alimony are the needs of one spouse for the
funds and the ability of the other spouse to provide the necessary funds. The
criteria to be used in establishing this need include the parties’ earning
ability, age, health, education, the duration of the marriage, the standard of
living enjoyed during its course, and the value of the parties’ estates.
[8] A spouse’s ability to pay may be determined not only from net
income, but also net worth, past earnings, and the value of the parties’
capital assets. Firestone v. Firestone, 263 So. 2d 223 (Fla.1972).
Responsibilities to other dependents for support needs may be considered
when determining a spouse’s financial capacity.
[9] While permanent periodic alimony is most commonly used to provide
support, in limited circumstances its use may be appropriate to balance such
inequities as might result from the allocation of income-generating
properties acquired during the marriage. Patterson v. Patterson, 315 So. 2d
104 (Fla. 4th DCA 1975).
[10] As a general rule, permanent periodic alimony is terminated upon
the death of either spouse or the remarriage of the receiving spouse. First
National Bank in St. Petersburg v. Ford, 283 So. 2d 342 (Fla.1973); In re
Estate of Freeland, 182 So. 2d 425 (Fla.1966).
[11] [12] [13] Once instituted, permanent periodic alimony is subject to
modification upon a substantial change of circumstances, Chastain v.

264
Chastain, 73 So. 2d 66 (Fla.1954), and may be converted to rehabilitative
alimony if the circumstances warrant such a change in the alimony scheme.
Although rehabilitative alimony is not at issue in these proceedings, it is
necessary to define its purpose in order to distinguish it from permanent
periodic alimony. The principal purpose of rehabilitative alimony is to
establish the capacity for self-support of the receiving spouse, either through
the redevelopment of previous skills or provision of the training necessary to
develop potential supportive skills. Reback v. Reback, 296 So. 2d 541 (Fla.
3d DCA 1974). Where appropriate, rehabilitative alimony may be converted
to permanent periodic alimony.
Judicial Discretion of the Trial Judge
Dissolution proceedings present a trial judge with the difficult problem of
apportioning assets acquired by the parties and providing necessary support.
The judge possesses broad discretionary authority to do equity between the
parties and has available various remedies to accomplish this purpose,
including lump sum alimony, permanent periodic alimony, rehabilitative
alimony, child support, a vested special equity in property, and an award of
exclusive possession of property. As considered by the trial court, these
remedies are interrelated; to the extent of their eventual use, the remedies are
part of one overall scheme. It is extremely important that they also be
reviewed by appellate courts as a whole, rather than independently.
In order to properly review orders of the trial judge, appellate courts must
recognize the distinction between an incorrect application of an existing rule
of law and an abuse of discretion. Where a trial judge fails to apply the
correct legal rule, as when he refuses to terminate periodic alimony upon
remarriage of the receiving spouse, the action is erroneous as a matter of
law. This is not an abuse of discretion. The appellate court in reviewing such
a situation is correcting an erroneous application of a known rule of law.
However, where the action of the trial judge is within his judicial
discretion, as in the establishment of the amount of alimony or award of
child custody, the manner of appellate review is altogether different.
Judicial discretion is defined as:
The power exercised by courts to determine questions to which no strict
rule of law is applicable but which, from their nature, and the circumstances
of the case, are controlled by the personal judgment of the court.
1 Bouvier’s Law Dictionary and Concise Encyclopedia 804 (8th ed.
1914). Our trial judges are granted this discretionary power because it is

265
impossible to establish strict rules of law for every conceivable situation
which could arise in the course of a domestic relation proceeding. The trial
judge can ordinarily best determine what is appropriate and just because
only he can personally observe the participants and events of the trial.
[14] We cite with favor the following statement of the test for review of
a judge’s discretionary power:
Discretion, in this sense, is abused when the judicial action is arbitrary,
fanciful, or unreasonable, which is another way of saying that discretion is
abused only where no reasonable man would take the view adopted by the
trial court. If reasonable men could differ as to the propriety of the action
taken by the trial court, then it cannot be said that the trial court abused its
discretion.
Delno v. Market Street Railway Company, 124 F.2d 965, 967 (9th Cir.
1942).
In reviewing a true discretionary act, the appellate court must fully
recognize the superior vantage point of the trial judge and should apply the
“reasonableness” test to determine whether the trial judge abused his
discretion. If reasonable men could differ as to the propriety of the action
taken by the trial court, then the action is not unreasonable and there can be
no finding of an abuse of discretion. The discretionary ruling of the trial
judge should be disturbed only when his decision fails to satisfy this test of
reasonableness.
The discretionary power that is exercised by a trial judge is not, however,
without limitation, and both appellate and trial judges should recognize the
concern which arises from substantial disparities in domestic judgments
resulting from basically similar factual circumstances. The appellate courts
have not been helpful in this regard. Our decisions and those of the district
courts are difficult, if not impossible, to reconcile. The trial court’s
discretionary power is subject only to the test of reasonableness, but that test
requires a determination of whether there is logic and justification for the
result. The trial courts’ discretionary power was never intended to be
exercised in accordance with whim or caprice of the judge nor in an
inconsistent manner. Judges dealing with cases essentially alike should reach
the same result. Different results reached from substantially the same facts
comport with neither logic nor reasonableness. In this regard, we note the
cautionary words of Justice Cardozo concerning the discretionary power of
judges:

266
The judge, even when he is free, is still not wholly free. He is not to
innovate at pleasure. He is not a knight-errant roaming at will in pursuit of
his own ideal of beauty or of goodness. He is to draw his inspiration from
consecrated principles. He is not to yield to spasmodic sentiment, to vague
and unregulated benevolence. He is to exercise a discretion informed by
tradition, methodized by analogy, disciplined by system, and subordinated to
“the primordial necessity of order in the social life.” Wide enough in all
conscience is the field of discretion that remains.
B. Cardozo, The Nature of the Judicial Process 141 (1921).
The Instant Case
In the instant case there was no rule of law which dictated the property
disposition and the alimony terms. The trial judge had the discretionary
authority to establish such terms as would be equitable under the
circumstances. We find that the award to the wife of $500 a week alimony
and the marital home as lump sum alimony, as well as the grant of attorney’s
fees, was not an abuse of discretion.
Given the factual circumstances of this cause, it is appropriate that we
consider the principles enunciated by the First District Court of Appeal in
Brown v. Brown, 300 So. 2d 719 (Fla. 1st DCA 1974), concerning the
distribution of accumulated material wealth in a dissolution proceeding. In
speaking for the court, Judge Rawls stated:
How shall the material wealth of a marriage which is being dissolved be
divided when one partner, the wife, has contributed her time to the marital
home and children of the parties while the husband has pursued the
accumulation of material goods. The evolution of the law of alimony that we
have reviewed in length shows that today the contributions of each party to
the accumulation of material assets must be considered in dissolving the
marital partnership. Either spouse may contribute either by working in the
market place or by working as a homemaker. The fact that in one marital
venture a spouse is gainfully employed in the market place and pays a
housekeeper to rear the children and keep house is not distinguishable from
the spouse who devotes his or her full time to the profession of homemaker.
The primary factual circumstance is each spouse’s contribution to the
marital partnership. In the case sub judice, the wife has been shortchanged.
The wife has not been adequately compensated for the contribution that she
made as a fulltime mother and homemaker to the equal partnership marriage.
We hold that the trial court abused its discretion in awarding the wife a

267
pittance of the material assets accumulated in the husband’s name during 21
years. In so holding, we emphasize even though the cited authorities on the
subject speak of “equal partners” and complete equality as partners, we are
not engrafting upon the jurisprudence of this state the law of community
property. On the question of alimony the judgment is reversed with
instructions to the trial court to enter an award of lump sum alimony
sufficient to compensate the wife for her contribution to the marriage. Id. at
726. The court emphasized that its policy was not grounded upon principles
of community property, but on basic fairness; a dissolution award should be
sufficient to compensate the wife for her contribution to the marriage.
[15] We recognize that a trial court need not equalize the financial
position of the parties. However, a trial judge must ensure that neither
spouse passes automatically from misfortune to prosperity or from
prosperity to misfortune, and, in viewing the totality of the circumstances,
one spouse should not be “shortchanged.” Brown v. Brown.
In the instant case, the parties were married when both were quite young.
The wife assisted to a limited extent in the profession and business of the
husband and maintained the home while the husband achieved substantial
success in his medical and business careers. All the wealth and property
involved in this proceeding were acquired during the marriage of the parties,
and a large part of these assets were acquired prior to the separation. The
total amount of property awards to the wife was approximately $385,000,
including the original home and a $50,000 lump sum cash award. The total
value of the property remaining with the husband exceeded $3.3 million.
[16] We find the district court erred in holding the lump sum alimony
award of the husband’s one-half interest in the marital home improper; we
believe its holding resulted from an improper application of the vested
interest “special equity” doctrine. This lump sum alimony award was clearly
within the trial court’s discretion and was justified as part of an equitable
distribution of the property of the parties acquired during their marriage.
[17] We find that the trial court’s periodic alimony award of $500 per
week was reasonable and should not be disturbed. Given the income of the
parties, the length of the marriage and the standard of living enjoyed by the
parties, the age and education of the wife, and the totality of the parties’
respective financial circumstances, this alimony award is neither
unreasonable nor arbitrary.[FN2] * * * We acknowledge that reasonable
persons might differ as to what is an appropriate sum for permanent periodic

268
alimony in this cause, but we find it is within the parameters of
reasonableness; therefore, there can be no finding of an abuse of discretion.
[18] [19] The final issue presented to us is whether the award of the
wife’s attorney’s fees, to be determined at a subsequent hearing, was error.
In Cummings v. Cummings, 330 So. 2d 134, 136 (Fla.1976), we cited Mertz
v. Mertz, 287 So. 2d 691 (Fla. 2d DCA 1973), as correctly stating that the
purpose of section 61.16, Florida Statutes, was to ensure that both parties
will have similar ability to secure competent legal counsel. Without
question, the financial positions of the parties in this proceeding are not the
same. The husband has a superior financial ability to secure and pay counsel.
It is not necessary that one spouse be completely unable to pay attorney’s
fees in order for the trial court to require the other spouse to pay these fees.
Given the complexity of the cause and the time necessary to appropriately
resolve the issues, the award of attorney’s fees in this case was proper to
avoid an inequitable diminution of the fiscal sums granted the wife in these
proceedings.
For the reasons expressed, the decision of the district court is quashed,
and the final judgment of the trial court is reinstated.
It is so ordered.
ENGLAND, C. J., and ADKINS, BOYD and ALDERMAN, JJ., concur.
SUNDBERG, J., concurs in result only.

Price Waterhouse Coopers LLP v. Cedar Resources, Inc.


761 So. 2d 1131 (Fla. 2d DCA 1999)
ALTENBERND, Acting Chief Judge.
The appellants, PricewaterhouseCoopers LLC (“PWC”) and Michael
Willis (“Willis”), appeal the trial court’s denial of their motion to transfer
venue from Pinellas County. We reverse. Although common sense suggests
that a major international accounting firm conducting statewide business
from an office a few miles from the Pinellas County Courthouse should be
subject to that venue, the venue statutes require resolution of this lawsuit
either in Orange County or Hillsborough County.
The appellee, Cedar Resources, Inc. (“Cedar”), filed a three-count
complaint against PWC and Willis in Pinellas County alleging three tort
claims: negligent misrepresentation, fraudulent misrepresentation, and

269
securities fraud under the Florida Securities and Investor Protection Act
(FSIPA). See § 517.301, Fla. Stat. (1993). The crux of this lengthy
complaint is that PWC and its agent, Willis, misrepresented the financial
health of a corporation, Quadrex Corporation, whose shares Cedar agreed to
take in exchange for shares of a subsidiary owned by Cedar. Cedar alleges
that had it known the true status of the corporation’s affairs, it never would
have agreed to the exchange of stock. Cedar claims that PWC and Willis
knew of the corporation’s problems and intentionally or negligently
produced financial statements concealing them. Relying on PWC’s
misrepresentations, Cedar ultimately purchased the shares at a closing held
in Orange County. After Cedar purchased the shares, the true state of the
corporation’s affairs became publicly known and the market value of these
shares dropped dramatically.
Cedar’s complaint admits that PWC’s office is in Hillsborough County,
but claims that it does business throughout the state from this location. Cedar
does not allege that Willis has any ties to Pinellas County. To justify venue
in Pinellas County, Cedar’s complaint alleges that its injuries occurred and
the causes of action accrued in Pinellas County.
PWC and Willis responded to the complaint by filing a motion to transfer
venue from Pinellas County. Thereafter various affidavits were filed on the
venue issue and the parties conducted discovery on this issue.FN1 The
discovery established that PWC and Willis had no contractual relationship
with Cedar. PWC provided independent auditing and accounting services to
Quadrex, which was a publicly traded Delaware Corporation with an office
in Alachua County. Willis is a certified public accountant who works for
PWC. Neither PWC nor Willis performed any services for Quadrex in
Pinellas County. Most, if not all, of the documents that Cedar relies upon as
containing misrepresentations or omissions were public information filings
and SEC filings. Cedar claims, for example, that PWC knew that Quadrex
had several serious financial problems and failed to disclose those problems
in SEC filings.
FN1. A related federal case had previously been filed and dismissed in
the Middle District of Florida, Tampa Division, so the parties had already
conducted some discovery in that case.
The only connections that Pinellas County can claim to this dispute are
that Cedar received copies of Quadrex’s allegedly fraudulent financial
statements at its principal place of business located in Pinellas County, and

270
someone mailed the actual stock certificates to Cedar in Pinellas County
after the closing in Orange County. The trial court determined that these
connections to Pinellas County were a sufficient “nexus” to justify venue
there and denied the motion to transfer. PWC and Willis then filed this
nonfinal appeal.
I. THE STANDARD OF REVIEW
Cedar urges us to review the trial court’s venue decision under an abuse
of discretion standard, citing to Instrumentation Services, Inc. v. Data
Management Associates, 708 So. 2d 1018 (Fla. 4th DCA 1998), and Air
South, Inc. v. Spaziano, 547 So. 2d 314 (Fla. 4th DCA 1989). Cedar’s
reading of Instrumentation Services and Air South overlooks the fact that
there are at least two different types of venue decisions a trial court may be
asked to make, each requiring a different standard of review.
[1] For example, when a party moves to transfer venue for the
convenience of the parties, the trial court is faced with more than one legally
acceptable venue and must chose a good location. The trial court exercises
discretion in making this venue decision. See Hu v. Crockett, 426 So. 2d
1275, 1281 (Fla. 1st DCA 1983). The trial court’s selection will not be
disturbed by an appellate court absent an abuse of discretion. Id; see J.L.S. v.
R.J.L., 708 So. 2d 293 (Fla. 2d DCA 1998). If the trial court transfers venue
to a location where the action could not have been brought, such a transfer is
probably both an error of law and an abuse of discretion. See Tindall v.
Smith, 601 So. 2d 627 (Fla. 2d DCA 1992); Bingham v. Manson, 363 So. 2d
370 (Fla. 1st DCA 1978).
[2] By contrast, when a trial court is presented with a motion to transfer
venue based on the impropriety of the plaintiff’s venue selection, the
defendant is arguing that, as a matter of law, the lawsuit has been filed in the
wrong forum. In order to rule on such a motion, the trial court needs to
resolve any relevant factual disputes and then make a legal decision whether
the plaintiff’s venue selection is legally supportable. A trial court’s factual
decisions in this context are reviewed to determine whether they are
supported by competent, substantial evidence or whether they are clearly
erroneous. See Holland v. Gross, 89 So. 2d 255, 258 (Fla.1956); Bradley v.
Waldrop, 611 So. 2d 31, 32 (Fla. 1st DCA 1992). The trial court’s legal
conclusions are reviewed de novo.
In this case, the trial court made the second type of venue decision. There
is no factual issue on appeal. The trial court was required to decide only

271
whether venue was proper in Pinellas County. It was not presented with
options over which it could exercise discretion.FN2 We review the trial
court’s legal decision de novo and conclude that it was incorrect.
FN2. We note that if the trial court had granted the motion, it appears that
it then may have had some discretion to decide whether to transfer the action
to Hillsborough County or Orange County. See Fla. R. Civ. P. 1.060(b).
II. THE CAUSES OF ACTION FIRST ACCRUED IN ORANGE
COUNTY
Venue is proper in Pinellas County only if at least one of the three causes
of action alleged in the complaint accrued in Pinellas County. See §§ 47.011,
.041, Fla. Stat. (1997). The plaintiff does not argue that PWC has an office
or agents in Pinellas County. See § 47.051, Fla. Stat. (1997). Willis does not
reside in Pinellas County, nor is there property in litigation in Pinellas
County. See § 47.011, Fla. Stat. (1997).
[3] [4] [5] The trial court came to the wrong legal conclusion in
resolving the motion to transfer because it applied the wrong law to this
case. The concept of “nexus” is not used to determine whether a tort action
accrued in Pinellas County for purposes of venue under sections 47.011 or
47.051.FN3 Rather, for venue purposes, a tort claim “is deemed to have
accrued where the last event necessary to make the defendant liable for the
tort took place.” Tucker v. Fianson, 484 So. 2d 1370, 1371 (Fla. 3d DCA
1986). Stated another way, a cause of action of this type accrues at the
moment the wrong and the injury both accrue. See Coggin Pontiac, Inc. v.
Putnam Auto Sales, Inc., 278 So. 2d 647, 648 (Fla. 1st DCA 1973). In this
case, that moment occurred in Orange County. Notwithstanding that the
plaintiff may suffer subsequent damages, even its greatest damage, in
another county, the tort accrues with the first compensable damage. See
Gaboury v. Flagler Hosp., 316 So. 2d 642 (Fla. 4th DCA 1975). Despite
Cedar’s argument to the contrary, this venue principle applies to all three of
its claims, including its securities fraud claim under section 517.301. See
Orlando Auto Auction, Inc. v. Crown Capital Corp., 171 So. 2d 30 (Fla. 1st
DCA 1965).
FN3. The concept of nexus is relevant when the trial court is considering
a motion to transfer venue on forum non conveniens grounds under section
47.122, Florida Statutes (1997), see Sullivan v. Klein, 691 So. 2d 21 (Fla. 3d
DCA 1997), or when the trial court is considering whether a corporation
transacts business in a particular county for venue purposes under section

272
47.051, Florida Statutes (1997), see U-Haul Co. of Northern Fla. v. Fuller,
417 So. 2d 1102 (Fla. 4th DCA 1982).
[6] According to Cedar’s allegations, it was first injured when it
obtained ownership of the stock. That occurred at the closing in Orange
County. The fact that it obtained financial documents prior to the closing in
Pinellas County or that the formal stock certificates were subsequently
mailed to it in Pinellas County does not alter where its causes of action
accrued.
For the first time on appeal, Cedar suggests that it sustained compensable
injury in Pinellas County prior to the closing because of its expenses in
preparing for that closing. This theory is akin to a claim for contractual
reliance damages. See Resorts Int’l v. Charter Air Ctr., 503 So. 2d 1293,
1296 (Fla. 3d DCA 1987) (explaining difference between reliance damages
and expectation damages). The problem with this argument is that Cedar is
not alleging fraud in the inducement of a contract to sell upon which it did
not close. Rather, Cedar is seeking damages similar to contractual
expectation damages caused by the actual sale.FN4
FN4. It does not appear that such reliance damages are a compensable
injury under its section 517.301 FSIPA claim. See § 517.211, Fla. Stat.
(1993) (providing remedies for violations of section 517.301).
Accordingly, venue is not proper in Pinellas County, and we reverse the
order denying the motion to transfer venue. On remand, the plaintiff should
be given the opportunity to choose the county to which the trial court will
transfer venue, either Orange County, where the causes of action accrued, or
Hillsborough County, where the defendants reside.
In closing, we note that we reach this result only because the venue
statutes require it. The venue statutes continue to reflect horse-and-buggy
economic policy in an era of electronic commerce conducted by
international corporations. PWC is a huge international accounting firm that
advertises on the Internet that “[n]o matter where you’re located, chances are
there’s a PWC office near you.” FN5 [ * * * ]It has an office in Hillsborough
County that is as convenient to the Pinellas County Courthouse in
Clearwater as to the Hillsborough County Courthouse in downtown Tampa.
Perhaps it should advertise that “[n]o matter where you're located, we don’t
want to litigate there.”
Reversed and remanded.
BLUE and DAVIS, JJ., Concur.

273
See also Management Computer Controls, Inc. v. Perry Constr., Inc., 743 So. 2d
627 (Fla. 1st DCA 1999).

D. Certiorari

1. Review of Appellate Decisions

Combs v. State
436 So. 2d 93 (Fla. 1983)
BOYD, Justice.
We have for review a decision of a district court of appeal, Combs v.
State, 420 So. 2d 316 (Fla. 5th DCA 1982), which announced as a rule of
law that a district court of appeal can review by certiorari appellate decisions
of circuit courts only if they have the effect of denying appellate review.
Since we find this rule of law to be in direct conflict with the rule announced
in Ellison v. City of Fort Lauderdale, 183 So. 2d 193 (Fla.1966), and
Mathews v. Metropolitan Life Insurance Co., 89 So. 2d 641 (Fla.1956), we
have jurisdiction. Art. V, § 3(b)(3), Fla. Const.; Mancini v. State, 312 So. 2d
732 (Fla.1975); Nielsen v. City of Sarasota, 117 So. 2d 731 (Fla.1960). We
therefore grant the petition for review for the purpose of expunging language
in the district court of appeal opinion that directly conflicts with our earlier
decisions. See Government Employees Insurance Co. v. Grounds, 332 So. 2d
13 (Fla.1976).
In this case Melvin Combs was convicted in county court of driving
while intoxicated. At the trial a police officer testified that while he was
working on an accident report concerning a parked car, Combs approached
him. The officer testified that when he asked Combs to state his business
Combs said he was the driver of the vehicle that ran into the parked car. In
addition, Nathan Hall, the owner of the parked car, testified that Combs
admitted to backing into his truck. Combs objected to the admission of these
statements on the grounds that they were made to fulfill his statutory duty to
make an accident report under sections 316.063 and 316.066, Florida
Statutes (1981), and were therefore privileged under section 316.066(4). The
trial court overruled the objections. On appeal the circuit court affirmed,

274
finding that the privilege conferred by Florida Statute 316.066 prohibits only
the introduction of the written report prepared by the officer investigating an
accident and does not protect statements made by the Appellant to the
officer. Of greater impact, however, is the fact that the statements made by
the Appellant were not in response to any question propounded by the
officer or Mr. Hall as the operator of the unattended, damaged vehicle; the
Appellant’s admissions were not in the form of a report and were
spontaneous and voluntary. The Appellant was not obligated to tell either the
officer or Mr. Hall that he was the driver of the vehicle involved in the
accident. Under these facts the admissions, therefore, are not privileged
under Florida Statute 316.066. Furthermore, it was not the intent of the
Legislature to extend the Accident Report Privilege to Florida Statute
316.063. Therefore, the Appellant’s admission to Nathan Hall was properly
admitted into evidence as was the testimony of Officer Watson.
Combs filed a petition for writ of certiorari with the District Court of
Appeal, Fifth District. In denying the petition, the district court stated:
Whether or not the county court and the circuit court were in error in
their determinations regarding the admissibility of these statements is not for
us to decide. Certiorari is not the vehicle for us to review alleged errors of
law made by a circuit judge sitting in review of county court judgments.
There is no vehicle for that review. The decision of the circuit court is final
and not reviewable....
....
The only thing we can take by certiorari in this type of case is an alleged
“departure from the essential requirements of law” which essentially
amounts to violations of due process rights; that is, violations which
effectively deny appellate review such as the circuit judge rendering a
decision without allowing briefs to be filed and considered, a circuit judge
making a decision without a record to support the decision, Lee v. State, 374
So.2d 1094 (Fla. 4th DCA 1979), or the circuit court dismissing an appeal
improperly. Lynch v. State, 409 So. 2d 133 (Fla. 5th DCA 1982).
420 So. 2d at 317.
We find that the district court has taken too narrow a view of what
constitutes a “departure from the essential requirements of law.” The use of
this phrase has generated much confusion; we recognize that some of the
confusion results from conflicting language in some earlier decisions of this
Court. See City of Winter Park v. Jones, 392 So. 2d 568 (Fla. 5th DCA

275
1980); see generally Haddad, The Common Law Writ of Certiorari in
Florida, 29 U.Fla.L.Rev. 207 (1977); Rogers and Baxter, Certiorari in
Florida, 4 U.Fla.L.Rev. 477 (1951). Some decisions have held that there is a
difference between the standard used in granting common-law certiorari-
departure from the essential requirements of law-and the standards used in
reviewing legal error on appeal. See, e.g., Chicken ‘N’ Things v. Murray,
329 So. 2d 302 (Fla.1976); American National Bank of Jacksonville v.
Marks Lumber & Hardware Co., 45 So. 2d 336 (Fla.1950). Other decisions
have ignored the distinctions between these two standards, using them
interchangeably. See, e.g., In re Camm, 294 So. 2d 318 (Fla.), cert. denied,
419 U.S. 866, 95 S.Ct. 121, 42 L.Ed.2d 103 (1974); Westerman v. Shell’s
City, Inc., 265 So. 2d 43 (Fla.1972); Goodkind v. Wolkowsky, 151 Fla. 62, 9
So.2d 553 (1942); Biscayne Beach Theatre, Inc. v. Hill, 151 Fla. 1, 9 So. 2d
109 (1942).
These conflicting decisions result from this Court’s efforts to clothe the
decisions of all appellate courts with finality while at the same time
providing a means for review in those few extreme cases where the appellate
court’s decision is so erroneous that justice requires that it be corrected. At
first, efforts were made to categorize errors reviewable by common-law
certiorari by their nature. The Court stated that a writ of common-law
certiorari should only be issued where there was an “illegality of procedure,
not an erroneous proceeding.” American National Bank of Jacksonville v.
Marks Lumber & Hardware Co., 45 So. 2d at 337. However, it soon became
apparent that procedural errors were not the only ones so egregious as to
require this Court to review an appellate court’s decision. In Mathews v.
Metropolitan Life Insurance Co., 89 So. 2d 641 (Fla.1956), this Court
espoused the view that the duty of a trial judge to apply to admitted facts a
correct principle of law is such a fundamental and essential element of the
judicial process that a litigant cannot be said to have had the “remedy, by
due course of law,” guaranteed by Section 4 of the Declaration of Rights of
our Constitution, F.S.A., if the trial judge fails or refuses to perform that
duty.... And, indeed, it is unthinkable that this court, in the exercise of its
supervisory jurisdiction over other courts, could not compel adherence to a
principle of law heretofore established by this court in the unlikely event that
a trial judge should deliberately and consciously refuse to follow our
decision, even though the trial judge’s arbitrarily erroneous action in this
respect had been approved and affirmed by a circuit court on appeal.

276
Id. at 642. Similarly, in Ellison v. City of Fort Lauderdale, 183 So. 2d
193 (Fla.1966), this Court held that the district courts of appeal should not
be limited to reviewing procedural errors in granting writs of certiorari to
review appellate decisions of the circuit courts. In that case, as in this one,
the petitioner’s conviction was affirmed by the circuit court and the district
court of appeal denied certiorari on the ground that the alleged error did not
pertain to the trial court’s jurisdiction or the regularity of procedure. The
Court quashed the district court’s decision, holding that common-law
certiorari can be used to review a conviction brought about by an
unconstitutional ordinance.
[1] [2] Hence the phrase “departure from the essential requirements of
law” should not be narrowly construed so as to apply only to violations
which effectively deny appellate review or which pertain to the regularity of
procedure. In granting writs of common-law certiorari, the district courts of
appeal should not be as concerned with the mere existence of legal error as
much as with the seriousness of the error. Since it is impossible to list all
possible legal errors serious enough to constitute a departure from the
essential requirements of law, the district courts must be allowed a large
degree of discretion so that they may judge each case individually. The
district courts should exercise this discretion only when there has been a
violation of a clearly established principle of law resulting in a miscarriage
of justice.
[3] It is this discretion which is the essential distinction between review
by appeal and review by common-law certiorari. See Scholastic Systems,
Inc. v. LeLoup, 307 So. 2d 166 (Fla.1974). A district court may refuse to
grant a petition for common-law certiorari even though there may have been
a departure from the essential requirements of law. See Mystan Marine, Inc.
v. Harrington, 339 So. 2d 200 (Fla.1976). The district courts should use this
discretion cautiously so as to avert the possibility of common-law certiorari
being used as a vehicle to obtain a second appeal.
[4] [5] [6] Although in this case the district court of appeal erred in
holding that it only had jurisdiction to review violations which effectively
deny appellate review, it did not err in denying the petition for common-law
certiorari. The petitioner asserted that the circuit court departed from the
essential requirements of law by affirming a judgment based upon
statements admitted into evidence in violation of section 316.066. Petitioner
correctly points out that the circuit judge was wrong in stating that the

277
privilege conferred by section 316.066 only applies to the introduction of the
written report of the investigating officer. See State v. Coffey, 212 So. 2d 632
(Fla.1968). The trial court also erred by concluding that petitioner was not
obligated to tell either the officer or Mr. Hall that he was the driver of the
vehicle involved in the accident.
However, the trial court did not err by concluding that the statements
were not privileged under section 316.066. That section specifically provides
that the privilege does not extend to statements disclosing “the identity of a
person involved in an accident.” See Wise v. Western Union Telegraph Co.,
177 So. 2d 765 (Fla. 1st DCA 1965). Since the trial court reached the right
result, albeit for the wrong reasons, the affirmance of the judgment by the
circuit court on appeal did not depart from the essential requirements of law.
Health Clubs Inc. v. Englund, 376 So. 2d 453 (Fla. 5th DCA 1979).
We therefore approve the district court’s decision to deny certiorari but
disapprove and expunge those statements in the opinion which conflict with
this opinion.
It is so ordered.
ALDERMAN, C.J., and ADKINS, OVERTON, McDONALD,
EHRLICH and SHAW, JJ., concur.

2. Review of Local Administrative Orders

Ed. Dev. Center, Inc. v. City of West Palm Beach Zoning Bd. of Appeals
541 So. 2d 106 (Fla. 1989)
BARKETT, Justice.
We have for review City of West Palm Beach Zoning Board of Appeals v.
Education Development Center, Inc., 526 So. 2d 775 (Fla. 4th DCA 1988),
in which the district court granted certiorari and quashed an order of the
circuit court overturning a decision of an administrative agency. Because the
district court’s opinion conflicts with City of Deerfield Beach v. Vaillant,
419 So. 2d 624 (Fla.1982), we have jurisdiction.FN*
FN* Art. V, § 3(b)(3), Fla. Const.
The issue here concerns the extent of the district court’s certiorari review.
We find that the district court exceeded the scope of review and quash the
decision below.

278
The petitioner, Education Development Center, Inc. (Center), owns
residential property. The Center appeared at a hearing before the respondent,
City of West Palm Beach Zoning Board of Appeals (Board), seeking to
convert its property to a private preschool and kindergarten.
The Board denied the Center’s application and the Center appealed to the
circuit court. The circuit court reversed the Board, concluding that there was
“substantially competent evidence” to support the Center’s application as
required by the zoning code.
In City of West Palm Beach Zoning Board of Appeals v. Education
Development Center, Inc., 504 So. 2d 1385 (Fla. 4th DCA 1987), the district
court granted the Board’s petition for writ of certiorari, concluding that the
circuit court had applied an incorrect standard of review. The district court
remanded for a redetermination and explained:
[T]he circuit court departed from the essential requirements of law by
applying an incorrect standard of review. The question is not whether, upon
review of the evidence in the record, there exists substantial competent
evidence to support a position contrary to that reached by the agency.
Instead, the circuit court should review the factual determination made by
the agency and determine whether there is substantial competent evidence to
support the agency’s conclusion.
Id. at 1386 (emphasis in original).
On remand, the circuit court again reversed, this time finding that “there
was no substantial competent evidence to support the City’s denial of the
petition.”
The Board returned a second time to the district court, which in the
opinion now before us, Education Development Center, 526 So. 2d at 775,
granted the petition for writ of certiorari and remanded to the circuit court
for further proceedings. The basis for the district court’s reversal was its
disagreement with the trial court’s finding that there was no substantial
competent evidence to support the Board’s decision. In contrast to the circuit
court, the district court found:
There was substantial evidence to support the denial of the application to
permit the operation of a preschool in this residential area. To find to the
contrary, we conclude that the lower tribunal either reinterpreted the
inferences which the evidence supported or reweighed the evidence; in
either event substituting its judgment for that of the zoning board, which it
may not properly do. Id. at 777 (emphasis supplied).

279
[1] In City of Deerfield Beach v. Vaillant, 419 So. 2d 624 (Fla.1982), the
Court clearly set forth the standards governing certiorari review. When the
circuit court reviews the decision of an administrative agency under Florida
Rule of Appellate Procedure 9.030(c)(3), there are three discrete
components of its certiorari review.
Where a party is entitled as a matter of right to seek review in the circuit
court from administrative action, the circuit court must determine whether
procedural due process is accorded, whether the essential requirements of
the law have been observed, and whether the administrative findings and
judgment are supported by competent substantial evidence. Vaillant, 419 So.
2d at 626. In so doing, the circuit court is not permitted to reweigh the
evidence nor to substitute its judgment for that of the agency. Bell v. City of
Sarasota, 371 So. 2d 525 (Fla. 2d DCA 1979).
[2] In turn, the standard of review to guide the district court when it
reviews the circuit court’s order under Florida Rule of Appellate Procedure
9.030(b)(2)(B) is necessarily narrower. The standard for the district court
has only two discrete components.
The district court, upon review of the circuit court’s judgment, then
determines whether the circuit court afforded procedural due process and
applied the correct law.
Vaillant, 419 So. 2d at 626. In Vaillant, the Court adopted the rationale of
the Fourth District Court of Appeal and quoted approvingly from its
decision:
“[C]ommon sense dictates that no one enjoys three full repetitive reviews
to,
1. a civil service board
2. a circuit court
3. a district court of appeal....”
Id. (quoting City of Deerfield Beach v. Vaillant, 399 So. 2d 1045, 1047
(Fla. 4th DCA 1981)).
We find the Board’s reliance on Skaggs-Albertson’s v. ABC Liquors, Inc.,
363 So. 2d 1082 (Fla.1978), to be misplaced. There, the issue concerned the
scope of review of the circuit court which had overturned the agency’s
decision, despite the existence of substantial competent evidence to support
it. Here, we are concerned with the scope of review of the district court and
find the definitive statements in Vaillant to be dispositive.

280
[3] We hold that the principles expressed by the Court in Vaillant clearly
define the standards of review applicable here. There was no contention of a
denial of due process and the district court of appeal did not find that the
trial judge applied an incorrect principle of law. The district court of appeal
simply disagreed with the circuit court’s evaluation of the evidence.
Accordingly, we reaffirm Vaillant and quash the decision of the district
court.
It is so ordered.
EHRLICH, C.J., and OVERTON, SHAW, GRIMES and KOGAN, JJ.,
concur.
McDONALD, J., dissents with an opinion.
McDONALD, Justice, dissenting. [ * * * ]

3. Review of Unappealable Pretrial Orders

Jaye v. Royal Saxon, Inc.


720 So. 2d 214 (Fla. 1998)
WELLS, Justice.
We have for review Jaye v. Royal Saxon, Inc., 698 So. 2d 940 (Fla. 4th
DCA 1997), which certified conflict with Johnson Engineering, Inc. v. Pate,
563 So. 2d 1122 (Fla. 2d DCA 1990); Quality Coffee Service, Inc. v.
Tallahassee Coca-Cola Bottling Co., 474 So. 2d 427 (Fla. 1st DCA 1985);
and Spring v. Ronel Refining, Inc., 421 So. 2d 46 (Fla. 3d DCA 1982). We
have jurisdiction. Art. V, § 3(b)(4), Fla. Const.
[1] The district courts are in conflict as to whether it is appropriate for an
appellate court to grant certiorari to review a nonfinal trial court order
striking a party’s demand for a jury trial. We hold that certiorari review is
inappropriate because a trial court’s order denying or striking a demand for a
jury trial does not cause an irreparable injury that cannot be remedied on
direct appeal.
In this case, Mildred Jaye petitioned the Fourth District Court of Appeal
to issue a writ of certiorari and review a trial court order striking her demand
for a jury trial. The district court, relying on Bared & Co. v. McGuire, 670
So. 2d 153, 156 (Fla. 4th DCA 1996); and Lindsey v. Sherman, 402 So. 2d
1349, 1349 (Fla. 4th DCA 1981), held that it was without subject-matter
jurisdiction to issue the writ and dismissed the petition. Jaye, 698 So. 2d at

281
941. The district court then certified conflict with the aforementioned cases.
Id.
Article V, section 4(b)(4) of the Florida Constitution vests district courts
of appeal with the discretionary jurisdiction to issue, inter alia, writs of
certiorari. Florida Rule of Appellate Procedure 9.030(b)(2)(A) authorizes
district courts to use the writ of certiorari to review nonfinal orders of lower
tribunals that are not directly appealable under Florida Rule of Appellate
Procedure 9.130 (the interlocutory appeal rule). This Court has emphasized,
however, that certiorari review in this instance “is an extraordinary remedy
and should not be used to circumvent the interlocutory appeal rule which
authorizes appeal from only a few types of non-final orders.” Martin-
Johnson, Inc. v. Savage, 509 So. 2d 1097, 1098 (Fla.1987).FN1 The judicial
policy in favor of limited certiorari review is based on the notion that
piecemeal review of nonfinal trial court orders will impede the orderly
administration of justice and serve only to delay and harass. See William A.
Haddad, The Common Law Writ of Certiorari in Florida, 29 U. Fla. L.Rev.
207, 222 (1977).
FN1. The committee notes to the interlocutory appeal rule also
emphasize this point:
The advisory committee was aware that the common law writ of
certiorari is available at any time and did not intend to abolish that writ.
However, because that writ provides a remedy only if the petitioner meets
the heavy burden of showing that a clear departure from the essential
requirements of law has resulted in otherwise irreparable harm, it is
extremely rare that erroneous interlocutory rulings can be corrected by resort
to common law certiorari. It is anticipated that because the most urgent
interlocutory orders are appealable under this rule, there will be very few
cases in which common law certiorari will provide relief. Fla. R.App. P.
9.130 (committee note, 1977 amend.).
[2] Turning to the issue in conflict, it is settled law that, as a condition
precedent to invoking a district court’s certiorari jurisdiction, the petitioning
party must establish that it has suffered an irreparable harm that cannot be
remedied on direct appeal. See Globe Newspaper Co. v. King, 658 So. 2d
518, 520 (Fla.1995); Martin-Johnson, Inc. v. Savage, 509 So. 2d 1097, 1098
(Fla.1987); Kilgore v. Bird, 149 Fla. 570, 6 So. 2d 541, 544 (1942); Bared &
Co., Inc. v. McGuire, 670 So. 2d 153 (Fla. 4th DCA 1996); Parkway Bank v.
Fort Myers Armature Works, Inc., 658 So. 2d 646 (Fla. 2d DCA 1995). The

282
district court in the instant case dismissed Jaye’s petition for certiorari
review based on its own precedent that an order striking a demand for a jury
trial does not cause the petitioning party an irreparable harm that cannot be
cured on direct appeal. Jaye, 698 So. 2d at 941.
In this review, Jaye advances several arguments as to why the direct
appeal remedy is inadequate. However, we agree with the district court
below and find that Jaye’s allegations of harm flowing from the trial court’s
order do not rise to the level of irreparable harm that permits certiorari
review.
Jaye initially argues that the denial of a constitutional right, i.e., the right
to a jury trial, should be sufficient to invoke a district court’s certiorari
jurisdiction. Jaye does not state how the alleged denial of this constitutional
right causes an irreparable injury. Jaye cites only to Spring, in which the
Third District concluded that it had certiorari jurisdiction to review an order
striking a demand for jury trial because “the denial of the right to jury trial is
more than the denial of a constitutional right; it is the denial of a
fundamental right recognized prior to the adoption of a written constitution.”
Spring, 421 So. 2d at 47. We disagree. Our disagreement is not because we
fail to respect the fundamental right to a jury trial. Our disagreement is
because we believe that an error in failing to provide a jury trial is an error
which can be corrected on direct appeal without the type of irreparable harm
necessary to support certiorari review.
Jaye argues that, as a result of the alleged error, she will suffer the
following irreparable harm that a direct appeal cannot remedy: (1) the
substantial injury caused when aggrieved parties must “show their hand” in a
preliminary nonjury trial; (2) a situation in which the age of the parties and
witnesses affects whether a second trial is feasible; and (3) the time, effort,
and expense of trying a case twice. We reject these arguments because we
do not find these contentions to be of the nature which demonstrates
irreparable harm. See Globe Newspaper Co., 658 So. 2d at 520; Martin-
Johnson v. Savage, 509 So. 2d at 1100; Parkway, 658 So. 2d at 650;
Whiteside v. Johnson, 351 So. 2d 759, 760 (Fla. 2d DCA 1977); Haddad,
supra, at 223. Moreover, the nonjury trial may result in a decision by the
trial judge that will cause the petitioner to conclude that there is no reason to
seek appellate review. We distinguish our decision in Wincast Associates,
Inc. v. Hickey, 342 So. 2d 77 (Fla.1977), because in that case we did not deal
with the issue of whether certiorari review was appropriate. We now make

283
clear that Hickey should not be read as approving certiorari review of the
denial of a demand for a jury trial.
Accordingly, we approve the dismissal of the petition by the district court
in this case. We disapprove Johnson Engineering, Quality Coffee Service,
and Spring to the extent that they are inconsistent with this opinion.
It is so ordered.
HARDING, C.J., and OVERTON, SHAW and KOGAN, JJ., concur.
ANSTEAD and PARIENTE, JJ., recused.

E. Harmless Error

1. Civil Cases

Stecher v. Pomeroy
253 So. 2d 421 (Fla. 1971)
DEKLE, Justice.
Petitioners claim conflict with the innovations in the law which were set
forth in this Court’s opinions in Beta Eta House Corp., Inc., of Tallahassee
v. Gregory, 237 So.2d 163 (Fla.1970), and Shingleton v. Bussey, 223 So.2d
713 (Fla.1969). After argument and further consideration, the alleged
conflict necessary for certiorari jurisdiction here[FN1] fails to appear and
accordingly the writ of certiorari which was earlier issued herein to the
Fourth District Court of Appeal to review its opinion in this cause at 244
So.2d 488 (Fla.App.1971), being improvidently entered, is discharged.
FN1. Fla.Const. art. V, s 4(2), F.S.A.
[1] The conflict asserted is not primarily founded upon the basic holding
in Beta Eta and in Bussey regarding the granting or not of severance to a
named insurance company defendant. This apparently continues to be a
troublesome question before the trial courts, which we hope we can clarify
later in this opinion. The principal conflict urged here is in the satellite areas
regarding the mention of insurance limits to the jury, refusal of a requested
instruction to disregard and whether in these circumstances such error, if
any, was harmless. It appears to be harmless here, in light of the fact that the
verdict was $19,000 despite policy limits of $100,000/$300,000; where there

284
was a disc involvement with serious and prolonged disability, traction and
hospitalization; and where the injuries were permanent.
[2] This recognition of harmless error in these particular circumstances is
not to be regarded as approval by this Court of the mention of policy limits
to a jury. This should not be done. Nor is it approval of the trial court’s
refusal to grant the requested instruction to disregard, which should have
been given. It is simply held to be harmless error here where an examination
of the entire record reflects a tone which indicates in no wise any adverse
effect upon the jury’s verdict.[FN2]
FN2. Southern Liquor Distributors v. Kaiser, 150 Fla. 52, 7 So.2d 600
(1942); Luster v. Moore, 78 So.2d 87 (Fla.1955). See also Compania
Dominicana de Aviacion and Underwriters at Lloyds, London v. Knapp et
al., 251 So.2d 18 (3d DCA Fla.1971) (Opinion filed July 20, 1971; rehearing
denied, August 20, 1971).
The record in this cause encompassed 416 pages without any other
mention whatever of insurance over a period of two days’ trial, the only
reference having been that which was set forth in the defendants’ answer to
interrogatories. Moreover, the Court here instructed the jury generally that
its verdict must be based solely on the evidence and the charges of the Court
in accordance with standard jury instructions. Liability was admitted.
One of the objectives of Beta and Bussey was to provide a disclosure of
policy limits between the parties which had not previously been allowed.
The reasons were for purposes of negotiation and to encourage settlement
between the parties and thus shorten litigation and speed up the courts’
heavy trial dockets. It was never intended that policy limits should go to the
jury and Beta Eta expressly said so. It is immaterial for the jury’s
consideration, because the principles still stand that its decision must rest
solely upon the evidence and the law as charged. Moreover, to reveal
defendants’ amount of insurance before the jury would equally entitle a
defendant to bring out his coverage when the limits are minimal and
advantageous to him. Neither one has relevancy and has no place before the
jury.
It was felt in reaching our decisions in Beta and Bussey that revealing the
existence of an insuror as a real party in interest justifiably reflects the true
fact that there is financial responsibility. This offsets any indulgence by
counsel or the jury with unfounded arguments like, ‘This poor, hard working
truck driver and his family’ approach, when in fact there is an ability to

285
respond. It is probably not a factor in other instances where there is an
obviously responsible principal defendant as in Compania Dominicana de
Aviacion.[FN3]
FN3. See footnote 2, supra.
It is fair to note also in this respect the holding of In re Rules Governing
Conduct of Attorneys in Fla., 220 So.2d 6 (Fla.1969), actively argued before
this Court shortly prior to Bussey. There it was asserted by the insurance
companies of this state then appearing (including three associations which
‘represent 659 insurance companies who write the bulk of the fire, casualty
and liability policies in Florida’), that they are the real parties in interest in
these negligence cases; that the lawyers they employ and provide under the
insurance policies to defend the cases, are really representing the Insurance
carriers in such litigation and ‘when an insurance company ‘has both the
financial stake in the outcome of the litigation and the burden of carrying the
costs of the defense, it is defending its own interests and is entitled to defend
by its own employee, so long as he is a member of the Bar and an officer of
the Court.‘‘ (p. 8)
One of the insurance companies there asserted to this Court that:
‘(T)he legal responsibility placed on the insurance company give(s)
pointed verification to the fact that the interest involved in defense of
liability suits Is primarily and ultimately The interest of the insurance
company.
‘It is the insurance company’s interest, As an entity, that the lawyer
represents, whether house counsel or fee counsel.’ (emphasis ours)
If this position of the carriers is to be recognized, as it was at their urging
by concurring with them in the position they asserted then, it surely follows
that such Real party in interest should be present and revealed when the
cases are tried. Consistency in the law, and certainly consistency of one’s
position, is essential to equal justice.
[3] This brings us to the principal point in Beta and Bussey as to granting
a severance. To be sure our opinion in Beta Eta employs the permissive
‘may’ in holding that the trial judge may grant a severance of a named
insurance defendant. This is lifted from the rule regarding severances, is
consistent with it and was used expressly in the context of the rule (1.270(b),
30 F.S.A.). The exact reference (p. 165) was ‘pursuant to’ the rule.

286
There is good reason for this adaptation of Civil Procedure Rule 1.270(b)
in Beta Eta. There are some instances where there is a question of coverage
when a severance would be quite proper to try those issues separately, and
the severance would be under this rule, as there pointed out. However, the
remainder of that opinion holds, and we hereby reaffirm, that absent a
justiciable issue relating to Insurance, such as a question of coverage or of
the applicability or interpretation of the insurance policy or other such valid
dispute on the matter of insurance coverage, there is no valid reason for a
severance and it should NOT be granted.
Bussey (223 So.2d p. 720) spoke-not of severing the trial on Negligence
liability-but of severing ‘issues between insurer and insured for separate trial
or adjudication.’
The insurance carrier’s position as a real party in interest is a position of
continuing interest which includes the trial of the cause which a third party
has asserted against its insured. To rule otherwise on a motion for severance
would be to defeat the purposes of the rule enunciated in these cases with
regard to the real party in interest so as to reflect the presence of financial
responsibility which should be left apparent before the jury (without other
express mention, of course) and the other bases set forth in those holdings.
The routine granting of such motions for severance except for such good
cause related to insurance coverage would be a misapprehension of this
Court’s holding.
The writ is discharged.
ROBERTS, C. J., and CARLTON and ADKINS, JJ., concur.
ERVIN and BOYD, JJ., concur in judgment.

2. Criminal Cases

State v. DeGuilio
491 So. 2d 1129 (Fla. 1986)
ON REHEARING GRANTED
SHAW, Justice.
[1] Respondent petitions for rehearing of our decision of August 29,
1985, wherein we held that comments on a defendant’s silence were subject
to harmless error analysis and remanded the case to the district court for
application of the harmless error analysis. We reaffirm our holding but grant

287
rehearing in order to apply harmless error analysis and to more fully
explicate the application of harmless error. We substitute this opinion for our
earlier opinion.
The following question has been certified as being of great public
importance:
Has the Florida Supreme Court, by its agreement in State v. Murray, 443
So. 2d 955 (Fla.1984), with the analysis of the supervisory powers of
appellate courts as related to the harmless error rule as set forth in United
States v. Hasting, 461 U.S. 499, 103 S.Ct. 1974, 76 L.Ed.2d 96 (1983),
receded by implication from the per se rule of reversal explicated in
Donovan v. State, 417 So. 2d 674 (Fla.1982); Shannon v. State, 335 So.2d 5
(Fla.1976); and Bennett v. State, 316 So.2d 41 (Fla.1975)?
DiGuilio v. State, 451 So. 2d 487, 491 (Fla. 5th DCA 1984). We have
jurisdiction. Art. V, § 3(b)(4), Fla. Const. We answer the certified question
in the affirmative and apply the harmless error doctrine to a comment on a
defendant’s remaining silent.
A jury convicted Angelo John DiGuilio of conspiracy to traffic in
cocaine. The district court reversed, finding that the prosecutor elicited
testimony from a witness which could be interpreted by the jury as a
comment on DiGuilio’s right to remain silent. Applying Donovan, Shannon,
and Bennett, the district court found the comment to be per se grounds for
reversal.
The comment in question arose during the prosecution’s examination of a
police officer to determine whether DiGuilio had been read his Miranda FN1
warnings. The following exchange then took place:
FN1. Miranda v. Arizona, 384 U.S. 436, 86 S.Ct. 1602, 16 L.Ed.2d 694
(1966).
Q. [Prosecutor] Did he indicate whether or not he would be willing to
answer any questions?
A. At that point, he didn’t say.
Q. Did Mr. DiGuilio make any statements to you at that time?
A. Only to the effect that the driver of the car picked him up at his home
and he had come directly to the Howard Johnson’s. That he lived in South
Daytona. He refused to give me an address. He refused to identify the name
of the driver. He also indicated to me that the driver had parked the car and
walked north to the southeast doors to the motel and had entered. After that,

288
he advised me he felt like he should speak to his attorney. And there was no
further questioning.
Q. No further questioning?
A. No.
[2] [3] [4] The district court found the statement, “After that, he advised
me he felt like he should speak to his attorney,” susceptible to the conclusion
that it was a comment on the right to remain silent. The fact that DiGuilio
answered a few questions first does not constitute a waiver of his fifth
amendment privilege. Miranda states that an individual can invoke his right
to remain silent “at any time prior to or during questioning.” Miranda v.
Arizona, 384 U.S. 436, 473-74, 86 S.Ct. 1602, 1627-28, 16 L.Ed.2d 694
(1966); Michigan v. Mosley, 423 U.S. 96, 96 S.Ct. 321, 46 L.Ed.2d 313
(1975). Thus, comment on a defendant’s invocation of his right to remain
silent after he has answered some questions is constitutional error. See
Peterson v. State, 405 So.2d 997 (Fla. 3d DCA 1981); Thompson v. State,
386 So. 2d 264 (Fla. 3d DCA 1980), review denied, 401 So.2d 1340
(Fla.1981). We agree that the comment here is fairly susceptible of being
interpreted by the jury as a comment on silence. State v. Kinchen, 490 So. 2d
21 (Fla.1985).
Florida has long followed a per se reversal rule when a prosecutor
comments on a defendant’s failure to testify. Gordon v. State, 104 So. 2d
524 (Fla.1958); Trafficante v. State, 92 So. 2d 811 (Fla.1957); Way v. State,
67 So. 2d 321 (Fla.1953); Rowe v. State, 87 Fla. 17, 98 So. 613 (1924). Prior
to Miranda, however, Florida followed the rule that a defendant’s silence,
when faced with accusatory statements while in custody, was admissible as
evidence tending to show guilt. Albano v. State, 89 So. 2d 342 (Fla.1956).
The per se reversal rule for comments on the right to remain silent was first
adopted in Jones v. State, 200 So. 2d 574 (Fla. 3d DCA 1967). This Court
adopted Jones and the per se rule in Bennett v. State, 316 So. 2d 41
(Fla.1975), and has approved the rule in other cases. E.g., Donovan v. State,
417 So. 2d 674 (Fla.1982); Shannon v. State, 335 So. 2d 5 (Fla.1976).
Because comment on a defendant’s failure to testify and comment on a
defendant’s silence violate the same constitutional provision and are
grounded on the same rationale, we reexamine both the Rowe and Jones line
of cases to determine if a rule of per se reversal should be followed.
The problem of prosecutorial comment on a defendant’s failure to testify
is of fairly recent vintage. Under the common law at the time the United

289
States and Florida Constitutions were adopted, an accused not only could not
be compelled to testify, but was considered to be incompetent to testify even
if he wished to do so. Because of this legal disability, no inference could be
drawn from a failure to testify and there could be no occasion for a
prosecutor to comment on the failure to testify. Obviously the framers of the
constitutions did not contemplate such prosecutorial comments when they
authored the constitutional right not to be compelled to testify against
oneself. In the late nineteenth century, a move developed to remove the
common law disability which prevented an accused from testifying. In 1878,
Congress passed an act FN2 granting the accused a statutory right, upon
request, to testify in federal courts. The act also provided that failure to make
such request would create no presumptions against the accused. The
meaning of the “no presumptions” language was tested in Wilson v. United
States, 149 U.S. 60, 13 S.Ct. 765, 37 L.Ed. 650 (1893), where the Court held
that a prosecutor’s comment on an accused’s failure to testify violated the
statutory provision and reversed the conviction.FN3 In 1895, the Florida
Legislature enacted chapter 4400, Laws of Florida FN4 which for the first
time not only granted an accused in Florida the right to testify FN5 but,
presumably in light of the Wilson decision, specifically provided that no
prosecutor would be permitted to comment before the court or jury on the
failure of the accused to testify.
FN2. Act of Mar. 16, 1878; currently codified as 18 U.S.C. § 3481
(1986).
FN3. Griffin v. California, 380 U.S. 609, 85 S.Ct. 1229, 14 L.Ed.2d 106
(1965), acknowledged that Wilson was grounded on statute but held that
prosecutorial comment on a defendant’s failure to testify also violated the
fifth amendment.
FN4. The act was later codified as section 3979, General Statutes (1906);
section 6080, Revised General Statutes (1920); section 8383, compiled
General Laws (1927); and section 918.09, Florida Statutes (1941). Section
918.09 was repealed by section 180, chapter 70-339, Laws of Florida,
following its incorporation in 1967 as Rule of Criminal Procedure 1.250. It
is currently contained in Florida Rule of Criminal Procedure 3.250.
FN5. From 1865 to 1895 an accused had the statutory right to make a
statement under oath to the jury. The accused was not a witness and could
not be examined on the sworn statement. Ch. 1472, No. 9, § 4, Laws of
Florida (1866); ch. 1816, No. 1, Laws of Florida (1870); Hart v. State, 38

290
Fla. 39, 20 So. 805 (1896); Hawkins v. State, 29 Fla. 554, 10 So. 822 (1892);
Miller v. State, 15 Fla. 577 (1876); Barber v. State, 13 Fla. 675 (1871).
It is from the 1895 legislative act that the Rowe line of cases sprung. In
Jackson v. State, 45 Fla. 38, 34 So. 243 (1903), this Court reversed a
conviction because of a prosecutor’s comment on an accused’s failure to
testify. In so doing, we grounded the reversal on violation of statute, not the
Florida Constitution, and noted that no curative instructions had been given
to the jury. Further, and even more significantly, although we held that the
particular comment in the case at hand was reversible error, we specifically
noted that comments on an accused’s failure to testify were not per se
reversible error:
There may be some circumstances where reference to the fact may be
made in such form as not to constitute reversible error, as in the case of State
v. Mosley, 31 Kan. 355, 2 Pac. 782, but the remarks made in this case are not
of that character.
Jackson, 45 Fla. at 39, 34 So. at 243 (citations omitted). The holding that
such comments were not per se reversible was made more explicit in
Steffanos v. State, 80 Fla. 309, 86 So. 204 (1920), where we held:
During the argument of counsel the prosecuting attorney commented
upon the failure of the accused to testify in his behalf. Exception was taken
to the remarks of counsel by the defendant, and the court corrected the
prosecuting attorney, and instructed the jury to disregard the statement; but
he did so in such words as to render the correction of little value to the
defendant. While we do not hold the transaction, as it appears to have
occurred, reversible error, we think that, when prosecuting attorneys do
violate the plain language of the statute, their remarks should be expunged
so far as possible, and removed from consideration by the jury.
80 Fla. at 315, 86 So. at 206. It is thus clear that a prosecutor’s comments
on an accused’s failure to testify was not per se reversible error as of 1920
when Steffanos was decided. This changed with the Rowe case.
In Rowe, the prosecutor made repeated references to an accused’s failure
to testify including one where the trial court failed to rebuke the prosecutors
and which we characterized as “an adroit and insinuating attempt, indirectly
to accomplish what could not have been accomplished by a direct statement.
The statute does not permit such evasions of its manifest purpose.” Rowe,
87 Fla. at 30, 98 So. at 618, quoting from State v. Moxley, 102 Mo. 374, 14
S.W. 969 (1890). We rejected the state’s argument that comments on failure

291
to testify could be cured by an instruction to the jury because “violation by
the prosecuting officer of a statute such as ours cannot be cured by the court
instructing the jury to disregard his comment.” Rowe, 87 Fla. at 29, 98 So. at
617. Accordingly,
For the violations of the statute by the prosecuting officers of the state, as
pointed out herein, and for that only, the judgment is reversed, and a new
trial granted.
87 Fla. at 32, 98 So. at 618 (emphasis supplied).
In a series of cases in the 1950’s, this Court again addressed the question
of whether a harmless error statute, section 54.23, Florida Statutes (1951),
FN6
could be applied to a comment on an accused’s failure to testify in
violation of section 918.09. In Way v. State, 67 So. 2d 321 (Fla.1953), we
concluded that section 54.23 was not applicable to a violation of section
918.09 and reversed the conviction. In Trafficante v. State, 92 So.2d 811
(Fla.1957), we relied on Way and Rowe and held that a prosecutor’s
comment violated section 918.09. We again addressed the issue of
prosecutorial comment on an accused’s failure to testify in Gordon v. State,
104 So. 2d 524 (Fla.1958). Obviously, however, we were feeling
considerable discomfort at our rule of per se reversal and commented at
length that we were only following such a rule because section 918.09
required that we do so:
FN6. Originally enacted by chapter 6223, § 1, Laws of Florida (1911),
currently codified as section 59.041, Fla.Stat. (1985).
Here again we have a specific legislative prescription of a right to be
accorded to those under prosecution for crime. Whether we as judges deem
the rule to be wise and salutary is of no consequence at all and we assume no
responsibility for it. The Legislature made the rule and we must follow it, at
least until the Legislature changes it.
....
Our responsibility as an appellate court is to apply the law as the
Legislature has so clearly announced it. We are not endowed with the
privilege of doing otherwise regardless of the view which we might have as
individuals. Way v. State, Fla.1953, 67 So. 2d 321. Also see Trafficante v.
State, Fla.1957, 92 So. 2d 811. The harmless error statute, Section 54.23,
Florida Statutes, F.S.A., does not apply to this type of error. Id. at 540-41.

292
It is clear that Rowe, Way, Trafficante, and Gordon rest on statutory
construction, i.e., did the legislature intend that the harmless error statute,
section 54.23, be applicable to the statutory prohibition against comment on
failure to testify, section 918.09. We concluded that the harmless error
statute did not apply for two reasons. First, the language in section 918.09
was mandatory-“nor shall any prosecuting attorney be permitted before the
court or jury to comment on the failure of the accused to testify in his own
behalf.” Second, section 54.23 was limited to errors relating to “misdirection
of the jury or the improper admission or rejection of evidence or for error as
to any matter of pleading or procedure.” These were the only two statutes at
issue when Rowe was decided and, arguendo, Rowe was correct in holding
that the legislature did not intend that harmless error analysis be applied to
prosecutorial comments on failure to testify. Way, Trafficante, and Gordon,
however, are another matter because, after Rowe issued, the legislature
enacted chapter 19554, section 309, Laws of Florida (1939), codified as
section 924.33 (1941 and thereafter).FN7 Section 924.33 differs from section
54.23 in two significant respects. First, it provides that harmless error
analysis is applicable to all judgments regardless of the type of error
involved. Second, it explicitly provides that there shall be no presumption
that errors are reversible unless it can be shown that they are harmful. Thus,
Way, Trafficante, and Gordon, which purport to rely on legislative intent, are
directly contrary to legislative intent as expressed in the plain words of
section 924.33.FN8
FN7. The statute reads:
924.33 When judgment not to be reversed or modified.-No judgment shall
be reversed unless the appellate court after an examination of all the appeal
papers is of the opinion that error was committed which injuriously affected
the substantial rights of the appellant. It shall not be presumed that error
injuriously affected the substantial rights of the appellant.
FN8. There is no reference in Way, Trafficante, or Gordon to section
924.33. A review of the briefs filed in these cases shows that the state did
not rely on, or even recognize the existence of, section 924.33. In Way, the
state relied on the general proposition that improper comments by the
prosecutor are not per se reversible; in Trafficante, that there was no
comment on failure to testify; and, in Gordon, that the issue had not been
preserved and there was no comment on failure to testify. In a petition for

293
rehearing on Way, which we denied, the state untimely sought rehearing and
reargument on the applicability of section 924.33.
[5] [6] Section 924.33 respects the constitutional right to a fair trial free
of harmful error but directs appellate courts not to apply a standard of review
which requires that trials be free of harmless errors. The authority of the
legislature to enact harmless error statutes is unquestioned.FN9 Contraposed
to this legislative authority, the courts may establish the rule that certain
errors always violate the right to a fair trial and are, thus, per se reversible.
To do so, however, we are obligated to perform a reasoned analysis which
shows that this is true, and that, for constitutional reasons, we must override
the legislative decision. It is clear that the rule of Way, Trafficante and
Gordon is not grounded on the constitution.FN10 Although we did not
explicitly say so, it is also clear that Rowe, Way, Trafficante and Gordon
were implicitly overruled by State v. Marshall, 476 So. 2d 150 (Fla.1985),
wherein we adopted the harmless error rule for comments on a defendant’s
failure to testify.
FN9. In this connection, see Chapman v. California, 386 U.S. 18, 22, 87
S.Ct. 824, 827, 17 L.Ed.2d 705 (1967), where the Court recognized that
Congress and the fifty states had the authority to enact harmless error
statutes or rules, and had done so. Note, also, that, although section 924.33
was enacted prior to Chapman, it is consistent with Chapman.
FN10. The prohibition of prosecutorial comment on failure to testify is
constitutional, Griffin v. California, 380 U.S. 609, 85 S.Ct. 1229, 14 L.Ed.2d
106 (1965). However, there is no constitutional right to per se reversal.
Chapman and progeny.
Florida’s per se reversal rule on comments on a defendant’s silence arose
from a separate line of cases. In Bennett v. State, 316 So. 2d 41 (Fla.1975),
relying on Jones v. State, 200 So. 2d 574 (Fla. 3d DCA 1967), we held that
comments on an accused’s post-arrest silence are per se reversible. Accord
Donovan v. State, 417 So. 2d 674 (Fla.1982); Shannon v. State, 335 So. 2d 5
(1976). The holding in these cases was grounded on the fifth amendment to
the United States Constitution and our interpretation of Miranda v. Arizona,
384 U.S. 436, 86 S.Ct. 1602, 16 L.Ed.2d 694 (1966). As it applies here,
Miranda stands for the proposition that comment on an accused’s post-arrest
silence is constitutional error; it does not stand for the proposition that such
error is per se reversible. This was made clear in Chapman v. California,
386 U.S. 18, 87 S.Ct. 824, 17 L.Ed.2d 705 (1967), and its progeny but,

294
although we cited Chapman in Bennett, we overlooked its holding that
automatic reversal of a conviction is only appropriate when the
constitutional right which is violated vitiates the right to a fair trial.
Chapman holds that comment on failure to testify is not constitutionally
subject to automatic reversal because it does not always vitiate the right to a
fair trial and the harmless error analysis should be applied. We followed our
interpretation of Miranda in Donovan and Shannon. It was not until we
issued State v. Marshall, 476 So. 2d 150 (Fla.1985), and State v. Murray,
443 So. 2d 955 (Fla.1984), that we adopted the correct rule from Chapman
and Hasting that constitutional errors, with rare exceptions, are subject to
harmless error analysis.
[7] [8] The harmless error test, as set forth in Chapman and progeny,
places the burden on the state, as the beneficiary of the error, to prove
beyond a reasonable doubt that the error complained of did not contribute to
the verdict or, alternatively stated, that there is no reasonable possibility that
the error contributed to the conviction. See Chapman, 386 U.S. at 24, 87
S.Ct. at 828. Application of the test requires an examination of the entire
record by the appellate court including a close examination of the
permissible evidence on which the jury could have legitimately relied, and in
addition an even closer examination of the impermissible evidence which
might have possibly influenced the jury verdict.
In comparing the per se reversible rule and the harmless error rule, and
determining their applicability, it is useful first to recognize that both rules
are concerned with the due process right to a fair trial. The problem which
we face in applying either rule is to develop a principled analysis which will
afford the accused a fair trial while at the same time not make a mockery of
criminal prosecutions by elevating form over substance.
The dissenters apparently believe that the rule of harmless error cannot
cope with comments on post-arrest silence or failure to testify and that only
a per se rule will suffice. This view ignores the far-ranging application of the
harmless error rule and does not recognize that a per se rule is nothing more
than a determination that certain types of errors are always harmful, i.e.,
prejudicial. Per se reversible errors are limited to those errors which are “so
basic to a fair trial that their infraction can never be treated as harmless
error.” Chapman, 386 U.S. at 23, 87 S.Ct. at 827-28. In other words, those
errors which are always harmful. The test of whether a given type of error
can be properly categorized as per se reversible is the harmless error test

295
itself. If application of the test to the type of error involved will always result
in a finding that the error is harmful, then it is proper to categorize the error
as per se reversible. If application of the test results in a finding that the type
of error involved is not always harmful, then it is improper to categorize the
error as per se reversible. If an error which is always harmful is improperly
categorized as subject to harmless error analysis, the court will nevertheless
reach the correct result: reversal of conviction because of harmful error. By
contrast, if an error which is not always harmful is improperly categorized as
per se reversible, the court will erroneously reverse an indeterminate number
of convictions where the error was harmless. See for example, Delaware v.
Van Arsdall, 475 U.S. 673, 106 S.Ct. 1431, 89 L.Ed.2d 674 (1986); United
States v. Mechanik, 475 U.S. 66, 106 S.Ct. 938, 89 L.Ed.2d 50 (1986);
United States v. Lane, 474 U.S. 438, 106 S.Ct. 725, 88 L.Ed.2d 814 (1986).
The unique and only function of the rule of per se reversal is to conserve
judicial labor by obviating the need to apply harmless error analysis to errors
which are always harmful. It is, in short, a rule of judicial convenience. The
unique function of the harmless error rule is to conserve judicial labor by
holding harmless those errors which, in the context of the case, do not vitiate
the right to a fair trial and, thus, do not require a new trial. Correctly applied
in their proper spheres, the two rules work hand in glove. Both provide an
equal degree of protection for the constitutional right to a fair trial, free of
harmful error.
[9] In Florida, we have adopted a very liberal rule for determining
whether a comment constitutes a comment on silence: any comment which
is “fairly susceptible” of being interpreted as a comment on silence will be
treated as such. Kinchen; David v. State, 369 So.2d 943 (Fla.1979). One
authority has said that “[c]omments or arguments which can be construed as
relating to the defendant’s failure to testify are, obviously, of almost
unlimited variety.” FN11 * * * The “fairly susceptible” test treats this variety
of arguable comments as comments on silence. We are no longer only
dealing with clear-cut violations where the prosecutor directly comments on
the accused’s silence and hammers the point home as in Rowe v. State, 87
Fla. 17, 98 So. 613 (1924). Comments on silence are lumped together in an
amorphous mass where no distinction is drawn between the direct or
indirect, the advertent from the inadvertent, the emphasized from the casual,
the clear from the ambiguous, and, most importantly, the harmful from the
harmless. In short, no bright line can be drawn around or within the almost

296
unlimited variety of comments that will place all of the harmful errors on
one side and the harmless errors on the other, unless the circumstances of the
trial are considered. We must apply harmless error analysis to the “fairly
susceptible” comment in order to obtain the requisite discriminatory
capacity.
The combination of the fairly susceptible test and the harmless error rule
is a happy union. It preserves the accused’s constitutional right to a fair trial
by requiring the state to show beyond a reasonable doubt that the specific
comment(s) did not contribute to the verdict. At the same time, it preserves
the public and state interest in finality of verdicts which are free of any
harmful error. In view of the heavy burden the harmless error rule places on
the state, it further serves as a strong deterrent against prosecutors
advertently or inadvertently commenting on an accused’s silence. It cannot
be rationally argued that commenting on an accused’s silence is a viable
strategy for obtaining convictions. By contrast, a union of the fairly
susceptible test and the rule of per se reversal is pernicious in that the former
has little, if any, discriminatory capacity and the latter has none. The union
which the dissenters urge substitutes mechanics for judgment in the style of
nineteenth century English and American appellate courts where error, no
matter how harmless, equaled reversal.FN12 * * *
The most perceptive analysis of harmless error principles of which we
are aware is that of former Chief Justice Traynor of the California Supreme
Court. See Roger J. Traynor, The Riddle of Harmless Error (1970), and the
dissent to People v. Ross, 67 Cal.2d 64, 429 P.2d 606, 60 Cal.Rptr. 254
(1967) (Traynor, C.J. dissenting), rev’d sub nom, Ross v. California, 391
U.S. 470, 88 S.Ct. 1850, 20 L.Ed.2d 750 (1968). In his dissent, Chief Justice
Traynor maintained that comments on Ross’s failure to testify were harmful
and that the majority misunderstood and misapplied the Chapman harmless
error test. Chief Justice Traynor argues, and we agree, that harmless error
analysis must not become a device whereby the appellate court substitutes
itself for the jury, examines the permissible evidence, excludes the
impermissible evidence, and determines that the evidence of guilt is
sufficient or even overwhelming based on the permissible evidence. In a
pertinent passage, Chief Justice Traynor points out:
Overwhelming evidence of guilt does not negate the fact that an error that
constituted a substantial part of the prosecution’s case may have played a
substantial part in the jury’s deliberation and thus contributed to the actual

297
verdict reached, for the jury may have reached its verdict because of the
error without considering other reasons untainted by error that would have
supported the same result. Ross, 60 Cal.Rptr. at 269, 429 P.2d at 621.
[10] It is clear that comments on silence are high risk errors because there
is a substantial likelihood that meaningful comments will vitiate the right to
a fair trial by influencing the jury verdict and that an appellate court, or even
the trial court, is likely to find that the comment is harmful under Chapman.
High risk that an error will be harmful is not enough, however, to justify
categorizing the error as always harmful (per se). In the case at hand, if the
accused had taken the stand and confessed guilt during cross examination,
we could say beyond a reasonable doubt that the officer’s comment on post-
arrest silence did not affect the jury’s verdict. Yet the dissenters would have
us declare in that instance that the comment FN13 is per se reversible error and
requires a retrial. It would be possible to set forth an infinite number of
realistic hypothetical cases where an analysis of the strength and nature of
the permissible evidence of guilt and of the strength and nature of the
impermissible comment on silence would show beyond any reasonable
doubt that the jury verdict was not affected by the comment on silence.
Accordingly, it cannot be said that comment on silence always denies the
accused a fair trial and is thus subject to per se reversal. By contrast, if a
defendant is denied counsel and takes the stand and confesses, we cannot say
beyond a reasonable doubt that the error, denial of counsel, was harmless.
Denial of counsel is always harmful, regardless of the strength of the
admissible evidence, and can be properly categorized as per se reversible.
FN13. The comment was “[a]fter that, he advised me he felt like he
should speak to his attorney.”
The suggestion is made that it is wise public policy to hold that
comments on failure to testify and post-arrest silence are per se reversible
error. This Court is not the forum for a debate on wise public policy. The
responsible branch of government has already established the public policy
through section 924.33 that appellate courts will not reverse trial court
judgments unless it is determined on the record that harmful error has
occurred. This legislative determination of public policy is not
constitutionally infirm. Accordingly, [o]ur responsibility as an appellate
court is to apply the law as the Legislature has so clearly announced it. We
are not endowed with the privilege of doing otherwise regardless of the view
which we might have an individuals. Way v. State, Fla.1953, 67 So. 2d 321.

298
Also see Trafficante v. State, Fla.1957, 92 So.2d 811. Gordon, 104 So. 2d at
541.
For the reasons set forth above, we answer the certified question in the
affirmative and hold that comments on a defendant’s silence are subject to
harmless error analysis as set forth herein.FN14
FN14. Our decision that comment on post-arrest silence is not per se
reversible error overturns the portion of Clark v. State, 363 So. 2d 331
(Fla.1978), to the contrary.
The district court below found that there was sufficient evidence to
support the conviction, absent the impermissible comment on post-arrest
silence, and concluded that, if the harmless error rule could be applied to the
facts of the case, the conviction would be affirmed because the error was
harmless beyond any reasonable doubt. The district court’s reference to a
sufficiency-of-the-evidence test suggests a misunderstanding of the harmless
error test. Because we wish to make it clear that the harmless error test is to
be rigorously applied, we examine the record ourselves rather than
remanding. We conclude that the error was harmful and the conviction
should be quashed.
[***]
The decision of the district court is approved for the reasons set forth
herein and this cause is remanded for further proceedings in light of this
opinion.
It is so ordered.
McDONALD, C.J., and BOYD and OVERTON, JJ., concur.
ADKINS, J., concurs in part and dissents in part with an opinion, in
which EHRLICH and BARKETT, JJ., concur.
ADKINS, Justice, concurring in part and dissenting in part.
I concur in the decision to reverse the conviction, but strongly dissent to
the ill-conceived reasoning which places an inordinate burden on the
appellate court and deprives defendants of a constitutional right.
EHRLICH and BARKETT, JJ., concur.

299
Ventura v. State
29 So. 3d 1086 ( 2010)
PER CURIAM.
We have for review Ventura v. State, 973 So. 2d 634 (Fla. 3d DCA
2008), in which the Third District Court of Appeal affirmed the admissibility
of a Department of Corrections release-date letter as a permissible means of
establishing the defendant's status as a prison-releasee reoffender. See id. at
638. In the process, the Third District relied upon the reasoning and rule of
law articulated in Yisrael v. State, 938 So. 2d 546 (Fla. 4th DCA 2006) (en
banc) (Yisrael I ), disapproved in part, 993 So. 2d 952 (Fla.2008). See
Ventura, 973 So. 2d at 638. We have jurisdiction. See art. V, § 3(b)(3), Fla.
Const.
***
[3][4] Next, the district court improperly utilized an “overwhelming
evidence” test when considering whether the impermissible testimony of a
witness which clearly undermined and violated the Fifth Amendment right
to remain silent constituted harmless error. Unlike some evidentiary errors,
such as the admission of a cumulative or irrelevant photograph, a comment
on the right to remain silent strikes at the heart of our criminal justice
system. This Court has clearly stated that it is constitutional error to penalize
an individual for exercising the Fifth Amendment privilege; therefore, the
prosecution may not introduce during trial the fact that an individual has
relied upon this protection in the face of accusation. See Simpson v. State,
418 So.2d 984, 984-85 (Fla.1982) (quoting Jones v. State, 200 So. 2d 574,
576 (Fla. 3d DCA 1967)); see also DiGuilio, 491 So. 2d at 1131. As we
stated in DiGuilio:
It is clear that comments on silence are high risk errors because there is a
substantial likelihood that meaningful comments will vitiate the right to a
fair trial by influencing the jury verdict and that an appellate court, or even
the trial court, is likely to find that the comment is harmful under Chapman
[v. California, 386 U.S. 18, 87 S.Ct. 824, 17 L.Ed.2d 705 (1967) ].
491 So. 2d at 1136-37. Accordingly, commenting on the silence of an
accused is not a viable strategy for obtaining convictions, *1089 and any
comment-direct or indirect-by anyone at trial on this right is constitutional
error that should be avoided. See DiGuilio, 491 So. 2d at 1136, 1139.

300
[5][6] During the trial proceedings below, a detective made two
comments with regard to Ventura's silence, as follows:
A. Along with the victims, the defendant's [sic] wouldn't give any
statements.
....
A. Yes. The suspects were in custody and the defendant then declined to
make statements.
(Emphasis supplied.) The Third District determined that the comments
were improper and stated: “We fail to see how the detective's comment,
twice repeated, could have been anything other than an intentional cheap
shot at Ventura's constitutional rights.” Ventura, 973 So. 2d at 637. The
court clearly and correctly recognized the egregious conduct. However, the
district court then held that the error was “harmless beyond a reasonable
doubt given the overwhelming evidence of guilt.” Id.
Unfortunately, the Third District expressed an incorrect harmless error
analysis. In DiGuilio, we fully explicated the application of the harmless
error doctrine to a comment on a defendant's right to remain silent. In doing
so, we explicitly expressed that the harmless error analysis is not an
“overwhelming-evidence test.” DiGuilio, 491 So. 2d at 1139.
[H]armless error analysis must not become a device whereby the
appellate court substitutes itself for the jury, examines the permissible
evidence, excludes the impermissible evidence, and determines that the
evidence of guilt is sufficient or even overwhelming based on the
permissible evidence. ...
Overwhelming evidence of guilt does not negate the fact that an error
that constituted a substantial part of the prosecution's case may have played
a substantial part in the jury's deliberation and thus contributed to the
actual verdict reached, for the jury may have reached its verdict because of
the error without considering other reasons untainted by error that would
have supported the same result.
....
The harmless error test ... places the burden on the state, as the
beneficiary of the error, to prove beyond a reasonable doubt that the error
complained of did not contribute to the verdict or, alternatively stated, that
there is no reasonable possibility that the error contributed to the
conviction. Application of the test requires not only a close examination of

301
the permissible evidence on which the jury could have legitimately relied,
but an even closer examination of the impermissible evidence which might
have possibly influenced the jury verdict ....
....
... The test must be conscientiously applied and the reasoning of the court
set forth for the guidance of all concerned and for the benefit of further
appellate review. The test is not a sufficiency-of-the-evidence, a correct
result, a not clearly wrong, a substantial evidence, a more probable than not,
a clear and convincing, or even an overwhelming evidence test. Harmless
error is not a device for the appellate court to substitute itself for the trier-of-
fact by simply weighing the evidence. The focus is on the effect of the error
on the trier-of-fact. The question is whether there is a reasonable possibility
that the error affected the verdict. The burden to show the error was
harmless must remain on *1090 the state. If the appellate court cannot say
beyond a reasonable doubt that the error did not affect the verdict, then the
error is by definition harmful.
DiGuilio, 491 So. 2d at 1136, 1138-39 (citations omitted) (emphasis
supplied) (quoting People v. Ross, 67 Cal.2d 64, 60 Cal.Rptr. 254, 429 P.2d
606, 621 (1967) (Traynor, C.J., dissenting), rev'd, 391 U.S. 470, 88 S.Ct.
1850, 20 L.Ed.2d 750 (1968)).
***
[7] The district court noted only one factor in the harmless error analysis
(i.e., permissible evidence of guilt), but that one factor is not the
determinative test. We have explicitly rejected the overwhelming evidence
test as a proper analysis of harmless error. Specifically, the decision of the
Third District does not address a proper analysis and does not discuss
whether there is a reasonable possibility that the constitutional error affected
the verdict. Our colleague in dissent suggests that our decision is based on
an erroneous assumption that the district court failed to give consideration to
the correct harmless error analysis. We cannot assume that an analysis was
conducted or review that which remains hidden behind the written opinion.
In other words, the decision does not reflect any consideration by the
appellate court of whether the impermissible comments contributed to the
conviction, as required in an analysis of harmless error. Instead, as written,
the appellate court appears to have “substitute[d] itself for the trier-of-fact by
simply weighing the evidence” instead of focusing on the “effect of the error
on the trier-of-fact.” DiGuilio, 491 So. 2d at 1139 (emphasis supplied). It is

302
important for the test to be “conscientiously applied and the reasoning of the
court set forth for the guidance of all concerned and for the benefit of further
appellate review.” Id.
[8] The harmless error rule functions to conserve judicial labor while
providing an “equal degree of protection for the constitutional right to a fair
trial, free of harmful error.” Id. at 1135. The analysis of the impermissible
comments in the decision below and the announced principle of law does not
demonstrate this balance.
Accordingly, we quash and remand to the Third District Court of Appeal
for reconsideration upon application of our decision in DiGuilio. In addition,
we approve the ultimate result reached by the Third District on the Yisrael
issue, but disapprove its analysis and reasoning based upon the rule
expressed in Yisrael I.
It is so ordered.
QUINCE, C.J., and PARIENTE, LEWIS, LABARGA, and PERRY, JJ.,
concur.
CANADY, J., dissents with an opinion, in which POLSTON, J., concurs.
***

F. The Two-Issue Rule

First Interstate Development Corp. v. Ablanedo


511 So. 2d 536 (Fla. 1987)
PER CURIAM.
This is a petition to review First Interstate Development Corp. v.
Ablanedo, 476 So. 2d 692 (Fla. 5th DCA 1985), concerning asserted fraud
claims in a land development. The relevant district court ruling (1) upheld a
compensatory damage judgment even though one of the fraud claims was
improperly submitted to the jury, and (2) held punitive damages claims must
be submitted to a jury where the underlying fraud claim is sufficient to
warrant jury consideration. We accepted jurisdiction under article V, section
3(b)(3), Florida Constitution, based upon apparent conflict with Winn &
Lovett Grocery Co. v. Archer, 126 Fla. 308, 171 So. 214 (1936), and Como
Oil Co. v. O’Loughlin, 466 So. 2d 1061 (Fla.1985). We quash that portion of
the district court’s opinion that denies petitioners a new trial on
compensatory damages and approve the district court in its determination

303
that punitive damages should have been submitted to the jury under the
circumstances of this case.
The petitioner First Interstate Development Corporation developed
Ocean Woods as a planned unit development consisting of single-family
homes, cluster homes, and villas, totaling 300 units. Substantial areas were
intentionally left undeveloped to create a natural environment. The
respondents, as property owners in the development, initially instituted this
action in 1980, primarily to obtain control over the homeowners association.
In 1983, respondents amended their complaint to include allegations that the
developers fraudulently misrepresented the entire project as an oceanfront
development and that the developers fraudulently misrepresented that they
would build a nature trail.
The record reflects that all of the individual respondents’ properties were
non-oceanfront. The oceanfront parcels were undeveloped at the time
respondents purchased their subject properties. Testimony was not uniform
regarding what the respondents were told or understood concerning the
beachfront property. Some were told the entire project was a beachfront
project; others stated that they relied exclusively on an advertising brochure,
which they believed represented the planned unit development, including
600 feet of ocean frontage; others stated they were told that high rise
condominiums were planned for that property; another group testified they
were told there were no plans for the oceanfront property at that time, but
that whatever was built would be different from the rest of Ocean Woods.
The petitioners’ advertising brochure is not clear whether the boundaries of
the planned unit development include the oceanfront parcel in question.
With regard to the nature trail, the record establishes that petitioners had
constructed part of the nature trail and were constructing the balance when
they were stopped by the city.
The trial court, at the end of all the evidence, found an insufficient basis
for punitive damages and directed a verdict on that issue for the petitioners.
The trial court submitted the alleged fraudulent claims on the oceanfront
misrepresentation and the failure to complete construction of the nature trail
to the jury for compensatory damages; the jury returned a compensatory
damage verdict for fifty respondents in the amount of $304,600.25.
The district court affirmed the compensatory damage award, but reversed
and remanded for a new trial on punitive damages. In so holding, the district
court found no evidence that First Interstate had fraudulently promised to

304
construct the nature trail and noted that petitioners had commenced
construction “until they were stopped by the City.” 476 So. 2d at 694. The
district court further stated, “[w]hile evidence on this failure to furnish the
trail as promised may have been the basis for a breach of contract case, it
does not in itself establish an intent to defraud.” Id. After so holding, the
district court rejected petitioners’ contention that the jury’s damage
determination must be reversed because the amount attributable to the nature
trail could not be separated from the alleged oceanfront fraud claim. The
district court held, “where a general verdict form is submitted to the jury
without objection, reversal is improper where no error is found as to one of
several issues submitted to the jury on which the verdict may be properly
based.” 476 So. 2d at 695 (citations omitted). The district court concluded,
“we have no way of knowing what, if any, weight the jury placed on the
testimony regarding the nature trail.” Id. With regard to the development’s
oceanfront status, the district court found sufficient evidence to submit that
issue to the jury. The district court then determined that, since the oceanfront
misrepresentation was a fraudulent claim, the question of whether punitive
damages were proper was for the jury, not the trial judge.
Two-Issue Rule
[1] [2] [3] We first address the district court’s holding that the
compensatory damage verdict should be affirmed even though the evidence
pertaining to the nature trail fraud and misrepresentation should not have
been submitted to the jury. We disagree and find that the two-issue rule does
not apply when two distinct claims for liability result in separate claims for
damages in the same action. We previously summarized the two-issue rule
as follows:
[W]here there is no proper objection to the use of a general verdict,
reversal is improper where no error is found as to one of two issues
submitted to the jury on the basis that the appellant is unable to establish that
he has been prejudiced.
Whitman v. Castlewood International Corp.,383 So. 2d 618, 619
(Fla.1980) (citing Colonial Stores, Inc. v. Scarbrough, 355 So. 2d 1181
(Fla.1977). This rule applies to those actions that can be brought on two
theories of liability, but where a single basis for damages applies. For
instance, in products liability, the claim can be brought on both negligence
and breach of implied warranty, but the measure of damages for the resulting
personal injury is the same. That is not the circumstance in the instant case.

305
Here, the fraud claim for failure to construct a nature trail and the claim for
damages because of reduction in value of respondents’ properties for the
failure to complete the trail is distinct from the claim of diminished property
values for the misrepresentation of the project as oceanfront. Each claim is
distinct and has a separate measure of damages. Finding liability on one
claim does not entitle the respondents to receive the total amount of damages
attributable to both theories of liability. We will not presume that petitioners
were not prejudiced by the improper submission of the nature trail issue to
the jury. Consequently, the jury’s compensatory damage award must be
reversed.
Punitive Damages Claim
[4] The district court correctly determined that the issue of punitive
damages in this case should have been submitted to the jury. In Winn &
Lovett we explicitly stated that punitive damages are appropriate for any
tortious conduct accomplished through fraud:
Exemplary damages are given solely as a punishment where torts are
committed with fraud, actual malice or deliberate violence or oppression, or
when the defendant acts willfully, or with such gross negligence as to
indicate a wanton disregard of the rights of others.
....
In order to recover exemplary or punitive damages the declaration must
[sic] allege some general facts and circumstances of fraud, malice, gross
negligence or oppression tending to show plaintiff’s right to recover such
damages in addition to damages by way of compensation.
126 Fla. at 327-28, 171 So. at 221-22 (emphasis added). The
overwhelming weight of authority in this state makes it clear that proof of
fraud sufficient to support compensatory damages necessarily is sufficient to
create a jury question regarding punitive damages.FN*
[5] This is so because intentional misconduct is a necessary element of
fraud. Indeed, to prove fraud, a plaintiff must establish that the defendant
made a deliberate and knowing misrepresentation designed to cause, and
actually causing detrimental reliance by the plaintiff. See Lance v. Wade,
457 So. 2d 1008, 1011 (Fla.1984). We conclude that the district court
correctly ruled that the punitive damage issue was for the jury, not the judge,
to decide once a case for fraud had been made.

306
For the reasons expressed, we quash that part of the district court
decision regarding the two-issue rule and remand for new trial. We approve
that part regarding punitive damages.
It is so ordered.
McDONALD, C.J., and EHRLICH, SHAW and BARKETT, JJ., and
ADKINS, J. (Ret.), concur.
OVERTON, J., concurs in part and dissents in part with an opinion.
OVERTON, Justice, concurring in part, dissenting in part.
I fully concur with the majority holding on the two-issue rule.
I dissent from the holding that every claim based on fraud which is
sufficiently established to go to the jury is automatically eligible for punitive
damages. [ * * * ]

Barth v. Khubani
748 So. 2d 260 (Fla. 1999)
SHAW, J.
We have for review Barth v. Khubani, 705 So. 2d 72 (Fla. 3d DCA
1997), which expressly and directly conflicts with Charlemagne v. Francis,
700 So. 2d 157 (Fla. 4th DCA), review dismissed, 703 So. 2d 476
(Fla.1997). FN1 We have jurisdiction. Art. V, § 3(b)(3), Fla. Const.
FN1. Additional conflict cases include Davidson v. Gaillard, 584 So. 2d
71 (Fla. 1st DCA 1991), A.G. Edwards & Sons, Inc. v. Weinreich, 572 So.
2d 993 (Fla. 2d DCA 1990), and LoBue v. Travelers Ins. Co., 388 So. 2d
1349 (Fla. 4th DCA 1980).
The issue presented by the conflict is whether an appellate court may
apply the “two issue rule” to bar review of a general defense verdict where
the appellee pled two or more defenses at trial and the appellant alleges error
only as to one such defense. Petitioner filed a complaint against respondents
alleging breach of contract and fraud.FN2 The trial court granted a directed
verdict for petitioner on the fraud count FN3 and submitted only the breach of
contract action to the jury. To the remaining count, respondents asserted
three defense theories: failure to prove that the alleged contract existed,
failure of a condition precedent, and the statute of frauds.FN4 The parties
agreed to a general verdict form and the jury found for respondents. On
appeal, the Third District held that by agreeing to the submission of a

307
general verdict petitioner was precluded from raising an alleged statute of
frauds instructional error. The district court explained as follows:
FN2. The claims arose out of a business transaction where respondent
Khubani was to acquire a property called the “Castle” by satisfying state tax
certificates and a federal tax lien, and by purchasing a judgment lien on the
property. Petitioner allegedly agreed to waive any homestead rights held in
the property as part of the deal. By acquiring the property, respondents
allegedly stood to make $2,000,000 out of which they allegedly agreed to
pay petitioner $300,000. Petitioner filed a complaint alleging breach of
contract and fraud based on the existence and nature of this alleged
agreement and respondents’ alleged failure to pay petitioner the $300,000.
FN3. The Third District reversed the trial court’s judgment on the fraud
count holding that the court erred in directing a verdict thereon since
petitioner established a prima facie case of fraud.
FN4. The record shows that the trial judge concluded the jury instruction
as to petitioner’s breach of contract action with the following instruction: f
the greater weight of the evidence does not support the claim of the plaintiff,
your verdict should be for the defendants. However, if the greater weight of
the evidence does support the claim for the plaintiff, then you shall consider
the defenses raised by the defendants.
[W]e find that the plaintiff did not properly preserve the statute of frauds
issue for review on appeal. Because a general verdict form was submitted to
the jury, it is unclear whether the jury found the underlying contract to be
unenforceable because it was barred by the statute of frauds, because the
plaintiff had failed to perform the conditions precedent, or because no valid
contract existed. Therefore, in the absence of an objection to the use of the
general verdict form, reversal is improper where no error is found as to one
of two issues submitted to the jury on the basis that the appellant is unable to
demonstrate prejudice. Barth, 705 So. 2d at 73. We agree.
[1] The district court correctly applied the “two issue rule” barring
appellate review of petitioner’s claim of error in the jury instruction as to
one defense where, as here, respondents relied on three separate defense
theories at trial. The “two issue rule” provides:
[W]here there is no proper objection to the use of a general verdict,
reversal is improper where no error is found as to one of two issues
submitted to the jury on the basis that the appellant is unable to establish that
he has been prejudiced.

308
Whitman v. Castlewood Int'l Corp., 383 So. 2d 618, 619 (Fla.1980). The
rule is based on the principle that reversal is improper where no error is
found as to one of the issues that can independently support the jury’s
verdict. See Colonial Stores, Inc. v. Scarbrough, 355 So. 2d 1181, 1186
(Fla.1977). In Colonial Stores, this Court cautioned that although it may
seem that injustice might result from application of the “two issue rule,” the
rule is an economical tool that limits appellate review to issues that actually
affect the case and that litigants may avoid application of the rule by simply
requesting a special verdict that would illuminate the jury’s decision making
process and the affect of any alleged error: “It should be remembered ... that
the remedy is always in the hands of counsel.” Id.
[2] [3] When a general verdict for the plaintiff is on review, the rule is
applied by focusing on the causes of action, such that an appellate claim of
error raised by the defendant as to one cause of action cannot be the basis for
reversal where two or more theories of liability (or causes of action) were
presented to the jury.FN5 On the other hand, when the jury returns a general
verdict for the defendant, the “two issue rule” is applied by focusing on the
defenses; thus, where two or more defense theories are presented to the jury
and it returns a verdict for the defense, an appellate claim of error as to one
defense theory will not result in reversal since the verdict may stand based
on another theory.FN6 The focus on the winning party’s actions or defenses,
as the case may be, is logical given that the opposing party has the burden of
establishing prejudice on appeal. See Whitman, 383 So. 2d at 619; Colonial
Stores, Inc., 355 So.2d at 1186; Rosenfelt v. Hall, 387 So. 2d 544, 546 (Fla.
5th DCA 1980).
FN5. See Whitman, 383 So. 2d at 619 (defendant’s appeal claiming error
as to one theory of liability cannot result in reversal of verdict since plaintiff
submitted two theories of liability to jury).
FN6. See, e.g., Treal Group, Inc. v. Custom Video Servs., Inc., 682 So. 2d
1230, 1231 (Fla. 4th DCA 1996) (“Because both [defense theories] were
presented to the jury by [defendant] Custom, and the jury could have found
for Custom on either ground, but entered a general verdict, the two-issue
rule ... applies here.”).
This Court adopted the rule from the case law of California, Connecticut,
Ohio and South Dakota, all of which focus on the number of defense
theories when applying the rule to general verdicts in favor of the defense.
See Colonial Stores, Inc., 355 So. 2d at 1186. See Philpott v. Mitchell, 219

309
Cal.App.2d 244, 32 Cal.Rptr. 911, 918-19 (1963) (“Every intendment is in
favor of the judgment of the lower court, and this means, among other
things, that this court, on appeal, may assume that the jury by its general
[defense] verdict found for [the defendant] on every issue submitted.”);
Johnson v. Pagano, 184 Conn. 594, 440 A.2d 244, 245-46 (1981) (“Thus, if
any of the court’s instructions are shown to be proper and adequate as to any
one of the defenses raised, the general verdict [for the defense] will stand
irrespective of any error in the charge as to the others.”); Bush v. Harvey
Transfer Co., 146 Ohio St. 657, 67 N.E.2d 851, 856 (1946) (“The two-issue
rule ... [has been] applied to sustain a verdict for ... defendants [where] there
were two separate and distinct defenses made, either one of which in itself, if
established, was sufficient to defeat the cause of action of the plaintiff.”);
McCrystal v. Trumbull Mem. Hosp., 115 Ohio App.3d 73, 684 N.E.2d 721,
728 (1996); Bankwest, Inc. v. Valentine, 451 N.W.2d 732, 736 (S.D.1990)
(“As [defendant] raised several affirmative defenses to the Bank’s claim, it
is difficult to determine how the jury reached its verdict.... [Thus] the
construction which sustains the [defense] verdict must be applied.”).
[4] In the present case, the Third District correctly applied the rule by
properly focusing on the three theories of defense pled at trial. On appeal,
petitioner attacked the verdict based solely on an alleged error as to the
statute of frauds instruction. Since respondents argued three theories of
nonliability-failure of proof, failure of a condition precedent, and the statute
of frauds, the appellate court did not violate the maxim that the “two issue
rule” only applies when the prevailing plaintiff submits more than one cause
of action or the prevailing defendant submits more than one separate and
distinct defense theory and the parties agree to a general verdict form.
Accordingly, we approve Barth and disapprove the conflict cases to the
extent they are inconsistent with our decision.FN7 [ * * * ]
It is so ordered.
HARDING, C.J., and WELLS, ANSTEAD, PARIENTE, LEWIS and
QUINCE, JJ., concur.

310
XV. Disposition

A. Applicable Law

Hendeles v. Sanford Auto Auction, Inc.


364 So. 2d 467 (Fla. 1978)
PER CURIAM.
We granted certiorari in this case for conflict with Vining v. Avis Rent-
A-Car Systems, Inc., 354 So. 2d 54 (Fla.1977). The estate of Carol Hendeles
Brady brought an action against respondents for wrongful death caused by
an automobile accident. The complaint sought damages from parties alleged
to have had ownership or possession of the offending automobile, who were
also alleged to have entrusted it to the driver who caused the fatal accident.
Each of the respondents as parties defendant denied ownership, possession,
and entrustment as to itself. Thus the question of the unauthorized use of the
car, along with the circumstances surrounding it the key was left in the
ignition was placed in issue. It appears from the record that the trial judge’s
order of summary judgment for the respondents was based on his belief that
the evidence showed the car to have been stolen. The district court of appeal
affirmed, per curiam, without opinion.
In Vining, we held that the chain of causation between the statutory
violation, under Section 316.097, Florida Statutes of one in charge of an
automobile who leaves it standing unattended, unlocked, with the key in the
ignition, and the injury caused by the operation of the automobile by a thief,
is not necessarily broken by the intervening criminal act of the driver in
stealing the car. The key to proximate cause, we said, is foreseeability. The
question of the foreseeability of the intervening theft under the
circumstances, and of the likelihood of subsequent danger to others, when
reasonable persons might differ, is for the jury.
[1] Vining had not been decided when this case was before the trial
court. But it controls now since disposition of a case on appeal should be
made in accord with the law in effect at the time of the appellate court’s
decision rather than the law in effect at the time the judgment appealed was
rendered. Florida East Coast Railway Co. v. Rouse, 194 So. 2d 260
(Fla.1967).
[2] There were disputed issues of material fact before the trial court. The
factual questions either placed in issue by the pleadings or upon which

311
evidence was adduced in the form of documents, affidavits, and depositions
included, among possible others: which of the respondents had ownership,
possession or control of the vehicle, or whether they had same jointly;
whether the party or parties in control of the vehicle entrusted it, negligently
or otherwise, to the driver; and whether it was foreseeable under the
circumstances that the car might be stolen with resultant danger to others.
[3] Therefore, the trial court erred in holding that there were no disputed
issues of material fact. And our Vining decision, filed since the time of the
trial court’s action, compels reversal of its implicit holding that, since it
appeared the car was driven without permission, there could be no liability
on the part of the person or persons having ownership or control.
The decision of the district court is quashed and it is directed that the
cause be remanded to the trial court for proceedings consistent with this
opinion.
It is so ordered.
ENGLAND, C. J., and ADKINS, BOYD, HATCHETT and
ALDERMAN, JJ., concur.

B. Affirmance

Florida Rules of Appellate Procedure

Rule 9.315. Summary Disposition


(a) Summary Affirmance.After service of the initial brief in appeals under
rule 9.110, 9.130, or 9.140, or after service of the answer brief if a cross-
appeal has been filed, the court may summarily affirm the order to be
reviewed if the court finds that no preliminary basis for reversal has been
demonstrated.

Florida Detroit Diesel v. Nathai


28 So. 3d 182 (Fla. 1st DCA 2010)
PADOVANO, J.
This is an appeal by the employer and its servicing agent from an order
awarding workers' compensation benefits to an injured worker. We conclude
that the appellants' initial brief fails to demonstrate a preliminary basis for

312
reversal and we therefore summarily affirm the order without the need for an
answer brief.
Yaadram Nathai tripped on a mat while working as a mechanic for his
employer Florida Detroit Diesel and he injured his knee and foot. The
employer and its servicing agent, Gallagher Bassett Services, Inc., accepted
the accident as compensable and provided Mr. Nathai medical treatment,
including arthroscopic knee surgery for a torn meniscus. Following the
surgery, the employer and servicing agent authorized treatment by Dr. Paul
Shirley, an orthopedist.
Dr. Shirley characterized the injury as “devastating” and “life altering”
and concluded that it would prevent Mr. Nathai from returning to his normal
functioning at work. Mr. Nathai wanted to obtain a second opinion to
determine whether there was any other course of treatment that might enable
him to regain the full use of his foot and return to work. The employer and
servicing agent denied this request, and then Mr. Nathai filed a petition for
benefits to litigate his entitlement to a second opinion.
The evidence presented at the hearing on the petition included Dr.
Shirley's deposition. Dr. Shirley's testimony on direct examination as it
pertained to the need for a second opinion was as follows:
Q. Have your recommendations for epidural blocks and a second opinion
with Dr. Murphy changed?
A. No.
Q. Do you still believe they are medically necessary?
A. Yes.
Q. And the basis for the medical necessity is the original workplace
injury?
A. Yes.
Q. All right. Doctor, are those recommendations within a reasonable
degree of medical certainty?
A. Yes.
On cross-examination, Dr. Shirley conceded that Mr. Nathai would not
benefit from additional surgery and, for that reason, he agreed that a second
opinion was not necessary from an orthopedic standpoint. However, he
stood by his conclusion that a second opinion was medically necessary to
determine whether there was anything that could be done that would enable
Mr. Nathai to return to work. He explained that a second opinion might

313
reveal a beneficial course of treatment other than additional surgery and that
it was important to assist Mr. Nathai in obtaining closure on the possibility
of significant improvement of his condition.
Based on Dr. Shirley's testimony, the judge of compensation claims
concluded that the second opinion was medically necessary and directed that
it be provided at the expense of the employer and servicing agent. This order
is the subject of the present appeal by the employer and servicing agent.
We have elected to review the order under the summary disposition
procedure in rule 9.315(a) of the Florida Rules of Appellate Procedure. This
rule authorizes an appellate court to summarily affirm an order of the lower
tribunal if “no preliminary basis for reversal has been demonstrated.” As
explained in the court commentary to rule 9.315, the purpose of the
summary affirmance procedure is to provide an expeditious method of
deciding an appeal that is unmeritorious.
[1] When used properly, the summary affirmance procedure is not only
faster than the ordinary practice of allowing full briefing by all parties, it is
also more economical. A decision to summarily affirm an order can be made
at any time after the filing of the appellant's initial brief, which means that
the appellate court may dispense with the answer brief. The expenses
incurred by the parties are not as great as they would be if the court required
an answer brief and then a reply brief.
[2][3][4] The summary affirmance procedure is appropriate in this case,
because it is clear from the record and the initial brief that the argument for
reversal is without merit. Whether a second opinion is medically necessary
is a question of fact. See Lombardi v. Southern Wine & Spirits, 890 So. 2d
1128 (Fla. 1st DCA 2004). In workers' compensation cases, as in other fields
of the law, a decision that turns on a finding of fact must be affirmed on
appeal if the finding is supported by competent substantial evidence. See De
Groot v. Sheffield, 95 So. 2d 912 (Fla.1957); Mylock v. Champion Int'l, 906
So. 2d 363 (Fla. 1st DCA 2005). It follows from these principles that the
order at issue here must be affirmed. The decision by the judge of
compensation claims that Mr. Nathai is entitled to a second opinion is fully
supported by the expert testimony given by Dr. Shirley.
[5][6] It makes no difference that there may have been some evidence in
the record to support the appellants' position. The pertinent question is
whether the evidence supports the order under review. It is an elementary
principle, but one apparently worth restating, that the facts of a case are not

314
reweighed in the appellate court. As the supreme court explained in Shaw v.
Shaw, 334 So. 2d 13, 16 (Fla.1976), “[i]t is not the function of the appellate
court to substitute its judgment for that of the trial court through re-
evaluation of the testimony and evidence from the record on appeal before
it.” Rather, the test is “whether the judgment of the trial court is supported
by competent evidence.” Shaw, 334 So. 2d at 16.
The employer and servicing agent have attempted to recast the issue in a
more favorable light by pointing out that there is no evidence that a second
opinion is “orthopedically” necessary. The problem with this argument is
that it would add a requirement that is not present in the Workers'
Compensation Law. The phrase “medically necessary” is defined in the
applicable statute as any medical service that is used to identify or treat an
illness or injury, and is appropriate to the patient's diagnosis and status of
recovery. See § 440.13(1)(m), Fla. Stat. (2003). This broad definition of the
phrase “medically necessary” could potentially include “any” medical
service that is shown to be necessary. The statute does not restrict the
concept of medical necessity to a particular field of medicine. Hence, we do
not accept the argument that Mr. Nathai was required to show that the
second opinion was “orthopedically” necessary. That is an artificial
construct not found in the text of the statute.
For these reasons, we conclude that the employer and servicing agent
have failed to present a preliminary basis for reversal. We have no need for
an argument in defense of the order by the judge of compensation claims, as
it clear to us from our reading of the record and the initial brief that we must
affirm the order.
Affirmed.
KAHN and WETHERELL, JJ., concur.

C. Modification

Belly Acres, Inc. v. Frankel


412 So. 2d 48 (Fla. 5th DCA 1982)
COWART, Judge.
Appellant, seller of a trailer park, appeals a final judgment in favor of the
buyer awarding various items of damages arising from the breach of express
contractual warranties relating to the condition of improvements on the real

315
property sold. The buyer cross-appeals the trial court’s denial of the buyer’s
claims for reformation, for punitive damages and for damages relating to
seller’s alleged misrepresentation that the premises were high and dry. We
have reviewed the record and the trial court’s findings as to all matters urged
as error on appeal and find no reversible error, with two exceptions.
The evidence is sufficient only to support a finding that there were two
broken urinals on October 22, 1976, rather than the three mentioned on the
plumber’s estimate establishing the cost of replacement at $200 each
(exhibit 25).
We note the discrepancies between the itemized damages allowed and the
total stated in the judgment. On page 2 of the final judgment damages for
exhibit 35 were itemized as follows: fan repairs, $226.20; pool motor,
$197.60; license fee, $185.00; and tile, $169.60, which items totaled
$778.40. However, on the summary on pages 3 and 4 of the final judgment,
the total for exhibit 35 is given as $818.40. Also, the itemized summary as
shown correctly totals $41,019.08, rather than $47,792.64 as shown in the
final judgment. We hereby reduce the $41,019.08 figure by $200 to
eliminate the cost of replacing one of the three urinals (exhibit 25) and by
the $40.00 necessary to correct the discrepancy relating to exhibit 35,
thereby modifying the final judgment to total $40,779.08. It is, hereby,
AFFIRMED as modified.
COBB and SHARP, JJ., concur.

D. Reversal

Florida Rules of Appellate Procedure

Rule 9.315. Summary Disposition


***
(b) Summary Reversal.After service of the answer brief in appeals under
rule 9.110, 9.130, or 9.140, or after service of the reply brief if a cross-
appeal has been filed, the court may summarily reverse the order to be
reviewed if the court finds that no meritorious basis exists for affirmance
and the order otherwise is subject to reversal.

316
E. Decision and Opinion

Horn v. Marine Hospitality Corp.


745 So. 2d 329 (Fla. 4th DCA 1998)
KLEIN, Judge.
After we affirmed this case without opinion, appellant filed a motion for
rehearing arguing that this case deserved an opinion. Appellant argues that
our affirmance of his allegedly inadequate jury verdict for $200, reduced by
99% comparative negligence, creates conflict with decisions from other
district courts of appeal.
Appellant’s sole argument is that the trial court erred in denying his
motion for new trial on the grounds that the verdict was inadequate. We
affirmed without opinion because there was evidence from which the jury
could have found that the knee injury, which he claimed he sustained in this
accident, did not result from this accident. An orthopedist testified that the
knee problems about which appellant was complaining resulted from an
injury he had sustained several years earlier, which required surgery, as well
as from arthritic changes which had occurred after that surgery.
The defendant paid appellant’s medical expenses incurred after this
accident. We concluded that the trial judge did not abuse its discretion in
denying appellant’s motion for a new trial where the jury only awarded him
$200 for loss of past income.
We recognize that appellants and their counsel often feel shortchanged
when we affirm without writing an opinion. In cases so lacking in merit as
this one, however, a written opinion would serve no useful purpose. An
opinion would not explain anything to the appellant that would not already
be apparent from the record in this case. Other cases in which we routinely
affirm without opinion are (1) those in which appellant is unable to establish
error because of the lack of a transcript of a trial, (2) those in which the
appellant has failed to preserve the error by making an objection in the trial
court, and (3) those in which litigants (often pro se) are unable to identify
any specific error made by the trial court but are simply unhappy with the
result. Those who would advocate requiring appellate courts to write
opinions in all cases would be surprised at how many cases fall in the above
categories.
We doubt that the brief opinion we have now written in this case will
give the appellant much more satisfaction than our original affirmance

317
without opinion. It will, however, unnecessarily enlarge the Southern
Reporter and “further burden attorneys with their research.” Whipple v.
State, 431 So.2d 1011, 1016 (Fla. 2d DCA 1983). Additional good reasons
for not requiring opinions in every case are set out in Whipple at 1016, 1017.
The motion for rehearing is denied.
DELL and FARMER, JJ., concur.

Elliot v. Elliot
648 So. 2d 137 (Fla. 4th DCA 1994)
MAGER, GERALD, Senior Judge.
In this court’s opinion on motion for rehearing, we ordered Appellant’s
counsel to show cause within twenty (20) days of the date of the entry of that
opinion, why monetary and other sanctions should not be imposed for
conduct determined to be a flagrant violation of Florida Rule of Appellate
Procedure 9.330(a). Appellant’s counsel mailed his response on the
twentieth day, indicating that he intended no disrespect to this court, the
lower court, or opposing counsel, and expressed his apologies. Had counsel
simply ended there (leaving well enough alone), the matter would have been
adequately addressed and put to rest. Instead, Appellant’s counsel proceeded
to explain what prompted his argumentative and overzealous motion for
rehearing, namely, the fact that the court’s opinion “was a simple per curiam
affirmance of the trial court’s Final Judgment, and the undersigned attorney
found it impossible to discern the Court’s reasoning.”
This is a most disturbing revelation from any practitioner, let alone one
who has practiced for fifteen (15) years and who admittedly has filed
numerous appeals during that period of time. The notion that an appellate
practitioner would view a per curiam disposition, without opinion, as lacking
in a meaningful review is absolutely astounding. (E.g., “the undersigned
attorney was extremely surprised at this Court’s per curiam affirmance and
presumed that his argument had been overlooked by this Court.”) It was not
until appellate counsel read a concurring opinion by another member of this
panel on motion for rehearing, that he felt that the court had properly
addressed issues originally raised in his appeal. Appellant’s counsel
seemingly tells this court, in effect, that had this court accompanied the per
curiam affirmance with a written opinion it would have been clearer to him
that the court had not overlooked the case authorities cited by him.

318
Perhaps appellate counsel should not be faulted for this misconceived
view of a per curiam affirmance, without opinion. Perhaps the fault lies with
the law school curriculum, the continuing legal education programs offered
by the Florida Bar, or by the appellate courts themselves, in not engendering
a sense of confidence that the absence of a written opinion is not akin to a
superficial treatment, and in leaving the bar with the unfounded notion that
the court “did not read the briefs.” It is fundamental black letter law that a
per curiam disposition affirming a trial court order without a written opinion,
occurs when the points of law raised are so well settled that a further writing
would serve no useful purpose. The sheer volume of appeals, in and of itself,
would seemingly indicate the impossibility of a written opinion on every
affirmance. Perhaps the bar might feel better if per curiam affirmances
contained “one-liners,” to the effect that, “the court has carefully reviewed
and considered the briefs and record in this appeal and finds no reversible
error.” It is doubtful whether this type of form over substance approach
would fully address the problem and the court’s concern about the
perception of per curiam affirmances, without opinion.
Perhaps the court needs simply to restate the fundamental proposition
that each and every appeal receives the same degree of attention and that a
per curiam affirmance without opinion is not an indication of any kind of
lesser treatment. We wonder whether this categorical statement truly puts the
issue to rest and restores some degree of confidence with the bar (and
particularly with the losing party) that the review is no less thorough,
whether the result is a per curiam affirmance with or without a written
opinion.
As the late Judge Rawls pointed out in Taylor v. Knight, 234 So.2d 156
(Fla. 1st DCA 1970):
This Court and not the attorney for the losing party is charged with the
responsibility of deciding which cases merit and warrant a full written
opinion upon the basis of that opinion’s contribution to the jurisprudence of
this State and those cases of great public interest. This Court is not now
denying and has not denied appellants herein any constitutional right and has
not overlooked or failed to consider the jurisprudence of this State in ruling
upon the merits of the appeal [by deciding a case without a written opinion].
Id. at 157.
Despite appellate counsel’s inexplicable misperception of per curiam
dispositions without opinion, the court nevertheless accepts counsel’s

319
explanatory apology and no sanctions will be imposed, this time. It is not the
court’s intention to discourage the good faith filing of motions for rehearing
and it is hoped that this opinion will be instructive to new as well as
seasoned counsel, who may find themselves in the appellate arena-and on
the losing side.
HERSEY, J., and HARRY LEE ANSTEAD, Associate Judge, concur.

State Dep’t of Highway Safety and Motor Vehicles v. Trauth


937 So. 2d 758 (Fla. 3d DCA 2006)
COPE, C.J.
The question before us is whether the circuit court appellate division
(“appellate division”) may grant certiorari and quash an administrative
decision without providing a statement of reasons for doing so. We conclude
that the answer is no. The appellate division must issue a reasoned opinion
when overturning an administrative order.
The underlying administrative proceedings were separate cases involving
the suspension of the driving privileges of respondents Jason Trauth and
Luis Llamas. Trauth and Llamas each requested a formal administrative
hearing pursuant to section 322.2615, Florida Statutes. The hearing officer
upheld the suspension in each case.
Trauth and Llamas each filed a petition for writ of certiorari in the
appellate division seeking to overturn the ruling of the hearing officer.
Because the two petitions raised the same legal issue, the appellate division
consolidated the proceedings.
After oral argument, the appellate division rendered an opinion which
states in its entirety:
(PER CURIAM). GRANTED. Petitioner’s motion for appellate
attorney’s fees is granted pursuant to Fla. R.App. P. 9.400, the amount to be
adjudicated by a duly designated judge of the county court.
Opinion, June 6, 2006.
The Department of Highway Safety and Motor Vehicles filed a motion
for clarification requesting a written opinion explaining the reasons for
quashing the decision of the hearing officer. In a motion for rehearing or
clarification, Trauth and Llamas joined that request. The appellate division
denied the motions without explanation.

320
The Department has filed a petition for writ of certiorari in this court,
seeking to quash the decision of the appellate division. The petition is well
taken.
[1] The appellate division departed from the essential requirements of
law because it quashed the decisions of the lower tribunal-in this case, the
decisions of the hearing officer-without any statement of reasons for doing
so.
[2] This court has previously explained that the appellate division cannot
issue what amounts to a “Per Curiam Reversal,” that is, a reversal without
written opinion. In Kates v. Millheiser, 569 So. 2d 1357 (Fla. 3d DCA
1990), this court said:
First, it is the responsibility of the appellate courts to guide the trial
courts as to questionable procedures or rulings. A per curiam reversal
opinion does not give the trial judge any guidance as to how to correct the
supposed error which was the basis of the reversal.
Second, to the extent that the reversal relates to evidentiary matters, it
fails to place the trial lawyers on notice as to what issues are open for retrial.
Finally, the need for an appellate court to announce the reason for a
reversal is essential to the integrity of the judicial process. It is important for
litigants and the public to recognize that determinations develop as the result
of a fair and just reasoning process as opposed to perceiving judicial
decisions as unjustified and arbitrary. Ultimately, it is the responsibility of
the judiciary to maintain the integrity of the legal system by ensuring that the
judgment processes in the appellate system involve scholarly and fair
deliberations which are open for the public to view. (FN 1) When a case is
appealed, the judgment under review is clothed with a presumption of
correctness. Applegate v. Barnett Bank of Tallahassee, 377 So. 2d 1150
(Fla.1979); Department of Transportation v. Morehouse, 350 So. 2d 529
(Fla. 3d DCA 1977), cert. denied, 358 So. 2d 129 (Fla.1978). If the appellate
court per curiam affirms the trial court decision, this means that the appellant
failed to rebut the presumption of correctness, but still leaves the parties with
the knowledge as to why the final judgment was reached.
However, in the situation where an appellate court per curiam reverses a
final order or judgment, without an opinion, neither party is apprised of why
that particular legal result was reached. Specifically, the parties are entitled
to know exactly why the result reached at the trial level, which was
presumed to be correct, was set aside by the appellate court.

321
It is for this reason that we find paramount among the responsibilities of
an appellate court, the responsibility of writing opinions in all reversals and
remands. See Whipple v. State, 431 So. 2d 1011, 1015 (Fla. 2d DCA 1983)
(“We write opinions in all reversals and remands”). In the interests of
propriety and fairness, litigants cannot be left to wallow in a sea of
confusion as to the rationale supporting a legal result.
....
(FN1.) As stated by one commentator:
In the broader view, appellate justice can be the last best effort of our
government and our law to gain the respect and acceptance of the people.
Appellate justice should be a model for the government’s dealings with
citizens. Appellate courts are the most dignified and receptive authorities to
which individuals can turn to express their legal dissatisfactions in a pointed
way, with assurance of a direct response. If these courts do not deal justly
with litigants, we cannot expect agencies or bureaucracies of lesser
sensitivity to legal rights to do so. It is therefore important that justice on
appeal be visible to all.
Carrington, Meador & Rosenberg, Preface to Justice on Appeal at v
(1976) [emphasis in original]. Id. at 1358. See also City of Kissimmee v.
Grice, 669 So. 2d 307, 308-09 (Fla. 5th DCA 1996); Campbell v. Vetter, 375
So. 2d 4, 5 (Fla. 4th DCA 1979).
The same principles apply here. As they did below, Trauth and Llamas
concede that the appellate division should have issued an opinion.
We conclude that the appellate division departed from the essential
requirements of law such that the granting of certiorari is called for in this
case. See Allstate Ins. Co. v. Kaklamanos, 843 So. 2d 885, 889-91
(Fla.2003); City of Kissimmee v. Grice, 669 So. 2d at 308-09; Kates v.
Millheiser, 569 So. 2d at 1358.
For the stated reasons, we quash the decision of the circuit court appellate
division and remand the matter for further proceedings consistent herewith.
Certiorari granted.

322
F. Precedents

Pardo v. State
596 So. 2d 665 (Fla. 1992)
BARKETT, Justice.
We have for review State v. Pardo, 582 So. 2d 1225 (Fla. 3d DCA 1991),
in which the district court certified express and direct conflict with Kopko v.
State, 577 So. 2d 956 (Fla. 5th DCA 1991), and certified the following
question of great public importance: FN1
FN1. We have jurisdiction pursuant to article V, section 3(b)(4) of the
Florida Constitution.
Where a child victim’s hearsay statements satisfy subsection 90.803(23),
Florida Statutes (1989), and the child is able to testify fully at trial, must the
hearsay statements be excluded solely because they are prior consistent
statement by the child, or is the test for exclusion that found in section
90.403, Florida Statutes (1989)? 582 So. 2d at 1228. In addition to the
certified question and conflict, we also find the district court’s opinion
conflicts with the Fourth District’s decision in State v. Hayes, 333 So. 2d 51
(Fla. 4th DCA 1976), and our decision in Weiman v. McHaffie, 470 So. 2d
682 (Fla.1985).
James Antonio Pardo is charged with seven counts of capital sexual
battery on a child seven years of age.FN2 Pursuant to subsection 90.803(23),
Florida Statutes (1989), the State filed notices of intent to rely on hearsay
statements made by the child victim to nine separate individuals.FN3 After
conducting a hearing as provided by the statute, the court found the
statements of three witnesses sufficiently reliable to be admissible.FN4
However, the court also found that the State intended to call the child to
testify at trial and that the child had the ability to testify fully concerning all
the elements of the alleged crimes. The court concluded that it was required
to exclude the hearsay statements under the authority of Kopko v. State, 577
So. 2d 956, 962 (Fla. 5th DCA 1991), which held that, even though the
criteria of section 90.803(23) are satisfied, where the child is able to testify
fully regarding the circumstances of the alleged abuse, hearsay statements
regarding the abuse are inadmissible prior consistent statements.
Accordingly, the trial court ordered the hearsay statements excluded. The
district court suggested that the trial court was entitled to disregard Kopko,
and in any event, determined that the holding in Kopko was inconsistent with

323
the plain language of the statute, and therefore quashed the trial court’s
order.
FN2. § 794.011(2), Fla.Stat. (1989).
FN3. They were the victim’s mother, the victim’s elementary school
counselors, North Miami Police Department Detective Quartiano, Rape
Treatment Center Doctor Karen Simmons, State Attorney Children’s Center
interviewer Merci Restani, Mental Health Counselor Dawn Bralow, Rape
Treatment Center worker Karen Weissman, Child Assault Program worker
Terry Vazquez, and Doctor Raquel Bild-Libbin.
FN4. Merci Restani, Dawn Bralow, and Dr. Bild-Libbin.
[1] Initially, we note that the district court erred in commenting that
decisions of other district courts of appeal were not binding on the trial
court. This Court has stated that “[t]he decisions of the district courts of
appeal represent the law of Florida unless and until they are overruled by
this Court.” Stanfill v. State, 384 So. 2d 141, 143 (Fla.1980). Thus, in the
absence of interdistrict conflict, district court decisions bind all Florida trial
courts. Weiman v. McHaffie, 470 So. 2d 682, 684 (Fla.1985). The purpose of
this rule was explained by the Fourth District in State v. Hayes:
The District Courts of Appeal are required to follow Supreme Court
decisions. As an adjunct to this rule it is logical and necessary in order to
preserve stability and predictability in the law that, likewise, trial courts be
required to follow the holdings of higher courts-District Courts of Appeal.
The proper hierarchy of decisional holdings would demand that in the event
the only case on point on a district level is from a district other than the one
in which the trial court is located, the trial court be required to follow that
decision. Alternatively, if the district court of the district in which the trial
court is located has decided the issue, the trial court is bound to follow it.
Contrarily, as between District Courts of Appeal, a sister district’s opinion is
merely persuasive.
333 So. 2d 51, 53 (Fla. 4th DCA 1976) (footnote and citations omitted).
FN5
Consequently, the trial court in this case was bound by the Fifth District’s
decision in Kopko.
FN5. See generally Taylor Mattis, Stare Decisis Among and Within
Florida’s District Courts of Appeal, 18 Fla.St.U.L.Rev. 143, 155-160
(1990).

324
[2] On the merits, we find that a child victim’s hearsay statement which
qualifies for the statutory exception in section 90.803(23) may be admissible
in evidence when the child is able to testify fully at trial notwithstanding its
characterization as a prior consistent statement.
[***]
Accordingly, we approve in part and quash in part the opinion of the
court below, disapprove the Fifth District’s opinion in Kopko, and remand
for proceedings consistent with this opinion.
It is so ordered.
SHAW, C.J., and OVERTON, McDONALD, GRIMES, KOGAN and
HARDING, JJ., concur.

Dep’t of Legal Affairs v. District Court of Appeal, 5th District


434 So. 2d 310 (Fla. 1983)
McDONALD, Justice.
The State of Florida Department of Legal Affairs complains of an
unreported order entered by the Fifth District Court of Appeal which reads:
Counsel for appellant shall not cite decisions of appellate courts which
have no written opinion.
The state’s petition, received pursuant to article V, section 3(b)(7),
Florida Constitution, requests an order commanding the district court to
rescind its order and to cease directing counsel for the state as to what
authorities may be cited in appellate briefs and other pleadings. After
receiving a response to the state’s petition and after considering this matter,
we deny the state’s petition and approve the order of the district court.
[1] The issue is whether a per curiam appellate court decision with no
written opinion has any precedential value. We hold that it does not.
In reaching this conclusion we note that this ruling is contrary to at least
one prior decision of this Court. In State Department of Public Welfare v.
Melser, 69 So. 2d 347 (Fla.1954), this Court grappled with the question of
whether naturopathic physicians had the authority to prescribe drugs. A trial
judge had apparently ruled in a prior case which had been affirmed without
opinion, In re Complaint of Melser, 160 Fla. 333, 32 So. 2d 742 (1947), that
a naturopath could prescribe drugs. In discussing that case the original
majority opinion stated:

325
“An affirmance without an opinion is an approval only of the point
decided or result reached by the court below, and not of the opinion and the
conclusions of law of the lower court, so as to establish a precedent for
future action.” 69 So. 2d at 350-51, quoting 21 C.J.S. Courts § 186(c)
(1940). On rehearing, by a four to three vote, the Court recognized the prior
per curiam affirmed judgment and followed it, stating:
It is the view of said Justices and the Associate Justice that not only is
that issue not presented for our consideration in this appeal but that since the
precise and only point decided by the Circuit Court of Pinellas County in “In
re Complaint of Melser, 160 Fla. 333, 32 So. 2d 742,” was the statutory right
of naturopathic physicians to prescribe narcotic drugs included in the term
phytotherapy, the affirmance of that judgment by this Court-even without
opinion-settled the law to that extent and that this Court, in this case, should
not be drawn into an unnecessary reconsideration of its former decision. Id.
at 353.FN1 Moreover, Foley v. Weaver Drugs, Inc., 177 So. 2d 221
(Fla.1965), in discussing whether this Court should construe a per curiam
decision of a district court to conflict with a written decision of another court
and thus grant review because of conflict jurisdiction, states:
FN1. Our research shows that Melser has been relied upon infrequently,
and never for the above-quoted proposition.
Nor can we escape that in common parlance, an affirmance without
opinion of a trial court by a district court is generally deemed to be an
approval of the judgment of the trial court, and becomes a precedent,
certainly, in the trial court rendering the judgment. Id. at 225-26.FN2
FN2. Using per curiam affirmed decisions without opinion as a basis for
conflict jurisdiction has been sharply restricted by the 1980 amendment to
this Court’s jurisdiction. Dodi Publishing Co. v. Editorial America S.A., 385
So. 2d 1369 (Fla.1980); Jenkins v. State, 385 So. 2d 1356 (Fla.1980).
The district courts of appeal which have addressed this issue, however,
have been firm in holding that per curiam affirmances without opinion have
no precedential value. In Schooley v. Judd, 149 So. 2d 587 (Fla. 2d DCA),
rev'd on other grounds, 158 So. 2d 514 (Fla.1963), the Second District Court
of Appeal noted that the trial judge had relied upon a per curiam decision
without opinion by this Court and stated: “We are of the view that such a
decision does not establish any point of law; and there is no presumption that
the affirmance was on the merits.” 149 So. 2d at 590. The district court,
therefore, declined to follow the trial court’s conclusions. In Acme Specialty

326
Corp. v. City of Miami, 292 So. 2d 379 (Fla. 3d DCA 1974), the third
district, in commenting on a prior decision, stated:
As noted before, this was a per curiam opinion with no reasons or
authorities given and, although this may be sufficient to support a plea of res
judicata as between the original parties, such per curiam affirmance opinion
does not stand for any general pronouncement of principles of law that
might have been urged by the parties in their pleadings and briefs. Id. at 380
(footnote omitted). That court reiterated its position in Berek v. Metropolitan
Dade County, 396 So. 2d 756, 759 n. 3 (Fla. 3d DCA 1981), approved on
other grounds, 422 So. 2d 838 (Fla.1982), stating: “We do not regard a per
curiam affirmance without opinion as having precedential significance for
other cases, notwithstanding that such an affirmance may establish the law
of the particular case.” (Citations omitted.) In Goldberg v. Graser, 365 So.
2d 770, 773 (Fla. 1st DCA 1978), the First District Court of Appeal stated:
“A per curiam affirmance without opinion does not bind the appellate court
in another case to accept the conclusion of law on which the decision of the
lower court was based.”
The fifth district obviously agreed with the other district courts’ views
when it rendered its order.FN3 The obvious weight of authority follows this
view. See 21 C.J.S. Courts § 186 (1940); 20 Am.Jur.2d Courts § 189 (1965).
The views expressed by the district courts are proper and correct
interpretations of the nonprecedential value of per curiam affirmances
without opinion.
FN3. The court reiterated its order in State v. A.D.H., 429 So. 2d 1316
(Fla. 5th DCA 1983), the case which prompted the instant order.
Perhaps justification existed for this Court’s giving precedential value to
an unwritten opinion thirty years ago. It was the only appellate court and had
access to all of its prior files for review, and its volume of work was small in
comparison to the number of appeals today. But even if those circumstances
prevailed today, we foresee great danger in applying the rule of stare decisis
to such a decision. No one can ascertain what was argued to the original
court first rendering a decision.FN4 More importantly, however, would be the
uncertainty of the law. All lawyers, and lay people also for that matter,
should be able to research and have available all existing law. If there is a
body of law floating around in unwritten or unpublished opinions, only those
persons privy to those cases know those pronouncements.FN5 This creates
unwarranted confusion and disparity in the orderly presentation of issues.

327
FN4. In dissent on rehearing in Melser Justice Mathews strongly argued
that the trial judge had not considered all of the statutes which were
applicable to the issue.
FN5. In State v. Fitzpatrick, 5 Wash.App. 661, 491 P.2d 262 (1971), an
appellate court found that unpublished opinions do not become part of the
common law. In Jones v. Superintendent, Virginia State Farm, 465 F.2d
1091 (4th Cir.1972), the court ruled that memorandum opinions could not
come under the stare decisis rule.
The rationale and basis for the decision without opinion is always subject
to speculation. In Newmons v. Lake Worth Drainage District, 87 So. 2d 49
(Fla.1956), the Court discussed several reasons for issuing per curiam
decisions, some with written opinions and some without written opinions,
and concluded: “In fine, there is no limit to the grounds that may prompt a
per curiam opinion.” Id. at 51. Such uncertainty in itself negates a basis for
reliance on an unwritten decision for guidance or precedence. We therefore
recede from the language in Melser on rehearing which implies that a per
curiam decision without opinion can be cited as a precedent.
[2] The second issue is whether it is proper to cite such a decision to a
court. It is evident that such a citation from another court has no relevance
for any purpose and is properly excluded from a brief or oral argument.
While the court from which the decision emanated has a record of that case
and may possess some unique knowledge underlying the decision, the court
to which it is being cited can only speculate as to the rationale of such a
decision and is not in a position to agree or disagree with the reason for the
decision.
[3] The situation is slightly different in reference to calling a court’s
attention to one of its own unwritten decisions. We reiterate that such a
decision is not a precedent for a principle of law and should not be relied
upon for anything other than res judicata. Usually, however, it would not be
improper for counsel, in an effort to persuade a court to adopt a certain
position, to refer to such a decision and thereby suggest to the court how it
previously viewed the proposition. That court has the records of its own
decisions and the judges have the opportunity to discuss such cases
collegially. Conversely, because such decisions have no precedential value,
a court may take the view that it desires not to consider such cases in any
circumstance, and it may properly disregard such a reference in briefs or
arguments presented to it.

328
This case commenced with the fifth district’s striking the citation of a
“per curiam affirmed” decision of the Second District Court of Appeal. As
such the fifth district’s action was clearly proper. In the context in which it
arose we believe that the order complained of referred to such citations from
other courts, but, even if it included citations from itself, the Fifth District
Court of Appeal had the prerogative to take the position it did.
The petition is denied.
It is so ordered.
ALDERMAN, C.J., and ADKINS, OVERTON, EHRLICH and SHAW,
JJ., concur.
BOYD, J., dissents with an opinion.
BOYD, Justice, dissenting. [ * * * ]

G. Mandate

Florida Rules of Appellate Procedure

Rule 9.340. Mandate


(a) Issuance of Mandate. Unless otherwise ordered by the court or
provided by these rules, the clerk shall issue such mandate or process as
may be directed by the court after expiration of 15 days from the date of
an order or decision. A copy thereof, or notice of its issuance, shall be
served on all parties.
(b) Extension of Time for Issuance of Mandate. If a timely motion for
rehearing, clarification, or certification has been filed, the time for issuance
of the mandate or other process shall be extended until 15 days after
rendition of the order denying the motion, or, if granted, until 15 days after
the cause has been fully determined.
(c) Entry of Money Judgment. If a judgment of reversal is entered that
requires the entry of a money judgment on a verdict, the mandate shall be
deemed to require such money judgment to be entered as of the date of
the verdict.

329
Blackhawk Heating & Plumbing Co., Inc. v. Data Lease Fin. Corp.
328 So. 2d 825 (Fla. 1975)
ADKINS, Chief Justice.
Petitioners Blackhawk and Machatta request that this Court enter an
order requiring the trial court to comply with our opinion and mandate
rendered in Blackhawk Heating & Plumbing Co., Inc. et al. v. Data Lease
Financial Corp., Fla.1974, and reported in 302 So. 2d 404.
This case involved an agreement relating to the purchase of 870,000
shares of the common stock of Miami National Bank for a purchase price of
$10,440.000. Upon rendition of our decision we remanded same to the
District Court of Appeal with instructions to further remand same to the trial
court for the purpose of determining the rights of the parties under the
contract. We also reinstated the temporary injunction which had been
entered by the trial court. The temporary injunction contained the following:
‘At this stage of the case it is not certain that plaintiff had a valid option
contract, or that it properly exercised it. On the other hand, plaintiff may
well have a valid option, and its contentions as to the optionee’s
performance upon exercise of the option may be valid. The purpose of a
temporary injunction being to maintain the status quo, North Dade Water
Co. v. Adken Land Co., (Fla.App.) 114 So. 2d 347, it would seem
appropriate here because there is a myriad of beneficial aspects to the
ownership of this stock which could be lost to plaintiff or compromised
should the stock be disposed of or otherwise affected during the pendency of
this litigation.’
Even at that stage of the proceeding the Court recognized the advantages
flowing from ownership of the stock, as outweighing the advantages of a
cash payment.
The agreement of the parties provides:
‘6. (a) In the event that either party shall at any time be in danger of
defaulting in any payments due in connection with the purchase of MNB
Stock, such party shall immediately give the other party timely notice of
such fact and shall afford the other party an opportunity to remedy such
default. Should the other party remedy such default then the party taking
such remedial action shall be entitled to an increase in its equity in MNB
Stock equivalent to the new proportion that payments made by it for such
stock bear to all payments properly made hereunder.

330
‘(b) Any cash flow benefit, including any tax benefits, derived by Data as
a consequence of its holding, hypothecation, assignment, pledge, etc. of
MNB Stock shall inure proportionately to Blackhawk in the calculation of
any payments due between the parties.’ (Emphasis supplied.)
[1] When the mandate was received by the trial court, such court should
have carried out and placed into effect the order and judgment of this Court.
Rinker Materials Corporation v. Holloway Materials Corporation, 175 So.
2d 564 (Fla.App.2d, 1965).
[2] [3] A trial court is without authority to alter or evade the mandate of
an appellate court absent permission to do so. Cone v. Cone, 68 So. 2d 886
(Fla.1953). If the trial court fails or refuses to comply with the appellate
court’s mandate, the latter may, generally speaking, take any steps or issue
any appropriate writ necessary to give effect to its judgment. State ex rel.
Dowling Co. v. Parks, 99 Fla. 1264, 128 So. 837 (1930).
It now appears that more than six months have elapsed and the trial court
has not yet determined the rights of the parties under the contract to purchase
the bank stock. Instead, on May 20, 1975, the trial court entertained a motion
for authorization to execute and perform an agreement by Data Lease to sell
the stock and to modify the temporary injunction reinstated by this Court.
On June 5, 1975, the trial court dissolved the temporary injunction which
order, says petitioner, was in violation of our mandate. We agree.
[4] The trial court should have related the findings of the special master
to the established Florida law in the construction of contracts and determined
the amount, if any, required for the exercise of the option agreement. The
order of the trial court dissolving the temporary injunction and authorizing
the sale of the stock violated the mandate of this Court and is quashed.
Data Lease has also filed a motion seeking authorization to execute and
perform the agreement to sell the stock and to modify the temporary
injunction, or, in the alternative, to obtain the leave or consent of this Court
to such modification and authorization. The record before this Court fails to
show that mere substitution of cash would truly maintain the petitioners in
the status quo posture mandated by this court pending outcome of the
litigation, or that status quo should be altered.
[5] [6] [7] In order to modify the mandate (which, in effect, would
modify the prior decision), the party seeking permission must show some
new relevant matter that would probably produce a different result had it
been considered by the court. Upon such showing, this court may then

331
amend its mandate or direct the lower court to make a factual determination
on the question of whether such an amendment should be made. After
permission from this Court, the trial court could modify or amend any
judgment mandated by this Court. For a general discussion of these
questions, see 5B C.J.S. Appeal and Error ss 1994, 2002, 2003.
It has not been shown that Blackhawk caused any delay in the
enforcement of the option agreement during the proceedings subsequent to
our mandate, so it cannot be held responsible for the interest obligations
accumulated by Data Lease. It has not been shown that the Bank has been
jeopardized in any way.
[8] This entire litigation involves the right of Blackhawk to purchase
bank stock and the purchase price to be paid for the stock. It should not be
deprived of this contractual right simply because a sale of the stock would be
to the advantage of Data Lease.
The motion of Data Lease for authorization to execute and perform the
stock sale agreement and modify the temporary injunction or, in the
alternative, to obtain leave or consent of this Court to such modification or
authorization is denied.
The trial court shall apply the established Florida law in the construction
of contracts and determine the amount, if any, required for the exercise of
the option and to preserve the assets in question, i.e., the shares of the
common stock of Miami National Bank. This was the mandate of our
decision in this case.
It is so ordered.
ROBERTS, BOYD and SUNDBERG, JJ., concur.
ENGLAND, J., dissents with opinion.
ENGLAND, Justice (dissenting). [ * * * ]

State Farm Mut. Auto. Ins. Co. v. Judges of Dist. Ct. of Appeal, Fifth Dist.
405 So. 2d 980 (Fla. 1981)
SUNDBERG, Chief Justice.
We here confront the issue of whether the District Court of Appeal, Fifth
District, may utilize the en banc rehearing rule as a toehold to extend ad
infinitum its jurisdiction whenever intradistrict conflict arises. We hold that
it may not.

332
On May 7, 1980, the district court affirmed without opinion the trial
court’s dismissal of plaintiff’s action in Rogers v. State Farm Mutual
Automobile Insurance Co., 383 So. 2d 1221 (Fla. 5th DCA 1980). Almost
two months after the end of the term [FN1] in which this PCA decision and
mandate were issued, Rogers filed a motion for rehearing. The court denied
the motion on November 12, 1980, because it was not filed within fifteen
days of the May 7th decision, the time period prescribed by Florida Rule of
Appellate Procedure 9.330(a). [FN2] Upon its own motion, however, the
court decided to reconsider the case, basing its authority to review on Florida
Rule of Appellate Procedure 9.331(c),[FN3] which governs rehearings en
banc. It then reversed Rogers as being in direct conflict with one of the
court’s later decisions. [FN4] Rogers v. State Farm Mutual Automobile
Insurance Co., 390 So. 2d 138 (Fla. 5th DCA 1980).
FN1. Two terms a year of the district courts are set by statute. Section
35.10, Florida Statutes (1979), reads in part: “Regular terms.-The district
court of appeal shall hold two regular terms each year at its headquarters,
commencing respectively on the second Tuesday in January and July.”
FN2. Fla.R.App.P. 9.330(a):Time for Filing; Contest; Reply. A motion
for rehearing or for clarification of decision may be filed within 15 days of
an order or within such other time set by the court. The motion shall state
with particularity the points of law or fact which the court has overlooked or
misapprehended. The motion shall not re-argue the merits of the court’s
order. A reply may be served within 10 days of service of the motion.
FN3. Fla.R.App.P. 9.331(a), (c):En Banc Proceedings: Generally. A
majority of the judges of a district court of appeal may order a proceeding
pending before the court be determined en banc. A district court of appeal en
banc shall consist of the judges in regular active service on the court. En
banc hearings and rehearings shall not be ordered unless necessary to
maintain uniformity in the court’s decisions.....Rehearings En Banc.(1)
Generally. A rehearing en banc may be ordered by a district court of appeal
on its own motion or on motion of a party. Within the time prescribed by
Rule 9.330 and in conjunction with the motion for rehearing, a party may
move for an en banc rehearing solely on the ground that such consideration
in necessary to maintain uniformity in the court’s decisions. A motion based
on any other ground shall be stricken. A vote will not be taken on the motion
unless requested by a judge on the panel that heard the proceeding, or by any
judge in regular active service on the court. Judges who did not sit on the

333
panel are under no obligation to consider the motion unless a vote is
requested.
FN4. State Farm Mutual Automobile Ins. Co. v. Bergman, 387 So. 2d
494 (Fla. 5th DCA 1980), cert. granted, No. 59,706 (Fla. Feb. 16, 1981).
State Farm filed a petition for common law certiorari with this Court
seeking to vacate the en banc court’s November 12th judgment because it
lacked jurisdiction. We have treated the petition as an application for writ of
mandamus. Our jurisdiction is found under article V, section 3(b)(8), Florida
Constitution (1980).
Respondents initially deny the propriety of a writ of mandamus as a
remedy in this case, claiming that the district court acted within its discretion
in reversing its prior judgment. Yet it is painfully clear that if the district
court had no jurisdiction to act as it did, the question of discretion is
irrelevant:
Where the action of a ... court in vacating a judgment is wholly void, he
has no discretion as to whether or not he will vacate the order vacating the
judgment and make an order of reinstatement.
....
And mandamus will lie to compel a court to vacate a judgment, order, or
decree which it was entirely without jurisdiction to make, there being no
adequate remedy by appeal, error, or otherwise.
State ex rel. Melbourne State Bank v. Wright, 107 Fla. 178, 184-85, 145
So. 598, 601 (1932) (citation omitted).
The thrust of respondents’ argument on the merits is that although
jurisdiction of the district court would normally end with the term, Florida
Rule of Appellate Procedure 9.331(c) extends the jurisdiction of the court
indefinitely for the sole purpose of correcting intradistrict conflict by means
of en banc rehearings. Respondents reason that because the Supreme Court
had jurisdiction to resolve intradistrict conflict on the date rule 9.331 became
effective,[FN5] and because the Committee Note [FN6] to the rule indicates
the en banc proceeding is to be utilized in situations analogous to the
Supreme Court’s conflict jurisdiction, therefore the district court is vested
with jurisdiction to resolve intradistrict conflict. Furthermore, it is asserted
that this jurisdiction is eternal because no time limit is specifically
prescribed by the rule.
FN5. January 1, 1980.

334
FN6. The Committee Note reads in part: This rule is patterned after the
en banc rule of the United States Court of Appeals for the Fifth Circuit and
should be used sparingly. Subsection (a) provides that a majority vote of a
district court is necessary to set a case for rehearing or rehearing en banc. All
judges in regular active service, not excluded for cause, will constitute the en
banc panel. Counsel are reminded that en banc proceedings are extraordinary
and will be ordered only in the enumerated circumstances. The ground,
maintenance of uniformity in the court’s decisions, is the equivalent of
decisional conflict as developed by Supreme Court precedent in the exercise
of its conflict certiorari jurisdiction. The district courts are free, however, to
develop their own concept of decisional uniformity.
[1] Without wading through the various shortcomings we perceive in the
argument, it will suffice to state that we simply do not view the jurisdictional
powers of an appellate court as encompassing such a grandiose temporal
spectrum. All things must have end, even a district court’s power to correct
inconsistencies. The reasons for this form the bedrock of Anglo-American
jurisprudence: “There must be an end of litigation. Public policy, as well as
the interests of individual litigants, demands it, and the rule just announced
is indispensable to such a consummation.” Lovett v. State, 29 Fla. 384, 401,
11 So. 176, 179 (1892). See also Martin v. Hunter’s Lessee, 14 U.S. (1
Wheat.) 304, 355, 4 L.Ed. 97 (1816). And the “rule” which determined this
end is set out in Lovett and several other Florida cases. An appellate court’s
power to recall its mandate is limited to the term during which it was issued.
See Chapman v. St. Stephens Protestant Episcopal Church, Inc., 105 Fla.
683, 138 So. 630 (1932); Washington v. State, 92 Fla. 740, 110 So. 259
(1926).
[2] We do not see rule 9.331(c) as abrogating this well established
common-law precedent that a court’s power to alter its mandate generally
ends with the term the mandate issued. Rule 9.331(c) is not a jurisdictional
grant; it is a rule of procedure. We therefore conclude that the precedent
limiting an appellate court’s jurisdiction is equally applicable to en banc
rehearing proceedings.
[3] Because the recall of mandate by the district court was made after the
term in which the original mandate issued, the court was without
jurisdiction. Its actions are thus void. See Wright. Accordingly, the petition
for writ of mandamus is granted, and the writ is issued directing the District

335
Court of Appeal, Fifth District, to vacate its last judgment in this cause and
to reinstate its former.
It is so ordered.
ADKINS, BOYD, OVERTON, ALDERMAN and McDONALD, JJ.,
concur.

Judges of Eleventh Judicial Circuit


in and for Dade County v. Janovitz
635 So. 2d 19 (Fla. 1994)
PER CURIAM.
We review Janovitz v. Judges of the Eleventh Judicial Circuit, 613 So. 2d
527 (Fla. 3d DCA 1993), based on conflict with Finkelstein v. North
Broward Hospital District, 484 So. 2d 1241 (Fla.1986). We have
jurisdiction pursuant to article V, section 3(b)(3) of the Florida Constitution
and quash Janovitz.
Pamela Janovitz leased a condominium in North Miami Beach to a
family named Machado. The lease agreement contained a provision that
cooperating brokers would receive ten percent of the lease amount as a
commission. Koslovsky Realty and the Keyes Company (the brokers) sued
Janovitz in county court to recover their commission. The county court
granted summary judgment for the brokers, and Janovitz appealed that order
to the appellate division of the circuit court. On June 19, 1992, the circuit
court entered a per curiam order affirming the trial court’s summary
judgment. The circuit court denied rehearing on July 22, 1992, and issued its
mandate on July 24. Fifty-four days later, on September 16, 1992, the circuit
court granted the brokers’ prior motion for appellate attorney’s fees and
remanded to the county court to determine the amount.
Janovitz petitioned the district court of appeal for a writ of mandamus
directing the circuit court to withdraw its award of attorney’s fees.
Following its earlier decision in Dyer v. City of Miami Employees’
Retirement Board, 512 So. 2d 338 (Fla. 3d DCA 1987), the district court of
appeal held that the circuit court had no jurisdiction to enter the order on
attorney’s fees after its mandate had issued. The circuit court’s appellate
division then petitioned this Court for review.
[1] An appellate court may recall a mandate before its term of court ends
in order to revisit the cause. State Farm Mut. Auto. Ins. Co. v. Judges of

336
Dist. Court of Appeal, Fifth Dist., 405 So. 2d 980 (Fla.1981). The petitioners
concede that the circuit court never withdrew its mandate in this case, but
they argue that a claim for attorney’s fees is an independent collateral claim
which may be determined after the plenary appeal has been adjudicated on
the merits regardless of whether the mandate has been issued.
In Finkelstein, we stated:
The issue before us is whether the trial court lacked jurisdiction to
entertain the plaintiffs’ motion for “prevailing party” attorney’s fees, where
the plaintiffs’ complaint contained a demand for attorney’s fees, where the
final judgment did not dispose of or explicitly retain jurisdiction over the
claim for attorney’s fees, and where the plaintiffs’ motion for attorney’s fees
was filed three days after the final judgment on the main claim became final.
484 So. 2d at 1242. We held that a motion for attorney’s fees raised a
collateral and independent claim which the trial court has continuing
jurisdiction to entertain within a reasonable time, notwithstanding that the
litigation of the main claim may have been concluded with finality. Even
though that case dealt with a trial court award rather than an appellate court
award, the principle is the same because it pertains to the jurisdiction of the
court to enter an order on attorney’s fees after it loses jurisdiction of the
claim on the merits.
[2] [3] Thus, we hold that when a motion for appellate attorney’s fees has
been timely filed, the court may enter an award of attorney’s fees within a
reasonable time after the issuance of the mandate. While the award in the
instant case was entered in the same term of court, we do not believe this to
be a requirement for such collateral orders so long as they are entered within
a reasonable time.
We disapprove Dyer and quash the decision below. We remand with
directions to reinstate the order awarding attorney’s fees.
It is so ordered.
BARKETT, C.J., and OVERTON, SHAW, GRIMES, KOGAN and
HARDING, JJ., concur.
McDONALD, J., concurs with an opinion. [ * * * ]

337
H. Law of the Case

Florida Dep’t of Transportation v. Juliano


801 So. 2d 101 (Fla. 2001)
PARIENTE, J.
We have for review Florida Department of Transportation v. Juliano,
744 So. 2d 477 (Fla. 3d DCA 1999) (“ Juliano II ”), a decision from the
Third District Court of Appeal that misapplies this Court’s holding in United
States Concrete Pipe v. Bould, 437 So. 2d 1061, 1063 (Fla.1983). Based on
the conflict created by this misapplication, we have jurisdiction under article
V, section 3(b)(3), of the Florida Constitution. See Vest v. Travelers Ins. Co.,
753 So. 2d 1270, 1272 (Fla.2000); Pender v. State, 700 So. 2d 664, 665
(Fla.1997).
FACTS
The underlying facts are recited in the Third District’s opinion:
Angelo Juliano, a former correctional officer, employed by the Florida
Department of Corrections (“DOC”),[note 1] was injured when he tripped on
a large bump in the floor of a mobile weigh station operated by the DOT
[Florida Department of Transportation]. At the time of Juliano’s accident,
the DOT had a contract with the DOC for the use of its inmates to clean the
DOT’s weigh station under the supervision of the DOC correctional officers.
Juliano was supervising the inmates at the weigh station when he tripped and
injured himself.
[Note 1: As a result of his injuries, Juliano was terminated as a
correctional officer.]
Juliano II, 744 So. 2d at 478.
Juliano received workers’ compensation benefits from DOC for his
injuries and subsequently filed a third-party tort action for personal injuries
against DOT for negligence in maintaining the weigh station. DOT moved
for summary judgment on the grounds that it was entitled to workers’
compensation immunity because the “unrelated works” exception under
section 440.11(1), Florida Statutes (1997),FN1 did not apply. DOT contended
that the exception did not apply because Juliano failed to name any specific
employee of DOT whom Juliano alleged to be negligent. In addition, DOT
argued that Juliano had neither alleged nor shown any specific actions on the
part of DOT’s employees that were negligent. In response, Juliano argued

338
that it was unnecessary to name a particular employee for the “unrelated
works” exception to apply. Moreover, in Juliano’s written response to
DOT’s summary judgment motion, he identified two specific DOT
supervisors who Juliano alleged had been negligent: a safety specialist
named Mary Lou and Sergeant Wyse. Juliano also argued that DOT was not
entitled to summary judgment because a disputed genuine issue of material
fact existed as to whether Sergeant Wyse was negligent. The trial court
denied the motion for summary judgment, concluding that disputed genuine
issues of material fact existed and that the jury was entitled to decide
whether there was negligence on the part of the DOT employees.
FN1. The “unrelated works” exception under section 440.11(1), Florida
Statutes (1997), provides that workers’ compensation immunity is not
“applicable to employees of the same employer when each is operating in
the furtherance of the employer’s business but they are assigned primarily to
unrelated works within private or public employment.”
DOT filed an interlocutory appeal of this order FN2 and it framed the issue
on appeal as whether a “claimant who has accepted workers’ compensation
benefits and has also brought suit in personal injury against his employer
must identify the fellow employee accused of negligence.” Juliano asserted
that under this Court’s opinion in Holmes County School Board v. Duffell,
651 So. 2d 1176 (Fla.1995), he was not required to identify the negligent
fellow employee.FN3 Moreover, Juliano contended that because he named
Mary Lou and Sergeant Wyse as the negligent coworkers, DOT’s appeal
was moot. The Third District affirmed per curiam with a citation to Holmes.
See Florida Dep’t. of Transp. v. Juliano, 664 So. 2d 77, 77 (Fla. 3d DCA
1995) (“ Juliano I ”).
FN2. Although prior case law allowed interlocutory appeals of denials of
summary judgment based on workers’ compensation immunity where
disputed issues of material fact existed, see Breakers Palm Beach, Inc. v.
Gloger, 646 So. 2d 237 (Fla. 4th DCA 1994), this Court amended Florida
Rule of Appellate Procedure 9.130(a)(3)(C)(vi) in 1996 to clarify that
appeals of orders denying summary judgment asserting workers’
compensation immunity may only be taken when the denial is as a matter of
law. See Hastings v. Demming, 694 So. 2d 718, 719 (Fla.1997). Therefore,
although DOT appealed its first motion for summary judgment in 1995, and
did not have the benefit of the 1996 amendment to rule 9.130(a)(3)(C)(vi), it
is clear that DOT would have been precluded from appealing the trial court’s

339
denial of its motion for summary judgment based upon the existence of a
genuine factual dispute had it done so after 1996.
FN3. In Holmes, this Court considered the interrelationship of the
sovereign immunity statute, section 768.28(9), Florida Statutes (1991), and
the “unrelated works” exception to workers’ compensation immunity under
section 440.11(1) as applied to a public employer. 651 So. 2d at 1177-78. As
this Court explained in Holmes, section 768.28(9) immunizes public
employees from personal liability for torts by requiring any civil action for
the employee’s negligence to be maintained against the governmental entity
itself. See id. at 1178. Thus, reading the two statutes together, this Court
concluded that an injured public employee’s exclusive remedy for a public
employee’s negligence was an action against the public employer “unless
such act or omission was committed in bad faith or with malicious purpose
or in a manner exhibiting wanton and willful disregard of human rights,
safety, or property.” Id. at 1178. The Court explained that, in essence, the
employer “is being sued as a surrogate defendant based on the negligent acts
of ... a fellow public employee.” Id. at 1179.
On remand, DOT filed a second motion for summary judgment. This
time, DOT argued that an employee could not sue a supervisor under the
“unrelated works” exception unless the employee could demonstrate that the
supervisor engaged in conduct that amounted to culpable negligence. The
trial court denied DOT’s second summary judgment motion as “a mere
relitigation of the first motion for summary judgment.” Juliano II, 744 So.2d
at 478. The case proceeded to trial with a verdict being entered in Juliano’s
favor. See id.
DOT appealed the final judgment. Among the issues raised on appeal,
DOT argued that the trial court erred in denying its second motion for
summary judgment. See id. In rejecting this argument, the Third District
concluded that the doctrine of res judicata precluded the DOT from raising
or reraising any aspect of its workers’ compensation defense on remand after
the first appeal of this cause. See Thomas v. Perkins, 723 So. 2d 293, 294
(Fla. 3d DCA 1998) (under the doctrine of res judicata, appellant is
precluded from raising any issues which were or should have been raised on
first appeal). Id. This appeal follows.
THE DOCTRINES OF RES JUDICATA AND THE LAW OF THE
CASE

340
In this case, we must determine whether the Third District properly
applied the doctrine of res judicata to preclude DOT from raising a distinct
aspect of its defense that it did not raise in the first non-final appeal. In
analyzing this issue, it is incumbent upon this Court to first review the
important differences regarding the doctrine of res judicata and the related
doctrine of the law of the case.
[1] [2] This Court has explained that under the doctrine of res judicata:
A judgment on the merits rendered in a former suit between the same
parties or their privies, upon the same cause of action, by a court of
competent jurisdiction, is conclusive not only as to every matter which was
offered and received to sustain or defeat the claim, but as to every other
matter which might with propriety have been litigated and determined in that
action.
Kimbrell v. Paige, 448 So. 2d 1009, 1012 (Fla.1984) (emphasis supplied)
(quoting Wade v. Clower, 94 Fla. 817, 114 So. 548, 552 (1927)). Based on
principles of res judicata, a judgment on the merits will thus bar “a
subsequent action between the same parties on the same cause of action.”
Youngblood v. Taylor, 89 So. 2d 503, 505 (Fla.1956) (emphasis supplied).
Importantly, the doctrine of res judicata not only bars issues that were raised,
but it also precludes consideration of issues that could have been raised but
were not raised in the first case. See id.
[3] As explained more fully in McGregor v. Provident Trust Company,
119 Fla. 718, 162 So. 323, 327 (1935):
Inhering in all courts of civilized nations and, as is said in one case, an
obvious rule of expediency and justice, res adjudicata is a fundamental
doctrine universally recognized. No better enunciation of it, perhaps, can be
found than that given by Black in his work on Judgments. He states it in two
main rules, as follows: First, a point which was actually and directly in issue
in a former suit, and was there judicially passed upon and determined by a
domestic court of competent jurisdiction, cannot again be drawn in question
in any future action between the same parties or their privies, whether the
causes of action in the two suits be identical or different; and, Second, a
judgment rendered by a court of competent jurisdiction, on the merits, is a
bar to any future suit between the same parties or their privies upon the same
cause of action, so long as it remains unreversed. Black on Judgments (2d
Ed.) vol. 2, § 504.

341
Thus, the doctrine of res judicata provides finality to judgments,
predictability to litigants, and stability to judicial decisions.
[4] [5] [6] [7] Where successive appeals are taken in the same case there
is no question of res judicata, because the same suit, and not a new and
different one, is involved. See Beverly Beach Props., Inc. v. Nelson, 68 So.
2d 604, 607 (Fla.1953). Under these circumstances, the doctrine of the law
of the case applies. The doctrine of the law of the case is also a principle of
judicial estoppel, but it is more limited and more flexible in scope. The
doctrine of the law of the case requires that questions of law actually decided
on appeal must govern the case in the same court and the trial court, through
all subsequent stages of the proceedings. See Greene v. Massey, 384 So. 2d
24, 28 (Fla.1980) (“All points of law which have been adjudicated become
the law of the case and are, except in exceptional circumstances, no longer
open for discussion or consideration in subsequent proceedings in the
case.”); Strazzulla v. Hendrick, 177 So. 2d 1, 3 (Fla.1965). Under the law of
the case doctrine, a trial court is bound to follow prior rulings of the
appellate court as long as the facts on which such decision are based
continue to be the facts of the case. See McGregor, 162 So. at 327.
Moreover, even as to those issues actually decided, the law of the case
doctrine is more flexible than res judicata in that it also provides that an
appellate court has the power to reconsider and correct an erroneous ruling
that has become the law of the case where a prior ruling would result in a
“manifest injustice.” Strazzulla, 177 So. 2d at 5.
[8] As to the scope of the law of the case doctrine, this Court in U.S.
Concrete, 437 So. 2d at 1063, explained that the doctrine is “limited to
rulings on questions of law actually presented and considered on a former
appeal.” (Emphasis supplied.) See also Two M. Dev. Corp. v. Mikos, 578 So.
2d 829, 830 (Fla. 2d DCA 1991). By reaffirming the principle articulated in
earlier decisions that the law of the case doctrine is limited to questions of
law actually presented and considered on a former appeal, U.S. Concrete
was consistent with prior cases from this Court. See, e.g., Greene, 384 So.
2d at 28; Strazzulla, 177 So. 2d at 3; Finston v. Finston, 160 Fla. 935, 37 So.
2d 423, 424 (1948). Additionally, the law of the case doctrine may foreclose
subsequent consideration of issues implicitly addressed or necessarily
considered by the appellate court’s decision. See Dade County Classroom
Teachers’ Ass’n v. Rubin, 238 So. 2d 284, 289 (Fla.1970); Dicks v. Jenne,
740 So. 2d 576, 578 (Fla. 4th DCA 1999).

342
[9] [10] A corollary of the law of the case doctrine is that a lower court is
not precluded from passing on issues that “have not necessarily been
determined and become law of the case.” Greene, 384 So. 2d at 27. As
stated in Wilder v. Punta Gorda State Bank, 100 Fla. 517, 129 So. 865, 866
(1930), the law of the case doctrine “has no applicability to, and is not
decisive of, points presented upon a second writ of error that were not
presented upon a former writ of error and consequently were not before the
appellate court for adjudication.”
Although the scope of the law of the case doctrine would appear to be
settled by this Court, several district courts have cited in dicta to this Court’s
prior decision in Airvac, Inc. v. Ranger Insurance Co., 330 So. 2d 467
(Fla.1976), for the proposition that the law of the case doctrine applies
where the issue could have been but was not raised.FN4 Indeed, based upon
this Court’s decisions in Airvac and U.S. Concrete, two appellate
commentators have termed the law of the case doctrine to be “a
misunderstood doctrine for which there is an irreconcilable conflict in
Florida law.” Raymond T. Elligett, Jr., & Charles P. Schropp, Law of the
Case, Fla. B.J. July-Aug.1985, at 23, 23.FN5
FN4. See, e.g., Ciffo v. Public Storage Mgmt., Inc., 622 So. 2d 1053,
1054 (Fla. 4th DCA 1993) (citing Airvac for the proposition that the law of
the case principle also applies where the issue could have been raised but
was not raised); Williams v. City of Minneola, 619 So. 2d 983, 987 (Fla. 5th
DCA 1993) (same); Federal Deposit Ins. Corp. v. Hemmerle, 592 So. 2d
1110, 1116 (Fla. 4th DCA 1991) (same); Valsecchi v. Proprietors Ins. Co.,
502 So. 2d 1310, 1311 (Fla. 3d DCA 1987) (same).
FN5. In a subsequent article, Elligett and Schopp elaborate on the
continuing confusion with the law of the case doctrine in the appellate
courts. See Raymond T. Elligett, Jr., & Charles P. Schropp, Law of the Case
Revisited, Fla. B.J. Mar. 1994, at 48.
Although this Court’s decision in Airvac may have caused some
confusion over the scope of the law of the case doctrine, an understanding of
the procedural posture of Airvac explains the result reached in that case. In
Airvac, the trial court had initially denied a request by the defendant to
amend his answer to include a fraudulent conveyance defense eighteen
months after answering and four days before trial. 330 So.2d at 468. The
first trial ended in a directed verdict for the defendant, and on appeal by the
plaintiff, the defendant failed to raise the trial court’s denial of its motion to

343
amend its pleadings on cross-appeal. See id. After the appellate court
reversed the directed verdict and remanded for a determination of several
factual issues, but before retrial, the defendant again sought to amend its
answer to include a fraudulent conveyance defense, which the trial court did
not permit. See id. at 469. After a verdict in favor of the plaintiff, the
defendant asserted as an issue on appeal error in the trial court failing to
allow the defendant to amend its answer. The appellate court reversed,
holding that the trial court erred in not allowing the defendant’s post-appeal
amendment. See id. This Court quashed the appellate court’s decision,
concluding that no error occurred and stating:
The record sub judice clearly shows that [defendant] had full knowledge
of the purported “fraudulent conveyance” prior to the initial trial; that it
sought by amendment to submit that issue to the court; that it had full
opportunity to perfect an appeal, assign as error or otherwise present the
issue of the denial of its motion to the appellate court on the initial appeal
but did not, thereby waiving any objection to the trial court’s rejection of its
amendment of the issue. Id. (emphasis supplied).
Despite the fact that the Court enunciated the law of the case doctrine in
concluding that on remand the defendant could not amend its complaint to
include a fraudulent conveyance defense, see id. at 469, this Court decided
Airvac on principles of waiver; i.e., the failure of a party to raise an issue on
appeal that was the subject of the trial court’s ruling. To the extent that
Airvac has been construed broadly to stand for the proposition that the law
of the case doctrine bars consideration of issues that were neither raised by
the parties nor decided by the appellate court in the prior appeal, it is in
conflict with our subsequently decided case of U.S. Concrete, which restricts
application of law of the case to issues that were decided in a prior appeal.
Therefore, we recede from Airvac to the extent it is inconsistent with U.S.
Concrete.
[11] In summary, the doctrines of the law of the case and res judicata
differ in two important ways. First, law of the case applies only to
proceedings within the same case, see Beverly Beach, 68 So. 2d at 607,
while res judicata applies to proceedings in different cases. See Strazzulla,
177 So. 2d at 3. Second, the law of the case doctrine is narrower in
application in that it bars consideration only of those legal issues that were
actually considered and decided in a former appeal, see U.S. Concrete, 437
So.2d at 1063, while res judicata bars relitigation in a subsequent cause of

344
action not only of claims raised, but also claims that could have been raised.
See Youngblood, 89 So. 2d at 505.
THIS CASE
[12] Because this case involves the issue of what preclusive effect the
prior appeal affirming the denial of summary judgment should have on the
trial court in the same case and in a subsequent appeal in the same case, the
doctrine of res judicata is inapplicable under these circumstances.
Accordingly, the Third District erred as a matter of law when it relied on the
doctrine of res judicata to preclude DOT from raising any aspect of its
workers’ compensation defense on remand after the first appeal.
Juliano argues, however, that the related doctrine of the law of the case
nevertheless precludes reconsideration of the standard of negligence.
Although DOT did not actually argue the appropriate standard of negligence
under the “unrelated works” exception in its first motion for summary
judgment, Juliano asserts that the Third District impliedly or necessarily
decided the issue in the first appeal of DOT’s motion for summary
judgment.
[13] We disagree. In this case, the scope of the issues in the first motion
for summary judgment and the non-final appeal of the denial of that motion
were limited to whether Juliano was required to specifically identify the
names of the negligent employees in order to be entitled to invoke the
“unrelated works” exception to section 440.11(1). Neither DOT’s first
summary judgment motion, the trial court’s order denying the summary
judgment motion, nor the issues raised or briefed by the parties on the first
appeal addressed the appropriate standard of negligence applicable to
supervisory employees who are sued under the “unrelated works” exception.
The issue regarding the appropriate standard of negligence was not before
the Third District when it affirmed the trial court’s order denying summary
judgment. FN6 Therefore, the matter of the appropriate standard of negligence
was neither necessarily nor implicitly decided in the first interlocutory
appeal. Compare Bakker v. First Federal Sav. & Loan Ass’n, 575 So. 2d
222, 224 n. * (Fla. 3d DCA 1991) (stating that because issue in original
motion for summary judgment involved whether a release barred the
plaintiff’s claims, the defendant’s subsequent motion for summary judgment
based on the merits of plaintiff’s claims was not precluded by the law of the
case doctrine), with Valsecchi, 502 So.2d at 1311 (holding that where trial
court initially concluded that Florida law applied in choice of law issue as

345
between Florida and North Carolina, and that decision was affirmed on
appeal, appellant was precluded from asserting that a different state’s law
applied on remand because the district court decided by implication that no
other state’s law was applicable). The law of the case doctrine thus did not
preclude either the trial court or the appellate court from considering the
issues raised in DOT’s second motion for summary judgment.
FN6. Indeed, there is a question as to whether the Third District in
Juliano I would have entertained an argument from DOT urging reversal of
the trial court’s order denying summary judgment based on a ground not
raised or argued below, i.e., that culpable negligence governed. Moreover, in
this case, the Airvac principle of waiver is inapplicable because the standard
of negligence under the unrelated works doctrine was not the subject of the
initial motion for summary judgment.
[14] [15] Further, where a previous appellate court has given no
explanation for its decision, a subsequent appellate court is not bound by the
law of the case unless a determination concerning the propriety of the trial
court’s order is necessarily inconsistent with every possible correct basis for
the earlier rulings of the appellate court. In Juliano I, 664 So. 2d at 77, the
Third District’s entire opinion simply stated “Affirmed. Holmes County
School Bd. v. Duffell, 651 So. 2d 1176 (Fla.1995).” One possible correct
basis for the Third District’s first decision affirming the denial of summary
judgment is that Holmes did not require Juliano to expressly name the fellow
employees who are alleged to be negligent. This determination is
independent of whether a culpable negligence standard was required for
supervisory employees engaged in unrelated works. In addition, “[t]here is
no prohibition on the presentation of successive motions for summary
judgment.” Bakker, 575 So. 2d at 224.
For all these reasons, on remand from the non-final appeal affirming the
denial of the motion for summary judgment on the ground that Juliano failed
to name specific employees under section 440.11(1), the law of the case
doctrine did not preclude either the trial court or the Third District from
considering whether section 440.11(1) requires that an employee
demonstrate that his or her supervisor was culpably negligent in order to be
entitled to relief pursuant to that section. Accordingly, we quash the decision
of the Third District and remand for proceedings consistent with this
opinion. FN7

346
FN7. Although both DOT and Juliano raise as an issue before this Court
whether culpable negligence is the proper standard under section 440.11(1),
because the Third District erroneously concluded that it was precluded from
addressing this issue based upon the doctrine of res judicata, the better
approach is for the Third District to consider this issue on remand.
It is so ordered.
WELLS, C.J., and SHAW, HARDING, ANSTEAD, LEWIS, and
QUINCE, JJ., concur.

Canty v. State
715 So. 2d 1033 (Fla. 1st DCA 1998)
LAWRENCE, Judge.
Alphonsia Canty (Canty) appeals the denial of his second motion filed
pursuant to Florida Rule of Criminal Procedure 3.850. We affirm.
A jury found Canty guilty of attempted first-degree premeditated murder.
Canty committed this crime on October 24, 1982, in Duval County, while he
was on parole from a ninety-nine year sentence for armed robbery. The trial
judge sentenced Canty, an habitual felon, to one hundred years, reciting the
fact that Canty shot his unarmed victim in the abdomen and, after the victim
fell to the ground, stood over the victim and shot the victim in the head at
point blank range. The judge retained jurisdiction for the first twenty-five
years of Canty’s sentence. Canty’s direct appeal was affirmed per curiam.
Canty v. State, 458 So. 2d 275 (Fla. 1st DCA 1984).
[1] [2] Canty filed the instant motion on October 1, 1997. The trial judge,
in a January 8, 1998 order, summarily denied the motion. Canty argues that
his sentence is illegal; he however admits on the face of his motion that he
raised this issue in his first postconviction motion. See Canty v. State, 636
So. 2d 509 (Fla. 1st DCA 1994) (affirming per curiam) (table). This issue
thus is barred by the law of the case: A per curiam decision even without
opinion establishes the law of the case on the same issue and facts which
were raised, or which could have been raised. White v. State, 651 So. 2d 726,
726 (Fla. 5th DCA 1995) (holding same), approved on other grounds, 666
So. 2d 895 (Fla.1996).
[3] Canty argues that his sentence is illegal under State v. Gray, 654 So.
2d 552 (Fla.1995) (holding that attempted felony murder is no longer a

347
crime in Florida). Canty was convicted of attempted premeditated, rather
than felony, murder; Gray thus is inapplicable to him.
[4] Canty argues that his lawyer was ineffective for failing to raise the
Gray issue. The Florida Supreme Court however tells us that “counsel’s
failure to raise a nonmeritorious issue is not ineffective assistance.”
Chandler v. Dugger, 634 So. 2d 1066, 1068 (Fla.1994).
The trial judge correctly denied Canty’s rule 3.850 motion. We affirm the
judge’s order.
ERVIN and JOANOS, JJ., concur.

Pompi v. City of Jacksonville


872 So. 2d 931 (Fla. 1st DCA 2004)
PADOVANO, J.
This is an appeal from an order denying a motion under rule 1.540(b) of
the Florida Rules of Civil Procedure. We conclude that the relief requested
in the motion is not barred by the doctrine of the law of the case, and that the
motion should have been granted on the ground of excusable neglect.
Therefore, we reverse.
The appellants were landowners and defendants in an eminent domain
proceeding in Jacksonville. A jury determined the value of their property,
and the trial judge rendered a final judgment on the verdict. The judgment
contains two stamps. One is marked “Filed” and it runs vertically along the
right edge of the page. This stamp is printed with narrow and closely spaced
characters, but it is legible by a careful examination. It states that the final
judgment was filed with the clerk of the court on January 24, 2003. The
other stamp is marked “Filed & Recorded” and it runs horizontally. This one
is printed in a recognizable typeface and it is easier to read. It reveals that
the final judgment was filed and recorded in the official records on January
30, 2003.
When the landowners decided to appeal the final judgment, their lawyers
in Jacksonville transferred the case to an appellate lawyer in the firm’s
Miami office. A secretary in the Jacksonville office called the clerk of the
court and was informed that the judgment had been rendered on January 30,
2003. She made a memorandum of this conversation and sent it along with a
copy of the judgment to appellate counsel in Miami. The appellate lawyer
noticed both stamps on the judgment, but she associated the date in the

348
secretary’s memorandum with the stamp marked “Filed & Recorded,” rather
than the one marked “Filed,” and, as a consequence, she calendared the
appeal time from January 30, 2003. She filed the notice of appeal on
February 28, 2003, in the mistaken belief that it was timely.
The City filed a motion in this court to dismiss the appeal for lack of
jurisdiction. Because the notice of appeal appeared to be late, we issued an
order to show cause. In response to this order, the landowners requested that
the time for appeal be recalculated from January 30, 2003. Alternatively,
they requested that the court dismiss the appeal without prejudice to their
right to pursue a rule 1.540(b) motion in the trial court. This court dismissed
the appeal without an opinion. See Pompi v. City of Jacksonville, 845 So. 2d
190 (Fla. 1st DCA 2003). The per curiam decision states in its entirety,
“Dismissed.”
Following the dismissal of the appeal, the landowners filed a motion in
the trial court under rule 1.540(b), contending that the final judgment should
be vacated on the ground of excusable neglect. The trial court found that the
stamp marked “Filed” plainly showed that the judgment had been rendered
on January 24, 2003. Based on this finding, the trial court concluded that the
landowners had proper notice of the final judgment and that they were not
entitled to relief under rule 1.540(b).
Additionally, the trial court determined that the relief requested in rule
1.540(b), was barred by the doctrine of the law of the case. The trial court
reasoned that further litigation of the issue was barred, because this court
had declined the opportunity to dismiss the appeal without prejudice. The
landowners filed a timely notice of appeal from the order denying the
motion to vacate.
[1] [2] At the outset we must determine whether the issue presented by
the appeal has been settled as the law of the case. When an appellate court
has decided an issue of law, the decision becomes the law of the case and it
is binding on the trial court in subsequent proceedings in the same case. See
Florida Dep’t of Transp. v. Juliano, 801 So. 2d 101 (Fla.2001); Brunner
Enters., Inc. v. Dep’t of Revenue, 452 So. 2d 550 (Fla.1984). With limited
exceptions, the decision is also binding on the appellate court in future
appeals in the same case. See Strazzulla v. Hendrick, 177 So. 2d 1
(Fla.1965).
The doctrine of the law of the case is based on the appellate court’s
decision, not its opinion. For this reason, the Florida courts have held that an

349
appellate decision that is accompanied only by the statement, “Affirmed,” is
the law of the case. See Canty v. State, 715 So. 2d 1033, 1033 (Fla. 1st DCA
1998); Comm’n on Ethics v. Sullivan, 430 So.2 d 928, 932 (Fla. 1st DCA
1983); New England Ins. Co. v. Int’l Bank of Miami, N.A., 537 So. 2d 1025,
1025-1026 (Fla. 3d DCA 1988); Russell v. Florida Ranch Lands, Inc., 441
So. 2d 190, 191 (Fla. 5th DCA 1983). An affirmance, even if unexplained, is
a decision on the merits of the case.
That is not true, however, with respect to a dismissal. By its nature, an
order dismissing an appeal signifies that the court did not reach the merits.
In the present case, the notice of appeal was not filed within the applicable
time limit, and we entered an order dismissing the appeal for lack of
appellate jurisdiction. This order resolves only the preliminary question
whether the court had judicial power to review the judgment. The order does
not address, much less adjudicate, the merits of the controversy, and it has
no impact on any remedy the parties may have in another forum.
[3] During the oral argument, counsel for the City pointed out that
appellate courts occasionally dismiss appeals without prejudice. That
observation is unfortunately correct. But in the context of an order
dismissing an appeal, the phrase “without prejudice” is unnecessary. It adds
nothing to that which is inherent in the order itself. The dismissal of an
appeal for lack of appellate jurisdiction is necessarily a decision made
without prejudice to the appellant’s right to pursue whatever remedies may
still exist. The doctrine of the law of the case assumes that an issue was
decided in the appellate court. Yet the very point of a dismissal for lack of
jurisdiction is that the court lacks judicial power to make a decision.
[4] We also reject the City’s alternative argument that the landowners
failed to establish excusable neglect. The case for a finding of excusable
neglect in this case is at least as strong as the one made in Williams v.
Roundtree, 464 So. 2d 1293 (Fla. 1st DCA 1985). There, we reversed the
denial of a rule 1.540(b) motion in part because a judicial assistant had given
counsel the incorrect date of rendition, and in part because the file stamp
was “more difficult to observe” than the subsequent recording stamp. Id. 464
So. 2d at 1294.
In the present case, the file stamp is legible on close examination, but the
form and placement of the stamp make it much less noticeable than the
recording stamp. Appellate counsel made a mistake in reading the two
stamps. She accepted full responsibility for her error and did not attempt to

350
blame the clerk of the court. We agree that the clerk bears no responsibility,
but the fact that a deputy court clerk made precisely the same mistake when
reporting the filing date on the telephone is at least some indication that
counsel’s error was justifiable. Under the circumstances, we think that the
error should be excused.
For these reasons, we conclude that the trial court erred in denying the
landowners’ rule 1.540(b) motion. The proper remedy on remand is to
vacate the original final judgment and render a judgment from which the
landowners may file a timely appeal.
Reversed.
BOOTH and BROWNING, JJ., concur.

Strazulla v. Hendrick
177 So. 2d 1 (Fla. 1965)
ROBERTS, Justice.
This cause is before the court on a petition for certiorari to review a
decision of the District Court of Appeal, Second District, on direct-conflict
grounds. We have assumed jurisdiction because of the apparent conflict,
generating confusion and uncertainty, in the decisions of this court referred
to hereafter concerning the doctrine of ‘the law of the case’; See Billingham
v. Thiefl, 109 So. 2d 763.
The decision which we here review, Hendrick v. Strazzulla,
Fla.App.1964, 168 So. 2d 156, was entered by the appellate court on the
second appeal of the case to it. On the first appeal, a verdict and judgment
for the defendants in a tort action arising out of an automobile collision was
affirmed. Hendrick v. Strazzulla, Fla.App.1960, 125 So. 2d 589. This court,
however, on direct-conflict certiorari quashed the judgment of affirmance on
a point no longer relevant (since the error did not recur on the second trial),
and the cause was remanded. Hendrick v. Strazzulla, Fla.1961, 135 So. 2d 1.
On the second trial, verdict and judgment were again for the defendants.
The plaintiff again appealed, challenging the propriety of the trial judge’s
charge to the jury on the provisions of Section 317.30, Fla.Stat.1959, F.S.A.,
which prohibits passing within 100 feet of an ‘intersection’, as defined in the
Act. The giving of antipass instructions under the statute had been assigned
as error on the first appeal; however, on that appeal the appellate court
without discussion declined to sustain the assignment, stating merely that

351
‘The remaining assignment has been examined and we find no error.’
Hendrick v. Strazzulla, supra, 125 So. 2d at p. 591. In the certiorari
proceedings in this court referred to above, the giving of anti-pass
instructions was not reviewed by this court.
On the second appeal, the Second District Court of Appeal reconsidered
the question of the propriety of the instruction concerning the anti-pass
statute, Section 317.30, supra, and held that the statute was inapplicable
under the facts and circumstances of the case and that the giving of the
instruction was reversible error. In refusing to sustain the defendant-
appellees’ contention that the doctrine of ‘the law of the case’ precluded a
reconsideration and reversal on this point, the appellate court said (168 So.
2d at page 161):
‘With this contention we cannot agree. The facts and circumstances on
this appeal are materially different from those on the former appeal; but even
if we now should find that we were in error on that point of the case, we
have the power to correct it.’
Cited in support of the above quoted statement was this court’s decision
in Beverly Beach Properties v. Nelson, Fla.1953, 68 So. 2d 604, 41
A.L.R.2d 1071.
It is strongly urged here by petitioners, who were the successful
defendants at the trial level, that the appellate court’s decision and the
Beverly Beach Properties decision are directly in conflict with prior
decisions of this court respecting the doctrine of the law of the case; and,
indeed, it does appear that there are two distinct lines of cases on this subject
which are sufficiently in conflict to invoke our direct-conflict jurisdiction.
Early in the jurisprudence of this state it was established that all points of
law adjudicated upon a former writ of error or appeal became ‘the law of the
case’ and that such points were ‘no longer open for discussion or
consideration’ in subsequent proceedings in the case. McKinnon v. Johnson,
1909, 57 Fla. 120, 48 So. 910, citing Wilson v. Fridenberg, 21 Fla. 386, and
other cases. In accord with this statement, either by the use of the same or by
the use of similar language, are the following decisions: Florida East Coast
Ry. v. Geiger, 1914, 66 Fla. 582, 64 So. 238; Utley v. City of St. Petersburg,
1935, 121 Fla. 268, 163 So. 523; Family Loan Co. v. Smetal Corporation,
1936, 123 Fla. 900, 169 So. 48, 49; United States Gypsum Co. v. Columbia
Casualty Co., 1936, 124 Fla. 633, 169 So. 532; Haddock v. State, 1940, 141
Fla. 132, 192 So. 802; Oates v. New York Life Ins. Co., 1940, 144 Fla. 744,

352
198 So. 681; Butler v. Major Holding Corporation, 1941, 145 Fla. 549, 200
So. 96; Westinghouse Electric Corp v. Carol Florida Corp., Fla.App.1963,
154 So. 2d 339.
The language of the Family Loan Co. v. Smetal decision, supra, 169 So.
48, 49, was even stronger than that used in the earlier cases. It was there
said:
‘Whether the holding in the opinion filed on the former appeal is sound
or erroneous as a precedent for other cases likely to arise and required to be
decided hereafter * * * is immaterial to the present consideration. This is so,
because the former opinion has conclusively settled the law of this case in so
far as it was duly put in issue for decision upon the assignments and cross-
assignments of error then presented. Therefore this court, on the present
appeal, is without authority to review or reverse what it has heretofore
decided as the law of this case, even if it were so inclined.’ (Emphasis
added.)
[1] Surprisingly, the Family Loan Company case was decided by this
court only a short time after the decision in McGregor v. Provident Trust Co.
of Philadelphia, 1935, 119 Fla. 718, 162 So. 323, was handed down. In the
McGregor case the court discussed at length the three principles of law-law
of the case, res judicata and stare decisis-which are adhered to by this court
and courts of other jurisdictions in order to lend stability to judicial decisions
and the jurisprudence of the state, as well as to avoid ‘piecemeal’ appeals
and to bring litigation to an end as expeditiously as possible. Respecting the
doctrine of ‘law of the case’, it was said:
‘By ‘law of the case’ is meant the principle that the questions of law
decided on appeal to a court of ultimate resort must govern the case in the
same court and the trial court, through all subsequent stages of the
proceedings, and will seldom be reconsidered or reversed, even though they
appear to have been erroneous.’ (Emphasis added.)
This particular statement made in the McGregor opinion-which clearly
implies authority to reconsider and reverse-has been quoted in the following
cases: Higbee v. Housing Authority of Jacksonville, 1940, 143 Fla. 560, 197
So. 479, 482; Lincoln Fire Insurance Co. v. Lilleback, 1938, 130 Fla. 635,
178 So. 394, 397; Leybourne v. Furlong, Fla.App.1964, 161 So. 2d 221. See
also Walker v. Atlantic Coastline Railroad Co., Fla.App.1960, 121 So. 2d
713, following the same rule and citing Lincoln Fire Insurance Co. v.
Lilleback, supra, in support thereof.

353
In 1953 the decision in Beverly Beach Properties v. Nelson, supra, 68 So.
2d 604, was rendered. In that case this court stated plainly that
‘We may change ‘the law of the case’ at any time before we lose
jurisdiction of a cause and will never hesitate to do so if we become
convinced, as we are in this instance, that our original pronouncement of the
law was erroneous and such ruling resulted in manifest injustice. In such a
situation a court of justice should never adopt a pertinacious attitude.’
There can be no doubt that the Beverly Beach Properties decision and the
line of cases following the McGregor decision, supra, are in conflict with the
holding in Family Loan Co. v. Smetal, supra, and the line of cases cited
above which are in accord with the decision in McKinnon v. Johnson, supra.
The Beverly Beach Properties decision, as well as the McGregor and similar
decisions, are, however, consistent with our decisions respecting the doctrine
of res judicata and stare decisis, see Wallace v. Luxmoore, 156 Fla. 725, 24
So. 2d 302, and with what appears to be the trend in other courts to
recognize that the administration of justice requires some flexibility in the
rule. See Johnson v. Cadillac Motor Car Co., 261 F. 878, 8 A.L.R. 1623;
Union Light, H. & P. Company v. Blackwell’s Adm’r. (Ky.), 291 S.W.2d
539, 87 A.L.R.2d 264; McGovern v. Kraus, 200 Wis. 64, 227 N.W. 300,
305, 67 A.L.R. 1381; Mangold v. Bacon, 237 Mo. 496, 141 S.W. 650, 654;
People v. Terry, Cal.1964, 390 P.2d 381; cases collected in the annotation in
87 A.L.R.2d, pp. 299-317.
In view of the apparent conflict, it is clear that the Beverly Beach
Properties decision must be held to have impliedly, if not expressly,
modified the earlier holding in Family Loan Co. v. Smetal, supra, and
similar decisions; and, insofar as these earlier decisions may be construed as
holding that an appellate court in this state is wholly without authority to
reconsider and reverse a previous ruling that is ‘the law of the case’, we
hereby expressly recede therefrom.
[2] [3] We think it should be made clear, however, that an appellate court
should reconsider a point of law previously decided on a former appeal only
as a matter of grace, and not as a matter of right; and that an exception to the
general rule binding the parties to ‘the law of the case’ at the retrial and at all
subsequent proceedings should not be made except in unusual circumstances
and for the most cogent reasons-and always, of course, only where ‘manifest
injustice’ will result from a strict and rigid adherence to the rule. Beverly
Beach Properties v. Nelson, supra.

354
The decision in the Beverly Beach Properties case exemplifies such an
exceptional case. There, a reconsideration and reversal of a previous
decision on a point of law was made mandatory be considerations of public
policy in order to give effect to the law of a sister state and judicial orders
regularly entered pursuant to such law, as required by the Full Faith and
Credit Clause of the federal constitution. Another clear example of a case in
which an exception to the general rule should be made results from an
intervening decision by a higher court contrary to the decision reached on
the former appeal, the correction of the error making unnecessary an appeal
to the higher court. See In re Reamer’s Estate, 1938, 331 Pa. 117, 200 A. 35,
119 A.L.R. 589; Louisville & N. R. Co. v. State (1914), 107 Miss. 597, 65
So. 881; People by and through Dept. of Public Works v. Lagiss,
Cal.App.1963, 30 Cal.Rptr. 852. Other examples which have appealed to
courts of other jurisdictions as proper exceptions to the general rule are set
out in the annotation in 87 A.L.R.2d, at pp. 299 et seq.
[4] But the exception to the rule should never be allowed when it would
amount to nothing more than a second appeal on a question determined on
the first appeal. Cf. Westinghouse Electric Corp. v. Carol Florida Corp.,
Fla.App.1963, 154 So. 2d 339. Judge Learned Hand in Higgins v. California
Prune & Apricot Grower, Inc., 3 F.2d 896, 898 (2 Cir. 1924), said that ‘the
‘law of the case’ does not rigidly bind a court to its former decisions, but is
only addressed to its good sense.’ And the same federal court, in Zdanok v.
Glidden Company etc., 327 F.2d 944 (2 Cir. 1964) some 40 years later
pointed out that
‘* * * where litigants have once bettled for the court’s decision, they
should neither be required, nor without good reason permitted, to battle for it
again. Perhaps the ‘good sense’ of which Judge Hand spoke comes down to
a calculus of the relative unseemliness of a court’s altering a legal ruling as
to the same litigants, with the danger that this may reflect only a change in
the membership of the tribunal, and of its applying one rule to one pair of
litigants but a different one to another pair identically situated. This explains
why a clear conviction of error on a point of law that is certain to recur, as in
this court’s well-known decision in Johnson v. Cadillac Motor Car Co., 261
F. 878, 8 A.L.R. 1623, (2 Cir. 1919) will prevail over ‘the law of the case’
whereas ‘mere doubt’ will not. In the former instance the court knows that
later litigants will be governed by a different rule; in the latter that is only a
possibility.’

355
[5] In the instant case, the appellate court followed the line of cases
stating the correct rule as to the doctrine of ‘the law of the case’ in this
jurisdiction-that is, that the court has the power to reconsider and correct an
erroneous ruling that has become ‘the law of the case’; and it is not
contended here that its decision in this respect was not justified, as an
exception to the general rule concerning the ‘law of the case’, under the
particular circumstances shown by this record.
Our conclusion makes it unnecessary to decide the questions posed by
the respondent as to the effect, on the doctrine of ‘the law of the case’, of
this court’s quashal of the judgment of affirmance relied upon by petitioners
as a basis for the application of the doctrine, as well as the question of
whether there was such a material difference in the facts adduced at the
second trial as to make the doctrine inapplicable.
Accordingly, the decision of the district court under review is correct and
the writ of certiorari heretofore issued should be and it is hereby discharged.
DREW, C. J., and THOMAS, THORNAL, O’CONNELL, CALDWELL,
and ERVIN, JJ., concur.

XVI. Rehearing

Florida Rules of Appellate Procedure

Rule 9.330. Rehearing; Clarification; Certification


(a) Time for Filing; Contents; Response. A motion for rehearing,
clarification, or certification may be filed within 15 days of an order or
within such other time set by the court. A motion for rehearing shall state
with particularity the points of law or fact that, in the opinion of the movant,
the court has overlooked or misapprehended in its decision, and shall not
present issues not previously raised in the proceeding. A motion for
clarification shall state with particularity the points of law or fact in the
court's decision that, in the opinion of the movant, are in need of
clarification. A response may be served within 10 days of service of the
motion. When a decision is entered without opinion, and a party believes
that a written opinion would provide a legitimate basis for supreme court
review, the motion may include a request that the court issue a written
opinion. If such a request is made by an attorney, it shall include the
following statement:

356
I express a belief, based upon a reasoned and studied professional
judgment, that a written opinion will provide a legitimate basis for supreme
court review because (state with specificity the reasons why the supreme
court would be likely to grant review if an opinion were written).
/s/ ____________
____________
Attorney for ____________
(name of party)
____________
(address and phone number)
Florida Bar No. ____________
(b) Limitation. A party shall not file more than 1 motion for rehearing or
for clarification of decision and 1 motion for certification with respect to a
particular decision.
(c) Exception; Bond Validation Proceedings. A motion for rehearing or
for clarification of a decision in proceedings for the validation of bonds or
certificates of indebtedness as provided by rule 9.030(a)(1)(B)(ii) may be
filed within 10 days of an order or within such other time set by the court.
A reply may be served within 5 days of service of the motion. The
mandate shall issue forthwith if a timely motion has not been filed. A timely
motion shall receive immediate consideration by the court and, if denied,
the mandate shall issue forthwith.
(d) Exception; Review of District Court Decisions. No motion for
rehearing or clarification may be filed in the supreme court addressing:
(1) the dismissal of an appeal that attempts to invoke the court's
mandatory jurisdiction under rule 9.030(a)(1)(A)(ii) when the appeal seeks
to review a decision of a district court of appeal without opinion, or
(2) the grant or denial of a request for the court to exercise its
discretion to review a decision described in rule 9.030(a)(2)(A), or
(3) the dismissal of a petition for an extraordinary writ described in
rule 9.030(a)(3) when such writ is used to seek review of a district court
decision without opinion.

357
A. Grounds

Whipple v. State
431 So. 2d 1011 (Fla. 2d DCA 1983)
PER CURIAM. Affirmed.
GRIMES, A.C.J., and SCHEB and RYDER, JJ., concur.
ON MOTION FOR REHEARING
PER CURIAM.
Appellant was convicted of drug trafficking and carrying a concealed
firearm. We affirmed the trial court and issued a per curiam affirmance,
commonly referred to as a PCA. Appellant has filed a motion for a rehearing
asking the court to reconsider points he previously briefed and argued.
Additionally, he complains of this court’s failure to write an opinion. The
motion for rehearing must be denied; however, we take the occasion to
address the function of a motion for rehearing and to explicate why counsel
cannot reasonably expect a written opinion from this court in all affirmances
of lower tribunals.
Appellant, on rehearing, focuses on two of the five points he previously
argued: first, that his own testimony established his defense of coercion and
duress as a matter of law; and second, that there was insufficient evidence of
his knowledge of the presence of the contraband to uphold the jury’s finding
that he possessed it. After reviewing the record and briefs and listening to
oral argument, we found no merit in appellant’s position on these or the
other points he raised. On the first point we concluded that while there was
evidence from which the jury could have found that appellant was coerced
or acted under duress, the jury did not err in rejecting that defense. The jury
was instructed as to the circumstances under which coercion and duress may
be a defense, and there was no attack on the correctness of the court’s
instruction. On the second point, we determined that the evidence was
sufficient for the jury to conclude that appellant had knowledge of the
contraband that was contained in his own plane which he was flying en route
from Jamaica to Florida.
[1] Having concluded that appellant’s conviction and sentence should be
affirmed, we decided that to write an opinion in this case would merely
serve to refute appellant’s arguments and would not show any conflict in law
which would merit an application for discretionary review to the supreme

358
court. Furthermore, an opinion would not have been of any significant
assistance to the bench or bar of this state.
At the outset of this discussion, we wish to point out that we have not
singled out appellant’s motion for rehearing as being any less meritorious
than many other similar motions filed in this court. Rather, we have taken
this occasion to speak generally to the bar, and particularly to those who
practice before this court, because our experience indicates that for many
practitioners a motion of this type has become a routine step in appellate
practice. In 1982, for example, motions for rehearing were filed in about one
out of every four cases we heard on the merits. With very few exceptions
they were denied.
Florida Rule of Appellate Procedure 9.330 governs motions for rehearing
or for clarification. Subsection (a) provides:
A motion for rehearing or for clarification of decision may be filed within
15 days of an order or within such other time set by the court. The motion
shall state with particularity the points of law or fact which the court has
overlooked or misapprehended. The motion shall not re-argue the merits of
the court’s order. A reply may be served within 10 days of service of the
motion.
From our experience, most motions for rehearing or clarification contain
a condensed version of all or some of the points previously argued.
Frequently, such motions urge the court to reconsider matters previously
considered, or to write an opinion to refute contentions of counsel.
Occasionally, the motion is misused by attorneys merely to express their
displeasure with the court’s judgment.
This leads to our first point: counsel should carefully and seriously
consider the necessity or desirability of asking the court to rehear a case.
Shortly after the district courts of appeal were established, Judge
Wigginton, in State v. Green, 105 So. 2d 817 (Fla. 1st DCA 1958), cert.
discharged, 112 So. 2d 571 (Fla.1959), addressed the function of a motion
(then petition) for rehearing and noted:
Certainly it is not the function of a petition for rehearing to furnish a
medium through which counsel may advise the court that they disagree with
its conclusion, to reargue matters already discussed in briefs and oral
argument and necessarily considered by the court, or to request the court to
change its mind as to a matter which has already received the careful

359
attention of the judges, or to further delay the termination of litigation. 105
So. 2d at 818, 819.
[2] We recommend that counsel carefully review Judge Wigginton’s well
articulated views in Green prior to filing a motion for rehearing. We
subscribe to those views and urge counsel to file a motion only where
careful analysis indicates a point of law or a fact which the court has
overlooked or misapprehended, or where clarification of a written opinion is
essential. Counsel should not use such motion as a vehicle to reargue the
merits of the court’s decision or to express displeasure with its judgment.
The second matter we address is the appellant’s contention, and one
frequently made by other counsel, that he cannot invoke the supreme court’s
discretionary review because of our failure to write an opinion.
[3] Appellant argues that by our having issued a PCA, we have thwarted
his right to obtain review in the supreme court. The fallacy of this contention
is that under our constitution he did not have such a “right” of review in the
supreme court in the first place. While Florida guarantees each litigant a
right of review, that guarantee does not extend to having the supreme court
hear each appeal. With Florida’s rapidly growing population and enormous
appellate caseload, if every litigant had a right of review in the supreme
court, the court would be so overwhelmed that it could not possibly focus on
the important cases.FN1 In fact, this was one of the principal reasons which
led to the creation of the district courts of appeal in 1957. When the supreme
court caseload reached 1,225 cases in 1955, the Judicial Council of Florida
proposed creation of the district courts of appeal. Their objective was to
restrict access to the Supreme Court of Florida in order to avoid double
appeals and to make appellate justice more readily available to litigants by
hearing appeals near the source.FN2 The legislature agreed, and on November
6, 1956, the voters overwhelmingly adopted an amendment to Article V of
the Florida Constitution providing for creation of the district courts of
appeal.FN3 It was originally intended that the district courts were to have final
appellate jurisdiction in most cases.FN4 However, this finality eroded as the
supreme court began looking to trial records, rather than to district court
holdings to establish conflict jurisdiction. FN5 The landmark case, of course,
was Foley v. Weaver Drugs, Inc., 177 So. 2d 221 (Fla.1965). As the finality
of the district courts continued to erode, the supreme court caseload, which
had decreased to 555 in 1959,FN6 became even more staggering with 2,676
cases filed in that court in fiscal year 1978-79.FN7 About half of those were

360
petitions for conflict certiorari. As a result of considerable study, an
amendment to Article V of the Florida Constitution was proposed to revise
the jurisdiction of the supreme court and district courts of appeal. The
amendment was adopted by the voters on March 11, 1980.FN8 The
amendment limits the supreme court’s mandatory review of district court of
appeal decisions to those declaring invalid a state statute or provision of the
state constitution. It provides for discretionary review of district court
decisions declaring valid state statutes or expressly construing a provision of
the state or federal constitution or affecting a class of constitutional or state
officers. Discretionary review is also given in those cases which expressly
and directly conflict with the decision of another district court of appeal or
the supreme court on the same question of law. The supreme court may no
longer search into the “record proper” to determine whether a district court
affirmance creates a necessary conflict.FN9 See, Jenkins v. State, 385 So. 2d
1356 (Fla.1980); Dodi Publishing Co. v. Editorial America, S.A., 385 So. 2d
1369 (Fla.1980). Thus, the district courts of appeal now have final appellate
jurisdiction in most cases, as was originally intended.
FN1. In 1982, for example, 13,924 appeals were filed and 13,976
disposed of in the district courts of appeal, while 1,465 cases were filed and
1,439 disposed of in the supreme court. Statistics for 1982 filings and
dispositions of the supreme court and district courts of appeal furnished by
Office of State Courts Administrator.
FN2. First Annual Report of the Judicial Council of Florida at 14 (June
30, 1955).
FN3. Fourth Annual Report of the Judicial Council of Florida, Ex. # # 3
(June 30, 1957).
FN4. See, e.g., Gaines, The Pending Amendment Will Enhance Regard
For Judicial System, 30 Fla.B.J. 142 (1956); Pennekamp, A Personal
Obligation, 30 Fla.B.J. 136 (1956); Ansin v. Thurston, 101 So. 2d 808, 810
(Fla.1958); Lake v. Lake, 103 So. 2d 639 (Fla.1958); Karlin v. City of Miami
Beach, 113 So. 2d 551 (Fla.1959).
FN5. See Note, The Erosion Of Final Jurisdiction In Florida’s Courts Of
Appeal, 21 U.Fla.L.Rev. 375 (1969).
FN6. Sixth Annual Report of Judicial Council of Florida, Ex. III (June
30, 1960).
FN7. Twenty-Fifth Annual Report of the Judicial Council of Florida at 22
(Feb. 1, 1980).

361
FN8. This amendment was actively supported by six of the seven justices
of the supreme court, the governor, the attorney general of Florida, The
Florida Bar, and by the Conferences of District Court, Circuit Court, and
County Court Judges. It was endorsed by many civic organizations, and
generally by the media of this state. See England, Hunter & Williams,
Constitutional Jurisdiction Of The Supreme Court Of Florida: 1980 Reform,
32 U.Fla.L.Rev. 147, 159-60 (1980).
FN9. Art. V, § 3(b)(3), Fla. Const., as amended 1980.
Under our present constitutional scheme, the district courts of appeal
engage primarily in the so-called error-correcting function to insure that
every litigant receives a fair trial. This frees the supreme court to discharge
its judicial policy-making function of clarifying the law and promulgating
new rules of law. These are matters which have an impact on those other
than the individual litigants. District courts can now certify matters “of great
public importance” or decisions “in direct conflict” with the decision of
another district court of appeal. Moreover, the district courts can also certify
trial court judgments which are “of great public importance” or which have
“a great effect on the proper administration of justice throughout the state”
as needing “immediate resolution by the supreme court.” FN10 As recently
pointed out by Justice Overton, “[t]he real effect of the amendment is the
substantial reduction in the amount of time the supreme court spends on its
screening responsibility,” relieving an overworked state supreme court. FN11
FN10. Art. V, § 3(b)(4) & (5), Fla. Const., as amended 1980.
FN11. Overton, Relieving An Overworked State Supreme Court: Florida
Asks Its Courts Of Appeal To Assist In Screening Cases, 66 Judicature 371,
374 (March 1983).
We recognize that if we decide a case without writing an opinion, the
losing party will be unable to obtain further review in the supreme court.
Therefore, we endeavor to write opinions in all cases in which we believe
that our decision can arguably be in conflict with a prior decision of the
supreme court or a district court of appeal. To be ever faithful to this
practice, there have been cases in which we first decided a case without
opinion but, upon rehearing, determined to write an opinion in order to
distinguish the cases relied on by the losing party. See, e.g., Fortman v.
Freedom Federal Savings and Loan Assoc., 403 So. 2d 985 (Fla. 2d DCA),
petition for review denied, 402 So. 2d 609 (Fla.1981). In Fortman we wrote
because we felt the cited cases were close enough on point that the losing

362
party could make a legitimate argument to the supreme court that we had
improperly distinguished them from the case at hand. The fact remains,
however, that most of the cases cited by zealous advocates as being in direct
conflict with our PCA decisions are simply not close enough to write about.
Appellant correctly observes that the decision of whether to write an
opinion rests with the assigned panel of three judges. However, he does this
court an injustice by saying that such decision is made “[u]pon the whim, or
caprice” of the assigned judge or panel. He characterizes this procedure as
“arbitrary, capricious, and irrational.”
We wish to apprise the bar of some of the problems of this court in
respect to requests by counsel for written opinions. In December 1982 a
tenth judge was invested in this court, and the supreme court has certified to
the legislature the need for two more judges during the current biennium. In
re Certificate of Judicial Manpower, 428 So. 2d 229 (Fla.1983). The
tremendous increase in this state’s population has led to an avalanche of
litigation. For example, in 1982 this court received 2,899 cases and disposed
of 2,789. This left 1,694 cases on file by the end of the year, an increase of
110 more than the previous year.
Each appeal is assigned to a panel of three judges before being disposed
of on the merits, so in 1982 each judge participated in the disposition of
approximately 900 cases. It is imperative to recognize the limitations on the
time judges have to write opinions. Commentators have suggested that a
three-judge panel be limited to a caseload of 300 decisions per year with a
participating judge expected to write about twenty-five opinions per year.
FN12
Judges on this court are each writing an average of sixty opinions per
year. For example, in 1982 the court wrote opinions in 559 cases, an
increase from 435 opinions written by the court in 1981. The fact is that
most appellate judges enjoy opinion writing and would prefer to spend a
much larger portion of their judicial time in researching and drafting than is
possible under existing caseloads. But appeals are filed here as a matter of
right, and in a court where each judge must participate in excess of 900
appeals per year, this luxury cannot be enjoyed.
FN12. Carrington, Meador & Rosenberg, Justice On Appeal, 143-45
(1976); Florida High School Activities Assoc. v. Bradshaw, 369 So. 2d 398,
401, n. 1 (Fla. 2d DCA 1979).
We write opinions in all reversals and remands and, as noted, in
affirmances where we believe an opinion will make a substantial

363
contribution to the law, or where necessary to disclose conflict or certify
questions. If it were not permissible to issue per curiam affirmances without
opinion, the processing of appeals would be materially delayed. Further, we
do not wish to write additional opinions to merely repeat well established
principles and further burden attorneys with their research.FN13 As it is, the
volumes of the Southern Reporter are already growing at an extremely rapid
rate, a rate which would be far greater if an opinion were written in every
case.
FN13. City of Miami Beach v. Poindexter, 96 Fla. 811, 119 So. 136
(1928).
We recognize that counsel would appreciate a reasoned opinion in
deference to the quality of their advocacy. However, we also realize that
“justice delayed is justice denied.” Over the past three years, we have more
strictly enforced time requirements and improved our administrative
procedures. We take pride in the fact that as of May 5, 1983, our inventory
of 1,477 cases included only forty, i.e., less than three percent of the cases
on file, where no decision has been issued within one year from the filing of
the notice of appeal. Today, the average processing time of cases orally
argued and disposed of on the merits is approximately nine months, and
those without oral argument are concluded in approximately eight months
from the notice of appeal.
We hope that our remarks will cause counsel to more carefully consider
the appropriateness of filing a motion for rehearing. Finally, we hope that
our discussion of the workings of the court will give members of the bar a
greater appreciation of our position when we affirm cases without writing an
opinion.
REHEARING DENIED.
GRIMES, A.C.J., and SCHEB and RYDER, JJ., concur.

Araujo v. State
452 So. 2d 54 (Fla. 3d DCA 1984)
DANIEL S. PEARSON, Judge.
The defendant appeals his convictions for trafficking in and unlawful sale
of marijuana. Although we find that the evidence against the defendant was
sufficient to sustain the convictions, it is hardly of such persuasive force to
render harmless the prosecutor’s closing argument comment on the

364
defendant’s failure to call a certain witness to corroborate the defendant’s
exculpatory explanation of his presence and actions at the time when and
place where the marijuana was seized by the police. Accordingly, we reverse
the convictions and remand the cause for a new trial.
[1] The applicable law is clear. An inference adverse to the defendant for
his failure to call witnesses is not permitted unless “it is shown that the
witnesses are peculiarly within the defendant’s power to produce and the
testimony of the witnesses would elucidate the transaction, that is, that the
witnesses are both available and competent.” Kindell v. State, 413 So.2d
1283, 1288 (Fla. 3d DCA 1982) (Pearson, J., concurring) (emphasis in text).
See Graves v. United States, 150 U.S. 118, 14 S.Ct. 40, 37 L.Ed. 1021
(1893); United States v. Blakemore, 489 F.2d 193 (6th Cir.1973); Gass v.
United States, 416 F.2d 767 (D.C.Cir.1969); Lane v. State, 352 So. 2d 1237
(Fla. 1st DCA 1977) (Rawls, J., dissenting), cert. denied, 362 So. 2d 1054
(Fla.1978).FN1 Cf. Buckrem v. State, 355 So. 2d 111 (Fla.1978) (both prongs
of rule satisfied); Jenkins v. State, 317 So. 2d 90 (Fla. 1st DCA 1975)
(same).
FN1. The majority in Lane acknowledged this well-settled rule, but found
its violation to be harmless error under the circumstances of the case.
In the present case, the missing witness was Frank DiCamillo, who was
originally joined as a defendant in the case. About two months before the
commencement of Araujo’s trial, DiCamillo entered a plea of guilty to the
charges, was sentenced to a term of imprisonment, and ordered to surrender
at a future date to begin serving his sentence. By the time of Araujo’s trial,
DiCamillo had failed to surrender as ordered and had become a fugitive
from justice. DiCamillo’s unavailability was, of course, well known to the
prosecutor.FN2
FN2. Indeed, all of the facts concerning DiCamillo’s fugitive status were
recited by the prosecutor to the trial judge at an early point in Araujo’s trial.
[2] Araujo testified on his own behalf. He explained that for a number of
years until early in 1979, he had worked with DiCamillo running an
automobile body shop. Thereafter, following brief employment at a similar
operation, Araujo decided to open his own body shop and requested
DiCamillo’s help. Their discussions were interrupted when Araujo took a
trip with his family. Upon Araujo’s return, he contacted DiCamillo to
resume the discussions. DiCamillo got back in touch with Araujo on May
11, 1979, and told Araujo that that evening he would be at the home of Mr.

365
Howard (known by Araujo to be a good friend of DiCamillo) and that
Araujo should meet DiCamillo there at 7:00 p.m. This, according to Araujo,
is what brought him to the Howard home on the night of the marijuana
raid.FN3,FN4
FN3. Araujo also explained that his act of opening the driveway gate to
allow a truck to enter was done at the request of DiCamillo and without
knowledge of the truck occupants’ purpose. The occupants’ purported
purpose was to pick up a delivery of marijuana; their real purpose was to
conduct a raid, seize the marijuana, and arrest everyone at the Howard
house. Before the night of the raid, the police had never met or talked to
Araujo, nor heard his name during the previous drug negotiations. It is clear,
however, that when Araujo came to the house and entered the garage area,
he saw the bales of marijuana and remained on the premises until the arrival
of the police.
FN4. Because this case must be retried, we note that the trial court
correctly overruled the prosecutor’s hearsay objections to Araujo recounting
what DiCamillo said to him. Such testimony was not offered to prove the
truth of the matter asserted by DiCamillo, but to show its effect upon Araujo
and is thus admissible as non-hearsay. See, e.g., United States v. Jackson,
621 F.2d 216 (5th Cir.1980) (defendant’s testimony concerning what he was
told about purpose of bank loan offered to prove lack of knowledge in
defense to making false entry); United States v. Herrera, 600 F.2d 502 (5th
Cir.1979) (defendant’s testimony concerning threats received admissible to
establish claim of duress); United States v. Rubin, 591 F.2d 278 (5th Cir.),
cert. denied, 444 U.S. 864, 100 S.Ct. 133, 62 L.Ed.2d 87 (1979)
(defendant’s testimony that he had been told by present and past union
presidents that union’s constitutional procedures for obtaining salary
increases did not have to be scrupulously followed admissible to establish
defense to charge of taking unauthorized salary increases); United States v.
Dunloy, 584 F.2d 6 (2d Cir.1978) (defendant’s testimony as to what he was
told as bearing upon whether he believed package contained valuables or
securities rather than narcotics); United States v. Wellendorf, 574 F.2d 1289
(5th Cir.1978) (defendant’s testimony concerning tax advice he received
admissible to establish good faith); United States v. Carter, 491 F.2d 625
(5th Cir.1974) (defendant’s testimony that cousin permitted him to use car
and did not tell him it was stolen); United States v. Anost, 356 F.2d 413 (7th
Cir.1966) (defendant’s testimony concerning contents of phone

366
conversations with co-defendant which bore upon his explanation for
possessing allegedly stolen goods).
In his closing argument to the jury, the prosecutor argued the
implausibility of Araujo’s testimony:
“You know the Defendant’s testimony that he gave from the stand. He
said he went to the Jeffrey Howard home to discuss a business deal with
Frank DeCamilla.FN5 Do you remember that?
FN5. DiCamillo’s name appears as DeCamilla throughout the transcript.
“You remember they were going to negotiate a business deal?
“Now, you know if you-if a fellow were going to discuss a business deal
with Frank DeCamilla, he certainly might go to DeCamilla’s place of
business. He might go to his home. He might have Frank DeCamilla come to
his own home. Why would he go to the Jeffrey Howard home?
“Frank DeCamilla doesn't have his books there. Why would he go there?
And, I think the Defendant might have suggested in answer to us in that
regard, ladies and gentleman [sic], because he said he went there-his own
testimony-because he expected-I even wrote this down-expected Frank
DeCamilla to provide some financing.”
Defense counsel argued in response:
“He says, is it reasonable they would meet at Jeffrey Howard’s house.
Nobody said it had to be reasonable, whether it is a likely thing to do. He
went to Mr. Howard’s house, because he is trying to close a deal with Mr.
DeCamilla. That is why he is at Mr. Howard’s house, because that is where
Mr. DeCamilla said to meet him.
“Are you going to put him in jail because Frank doesn’t do business in
his office?”
The prosecutor rebutted:
“If Frank DeCamilla is not just the sort of fellow that does business in his
home, maybe that is so, and maybe not. Where is Frank DeCamilla?
“We both called witnesses. I called everybody I wanted.”
The defendant objected and moved for a mistrial. The trial court
overruled his objection and denied his motion for mistrial on the ground,
urged by the State, that defense counsel’s argument had “opened the door”
to the State’s remark. The State makes the same contention here with far less
success.

367
[3] The prosecutor argued to the jury the unlikelihood of Araujo meeting
DiCamillo at Howard’s house to discuss business. Defense counsel’s
rejoinder was simply that whether it was unlikely or not, Araujo said that is
what he was doing there. Defense counsel’s comment neither suggests that
had DiCamillo been called, he would have corroborated the defendant’s
testimony, nor attempts to take advantage of DiCamillo’s absence. The
defendant’s argument cannot therefore be said to have provoked the State’s
impermissible reply. Where a defendant testifies as to what another told him
to establish his lack of knowledge or intent, the State, having argued that the
defendant’s story is unreasonable, will not be heard to say that the
defendant, by thereafter arguing the reasonableness of his testimony, ipso
facto, invites the State’s “missing witness” reply. Thus, no door was opened
to the State’s pernicious comment.
It being undisputed in this record that DiCamillo, although presumably
competent to testify, was not available,FN6 it was error to overrule the
defendant’s objection to the State’s comment, “Where is Frank DeCamillo?
We both called witnesses. I called everybody I wanted.” Because this
comment inescapably invited the jury to infer that the reason the defendant
did not call DiCamillo as a witness is that DeCamillo’s testimony would not
corroborate the defendant’s (despite the fact, which the prosecutor well
knew, that the defendant was wholly incapable of procuring DiCamillo’s
attendance as a witness) and because the evidence against the defendant is
far from overwhelming, the judgment is reversed and the cause remanded
for a new trial.
FN6. Additionally, there is no showing in this record that DiCamillo, had
he not been a fugitive, was peculiarly within the defendant’s power to
produce.
Reversed and remanded.
ON MOTION FOR REHEARING
The Assistant Attorney General, in a motion for rehearing filed on behalf
of the State, has the effrontery to tell us that in deciding this case we
“overlooked” or “misapprehended” a critical fact, namely, that DiCamillo,
whom we described in our opinion as a fugitive by virtue of not having
surrendered to begin the service of his sentence, was not a fugitive at all, but
was safely nestled in one of the State’s correctional institutions at the time of
Araujo’s trial.

368
The transcript of trial in this case reflects that the prosecutor informed the
court that “[s]hortly after he pled guilty, Mr. DiCamillo did not surrender
himself to commence the sentence after the plea was taken.” (emphasis
supplied). Assuming, generously, that the Assistant Attorney General
overlooked that statement when he first read it, surely its existence could not
have been missed when the Assistant Attorney General read the appellant’s
initial brief, which, referring to the transcript, informed his adversary that
“the prosecutor himself had earlier informed the court that Mr. DeCamillo
was a fugitive (2/25 Tr. 46). Given that status, he certainly was not available
to be called by the defense. The argument was improper and unfair.”
Appellant’s Initial Brief, p. 23.
Yet, in the face of this, the Assistant Attorney General filed a brief in
which he announced his acceptance of the appellant’s statement of the facts
“as a substantially accurate account of the proceedings below” (Appellee’s
Answer Brief, p. 1) and at no time took issue with the fact appearing in the
record, that is, that DiCamillo was a fugitive.
After appellant’s reply brief arrived and informed us once again that “it is
not fair reply to fault the defense for not calling a witness whom the
prosecutor knows is a fugitive and unavailable to testify” (Appellant’s Reply
Brief, p. 5), there was no protest from the Assistant Attorney General. And
when the fugitive status of DiCamillo was brought up time and again at oral
argument of this case, the fact remained unchallenged by the Assistant
Attorney General. We think, therefore, it is fair to say that any overlooking
and misapprehending that occurred in this case was done by the Assistant
Attorney General.
Our opinion in this case issued on May 22, 1984, nineteen days after oral
argument. We recounted therein as a fact that:
“By the time of Araujo’s trial, DiCamillo had failed to surrender as
ordered and had become a fugitive from justice. DiCamillo’s unavailability
was, of course, well known to the prosecutor.”
To the extent that the Assistant Attorney General had not previously
overlooked this fact, but had apprehended it as a fact of little importance, his
misapprehension was now brought home to him by our opinion.
[4] [5] The motion for rehearing alleges for the first time that the
transcript of the trial upon which we relied is inaccurate and that DiCamillo
was in fact not a fugitive. Quite apart from the fact that no effort to correct
the transcript has ever been made below, it appears from the court reporter’s

369
affidavit (appended to the appellant’s response to the State’s motion for
rehearing) that the transcript has been compared to the court reporter’s notes
and is absolutely accurate. It appears from an affidavit submitted by the
Department of Corrections that DiCamillo was a prisoner in State custody at
the time of Araujo’s trial and that, therefore, the prosecutor’s statement at
trial to the contrary was erroneous. However, since DiCamillo’s being in fact
not a fugitive does not change the impropriety of the prosecutor’s argument,
see n. 6,FN1 and his being a fugitive would serve only, as it did in our
opinion, to exacerbate the unfairness of the argument, the Assistant Attorney
General’s belated effort to save this case from reversal must fail.
FN1. Even if DiCamillo was within the reach of a subpoena, he was not
shown by this record to be available to either party, much less the defendant,
where, as here, he might, if called, have invoked his privilege against self-
incrimination and refused to testify. See United States v. Chapman, 435 F.2d
1245 (5th Cir.1970), cert. denied, 402 U.S. 912, 91 S.Ct. 1392, 28 L.Ed.2d
654 (1971). See also United States v. Seavers, 472 F.2d 607 (6th Cir.1973);
Maristany v. State, 414 So. 2d 206 (Fla. 3d DCA 1982) (a defendant’s plea
of guilty to a particular offense does not waive privilege of self-
incrimination as to other offenses arising out of same transaction).
[6] Accordingly, the State’s motion for rehearing is denied. For the
Assistant Attorney General’s benefit, we add these words, written by another
court on the occasion of being confronted with like behavior:
“This is not a case of a party caught in some manner by surprise .... For a
governmental agency best familiar with its own practice with respect to a
matter directly in issue, and now said to be of paramount importance, to
make no mention of the subject until after it had lost the case on another
ground, if deliberate, is a breach of duty to the court and, if inadvertent, is
still inexcusable. The Commission’s petition for rehearing raising this
allegedly vital point contains no mention of why it was first developed at
this late date, let alone any apology for so doing.” Carr v. Federal Trade
Commission 302 F.2d 688, 692 (1st Cir.1962) (on rehearing).
The same holds true here.
Motion for rehearing denied.

370
B. Time for Filing Motion

Orange Federal Sav. & Loan Ass’n v. Dykes


444 So. 2d 1152 (Fla. 5th DCA 1984)
COBB, Judge.
The appellant, Orange Federal Savings & Loan Association, has filed a
motion pursuant to Florida Rule of Appellate Procedure 9.300(a), seeking
leave to file a belated motion for rehearing and/or clarification. The basis for
the motion is the representation by the movant that it has been informed that
the United States Supreme Court erred in its de la Cuesta FN1 opinion in
regard to its designation of the effective date of a 1976 regulation
promulgated by the Federal Home Loan Bank Board. That date was critical
to the result of our opinion in this case.
FN1. Fidelity Federal Savings & Loan Assn. v. de la Cuesta, 458 U.S.
141, 102 S.Ct. 3014, 73 L.Ed.2d 664 (1982).
In our opinion in Orange Federal Savings & Loan Assn. v. Dykes, 433
So. 2d 642 (Fla. 5th DCA 1983), we relied, as did the appellant, on the
references in the slip opinion of the de la Cuesta 102 S.Ct. at 3019, 3031 (fn.
24), designating July 31, 1976, as the effective date of the due-on-sale clause
regulatory amendment now codified as 12 C.F.R. § 545.8-3(f). It now
appears that, prior to publication in the United States Reports, these
references were deleted when it was called to the attention of Justice
Blackmun by counsel for the Federal Home Bank Board that while the
provisions of section (g) of the new regulation became effective on July 31,
1976, those of section (f) became effective on June 8, 1976. This minor
disparity had no bearing on the outcome of the de la Cuesta opinion, but it
would have been determinative in our case and would have mandated a
contrary result. We dealt with a mortgage executed on July 30, 1976.
But there is no relief available to the movant in this Court at this time.
Our opinion was filed on June 23, 1983, and the mandate issued on July 11,
1983, during the January, 1983 term of this Court. See § 35.10, Fla.Stat.
(1981). The July, 1983 term began on July 12, 1983 and we are now in the
January, 1984 term. We have no jurisdiction over the cause at this time. See
State Farm Mutual Automobile Ins. Co. v. Judges of the District Court of
Appeal, Fifth District, 405 So.2d 980 (Fla. 5th DCA 1981). As the Florida
Supreme Court said in that case: “All things must have an end.” This case
has ended. The motion is denied for lack of subject matter jurisdiction.

371
DAUKSCH and SHARP, JJ., concur.

Portu v. State
654 So. 2d 169 (Fla. 3d DCA 1995)
PER CURIAM.
On January 18, 1995, this court filed an opinion in the captioned case.
Nine days later, on January 27, 1995, the state filed a copy of a notice to
invoke the discretionary jurisdiction of the Florida Supreme Court pursuant
to Florida Rule of Appellate Procedure 9.120 (1994). Following that event,
on February 2, 1995, the appellant’s timely motion for clarification arrived
and was ultimately determined and granted. In response to the appellant’s
motion, the state contended that its filing of the notice to activate the
discretionary jurisdiction of the Florida Supreme Court stripped this court of
the power to act on the timely filed motion for rehearing. On March 8, 1995,
this court granted and filed an opinion on the appellant’s motion for
clarification.
The state continues to assert that we did not have jurisdiction to so act,
citing to State v. Smulowitz, 486 So. 2d 587 (Fla.1986). We do not
understand that decision to so hold. Instead, it appears that our jurisdiction to
rule on timely filed motions does not expire until we render an order
disposing of the motion(s).
Florida Rule of Appellate Procedure 9.020(g) states that an order is
“rendered” when a signed, written order is filed with the clerk of the court. If
a motion for rehearing or clarification has been filed on that order, “the final
order shall not be deemed rendered with respect to any claim ... until the
filing of a signed, written order disposing of all such motions between such
parties.” Rule 9.020(g)(1). Here, the appellant appropriately filed within the
rehearing period; he had until February 2, 1995, fifteen days from the
January 18, 1995, rendering of the written opinion, to file a motion for
clarification or rehearing. Rule 9.330, Fla.R.App.P. (1994). Until this court
rendered a written order disposing of that timely filed motion, the state’s
time period for filing a notice to invoke discretionary jurisdiction of the
Supreme Court had not yet begun.FN1 The jurisdiction of the Florida
Supreme Court is invoked, according to rule 9.120(b), upon the filing of a
notice with the clerk of the district court within thirty days of rendition of
the order to be reviewed, in this case, within thirty days of the March 8,

372
1995 order granting the appellant’s motion for clarification. See also rule
9.120, Fla.R.App.P., Committee Notes (1993).
FN1. We remind the state that the face of orders and opinions issued by
this court clearly provides that the order or opinion is “NOT FINAL UNTIL
TIME EXPIRES TO FILE REHEARING MOTION AND, IF FILED,
DISPOSED OF.”
The State argues that our jurisdiction was cut off nine days from
rendition of the opinion, on January 25, 1995, when the State filed its notice
to invoke discretionary jurisdiction, six days prior to the appellant’s timely
filing of its motion for clarification pursuant to rule 9.330. Such an
interpretation of the rules of procedure would entirely preclude a party from
filing a timely motion for rehearing or clarification pursuant to rule 9.330 if
the opposing party merely files a notice of discretionary review immediately
after rendition of an order or opinion.FN2
FN2. Nevertheless, even if we are powerless to act as the state contends,
its position is no worse than it was prior to our original opinion. If the rule is
merely one of policy, it would seem desirable for a district court to continue
to act to clarify and rectify errors in an original opinion rather than cluttering
the Supreme Court with needless plain conflict.
The state’s notice to invoke the Supreme Court’s discretionary
jurisdiction was premature. Moreover, it affirmatively appears from the files
and records pending in this court that the Supreme Court of Florida has not
yet accepted jurisdiction of the state’s original petition.
For the foregoing reasons, the state’s motion filed March 19, 1995 FN3 is
denied.
FN3. This motion may be deemed to constitute an abandonment of the
state’s petition for or discretionary review in the Florida Supreme Court. It is
analogous to filing a notice of appeal and then seeking rehearing in the lower
tribunal. Of course, this is not our prerogative to decide.
SCHWARTZ, C.J., and NESBITT, J., concur.
BASKIN, J., concurs in the denial of the motion to reinstate original
opinion.

373
C. En Banc Review

Florida Rules of Appellate Procedure

Rule 9.331. Determination of Causes in a District Court of Appeal En


Banc
(a) En Banc Proceedings: Generally. A majority of the judges of a
district court of appeal participating may order that a proceeding pending
before the court be determined en banc. A district court of appeal en banc
shall consist of the judges in regular active service on the court. En banc
hearings and rehearing shall not be ordered unless the case is of
exceptional importance or unless necessary to maintain uniformity in the
court's decisions. The en banc decision shall be by a majority of the active
judges actually participating and voting on the case. In the event of a tie
vote, the panel decision of the district court shall stand as the decision of
the court. If there is no panel decision, a tie vote will affirm the trial court
decision.
(b) En Banc Proceedings by Divisions. If a district court of appeal
chooses to sit in subject-matter divisions as approved by the Supreme
Court, en banc determinations shall be limited to those regular active
judges within the division to which the case is assigned, unless the chief
judge determines that the case involves matters of general application and
that en banc determination should be made by all regular active judges.
However, in the absence of such determination by the chief judge, the full
court may determine by an affirmative vote of three-fifths of the active
judges that the case involves matters that should be heard and decided by
the full court, in which event en banc determination on the merits of the
case shall be made by an affirmative vote of a majority of the regular
active judges participating.
(c) Hearings En Banc. A hearing en banc may be ordered only by a
district court of appeal on its own motion. A party may not request an en
banc hearing. A motion seeking the hearing shall be stricken.
(d) Rehearings En Banc.
(1) Generally.A rehearing en banc may be ordered by a district
court of appeal on its own motion or on motion of a party. Within the time
prescribed by rule 9.330, a party may move for an en banc rehearing
solely on the grounds that the case is of exceptional importance or that
such consideration is necessary to maintain uniformity in the court's
decisions. A motion based on any other ground shall be stricken. A
response may be served within 10 days of service of the motion. A vote
will not be taken on the motion unless requested by a judge on the panel

374
that heard the proceeding, or by any judge in regular active service on the
court. Judges who did not sit on the panel are under no obligation to
consider the motion unless a vote is requested.
(2) Required Statement for Rehearing En Banc.A rehearing en
banc is an extraordinary proceeding. In every case the duty of counsel is
discharged without filing a motion for rehearing en banc unless one of the
grounds set forth in (1) is clearly met. If filed by an attorney, the motion
shall contain either or both of the following statements:
I express a belief, based on a reasoned and studied professional
judgment, that the panel decision is of exceptional importance.Or
I express a belief, based on a reasoned and studied professional
judgment, that the panel decision is contrary to the following decision(s) of
this court and that a consideration by the full court is necessary to
maintain uniformity of decisions in this court (citing specifically the case or
cases).
____________

____________
Attorney for ____________
(name of party)
____________
(address and phone number)
Florida Bar No. ____________
/s/ ____________
____________
Attorney for ____________
(name of party)
____________
(address and phone number)
Florida Bar No. ____________
(3) Disposition of Motion for Rehearing En Banc.A motion for
rehearing en banc shall be disposed of by order. If rehearing en banc is
granted, the court may limit the issues to be reheard, require the filing of

375
additional briefs, and may require additional argument.

XVII. Costs and Attorney Fees

Florida Rules of Appellate Procedure

Rule 9.400. Costs and Attorneys' Fees


(a) Costs. Costs shall be taxed in favor of the prevailing party unless the
court orders otherwise. Taxable costs shall include
(1) fees for filing and service of process;
(2) charges for preparation of the record;
(3) bond premiums; and
(4) other costs permitted by law.Costs shall be taxed by the lower
tribunal on motion served within 30 days after issuance of the mandate.
(b) Attorneys' Fees. A motion for attorneys' fees may be served not later
than the time for service of the reply brief and shall state the grounds on
which recovery is sought. The assessment of attorneys' fees may be
remanded to the lower tribunal. If attorneys' fees are assessed by the
court, the lower tribunal may enforce payment.
(c) Review. Review of orders rendered by the lower tribunal under this
rule shall be by motion filed in the court within 30 days of rendition.

A. Appellate Court Costs

American Medical Int’l, Inc. v. Scheller


484 So. 2d 593 (Fla. 4th DCA 1985)
DOWNEY, Judge.
After our mandate went down, appellant, American Medical
International, Inc. (AMI), filed a motion to tax costs involved in the appeal.
Among the items listed as taxable costs was a bond premium of $132,348
paid by appellants for a supersedeas bond. The trial court entered a cost
judgment for appellants, but only allowed $100,000 for the bond premium
although there was no dispute as to the amount paid by appellant or its

376
reasonableness. AMI has filed a motion to review that aspect of the cost
judgment pursuant to Rule 9.400(c), Florida Rules of Appellate Procedure.
[1] The pertinent portions of the appellate rule in question read:
Rule 9.400. Costs and Attorney’s Fees
(a) Costs. Costs shall be taxed in favor of the prevailing party unless the
court orders otherwise. Taxable costs shall include:
(1) fees for filing and service of process;
(2) charges for preparation of the record;
(3) bond premiums; and
(4) other costs permitted by law.
Costs shall be taxed by the lower tribunal on motion served within 30
days after issuance of the mandate.
Thus, it appears that the specified costs, including bond premiums, are
taxable in favor of the prevailing party “unless the court orders otherwise.”
For our purposes the operative word is “court.” Appellant contends that, in
the present appellate rules, the term “court” refers to the tribunal in which
the referenced appeal is pending. In applying the appellate rules to a case
that is pending in the supreme court to review a decision of a district court of
appeal, the “court” is the supreme court and the district court of appeal is the
“lower tribunal” or lower court. If a matter is pending in the district court of
appeal to review a decision of the circuit court, the “court” is the district
court of appeal and the “lower tribunal” is the circuit court, and so on down
the judicial ladder.
This analysis is made quite clear in Rule 9.020, Florida Rules of
Appellate Procedure, and the attendant committee note. The note in pertinent
part states:
The term “court” retains the substance of the term “court” defined in the
former rules, but has been modified to recognize the authority delegated to
the Chief Justice of the Supreme Court and to the chief judges of the district
courts of appeal. This definition was not intended to broaden the scope of
these rules in regard to the administrative responsibilities of the mentioned
judicial officers. The term is used in these rules to designate the court to
which a proceeding governed by these rules is taken. When Supreme Court
review of a district court of appeal decision is involved, the district court of
appeal is the “lower tribunal”. (Emphasis added.)

377
[2] [3] [4] [5] Thus, in subsection (a) of the rule on Costs and Attorney’s
Fees, where the rule states that costs shall be taxed in favor of the prevailing
party unless the court orders otherwise, it means that costs falling in the
named categories shall be taxed in favor of the prevailing party unless the
court where the appeal was pending orders otherwise. For example, in the
present case that means the district court of appeal-this court-must order
otherwise in order for a trial court to refuse to tax costs in the named
categories. However, the court actually determining the amount to be
assessed must be allowed some discretion in arriving at the amount,
otherwise anything the movant asks for would be automatically granted. The
lower tribunal making the assessment must determine that the amounts set
forth are necessary and reasonable. In making that determination a myriad of
factors may be considered. Here, the trial judge heard evidence that the bond
premium was so high because the appellant obtained extensions of time to
file its briefs amounting to 400 days. Appellant pointed out to the lower
court that appellee was not exactly without fault himself because he obtained
extensions of 253 days. Be that as it may, it was a difficult case from any
standpoint and the question presented is a classic one for the exercise of a
trial judge’s discretion.
Therefore, we hold that, while under Rule 9.400, Florida Rules of
Appellate Procedure, the trial judge may not disregard the items of cost
listed in the rule, he does have discretion as to the amount to be allowed
therefor, which discretion, of course, must be exercised in the light of
competent evidence contained in the record.
Accordingly, we find no reversible error demonstrated and thus affirm
the order appealed from.
GLICKSTEIN, J., and SMITH, RUPERT J., Associate Judge, concur.

Fleitman v. McPherson
704 So. 2d 587 (Fla. 1st DCA 1997)
JOANOS, Judge.
Petitioner sought a writ of certiorari to quash a trial court order denying
his motion to disqualify counsel Fred Johnson and his law firm, from
continuing to represent the defendants named in petitioner’s defamation
complaint. The underlying litigation involves a suit against a St. George
Island homeowners’ association. The petition was granted to the extent that

378
the order was quashed with respect to disqualification of Mr. Johnson at trial
of the underlying suit, because it appeared Mr. Johnson would be a featured
witness at the trial. The trial court’s order was affirmed with respect to the
denial of disqualification of Mr. Johnson’s law firm. See Fleitman v.
McPherson, 691 So. 2d 37 (Fla. 1st DCA 1997).
After issuance of the mandate, petitioner moved to tax costs pursuant to
Florida Rule of Appellate Procedure 9.400(a). On June 12, 1997, the trial
court issued an order, taking the motion to tax costs under advisement until
the case is ultimately decided. The trial court found that since this court’s
ruling affirmed in part and reversed in part, it could not be determined which
side prevailed. The trial court then relied upon Rogers v. State Board of
Medical Examiners, 364 So. 2d 1239 (Fla. 1st DCA 1978), for the
proposition that since there was an absence of an ability to identify the
prevailing party with a degree of certainty, it would be inappropriate to
assess costs. The court further found it must consider the results of the entire
litigation, not merely an interlocutory certiorari review, and thus it would be
inappropriate to tax costs at this time.
Petitioner cites Reliable Reprographics Blueprint & Supply, Inc. v.
Florida Mango Office Park, Inc., 645 So. 2d 1040 (Fla. 4th DCA 1994), for
the proposition that the order is an appealable non-final order, and not
prefatory to another order, because the trial court was not required to rule on
any other issues regarding petitioner’s motion to tax appellate costs.
Petitioner asserts he was the prevailing party on the significant issues on
appeal, he timely filed a motion to tax appellate costs, the trial court need
not take any additional action on the issues appealed, and the applicable
rules and case law preclude the trial court from deferring its ruling.
Respondent contends there has been no ruling either granting or denying
petitioner’s motion to tax appellate costs. Thus, there is nothing for this
court to review, other than whether the trial court abused its discretion in
taking the motion under advisement until a prevailing party could be
determined with certainty.
The trial court’s order is reversed and remanded with directions to make
a determination as to the prevailing party on the significant issues on appeal
in the interlocutory proceeding, for purposes of Florida Rule of Appellate
Procedure 9.400(a). Florida Rule of Appellate Procedure 9.400(a) provides:
(a) Costs. Costs shall be taxed in favor of the prevailing party unless the
court orders otherwise. Taxable costs shall include

379
(1) fees for filing and service of process;
(2) charges for preparation of the record;
(3) bond premiums; and
(4) other costs permitted by law.
Costs shall be taxed by the lower tribunal on motion served within 30
days after issuance of the mandate.
[1] [2] A trial court’s ruling on a motion to tax appellate costs is reviewed
to determine whether the trial court applied the correct law, and whether it
abused its discretion. See Florida Power & Light Co. v. Polackwich, 705 So.
2d 23 (Fla.1997). See also Moritz v. Hoyt Enterprises, Inc., 604 So. 2d 807,
810 (Fla.1992). “[T]he trial court must determine which party prevailed on
the significant issues on appeal, not at trial.” FP & L v. Polackwich, 705 So.
2d at 25.
The trial court’s reliance upon Rogers v. State Board of Medical
Examiners, 364 So. 2d 1239 (Fla. 1st DCA 1978), is misplaced. Rogers
involved an appeal from an order of an administrative agency, thus
implicating section 120.57(1)(b)6, which “requires an agency to preserve all
testimony in a proceeding and, on request of any party, to make a full or
partial transcript available ‘at no more than actual cost.’ ” 364 So. 2d at
1240. When Rogers failed to pay for preparation of the appellate record,
prepared in accordance with his directions, the agency moved for dismissal.
Rogers filed a reply to the motion to dismiss, taking the position that he was
not required to pay for preparation of the record on appeal, but attached a
check payable to the agency as payment of costs of the record. Id. In that
context, this court determined that the prevailing party could not be
determined at that juncture of the case.
[3] In contrast, in this case, petitioner prevailed on the question of
disqualification of the attorney, while respondents prevailed on the question
of disqualification of the law firm. Petitioner’s effort to disqualify Mr.
Johnson involved the attorney’s actions during discovery. Since no further
action need be taken by the trial court with respect to the attorney
disqualification issue, it appears the trial court failed to apply the correct
law, and abused its discretion, in declining to make a determination as to the
prevailing party for purposes of an award of appellate costs with respect to
the petition for certiorari. See FP & L v. Polackwich, and Stringer v. Katzell,
695 So. 2d 369 (Fla. 4th DCA 1997).

380
[4] The trial court also erred in delaying a decision as to costs pertaining
to an interlocutory appeal, based on a determination that the court must
consider the results of the entire litigation. In Stringer v. Katzell, the Fourth
District reversed a directed verdict in favor of appellees/defendants in a
medical malpractice action, and remanded the case for a new trial. Stringer
then moved for assessment of appellate costs pursuant to Florida Rule of
Appellate Procedure 9.400(a). The trial court awarded Stringer $7,397.68 in
appellate costs, and denied appellees’ motion for rehearing and stay of
execution.
Appellees then sought review of the order awarding appellate costs. The
court affirmed the award of appellate costs, holding in part: “Because
appellant was the prevailing party in the appeal, she was entitled to the
award of appellate costs under rule 9.400(a). Department of Labor and
Employment Sec., Div. Of Workers’ Compensation v. American Building
Maintenance, 449 So. 2d 932 (Fla. 1st DCA 1984).”
The cases relied upon by respondent are distinguishable, in that they
involve appellate attorney’s fees under Florida Rule of Appellate Procedure
9.400(b), rather than costs pursuant to rule 9.400(a). An attorney’s fee award
cannot be made until the prevailing party in the underlying litigation is
determined. However, the prevailing party under rule 9.400(a) is the party
who prevailed in the appellate proceeding that was the subject of the motion
to tax costs. Stringer.
Therefore, the trial court’s order which deferred ruling on the motion to
tax appellate costs is reversed and remanded, with directions to make a
determination as to the party who prevailed on the significant issue in the
petition for certiorari review, and for an award of costs pursuant to Florida
Rule of Appellate Procedure 9.400(a). See FP & L v. Polackwich, citing
Moritz v. Hoyt Enterprises, Inc., 604 So. 2d 807 (Fla.1992).
Reversed and remanded.
WOLF and VAN NORTWICK, JJ., concur.

Mulato v. Mulato
734 So. 2d 477 (Fla. 4th DCA 1999)
WARNER, J.
Pursuant to Florida Rule of Appellate Procedure 9.130(a)(4), this case
was brought as an appeal of three orders of the trial court entered after the

381
issuance of our mandate on the appeal from the original final judgment. See
Mulato v. Mulato, 705 So. 2d 57 (Fla. 4th DCA 1997), rev. denied, 717 So.
2d 535 (Fla.1998). Appellant challenges the trial court’s denial of her motion
to tax appellate costs FN1 and the trial court’s refusal to vacate the cost
judgment in favor of appellee entered after the original final judgment,
which has now been reversed. Appellee has cross-appealed the trial court’s
denial of his motion for trial attorney’s fees in connection with the original
final judgment. We affirm the denial of appellate costs because the motion to
tax costs in the trial court was not filed within thirty days of issuance of the
mandate as required by the Rule 9.400(a). We reverse the order denying the
motion to vacate, as the trial court did not determine who was the ultimate
prevailing party. We affirm the denial of attorney’s fees.
FN1. Review of an order on appellate costs is by motion under Florida
Rule of Appellate Procedure 9.400(c). However, had this been a timely filed
motion we could have entertained this as a non-final appeal because it was
combined with other points on appeal other than those concerning appellate
costs or attorney’s fees. See Starcher v. Starcher, 430 So. 2d 991, 993 (Fla.
4th DCA 1983). This is a very limited exception to the method of review
required under Rule 9.400(c). See id.
[1] [2] [3] [4] As to the motion for appellate costs, Rule 9.400(a)
requires that the motion be filed within 30 days after issuance of the
mandate. Appellant admits that she failed to file the motion for costs within
that time limitation but claims that appellee waived compliance. Although
the trial court has jurisdiction to conduct further trial court proceedings, the
assessment of appellate costs is actually part of the appellate function. The
fact-finding portion of that function is delegated to the trial court under Rule
9.400(a). Just as filing an appeal within thirty days of entry of a final order is
necessary to invoke the jurisdiction of the appellate court, the filing of a
motion for appellate costs within the time required by the rule is mandatory
to invoke the trial court’s jurisdiction to determine appellate fees and costs.
See Abraham v. S.N.W. Corp., 549 So. 2d 776, 777 (Fla. 4th DCA 1989);
Executive Motors, Inc. v. Strack, 527 So. 2d 286, 287 (Fla. 1st DCA 1988);
Thornburg v. Pursell, 476 So. 2d 323, 324 (Fla. 2d DCA 1985). Because the
subject time requirements are jurisdictional, they cannot be corrected, nor
can compliance with them be waived. Cf. Hawks v. Walker, 409 So. 2d 524,
525 (Fla. 5th DCA 1982). We thus affirm the order denying the appellate
costs because the trial court had no jurisdiction to award them.

382
[5] [6] [7] Appellant also filed a motion to vacate the previously entered
cost judgment in favor of appellee on the ground that after the appeal, she
was now the prevailing party. At the hearing on the motion to vacate, the
trial court denied the relief, explaining that because appellee was successful
in some of the claims he brought against appellant, even after the appeal, the
cost award should not be vacated. This was error. First, where the judgment
on which a cost judgment is predicated is reversed, the original cost
judgment also cannot stand. See Thornburg, 476 So. 2d at 324 . Second, the
trial court must re-assess who the prevailing party is on the significant issues
in litigation. In Moritz v. Hoyt Enterprises, Inc., 604 So. 2d 807, 810
(Fla.1992), the supreme court held that the test for determining the
prevailing party is whether the party prevailed on the significant issues of
the litigation. See also Prosperi v. Code, Inc., 626 So. 2d 1360, 1363
(Fla.1993). In the instant case, the trial court only determined that appellee
prevailed on some issues. It did not determine a prevailing party in line with
the direction of Moritz. We therefore remand to the trial court to determine
which party, if any, is the prevailing party and to assess the costs of the trial
litigation accordingly.
[8] As to the cross-appeal, we affirm because appellee has waived the
claim to trial fees that he raises in this appeal. In the original final judgment
the court denied appellee’s prayer for attorney’s fees. Appellee cross-
appealed that denial to this court, arguing he was entitled to attorney’s fees
on his civil theft claim. We affirmed. See Mulato, 705 So. 2d at 62. Some six
months after the completion of the appeal, appellee again moved for fees in
connection with another count of the complaint on which he prevailed, even
though he had made no argument in the prior appeal that he was entitled to
fees on this count. This claim is barred by the doctrine of law of the case .
See Brunner Enters., Inc. v. Department of Revenue, 452 So. 2d 550, 552
(Fla.1984); Adams v. Dickinson, 672 So. 2d 874, 876 (Fla. 5th DCA 1996).
Clearly, appellee litigated the issue of fees in the prior appeal. He could have
and should have brought this claim at that time. He does not get a second
bite at the apple.
Affirmed in part; reversed in part; and remanded for proceedings
consistent with this opinion.
POLEN and FARMER, JJ., concur.

383
B. Appellate Attorneys’ Fees

Boca Burger, Inc. v. Forum


912 So. 2d 581 (Fla. 2005)
CANTERO, J.
We review Forum v. Boca Burger, Inc., 788 So. 2d 1055 (Fla. 4th DCA
2001), which expressly and directly conflicts with Volpicella v. Volpicella,
136 So. 2d 231, 232 (Fla. 2d DCA 1962). The conflict concerns whether a
trial court has discretion to deny a plaintiff leave to amend the complaint
once before a responsive pleading is served. We have jurisdiction. See art.
V, § 3(b)(3), Fla. Const.; Boca Burger, Inc. v. Forum, 817 So. 2d 844
(Fla.2002) (granting review). While we grant jurisdiction to review this
issue, we also have authority to address other issues properly raised. See
Savoie v. State, 422 So. 2d 308, 310 (Fla.1982) (stating that once the
Supreme Court accepts jurisdiction over a case to resolve the legal issue in
conflict, it may, in its discretion, consider other issues properly raised and
argued before the Supreme Court). For the reasons we explain below, we
hold that a plaintiff has the absolute right to amend a complaint once as a
matter of course before a responsive pleading is served, and a trial court has
no discretion to deny such an amendment; that a defendant may assert an
affirmative defense, including the defense of federal preemption, in a motion
to dismiss; and that an appellate court may, in appropriate circumstances,
impose sanctions on an appellee or its lawyer for its frivolous defense of a
patently erroneous trial court order.
I. BACKGROUND AND FACTS
Respondent Richard Forum filed an action against petitioner Boca
Burger, Inc., seeking declaratory judgment, injunctive relief, and damages
under the Florida Deceptive and Unfair Trade Practices Act (the
“FDUTPA”). See §§ 501.201-.213, Fla. Stat. (2000). Boca Burger filed a
motion to dismiss with prejudice, arguing that Forum’s complaint failed to
state a cause of action and was otherwise preempted by federal and state
law. For purposes of the motion, Boca Burger assumed that Forum purported
to allege an FDUTPA violation in connection with the manufacture and
distribution of a food product, which is Boca Burger’s sole line of business.
Boca Burger argued that any such claim was preempted by Florida’s Food
Safety Act, sections 500.01-.601, Florida Statutes (2000), which grants the
Department of Agriculture and Consumer Services exclusive jurisdiction to

384
protect the public from injury flowing from intrastate commerce in food. It
further argued that any claim not preempted by Florida law was preempted
by the Federal Food, Drug and Cosmetic Act, which governs the
manufacture and distribution of food through interstate commerce. See 21
U.S.C. §§ 301-397 (2000).
Boca Burger scheduled a hearing on its motion to dismiss. On the
morning of the hearing, Forum, through new (though not substitute) counsel,
filed an amended complaint without leave of court. The amended complaint
asserted multiple claims for relief arising from the alleged “intentional and
negligent misrepresentations and omissions by [Boca Burger] concerning the
nature and ingredients of their purportedly ‘all natural’ vegetarian
hamburger patty substitute.” Forum accused Boca Burger of omitting
ingredients from the product label. Based on these facts, Forum alleged both
statutory and common law violations.FN1 The district court summarized what
transpired at the hearing:
FN1. The amended complaint alleges causes of action under the federal
Racketeer Influenced and Corrupt Organizations Act (“RICO”), 18 U.S.C. §
1962(a)-(c) (2000), the FDUTPA, common law fraud and deceit, breach of
contract, and breach of warranty.
The amended complaint was filed over the names of three sets of lawyers
who had not appeared on the original complaint, but one had the same
address as counsel who filed the original complaint. At the hearing the trial
judge first mentioned both the filing of the amended complaint and the fact
that the lawyer appearing at the hearing on behalf of plaintiff was not the
lawyer who had signed the original complaint. Counsel responded that he
had filed a notice of appearance that morning along with the amended
complaint. The trial judge replied that he had a “fundamental problem” with
both aspects: the complaint had been amended without leave of court, and
the lawyer at the hearing was not the lawyer who had signed the original
complaint on plaintiff’s behalf.
The trial judge asked defense counsel if they had any comments on both
matters raised by the court. Defense counsel first addressed the appearance
of different attorneys who were now representing plaintiff. He stated that the
lawyer who had filed the original pleading had his own firm, and that one of
the lawyers signing the amended pleading had a different firm but at the
same address as the first. He admitted that he had communications before
the hearing with one of the new lawyers listed on the amended pleading but

385
added that “he’s never defined exactly-to my knowledge-his role....” The
tone and tenor of his remarks was to hint that there was some impropriety in
these new lawyers appearing on plaintiff’s behalf, notably failing to say
what it might be. Defense counsel failed to acknowledge that the rules
expressly permit the appearance of additional attorneys for a party without
leave of court. The judge again suggested disapproval of the appearance but
decided to “skip that one for a minute.”
Turning to the issue of amending the complaint without leave of court,
the trial judge next noted that the hearing had been set for nearly four weeks
and that the amended complaint had been filed only on the day of the
hearing. Plaintiff’s counsel pointed out that the rules permit an amendment
without leave of court before a responsive pleading is filed and that the
pending motion to dismiss was not a responsive pleading. Again the court
turned to defense counsel.
Defense counsel argued case authority from a federal trial court in
Michigan that, as he put it, “suggests that attempting to amend a complaint
while a motion to dismiss is pending is procedurally improper and causes
prejudice to the party seeking to dismiss the complaint.” He also cited a
bankruptcy case which he claimed was to the same effect. He then added
that our decision in Life General Security Insurance Co. v. Horal, 667 So.2d
967 (Fla. 4th DCA 1996), “suggests that leave to amend or attempting to
amend a complaint at the eleventh hour would cause prejudice to the
opposing party, and it is within the discretion of the trial court to deny.”
While conceding that “the rule is what it is,” he went on to contend “it is
within the court’s discretion to deny leave to amend.”
....
The trial judge thereupon announced that he would proceed with the
hearing for argument on the pending motion to dismiss-but only as to the
initial complaint. The judge added that he would not “recognize” the
amended complaint “as filed.” The outcome was that the court granted the
motion to dismiss the original complaint with prejudice, thereby refusing to
permit any amendments to the pleading, and finding that the complaint
raised claims that are “pre-empted” by federal and Florida law. Forum, 788
So.2d at 1057-59 (footnotes omitted).
On appeal, the Fourth District reversed. The court did not “[overlook]
defendant’s argument that plaintiff’s claims [were] pre-empted by federal
law,” but concluded that the “essential problem with that argument is that it

386
was being raised at the wrong time, under the auspices of the wrong
motion.” Id. at 1061. According to the district court, Boca Burger’s
preemption argument constituted an avoidance, not a real defense, that
should have been pled only as an affirmative defense and resolved on
motion for summary judgment.
The district court also reversed the trial court’s decision to dismiss the
complaint, holding that Forum could amend his complaint by right, and that
the amended complaint superseded the initial complaint. According to the
district court, Boca Burger’s counsel misled the trial court into believing that
it had discretion to refuse Forum’s amended complaint. Id. at 1059.
Concluding that counsel could not have made such an argument in good
faith at either the trial or appellate levels, the district court applied the
recently amended attorney’s fees statute and imposed trial and appellate
court sanctions against Boca Burger’s counsel. See § 57.105, Fla. Stat.
(2000). We granted review. Boca Burger, 817 So. 2d at 844.
II. ANALYSIS
We focus on three issues discussed in the district court’s opinion: (A)
whether a trial court has any discretion to deny a plaintiff the right to amend
the complaint once before an answer is served; (B) whether a defendant may
assert federal preemption in a motion to dismiss; and (C) whether the district
court properly imposed sanctions against Boca Burger’s counsel for both its
trial court and appellate court conduct.
A. The Right to Amend a Complaint
[1] We first address the issue on which conflict exists: whether a trial
court has any discretion to deny a plaintiff’s first amendment to the
complaint before a responsive pleading is served. We hold that the plain
language of the rule grants trial courts no such discretion.
The Fourth District held that it is “incontrovertible that plaintiff had
every right under the rule-so early in the case-to amend his complaint
without leave of court, and therefore the legal sufficiency of the original
complaint was clearly moot.” 788 So. 2d at 1059. The court then discussed
other cases:
There were some earlier cases outside this district finding a residual
discretion in the trial judge to deny leave to amend when sought by a party
before the filing of a responsive pleading, but even they go on to hold that it
is an abuse of discretion to deny the amendment unless as a matter of law
plaintiff clearly could never state a cause of action.

387
Id. at 1059. The court cited Volpicella, 136 So. 2d at 232, as one of those
cases. In Volpicella, the trial court granted the defendant’s motion to dismiss
without explaining the grounds for dismissal and without granting plaintiff
leave to amend. The plaintiff appealed, arguing that the court should have
granted leave to amend. The Second District reversed, holding that the trial
court abused its discretion in denying leave to amend. In doing so, the court
stated:
Under [rule 1.190], a party may amend his pleading once as a matter of
course at any time before a responsive pleading is served. This rule has not
yet been construed as depriving a trial court of discretion to withhold leave
to amend a pleading to which no response has been served. The rule does
however indicate that a denial of leave to amend in such a case amounts to
an abuse of discretion unless a complaint is clearly not amendable. Id.FN2
FN2. The Fourth District also cited Nenow v. Ceilings & Specialties, Inc.,
151 So. 2d 28 (Fla. 2d DCA 1963), as another of the “earlier cases.” Nenow
followed Volpicella’s holding that the right to amend is not absolute. It
interpreted Volpicella as holding that the right terminates upon a judgment
of dismissal and that subsequent amendment is committed to the trial court’s
discretion. Id. at 30-31. In this case, Forum clearly amended the complaint
before dismissal, and the Fourth District did not address whether the
absolute right to amend the complaint terminates with dismissal. Neither do
we.
In Forum, the Fourth District “doubt[ed] the correctness of the second
district’s assertion of residual discretion. Rule 1.190(a) states a rule, not a
discretion, as regards to amending before a responsive pleading is filed.” 788
So.2d at 1059. Thus, the two cases conflict on whether the rule grants trial
courts any discretion to deny a plaintiff’s first amendment to the complaint
before an answer is served. We agree with the Fourth District, as we explain
below.
Rule 1.190(a) of the Florida Rules of Civil Procedure permits the
amendment of a pleading “once as a matter of course at any time before a
responsive pleading is served.” Fla. R. Civ. P. 1.190(a). The rule reads in its
entirety:
A party may amend a pleading once as a matter of course at any time
before a responsive pleading is served or, if the pleading is one to which no
responsive pleading is permitted and the action has not been placed on the
trial calendar, may so amend it at any time within 20 days after it is served.

388
Otherwise a party may amend a pleading only by leave of court or by written
consent of the adverse party. If a party files a motion to amend a pleading,
the party shall attach the proposed amended pleading to the motion. Leave of
court shall be given freely when justice so requires. A party shall plead in
response to an amended pleading within 10 days after service of the
amended pleading unless the court otherwise orders.
Fla. R. Civ. P. 1.190(a).FN3 Thus, by its terms the rule provides for
amendment as of right (first sentence) and amendment by agreement or
leave of court (second sentence), depending on the circumstances.
FN3. The third sentence of the rule was added in 2003 and is not at issue
in this case.
[2] Several courts have recognized that the first sentence of the rule
grants plaintiffs an automatic right to amend the complaint once before a
responsive pleading is served. See, e.g., Vanderberg v. Rios, 798 So. 2d 806,
807 (Fla. 4th DCA 2001); Fusilier v. Markov, 676 So. 2d 1053, 1054 (Fla.
3d DCA 1996); Posey v. Magill, 530 So. 2d 985, 986 (Fla. 1st DCA 1988);
Abston v. Bryan, 519 So. 2d 1125, 1127 (Fla. 5th DCA 1988); Fla. Power &
Light Co. v. Sys. Council U-4 of Int’l Bhd. of Elec. Workers, 307 So. 2d 189,
191 (Fla. 4th DCA 1975); Bryant v. Small, 271 So. 2d 808, 809 (Fla. 3d
DCA 1973). Moreover, a motion to dismiss is not a “responsive pleading”
because it is not a “pleading” under the rules. See Fla. R. Civ. P. 1.100(a)
(designating permissible pleadings and providing that “[n]o other pleadings
shall be allowed”); see also Vanderberg, 798 So. 2d at 807 (noting that a
motion to dismiss is not a pleading) (citing Forum, 788 So. 2d at 1057).
Therefore, the filing of a motion to dismiss does not terminate a plaintiff’s
absolute right to amend the complaint “once as a matter of course.”
Thus, the district court was correct in determining that Forum could
amend his complaint without leave of court, regardless of the pending
motion to dismiss, and that the trial court erred in refusing to recognize the
amended complaint. Under the plain language of the first sentence of rule
1.190(a), a plaintiff has an absolute right to amend the complaint before a
responsive pleading is served. Because Boca Burger had not served its
answer, and had only filed a motion to dismiss, Forum had the right to file
an amended complaint, even if that amendment was filed on the day of-or
even just before-the hearing on Boca Burger’s motion to dismiss the original
complaint. The rule clearly grants a plaintiff one free amendment to perfect
the complaint before an answer is served.

389
[3] A judge’s discretion to deny amendment of a complaint arises only
after the defendant files an answer or if the plaintiff already has exercised
the right to amend once. At that time, the second and fourth sentences of rule
1.190(a) apply: “ Otherwise a party may amend a pleading only by leave of
court or by written consent of the adverse party.... Leave of court shall be
given freely when justice so requires.” (Emphasis added.)
Although Volpicella implied that a trial court may deny leave to amend
where the complaint is clearly not amendable, a court only has such
discretion under the second sentence of the rule, not under the first. The
cases that have recognized a court’s discretion to deny amendment in those
circumstances concerned either a plaintiff’s second (or subsequent)
amendment or an amendment requested after the answer was filed. See, e.g.,
Fla. Nat’l Org. for Women, Inc. v. State, 832 So. 2d 911, 915 (Fla. 1st DCA
2002) (holding that where the plaintiff had amended once before a
responsive pleading had been served and once again after the defendant filed
an answer, the trial court abused discretion in refusing leave to amend the
second amended complaint); Kohn v. City of Miami Beach, 611 So. 2d 538,
539 (Fla. 3d DCA 1992) (holding that the trial court did not abuse its
discretion where the plaintiff failed in four attempts to cure the defects in the
complaint); Bouldin v. Okaloosa County, 580 So. 2d 205, 207 (Fla. 1st DCA
1991) (stating that when a party seeks to amend a complaint after a
responsive pleading has been served, leave should be granted unless the
court finds a clear abuse of the privilege to amend or the complaint is clearly
not amendable); see also Dimick v. Ray, 774 So. 2d 830, 835 (Fla. 4th DCA
2000) (holding that the trial court abused its discretion in denying motion for
leave to amend plaintiff’s first amended complaint); Adams v. Knabb
Turpentine Co., 435 So. 2d 944, 946 (Fla. 1st DCA 1983) (same); Highlands
County Sch. Bd. v. K.D. Hedin Constr., Inc., 382 So. 2d 90, 91 (Fla. 2d DCA
1980) (same).
[4] As the Fourth District held, a court has no discretion to deny an
amendment under the first sentence of the rule. A defendant may contest the
legal viability of a first amended complaint by moving to dismiss the
amended complaint, not by contesting the plaintiff’s right to amend. We
disapprove Volpicella to the extent it holds that a trial court retains any
discretion to deny an amendment under such circumstances-regardless of
whether the plaintiff simply files an amended complaint or requests leave of
court to file one.

390
B. Pleading Preemption in a Motion to Dismiss
[5] We next discuss whether the defense of federal preemption may
properly be asserted in a motion to dismiss. The district court held that Boca
Burger could only plead the preemption defense as an affirmative defense,
and therefore the issue could only be resolved on motion for summary
judgment. 788 So.2d at 1062. On this point we disagree. Florida courts,
including this Court, have held that the issue of federal preemption is a
question of subject matter jurisdiction. See Jacobs Wind Elec. Co. v. Dep’t
of Transp., 626 So. 2d 1333, 1335 (Fla.1993); Bankers Risk Mgmt. Servs.,
Inc. v. Av-Med Managed Care, Inc., 697 So. 2d 158, 160 (Fla. 2d DCA
1997); Fla. Auto. Dealers Indus. Benefit Trust v. Small, 592 So. 2d 1179,
1183 (Fla. 1st DCA 1992). Lack of subject matter jurisdiction may be
properly asserted in a motion to dismiss. See Fla. R. Civ. P. 1.140(b). Florida
courts regularly review arguments concerning subject matter jurisdiction on
motions to dismiss. See, e.g., Bradshaw v. Ultra-Tech Enters., Inc., 747 So.
2d 1008, 1009 (Fla. 2d DCA 1999) (affirming dismissal of complaint based
on ERISA preemption of state law); Doe v. Am. Online, Inc., 718 So. 2d
385, 388 (Fla. 4th DCA 1998) (rejecting the argument that a federal
preemption defense constituted an affirmative defense that should have been
raised in an answer, not on a motion to dismiss); Bankers, 697 So. 2d at 160
(addressing an issue raised in defendant’s motion to dismiss regarding
federal preemption of plaintiff’s claims).
[6] [7] Therefore, we reject the Fourth District’s holding in this case that
Boca Burger could not raise its preemption defense in a motion to dismiss. A
defendant may, at its option, raise any affirmative defense, including the
defense of federal preemption, in a motion to dismiss. Of course, when a
defendant asserts such a defense in a motion to dismiss, a trial court must
determine the issue as a matter of law based only on the well-pleaded
allegations in the complaint, assuming the truth of the facts asserted. See
Hanft v. Phelan, 488 So. 2d 531, 532 n. 1 (Fla.1986).
C. The Imposition of Sanctions for Defending a Trial Court Order
[8] [9] The district court imposed sanctions on Boca Burger both for its
conduct in the trial court and its conduct on appeal. The trial court had not
imposed sanctions for such actions, and no authority exists for an appellate
court’s imposition of sanctions for conduct occurring in the trial court.
“Where the trial court has failed to make ... findings [under section 57.105],
[the appellate court is] without authority to do so in the first instance on

391
appeal.” Kurzweil v. Larkin Hosp. Operating Co., 684 So. 2d 901, 903 (Fla.
3d DCA 1996). If the district court was concerned with counsel’s conduct in
the trial court, the proper procedure would have been to remand for the
circuit court to allow the trial court to determine for itself whether to impose
sanctions.FN4
FN4. This approach is consistent with the rationale underlying the “abuse
of discretion” standard that governs appellate review of a trial court’s
decision regarding sanctions. See, e.g., Mercer v. Raine, 443 So. 2d 944,
945-46 (Fla.1983) (holding that a trial court’s order granting sanctions
should be reviewed using an abuse of discretion standard). That rationale
emphasizes “the superior vantage point of the trial judge” as “essential to the
just and proper application of procedural rules,” id., and applies equally to
our holding that appellate courts do not have authority to award sanctions for
conduct that occurred in the trial court. “[I]t is the responsibility of the trial
court, and not the appellate courts, to manage and control the trial process,
including the application of sanctions for serious abuses.” Tramel v. Bass,
672 So. 2d 78, 83 (Fla. 1st DCA 1996).
Regarding sanctions for counsel’s conduct on appeal, however, we hold
that a district court may, in appropriate circumstances, impose sanctions for
counsel’s defense of a patently erroneous order. As we explain below, (1)
under the new version of section 57.105, Florida Statutes, an appellee as
well as an appellant may be subject to sanctions; (2) allowing appellate
courts to impose sanctions on appellees for frivolous defense of trial court
orders will not chill representation, but instead will emphasize that counsels’
obligations as officers of the court override their obligations to zealously
represent their clients; and (3) because the district court’s opinion addressed
almost exclusively counsel’s conduct in the trial court, we cannot determine
to what extent, if any, the court intended to impose sanctions for conduct
that occurred in the appellate court.
1. An appellee may be subject to sanctions
The petitioner argues that an appellee-who by definition is defending a
trial court’s order-cannot be sanctioned for asserting a frivolous defense on
appeal. Some Florida courts agree with the petitioner’s position. See State
Dep’t of Highway Safety & Motor Vehicles v. Salter, 710 So. 2d 1039, 1041
(Fla. 2d DCA 1998) (holding that because the judgment of a trial court
carries a presumption of correctness, defense of that judgment necessarily
presents a justiciable issue); Coral Springs Roofing Co. v. Campagna, 528

392
So. 2d 557, 558 (Fla. 4th DCA 1988) (same); McNee v. Biz, 473 So. 2d 5, 6
(Fla. 4th DCA 1985) (holding that the appellate court could not award fees
under section 57.105 against an appellee that did not cross-appeal the trial
court’s order because, as a matter of law, the appellee’s position had to
embody a justiciable issue of law or fact). At least one appellate court,
however, has awarded fees to the appellant under section 57.105. See Rapid
Credit Corp. v. Sunset Park Centre, Ltd., 566 So. 2d 810, 812 n. 2 (Fla. 3d
DCA 1990) (noting that the court had found counsel’s attempt to defend a
default on appeal so frivolous that it had granted appellant fees under section
57.105) (Schwartz, C.J., specially concurring).
All these cases were decided under a prior version of section 57.105. In
1999, the Legislature substantially rewrote that statute to significantly
broaden the courts’ authority to award attorneys’ fees under that section. As
revised, the statute reads:
(1) Upon the court’s initiative or on motion of any party, the court shall
award a reasonable attorney’s fee to be paid to the prevailing party in equal
amounts by the losing party and the losing party’s attorney on any claim or
defense at any time during a civil proceeding or action in which the court
finds that the losing party or the losing party’s attorney knew or should have
known that a claim or defense when initially presented to the court or at any
time before trial:
(a) Was not supported by the material facts necessary to establish the
claim or defense; or
(b) Would not be supported by the application of then-existing law to
those material facts.
However, the losing party’s attorney is not personally responsible if he or
she has acted in good faith, based on the representations of his or her client
as to the existence of those material facts. If the court awards attorney’s fees
to a claimant pursuant to this subsection, the court shall also award
prejudgment interest.
(2) Paragraph (1)(b) does not apply if the court determines that the claim
or defense was initially presented to the court as a good faith argument for
the extension, modification, or reversal of existing law or the establishment
of new law, as it applied to the material facts, with a reasonable expectation
of success.
(3) At any time in any civil proceeding or action in which the moving
party proves by a preponderance of the evidence that any action taken by the

393
opposing party, including, but not limited to, the filing of any pleading or
part thereof, the assertion of or response to any discovery demand, the
assertion of any claim or defense, or the response to any request by any other
party, was taken primarily for the purpose of unreasonable delay, the court
shall award damages to the moving party for its reasonable expenses
incurred in obtaining the order, which may include attorney’s fees, and other
loss resulting from the improper delay.
§ 57.105(1)-(3), Fla. Stat. (2000). As the district court noted in this case,
788 So.2d at 1061, the statute no longer applies only to an entire action; it
now applies to any claim or defense. The standard for granting fees also has
changed. Previously, a movant had to show “a complete absence of a
justiciable issue of either law or fact raised by the losing party.” § 57.105,
Fla. Stat. (Supp.1978). Under the revised version, however, a movant need
only show that the party and counsel “knew or should have known” that any
claim or defense asserted was (a) not supported by the facts or (b) not
supported by an application of “then-existing” law. § 57.105, Fla. Stat.
(2000). The amendments therefore greatly expand the statute’s potential use.
[10] [11] We hold that an appellee is not shielded as a matter of law from
the imposition of sanctions in an appropriate case. We acknowledge that an
appellee, by definition, is defending an order of the trial court. Appellate
courts, therefore, should impose sanctions against an appellee only in rare
circumstances. Moreover, because a district court of appeal is, in the vast
majority of cases, the court of last resort, it should exercise great restraint in
imposing appellate sanctions. We also warn, however, that an appellee
cannot hide behind the “presumption of correctness” of an order that the
appellee itself procured by misrepresenting the law or the facts. The
presumption of correctness is necessarily based on another presumption: that
the appellee correctly informed the trial court of the facts and applicable law.
Busy judges managing overloaded motion calendars often depend on the
attorneys appearing before them to provide them with accurate information
about the issues involved, the facts relevant to those issues, and the law
applicable to those facts. When it becomes apparent that counsel
misrepresented this information, counsel cannot later hide behind the
presumption of correctness to avoid sanctions.
[12] Appellate courts, too, must manage heavy caseloads. They depend
on counsel to accurately state both the facts and the applicable law.
Therefore, regardless of trial counsel’s conduct or representations, appellate

394
counsel (who often is separate from trial counsel) has an independent ethical
obligation to present both the facts and the applicable law accurately and
forthrightly. This will sometimes require appellate counsel to concede error
where, although trial counsel obtained a favorable result, either the facts
were not as represented to the trial court or the law is clearly contrary to the
appellee’s position and no good-faith basis exists to argue that it should be
changed.
2. Counsel’s ethical obligations
Petitioner warns that adopting a rule allowing sanctions against appellees
will require “the extreme, indeed unprofessional, act of ‘throwing in the
towel’ when there is any chance that an order may be reversed on appeal.”
This argument overlooks counsel’s professional responsibilities as officers
of the court. We do not hold that appellate counsel should concede error in
all or even many cases. And whether counsel should concede error does not
depend on the statistical chances for reversal. In (we hope) rare cases,
however, the trial court, whether because of its own misconceptions or
counsel’s misrepresentations, may incorrectly assume the relevant facts or
apply the wrong law. In such circumstances, appellate counsel has a duty to
recognize and apprise the appellate court of that fact.
Contrary to petitioner’s arguments, allowing sanctions against appellees
or their counsel for defending indefensible orders requires the
quintessentially professional act of admitting defeat when there is no chance
of victory, or when victory will have been obtained at the price of integrity
and truth. “While counsel does have an obligation to be faithful to [his]
[client’s] lawful objectives, that obligation cannot be used to justify
unprofessional conduct by elevating the perceived duty to zealously
represent over all other duties.” Lingle v. Dion, 776 So. 2d 1073, 1078 (Fla.
4th DCA 2001) (alterations in original) (quoting Visoly v. Sec. Pac. Credit
Corp., 768 So. 2d 482, 492 (Fla. 3d DCA 2000)). Section 57.105, as well as
the Florida Bar rules of professional conduct and even the oath of admission
to the Florida Bar, all warn-if any warning were needed-that counsel must be
governed by considerations other than mere zealous advocacy for the client.
See § 57.105, Fla. Stat. (2002) (allowing a court to sanction the losing party
and the losing party’s attorney if the court finds the losing party’s attorney
knew or should have known that a claim or defense was not supported by the
application of then-existing law); R. Regulating Fla. Bar 4-3.3(a)(1) ( “A
lawyer shall not knowingly make a false statement of material fact or law to

395
a tribunal.”); Oath of Admission, Fla. Bar J., Sept. 2004, at 2 (“I will employ
for the purposes of maintaining the causes confided to me such means only
as are consistent with truth and honor, and will never seek to mislead the
judge or jury by any artifice or false statement of fact or law.”). Rule 4-
3.3(a)(3) of the Rules Regulating the Florida Bar specifically prohibits an
attorney from knowingly “fail[ing] to disclose to the tribunal legal authority
in the controlling jurisdiction known to the lawyer to be directly adverse to
the position of the client and not disclosed by opposing counsel.” Therefore,
contrary to the petitioner’s assertions, the rules already require counsel to
concede error on appeal when appropriate.
This is not a new concept. Appellees regularly concede error. Cases in
which the State has conceded error on appeal are legion.FN5 * * *
Confessions of error in civil cases also occur frequently.FN6 * * * The sheer
number of such cases demonstrates that not only do the rules require counsel
to concede error in appropriate cases; counsel can and do adhere to these
rules in practice. Lawyers recognize every day when the defense of a trial
court’s order is untenable. The petitioner’s protestations that such
concessions would be “throwing in the towel” reflect an attitude that Chief
Judge Schwartz has lamented:
Too many members of the Bar practice with complete ignorance of or
disdain for the basic principle that a lawyer’s duty to his calling and to the
administration of justice far outweighs-and must outweigh-even his
obligation to his client, and, surely what we suspect really motivates many
such inappropriate actions, his interest in his personal aggrandizement.
Rapid Credit Corp., 566 So. 2d at 812 n. 1 (Schwartz, C.J., specially
concurring).
Finally, the district court’s opinion aptly responds to the petitioner’s
proposition that conceding error on appeal is unprofessional:
The heart of all legal ethics is in the lawyer’s duty of candor to a tribunal.
It is an exacting duty with an imposing burden. Unlike many provisions of
the disciplinary rules, which rely on the court or an opposing lawyer for their
invocation, the duty of candor depends on self-regulation; every lawyer must
spontaneously disclose contrary authority to a tribunal. It is counter-
intuitive, cutting against the lawyer’s principal role as an advocate. It also
operates most inconveniently-that is, when victory seems within grasp. But
it is precisely because of these things that the duty is so necessary.

396
Although we have an adversary system of justice, it is one founded on the
rule of law. Simply because our system is adversarial does not make it
unconcerned with outcomes. Might does not make right, at least in the
courtroom. We do not accept the notion that outcomes should depend on
who is the most powerful, most eloquent, best dressed, most devious and
most persistent with the last word-or, for that matter, who is able to
misdirect a judge. American civil justice is so designed that established rules
of law will be applied and enforced to insure that justice be rightly done.
Such a system is surely defective, however, if it is acceptable for lawyers to
“suggest” a trial judge into applying a “rule” or a “discretion” that they
know-or should know-is contrary to existing law. Even if it hurts the strategy
and tactics of a party’s counsel, even if it prepares the way for an adverse
ruling, even though the adversary has himself failed to cite the correct law,
the lawyer is required to disclose law favoring his adversary when the court
is obviously under an erroneous impression as to the law’s requirements.
Forum, 788 So. 2d at 1062 (footnote omitted).
3. Boca Burger’s defense of the order
[13] We now examine counsel’s actions in this case. A lower court’s
decision to impose sanctions is reviewed under an abuse of discretion
standard. See Harless v. Kuhn, 403 So. 2d 423, 425 (Fla.1981) (noting that
“[i]n the absence of an abuse of discretion the sanctions imposed by [a]
judge should stand”). As explained above, an appellate court only has
authority to impose sanctions under section 57.105 for conduct occurring in
the appellate court. Conversely, the trial court should determine any
sanctions for conduct occurring in that court. Therefore, we quash that part
of the district court’s decision that imposes sanctions for conduct that
occurred in the trial court. The district court is free to remand the issue of
trial court sanctions to the circuit court for consideration.
[***]
[14] We must now determine whether the district court abused its
discretion in imposing sanctions for conduct that occurred on appeal. We
have encountered difficulty in doing so, however. The vast majority of the
district court’s opinion discusses counsel’s representations in the trial court,
not on appeal; it appears that the district court imposed sanctions primarily
for conduct that occurred in the trial court. Therefore, we cannot determine
whether the district court intended to impose sanctions for any conduct that
occurred on appeal. We therefore remand to the district court for it to

397
reconsider its order imposing sanctions, addressing only conduct on appeal.
In so doing, we neither approve nor disapprove the imposition of sanctions.
[***]
III. CONCLUSION
We approve that part of the district court’s opinion holding that a plaintiff
has an absolute right to amend the complaint once as a matter of course
before a responsive pleading is served. It therefore follows that leave of
court is unnecessary to amend a complaint for the first time before the
defendant has filed an answer. We disapprove the district court’s opinion,
however, to the extent it holds that a preemption argument cannot be
presented in a motion to dismiss. To the contrary, we hold that it can.
Finally, we conclude that the district court had no authority to impose
sanctions for the conduct of Boca Burger and its counsel in the trial court.
Because we cannot determine how much, if any, of the district court’s
decision was based on counsel’s conduct on appeal, we remand to the
district court to consider that issue. If the district court does impose
sanctions, it should state clearly whether its sanctions are to be imposed
against Boca Burger itself, its counsel, or both. The court may also remand
to the trial court to determine whether to impose sanctions for conduct that
occurred in that court.
It is so ordered.
PARIENTE, C.J., and WELLS, ANSTEAD, and BELL, JJ., concur.
LEWIS, J., dissents with an opinion, in which QUINCE, J., concurs.
LEWIS, J., dissenting. [ * * * ]

Brass & Singer, P.A. v. United Auto. Ins. Co.


944 So. 2d 252 (Fla. 2006)
BELL, J.
We have for review the decision of the Third District Court of Appeal in
Brass & Singer, P.A. v. United Automobile Insurance Co., 919 So. 2d 473
(Fla. 3d DCA 2005), which the Third District certified to be in direct conflict
with the decision of the Fourth District Court of Appeal in Gedeon v. State
Farm Mutual Automobile Insurance Co., 805 So. 2d 119 (Fla. 4th DCA
2002). The conflict issue is a pure question of law: whether the language of
section 627.428, Florida Statutes (2004), authorizes an appellate court to

398
conditionally award appellate attorney’s fees to an insured that loses on
appeal on the basis that the insured may be the prevailing party at the close
of litigation.FN1 The Third District held that the plain meaning of section
627.428 does not authorize an appellate court to grant conditional appellate
attorney’s fees to an insured or, in this case, the insured’s assignee who lost
on appeal. Brass & Singer, P.A., 919 So. 2d at 475. FN2 In so holding, the
Third District endorsed the Fifth District’s decision in Nationwide Mutual
Insurance Co. v. Nu-Best Diagnostic Labs, Inc., 810 So. 2d 514 (Fla. 5th
DCA 2002). 919 So. 2d at 475. In Gedeon, the Fourth District held that a
circuit court acting in its appellate capacity erred in refusing to conditionally
grant appellate attorney’s fees pursuant to section 627.428 to an insured who
lost on appeal. 805 So. 2d at 119. The Fourth District reasoned that the
circuit court failed to follow the law set forth in Aksomitas v. Maharaj, 771
So. 2d 541 (Fla. 4th DCA 2000), which requires that “an appellate court
should ordinarily grant a motion for prevailing party attorney’s fees
contingent on that party ultimately prevailing in the case below.” Gedeon,
805 So.2d at 120 (citing Aksomitas, 771 So. 2d at 543-44).FN3
FN1. We have jurisdiction to resolve the certified conflict. See art. V, §
3(b)(4), Fla. Const.
FN2. Brass & Singer, P.A. are the insured’s physicians. Brass & Singer,
P.A., 919 So. 2d at 474. The insured assigned her benefits to the physicians,
and they filed suit directly against the insurer, United Automobile Insurance
Company. Id.
FN3. In Aksomitas, the Fourth District conditionally granted attorney’s
fees to the losing party on appeal, reasoning that both parties could
potentially be deemed the prevailing party at the end of all litigation under
the prevailing party test of Moritz v. Hoyt Enters., Inc., 604 So.2d 807
(Fla.1992). 771 So. 2d at 543-45. Although the Fourth District’s decision in
Aksomitas does not involve the proper interpretation of section 627.428(1),
in its analysis, it mentions that the intent behind section 627.428(1) is to
make the insured whole. Id. at 544.
For the reasons stated below, we approve the decisions of the Third and
Fifth Districts and disapprove the decision of the Fourth District.
DISCUSSION
[1] We review the district courts’ interpretation of section 627.428,
Florida Statutes (2004), de novo. B.Y. v. Dep’t of Children & Families, 887
So. 2d 1253, 1255 (Fla.2004) (“The standard of appellate review on issues

399
involving the interpretation of statutes is de novo.”). Florida Rule of
Appellate Procedure 9.400(b) is the procedural vehicle by which a party
seeks attorney’s fees from an appellate court. United Servs. Auto. Ass’n v.
Phillips, 775 So. 2d 921, 922 (Fla.2000).FN4 Pursuant to rule 9.400(b), “a
party seeking attorney’s fees in an appellate court must provide substance
and specify the particular contractual, statutory, or other substantive basis
for an award of fees on appeal.” Id. at 922.
FN4. Rule 9.400(b) provides:
(b) Attorneys’ Fees. A motion for attorneys’ fees may be served not
later than the time for service of the reply brief and shall state the grounds on
which recovery is sought. The assessment of attorneys’ fees may be
remanded to the lower tribunal. If attorneys’ fees are assessed by the court,
the lower tribunal may enforce payment.
Section 627.428 is the “statutory basis for an award of fees on appeal” to
an insured who must file suit against an insurer to enforce payment. Section
627.428(1), Florida Statutes (2004), provides as follows:
(1) Upon the rendition of a judgment or decree by any of the courts of
this state against an insurer and in favor of any named or omnibus insured or
the named beneficiary under a policy or contract executed by the insurer, the
trial court or, in the event of an appeal in which the insured or beneficiary
prevails, the appellate court shall adjudge or decree against the insurer and
in favor of the insured or beneficiary a reasonable sum as fees or
compensation for the insured’s or beneficiary’s attorney prosecuting the suit
in which the recovery is had. (Emphasis added.)
[2] [3] [4] The conflict issue here involves the proper interpretation of
this statutory language. “Although legislative intent guides our analysis, to
determine that intent we look first to the statute’s plain meaning.” V.K.E. v.
State, 934 So. 2d 1276, 1286 (Fla.2006) (citation omitted) (citing Knowles v.
Beverly Enters.-Fla., Inc., 898 So. 2d 1, 5 (Fla.2004); State v. Dugan, 685
So. 2d 1210, 1212 (Fla.1996)). “[T]he statute’s text is the most reliable and
authoritative expression of the Legislature’s intent.” Id. “[I]t is also a well-
established rule in Florida that ‘statutes awarding attorney’s fees must be
strictly construed.’ ” Dade County v. Pena, 664 So. 2d 959, 960 (Fla.1995)
(quoting Gershuny v. Martin McFall Messenger Anesthesia Prof’l Ass’n,
539 So. 2d 1131, 1132 (Fla.1989)); see also Nu-Best, 810 So. 2d at 516.
Because the text of section 627.428(1) must be strictly construed to give
effect to its plain meaning, we approve the Fifth District’s interpretation and

400
reasoning in Nu-Best, as endorsed by the Third District below. In Nu-Best,
the Fifth District strictly construed the plain meaning of the underlined
language in section 627.428(1) above to mean that an insured that loses on
appeal “is not entitled to a conditional award of fees because this is not an
appeal ‘in which the insured or beneficiary prevails.’ ” Nu-Best, 810 So.2d at
516 (quoting § 627.428, Fla. Stat.). The Third District adopted this
construction of section 627.428(1) and properly denied Brass & Singer’s
motion for a conditional award of appellate attorney’s fees. Brass & Singer,
P.A., 919 So. 2d at 475. On the other hand, we find that the Fourth District
did not give effect to the plain meaning of section 627.428(1) which
authorizes appellate attorney fees for an insured only “in the event of an
appeal in which the insured or beneficiary prevails.”
[5] Accordingly, we hold that under the plain language of section
627.428(1), an appellate court may not award attorney’s fees to an insured
unless the insured prevails on appeal. In so holding, we approve the decision
of the Third District below and disapprove the decision of the Fourth District
in Gedeon.
It is so ordered.
LEWIS, C.J., and WELLS, ANSTEAD, PARIENTE, QUINCE, and
CANTERO, JJ., concur.

Joseph Land & Co. v. Green


486 So. 2d 87 (Fla. 1st DCA 1986)
PER CURIAM.
[1] The claimant’s attorney filed a motion for attorney’s fee on March
20, 1986, alleging that this court had affirmed an order awarding claimant
benefits by a decision filed on February 19, 1986. Mandate pursuant to that
opinion was issued on March 7, 1986. The motion, having been served and
filed after the time for service of the reply brief, is not timely and is denied.
Florida Rule of Appellate Procedure 9.400.
[2] Incorporated with the motion is statement of services performed on
appeal and a claim for a fee of $1,600. Had the motion been timely filed in
accordance with the rule, this court, consistent with its policy and practice,
would have awarded a fee in that amount. Since claimant’s attorney has
negligently failed to preserve the claimant’s right to recover from the
employer and carrier the amount of the fee due for the services of his

401
attorney on appeal, no fee for such services on appeal will be approved and
the attorney shall not charge the claimant any fee for such services.
A copy of this decision shall be delivered by claimant’s attorney to his
client.
THOMPSON, ZEHMER and BARFIELD, JJ., concur.

C. Review of Cost and Fee Awards

Pellar v. Granger Asphalt Paving, Inc.


687 So. 2d 282 (Fla. 1st DCA 1997)
PADOVANO, Judge.
Joseph R. Pellar and Sweetwater Development, Inc., appeal a final order
of the trial court determining the amount of appellate attorney’s fees for their
successful defense of a prior appeal. The sole argument for reversal is that
the amount of the attorney’s fee award is inadequate. We treat the present
appeal as a motion for review under Rule 9.400(c), Florida Rules of
Appellate Procedure, and, finding no abuse of discretion, we affirm.
The original controversy arose from a dispute concerning the
performance of a road paving contract. Granger Asphalt Paving, Inc., sued
Pellar for breach of contract and unjust enrichment and to foreclose a
mechanic’s lien. In a separate lawsuit, Sweetwater and Pellar asserted a
claim against Granger for breach of the same contract. By stipulation, the
two cases were consolidated and presented in a single nonjury trial. The trial
court found that Granger had breached its contract with Sweetwater and
awarded Sweetwater damages of $53,581 against Granger. The court also
found that Sweetwater had been unjustly enriched by the labor, services, and
materials provided by Granger, and awarded Granger damages of
$44,312.80 against Sweetwater. As a result of these findings, the court
rendered judgment for Sweetwater for $9,268.20, the difference between the
two awards. The court then ruled that Granger had not prevailed against
Pellar on the mechanic’s lien claim and was therefore not entitled to any
relief against Pellar.
Granger appealed to this Court contending that the trial court erred in
consolidating the two cases and adopting a “net winner” approach to the
damages awards. Sweetwater filed a cross appeal asserting as error the trial
court’s failure to include prejudgment interest in the amount due from

402
Granger. We affirmed the appeal without discussion, but reversed on the
cross appeal and remanded with directions to include prejudgment interest.
Granger Asphalt Paving, Inc. v. Pellar, 668 So. 2d 345 (Fla. 1st DCA 1996).
We also granted the motion by Pellar and Sweetwater for appellate
attorney’s fees and remanded the case to the trial court for determination of
the amount.
Following the appeal, Pellar and Sweetwater presented their claim in the
trial court for attorney’s fees and costs totaling $10,351.51. This figure
includes $631 for reproducing the trial transcript, an expense caused by the
trial lawyer’s alleged refusal to cooperate with appellate counsel. Pellar and
Sweetwater called their appellate counsel as a witness to verify the time
records and they presented the expert testimony of another lawyer who
rendered an opinion that the fees were reasonable. Granger presented no
testimony on the issue of the reasonableness of the amount.
The trial court ruled that Granger should not be required “to repay Pellar
for the problems with his attorney” and therefore did not include the cost of
the transcript or eight hours of paralegal time attributed to preparation of the
transcript. As for the value of the legal services, the trial court concluded
that the appeal involved only a minor issue. Consequently, the court reduced
the amount requested and ordered Granger to pay Pellar and Sweetwater
$2,500 in fees with $344.61 in costs for a total of $2,844.61, with interest.
[1] Rule 9.400(c) of the Florida Rules of Appellate Procedure provides
that review of an order determining appellate costs or fees “shall be by
motion filed in the court within 30 days of rendition.” The term “court” is
used throughout the appellate rules to mean the appellate court. Fla.R.App.P.
9.020(c). Thus, the correct method of seeking review of an order on
appellate costs or attorney’s fees is to file a motion for review in the
appellate court in the proceeding that was the subject of the award, within 30
days of rendition of the order in the lower tribunal. See Cline v. Gouge, 537
So. 2d 625 (Fla. 4th DCA 1988); Altamonte Hitch and Trailer Service, Inc.
v. U-Haul Company of Eastern Florida, 483 So. 2d 852 (Fla. 5th DCA
1986); Starcher v. Starcher, 430 So. 2d 991 (Fla. 4th DCA 1983). Failure to
file a motion for review in the appellate court as required by rule 9.400(c)
may even result in a denial of the right of review. See, e.g., Magner v.
Merrill Lynch Realty/MCK, Inc., 585 So. 2d 1040 (Fla. 4th DCA 1991).
[2] An order determining the amount of appellate attorney’s fees on
remand from an appellate court has the characteristics of a final order but

403
review of such an order by a plenary appeal is not necessary or even proper.
Rule 9.400(c) enables the parties to pursue a one-step method of review that
is more practical than an appeal and much less expensive. The procedure
does not require the payment of a filing fee or the preparation of a formal
record. Testimony and other evidence before the trial court can be presented
in an appendix to the motion filed in the appellate court.
[3] Although an appeal is not the proper remedy, the filing of an appeal
in this case does not foreclose the right of review. Rule 9.040(c), Florida
Rules of Appellate Procedure, provides in material part that “[i]f a party
seeks an improper remedy, the cause shall be treated as if the proper remedy
had been sought.” In General Accident Insurance Company v. Packal, 512
So. 2d 344 (Fla. 4th DCA 1987), the court treated a notice of appeal from an
order awarding appellate attorney’s fees as a motion for review of the order
under rule 9.400(c). The same result is required here. The notice of appeal
was filed within the time for filing a motion for review and the error in the
form of the remedy did not prejudice the rights of any party. Applying the
general principle of rule 9.040(c), we treat the notice of appeal as a motion
for review of the order on appellate costs and fees.
[4] [5] An order setting the amount of appellate attorney’s fees is
reviewable on the merits by the abuse of discretion standard. G.H. Johnson
Construction Co. v. A.P.G. Electric Inc., 656 So. 2d 566 (Fla. 2d DCA
1995). As with other discretionary decisions, we must affirm the order of the
trial court if reasonable people could differ as to the propriety of the action
taken. Canakaris v. Canakaris, 382 So. 2d 1197 (Fla.1980). There is,
however, one unique aspect of the court’s review of an order setting
appellate attorney’s fees. Because the legal work was done in the appellate
court, the court can examine the file as a part of its review of the trial court
order. Dalia v. Alvarez, 605 So. 2d 1282 (Fla. 3d DCA 1992). Our
examination of the file in the previous appeal and the record of the
proceedings in the trial court on remand does not reveal an abuse of
discretion.
[6] Pellar and Sweetwater easily refuted Granger’s argument in the
previous appeal. Granger could not prevail on his argument that the trial
court erred in adopting a “net winner” approach because the parties had
stipulated to the consolidation of the cases and because the trial court had
properly segregated the individual claims. Granger’s initial appellate brief
was fifteen pages long. The answer brief filed by Pellar and Sweetwater was

404
eighteen pages long, including the cross appeal which consumed two of
those pages. For this work, the appellate attorney claimed 90.6 hours at $110
per hour. The amount of fees requested includes such items as $88 for
refiling an amended answer brief with a proper conclusion not exceeding
one page, many telephone conferences of .20 hours at $22 each, and
approximately 73 hours of work in research and preparation of the answer
brief. Because Pellar and Sweetwater were the appellees, they were not
required to initiate the appeal or prepare the record. There was no oral
argument and consequently no expense incurred for preparation or travel to
the appellate court. In similar circumstances the court in Dalia reversed an
award of $10,000 in appellate fees as clearly excessive given the
uncomplicated nature of the appeal.
The fact that the award of appellate attorney’s fees was only one fourth
the amount requested does not in itself establish an abuse of discretion. The
fallacy of this argument is that it assumes the requested amount is a
reasonable starting point in the analysis. Nor can we say that an abuse of
discretion is shown merely because the opposing party did not present
evidence to refute the claimant’s expert testimony regarding the amount of
the fees. The amount of attorney’s fees requested should be reduced if the
trial court finds that it is excessive based on the record as a whole.
We cannot substitute our judgment for that of the trial court. Pellar and
Sweetwater have not shown that the court abused its discretion in setting the
amount of appellate attorney’s fees for services rendered in the previous
appeal. Therefore we affirm.
Affirmed.
BOOTH and VAN NORTWICK, JJ., concur.

405

S-ar putea să vă placă și